ebook-batbien.pdf

July 16, 2017 | Author: Nguyễn Hữu Điển | Category: N/A
Share Embed Donate


Short Description

Download ebook-batbien.pdf...

Description

, ˜ HU ˜,U ÐIÊN NGUYÊN

, , , ˘` ´ BANG ´ GIAI TOAN PHUONG PHAP

, , ´ BIÊN ´ ÐAI LU O NG B ÂT . . , , (T´ ai ban lâ`n thu´ hai)

, ´ BAN GIAO ` XUÂT ´ DUC NHA .

517 21/727-05 GD − 05

M˜ a sô´: 8I004T5-TTS

, ` ` ´ ÐÂU LOI NOI

, D˜ı bâ´t biê´n, u´ng van . biê´n ,, Tôn tu (Binh ph´ ap)

, , , ,, ,, ´ ta thu. c hiên o môt Ta h˜ ay tuong tuo. ng c´ . hê. thô´ng trên d¯o . nh˜ung thao , , , t´ ac kh´ ac nhau. Ta c´ o thê phân t´ıch trang th´ ai cua hê. thô´ng d¯ê x´ ac d¯inh . vi. . , , , , , tr´ı (muc ac. Môt . d¯´ıch) câ`n d¯at . d¯uo. c t`u nh˜ung vi. tr´ı kh´ . trong nh˜ung công cu. , râ´t manh cho viêc a t´ınh châ´t bâ´t biê´n cua môt ¯ai . . phân t´ıch hê. thô´ng l` . sô´ d . , ,, ,, , , , , ´ ´i nh˜ung luo. ng trong hê. thông. Nh˜ung d ay không thay d¯ôi duo ¯ai . luo. ng n` , , , , `˘ng thao t´ ac kh´ ac nhau trong hê. thô´ng. Su. bâ´t biê´n c´ o thê d` ung d¯ê chı ra ra , , , ´i môt t`u môt ac. Xuyên suô´t cuô´n . câ´u h`ınh không thê d¯at . to . câ´u h`ınh kh´ , , , ,, s´ ach n` ay l` a tu tuong bâ´t biê´n thông qua c´ ac chu d¯ê` trong sô´ hoc, . d¯ai . sô´, , h`ınh hoc a tr` o choi to´ an hoc,... . v` . , ,, ,, , ˘. t tên l` Cuô´n s´ ach d¯uo. c d¯a a Phuong ph´ ap d a mong ¯ai ¯ây l` . luo. ng bâ´t biê´n, d , , ,, ´ ´ muôn cua t´ ac gia muôn biên soan ac phuong ph´ ap to´ an hoc . môt . loat . c´ . trong , , ,´, ,, ´ ´ ˜ biên soan hoc a nghiên cuu. Truoc d¯ây t´ ac gia d¯a ap . tâp . v` . môt . sô phuong ph´ , ,, , giai to´ an trong c´ ac cuô´n s´ ach [6], [7], [8], [9], [10], [11]. Ngu`oi xua l` am , , , , ´p dung ra Binh ph´ ap d¯ê a v` a giai quyê´t nh˜ung cuôc . . chiê´n tranh, d¯iên h`ınh , , ,, ˜ trong t`u,ng nhâ´t l` a bô. Binh ph´ ap cua Tôn tu v` a bô. Binh ph´ ap cua Tôn Tân , , , , ˜ trai nghiêm ´p dung th`oi k`ı m` a hai soan aa v` ao thu. c ti˜ên. ¯a . gia trên d . qua v` . , , , , ,, ˜ biê´t Binh ph´ Nhu ta d ap chı câ`n c´ o 36 muu kê´ m` a ho´ a giai d ¯a ¯uo. c hâ`u hê´t , ,, ´ ,, ˘. t ra. Ða ˘. c biêt ˘m d¯uo. c Binh c´ ac t`ınh thê´ cua c´ ac cuôc ¯a . chiê´n tranh d . ngu`oi na , , ´ , ´ ´p dung ph´ ap v` aa n´ o v` ao thu. c tê nhu thê n` ao l` a môt ang tao . . vâ´n d¯ê` s´ . cua , , , ,, , ,, , , t`ung ngu`oi v` a t`ung th`oi d o thê n´ oi c´ ac phuong ph´ ap giai to´ an l` a nh˜ung ¯ai. . C´ 3

4

,, ,, Phuong ph´ ap d ¯ai . luo. ng bâ´t biê´n

, , , ai tâp an. T´ ac gia cuô´n s´ ach mao muu kê´ trong khi giai b` . to´ . muôi . biên soan . , , , , nh˜ung phuong ph´ ap giai to´ an v` a hoc an c˜ ung hi vong th` anh bô. To´ an . to´ . , , ´ ` ´ ` ph´ ap cho m`ınh v` a c´ ac ban tham kh ao. Râ t nhiê u vâ n d ê trong c´ a ch gi ai ¯ . , , , , , ˜ suu tâ`m v` to´ an v` a hoc an c´ o thê tông kê´t lai, ac gia d a chon ¯a . to´ . t´ . loc . nh˜ung ,, , , ´,c kh´ ˘. c trung nhâ´t, nh˜ung b` phuong ph´ ap d¯a ai tâp a c´ o mu ai qu´ at cao . hay v` , , ´ ´ ` ˘c. Ban mang nôi an hoc a sâu sa ao c´ ac . dung to´ . co ban v` . d¯oc . t`ım thây phân n` , ,, ˘ phuong ph´ ap h´ oa giai c´ ac b` ai tâp ho a c m ôt c´ a ch nh` ın trong h oc t âp c˜ u ng . . . . . , , , , nhu thu. c h` anh tu. m`ınh giai b` ai tâp. . , , , , ,, , ´ ˜ ´ ˜ d¯uo. c xuâ´t ban, cuô´n s´ ach n` ay C` ung voi nhung cuôn s´ ach cua t´ ac gia d ¯a ,, ,, ˜ vâ´n d¯ê` d ´,i 5 vâ´n d¯ê`, môi quan tâm to oi gon trong m ôt chu o ng v` a l` a ¯uo. c g´ . . , ,, ,, , , , , ˜ ´p dung môt ap giai a d¯ai ¯ô´i . phuong ph´ . . luo. ng bâ´t biê´n. Môi chuong tuong d , , , , , ˜ ´ ` ´ ` ´i nhau, sau môi tiêt nho trong chu d¯ê l` ˘. t ra d¯ôc a môt . lâp . vo . vân d¯ê d¯uo. c d¯a , , , , ,, ˜ ´ ´ v` a c´ o v´ı du. minh hoa, sau d o l` a b` a i t âp a p d ung nh u ng tu tu o ng c ua tiê´t. ¯ . . . , , , , ˜ chuong c´ ˜ ˘. c trung kh´ ˘. c biêt Môi o nh˜ung tiê´t d¯a ac nhau. Ða . phâ`n cuô´i môi , , ,, ,, , , , , chuong l` a môt phuong ph´ ap cua chuong ¯ê` hay thê hiên . chuyên d . su dung . , , , ,, ˜ ´ ` ´, môi vân d¯ê o d¯ây c´ d o thê ph´ at triên th` anh nôi oi chuyên ¯o . dung môt . buôi n´ . , , , ,, ˜ ˜ ´ ngoai kh´ o a. Nh u ng b` a i t âp d u o c a p d ung c´ a ch gi ai c ua c´ a c b` a i m âu nên . . ¯ . . , , , ,, ,´, , ˜ v` không c´ o giai chi tiê´t. Chı c´ o hai chuong c´ o huo ng dân a go. i ´y cua b` ai tâp . , , , , , ˜ ´ ˘t nhu sau: trong ca chuyên d¯ê`. Nôi om ta ¯ê` d ¯uo. c t´ . dung cua môi chuyên d ,, , Chuong 1: Nguyên l´ı bâ´t biê´n. Nhiê`u b` ai to´ an cho biê´t thu. c hiên . môt . sô´ , , , , ˘. c ´ nhu c´ thao t´ ac trên môt ao d¯o ac sô´, quân b` ai, quân c`o hoa . hê. d¯ô´i tuo. ng n` , , , , , , ´ c tap ´ a nh˜ung d¯ai ˜ cho. Tuy b` nh˜ung biê´n d ai to´ an c´ o phu ¯a . nhung ân chu . , , , ,, , ˜ ´ ´ ´ ˘n le hoa ˘. c tông, t´ıch cua c´ luo. ng bât biên nhu t´ınh cha ac biên không thay , , , , ´ ´ ˜ ` ˘ d ô i. Nh o ph´ a t hi ên ra ho a c cô t` ınh d u a ra nh u ng biê n c´ o t´ınh châ´t bâ´t biê´n ¯ ¯ . . , , , , , ˘. c d¯on d¯iêu hoa ao nh˜ung d˜u kiên ¯ê´n kê´t . bâ´t biê´n, nh`o v` . bâ´t biê´n d¯ua ta d , , , , , , luân ai to´ an. Nh˜ung b` ai to´ an cua chuong n` ay l` a du. a v` ao c´ ac t´ınh châ´t . cua b` sô´ hoc, ai to´ an c˜ ung mang t´ınh c´ ac b` ai tâp a thiê´t lâp . b` . sô´ hoc. . Chuyên d¯ê` l` . , , ´ ng dung c´ ac h` am bâ´t biê´n v` au c ua n´ o . . , ,, ´,c d ´,ng hai biê´n. T´ınh bâ´t biê´n thê hiên Chuong 2: Ða thu o viêc ¯inh ¯ô´i xu . . r˜ . d

, L`oi n´ oi d ¯â`u

5

, ´,c d¯ô´i xu ´,ng: Môt ´,c hai biê´n goi ´,ng nê´u ta ngh˜ıa nh˜ung d¯a thu ad ¯ô´i xu . d¯a thu . l` , , , ´,c không thay d¯ôi vai tr` o v` a vi. tr´ı gi˜ua hai biê´n cho nhau, gi´ a tri. cua d¯a thu , , ,, ´,ng minh d ´,c d¯ô´i xu ´,ng hai thay d ¯ôi. T`u d ¯inh ¯uo. c moi ¯a thu . ngh˜ıa trên ta chu . d , , , , ´,c cua c´ ´,c d ´,ng co, so,, m` biê´n d¯ê`u biêu di˜ên nhu d ac d a c´ ac ¯a thu ¯a thu ¯ô´i xu , , , , , , , , 1 ´ ´ ´ c d¯ôi xu ´ ng co so liên quan d ´ c nghiêm ´ d¯a thu ¯ên công thu . cua Vi`ete . T`u d¯o , , , , ´ ´ ` ´ ´ ´p dung a d ang loat ac vân d ¯a thuc cho h` ¯ê trong d¯ai . . c´ . sô so câp nhu giai hê. , ,, ,, , ˜ vê´ cua ´,c d¯ô´i xu ´,ng; ´,i môi phuong tr`ınh vo c´ ac phuong tr`ınh l` a nh˜ung d¯a thu , , `˘ng c´ ´,c ra th`u,a sô´; giai phu,o,ng tr`ınh ba ˘. t ân sô´ phu; phân t´ıch d ach d¯a ¯a thu . , ,, , , , ´ ´ ` ´ ng, ... Chuyên d¯ê l` ´i hê. sô d¯ôi xu ´p b` giai phuong tr`ınh vo a môt ai to´ an d ¯a . lo , ,, ´,c bâc ´,ng, u ´,ng dung thu o c´ ac hê. sô´ d¯ô´i xu . cao c´ . chuyên d¯ê` giai phuong tr`ınh `˘ng c´ bâc ach ha. bâc. . cao ba . , , ,, ´, tu.,. Khi chu ´,c cua c´ ´,ng minh ˘ng thu Chuong 3: Bâ´t d¯a ac d˜ ay sô´ d¯ô`ng thu , , ´,c th`ı vai tr` ˘ng thu ˘. c c´ ˘ng bâ´t d¯a o c´ ac biê´n hoa ac biê´n sô´ tham gia trong bâ´t d ¯a , , , ´,c ngang nhau. Nhiê`u khi su., chuyên d thu o v` a vi. tr´ı gi˜ua c´ ac biê´n ¯ôi vai tr` , , , ˜ không thay d¯ôi, do d¯o `˘ng nh˜u,ng ´ c vân ˘ng thu ´ ta cho ra cho nhau th`ı bâ´t d¯a ,, ,, , ´, tu., n` ´ v` biê´n sô´ n` ay d¯uo. c xê´p theo môt u ao d¯o a nh`o thêm d¯iê`u kiên . th . d¯uo. c , , , , , , , ´˘p m` ´ ng minh d ´ c d˜ê d` ˘ng thu sa a chu ang hon. Trong chu d¯ê` n` ay ¯uo. c bâ´t d¯a , ,, , , , , , ´ ´ ´ ´ ´ ´ ˘ng thuc d¯uo. c tao ˘p xêp theo môt ˜ o sa ta x´et bât d ¯a . boi hai bô. sô c´ . thu tu. d¯a , , ,, ´ ´,c vê` tông c´ ˘ng thu ˘. p sô´ trong hai bô. sô´ d¯uo. c sa ˘p xê´p ac ca biê´t, d a bâ´t d ¯ây l` ¯a , , , , , , ´,ng minh bâ´t tô´i uu nhâ´t. T`u d l´ı co ban cua chuyên d o thê chu ¯inh ¯ê`, ta c´ . , , , ´ c Cauchy2 , Chebyshev3 v` ´ ng dung ˘ng thu d¯a a nhiê`u u kh´ ac. Chuyên d a ¯ê` l` . , , , , , , , , , ´ ´ ´ ´ ´ ˘ ˘ a ng th u c nô i môt bâ t d a ng th u c râ t tô ng qu´ a t, c´ o thê nh ân d u o c c´ a c bâ t d ¯ ¯ . . ¯ . , , , tiê´ng kh´ ac. Kê´t qua cua chuyên d a c´ ac ban o thê s´ ang tao ¯ê` l` . c´ . ra râ´t nhiê`u , , , ´ c mo ´i v` ˘ng thu bâ´t d¯a a râ´t d ¯ep. . , , ,, ,, ,, ,, Chuong 4: Phuong tr`ınh h` am. Ðê giai môt am, ngu`oi ta . phuong tr`ınh h` , , , ,, ,, , ,, thu`ong thay d a tri. cua d¯ô´i sô´ d ¯ôi gi´ ¯ê nhân ¯uo. c nh˜ung phuong tr`ınh th´ıch . d , , , , ´,c cua ´,i nh˜u,ng gi´ ˘ng thu ho. p rô`i suy ra h` am phai t`ım. Vo a tri. kh´ ac nhau, d ¯a ,, Vi`ete (1540-1603): Nh` a to´ an hoc ap. . ngu`oi Ph´ ,, , Louis Cauchy (1789-1857): Nh` a to´ an hoc uc. . ngu`oi д ,`, 3 Pafnuty Lvovich Chebyshev (1821-1894): Nh` a to´ an hoc . nguoi Nga. 1 Francois

2 Augustin

6

,, ,, Phuong ph´ ap d ¯ai . luo. ng bâ´t biê´n

, ,, , ,, ˘. c bâ´t biê´n, ta d¯ua ra phuong ph´ phuong tr`ınh h` am không d¯ôi, hoa ap thê´ , , ,, , , , , , ´ hai d c´ ac gi´ a tri. ap thu am l` a phuong ph´ ap ¯ê giai phuong tr`ınh h` . Phuong ph´ , , , , ` ˜ ´ ˜ d¯iêm bâ´t d ông. Chuyên d ê l` a nh u ng d a th u c giao ho´ a n, c´ a ch t` ım nh u ng d ¯. ¯ ¯ ¯a , ´ c giao ho´ thu an. ,, , , , Chuong 5: Nh˜ ung tr` o choi to´ an hoc. ai to´ an tr` o choi t`ım kiê´m . sô´ b` . Môt , , ,, , , ´˘ng v` nh˜ung chiê´n thuât a ho` a cua ngu`oi choi c˜ ung d¯ê`u du. a v` ao t´ınh bâ´t . tha , , , , ˜ cho. T`ım kiê´m môt biê´n cua nh˜ung d¯iê`u kiên ai to´ an tr` o . d¯a . l`oi giai cho b` , , , , , , , ´ ´ ˜ ` ` choi l` a môt vi êc râ t kh´ o , ngu o i ta thu o ng ph´ a t hi ên ra nh u ng biê n d o n ¯ . . . , ,, ´ ´ ˘ ´ theo môt d¯iêu bâ t biê n, ngh˜ ıa l` a c´ o t˘ a ng ho a c gi am m ôt d ai lu o. ng n` ao d¯o ¯ . . . . . , ,, ,´, ´ ` ´ quy luât Nhiêu b` ai thi hoc ac nuo c, v`ı n´ o . . cô d¯inh. . sinh gioi rât hay ra o c´ , ` ` d¯oi hoi hoc ach logic v` a thông minh. Chuyên d¯ê l` a c´ ac . sinh suy luân . môt . c´ , `, , , ´ ´ ´ chiê´n thuât trong tr` o cho i Nim, m ôt tr` o cho i xuâ t x u t u Trung Quô c. Tr` o . . , , , ,´, , ,, ´ ˘ng. choi d a chı ra buoc d ¯uo. c phân t´ıch k˜ı v` ¯i d ¯ê cho kê´t qua tha , , , , , ˜ d¯uo. c lu. a chon ac nhau, viêc Nhiê`u b` ai tâp . t`u nhiê`u nguô`n kh´ . d¯a . giai , , `˘m mô ta tô´t phu,o,ng ph´ ˘. t ra, c` ˜d v´ı du. v` a b` ai tâp ap d¯a on nh˜ung c´ ach ¯a . nha , , , , , , ´ ng minh kh´ ˜ bo qua (v´ı du. nhu bâ´t chu ac c´ o thê c´ o v` a hay hon t´ ac gia d ¯a , , ´ c Cauchy). ˘ng thu d ¯a , , Cuô´n s´ ach d` anh cho hoc an, hoc a gioi . sinh phô thông yêu to´ . sinh kh´ môn to´ an, c´ ac thâ`y cô gi´ ao, sinh viên d anh to´ an, ng` anh tin hoc ¯ai . hoc . ng` . , , , ,`, ˜ v` a nhung nguoi yêu th´ıch to´ an hoc . phô thông. Trong biên soan . không thê , ,, ˜ mong ban tr´ anh khoi sai s´ ot v` a nhâ`m lân op ´y gui vê` ¯oc . d . cho ´y kiê´n. Moi . g´ , , , d a xuâ´t ban Gi´ ao duc, a Nôi. ¯ia . chı: Nh` . 81 Trâ`n Hung Ðao, . H` . H` a Nôi, ang 9 n˘ am 2004 . th´ , , Nguy˜ên H˜uu Ðiên

, , CHUONG

1

´ BIÊN ´ NGUYÊN L´I BÂT

,, ,, , ´ d¯ai ´ i thiêu 1.1. Gio . phuong phap . luo. ng bâ´t biê´n . . . . . . . . . . . . . . . . . . . . . . . . . . . . . ´ hiên ` toan ´ ........................................ 1.2. Phat . bâ´t biê´n trong bai , ,, ´ ba ˘` ng d¯ai 1.3. Giai toan . luo. ng bâ´t biê´n . . . . . . . . . . . . . . . . . . . . . . . . . . . . . . . . . . . . . . , 1.4. Bâ´t biê´n do ¯ n di ¯ êu . ........................................................ , ` toan ´ nâng cao . . . . . . . . . . . . . . . . . . . . . . . . . . . . . . . . . . . . . . . . . . . . . . . 1.5. Nh˜ung bai ` bâ´t biê´n . . . . . . . . . . . . . . . . . . . . . . . . . . . . . . . . . . . . . . . . . . . . 1.6. Chuyên dê ¯ ` vê` ham

1.6.1. Ðinh am bâ´t biê´n trên trang ai . . . . . . . . . . . . . ngh˜ıa h` . th´ , ´ ´ ´ ` 1.6.2. Hê. thông bât biên d¯ây d¯u . . . . . . . . . . . . . . . . . . . . . . . . . .

7 9 24 27 43 51

52 58

,, ,, , ´ d¯ai ´ i thiêu 1.1. Gio . phuong phap . luo. ng bâ´t biê´n , , ˜ , , Cho a, b, c l` a nh˜ung sô´ thu. c. Ta x´et tông S = a + b + c. Nê´u ta d a ¯ôi chô , , , cho b, b cho c v` a c cho a, th`ı tông S luôn luôn chı l` a môt. ay không . Tông n` , , , , , , ´ ´ tu. thu. c hiên ´, tu., ´i thu thay d¯ôi d¯ôi vo D` u a, b, c c´ o thay d ¯ôi thu . ph´ep công. . , , , ˜ không thay d ´,i nhu thê´ n` ao ch˘ ang n˜ua S vân a S bâ´t biê´n d ¯ôi, ngh˜ıa l` ¯ô´i vo , , , viêc ac biê´n kh´ ac. Trong thu. c tê´ c˜ ung nhu trong to´ an hoc, . thay d¯ôi c´ . râ´t ,, ´,u lai nhiê`u vâ´n d¯ê` liên quan d ¯ê´n môt . sô´ d¯ô´i tuo. ng nghiên cu . bâ´t biê´n d¯ô´i , , , ,, ˜ theo ´,i su., thay d¯ôi cua nhiê`u d vo ac. V´ı du, ¯ô´i tuo. ng kh´ . nê´u ta chuyên chô , ,´, ˘. t pha ˘ng hoa ˘. c trong không gian, ´ trong môt huo ng môt ao d ¯o . h`ınh khô´i n` . ma

,, Chuong 1. Nguyên l´ı bâ´t biê´n

8

, , ´,n c´ th`ı d ai c´ ac canh, d¯ô. lo ac g´ oc cua h`ınh, khô´i không thay d¯ôi... ¯ô. d` . , , ,, ˜,a Ta c´ o thê hiêu l` a : Moi a quan hê. giu ¯ai ¯inh . d . luo. ng d . t´ınh hay t´ınh châ´t v` , , , , , ˜,ng phâ`n tu, cua môt nhu ˘. c môt a không thay d oi môt ¯ôi v´ . hoa . sô´ tâp . ho. p m` . biê´n , ,, ´d d ao d a bâ´t biê´n. ¯ôi n` ¯o ¯uo. c goi . l` , `˘ng su., bâ´t biê´n c´ Nhiê`u v´ı du. chı ra ra o trong môn sô´ hoc, a h`ınh . d¯ai . sô´ v` , hoc,... Nh˜ung b` ai to´ an c´ o liên quan d¯ê´n bâ´t biê´n chia l` am hai loai . . : , , , ˘. c kê´t qua) phai t`ım. 1. Nh˜ung b` ai to´ an lâ´y bâ´t biê´n l` am kê´t luân . (hoa , , ,, ,, Nh˜ung b` ai to´ an loai ay d¯uo. c liêt . n` . kê râ´t nhiê`u trong chuong 1 cua , , , , cuô´n s´ ach [11]. Nh˜ung bâ´t biê´n cua b` ai to´ an l` a kê´t qua cua qu´ a tr`ınh ˘. p trong nôi la ai to´ an. . dung b` , , ,, 2. Nh˜ung b` ai to´ an lâ´y t´ınh bâ´t biê´n l` am phuong ph´ ap giai. Trong cuô´n , ,, s´ ach [11] c´ o môt ay. Chuong n` ay . phâ`n râ´t nho d¯ê` câp . d¯ê´n vâ´n d¯ê` n` , ta d ac kh´ıa canh cua loai ai to´ an n` ay, c˜ ung l` a tr`ınh b` ay ¯ê` câp ¯ê´n c´ . d . . b` , ,, , ´p b` phuong ph´ ap giai cho môt ai to´ an. . lo , , B` ai to´ an xuâ´t ph´ at nhu môt . chuyên . cô t´ıch : , ,`, , , , Nguoi Nông dân trô`ng d o 99 qua chua ch´ın m` au ¯uo. c môt . cây khê´ thâ`n c´ , , ˜ ´ ˜ ch´ın m` xanh v` a 1000 qua d au v` ang. Môt ˘n môi ng` ay hai qua ¯a ¯ê n a . con Qua. d , , ,, , ˘n môt khê´ v` a n´ oi v´ oi ngu`oi nông dân: "A ang, may t´ ui ba gang . qua tra cuc . v` , , , ´ ´ ´ mang d ad ˘n hai qua khê bât k`ı không phân biêt ¯i m` ¯u. ng". Qua. d ¯ên a . qua xanh , , , v` a qua v` ang. Nê´u Qua. a ˘n môt ang v` a môt . qua v` . qua xanh th`ı cây khê´ lai . sinh , , , ´ ´ ra môt ˘n hai qua v` ang, th`ı cây khê lai . qua xanh. Nêu Qua. a . sinh ra môt . qua , , , v` ang. Nê´u Qua. a ˘n hai qua xanh th`ı cây khê´ lai ung qua v` ang. Hoi c´ o . sinh c˜ , , , ,`, , ´ ´ thê xay ra truong ho. p qua khê cuôi c` ung c` on lai a m` au v` ang không? . trên cây l` , , , , Ðê thuân a X; qua khê´ . tiên . cho viêc . giai ta k´ı hiêu . : Qua khê´ xanh l` , , ˘n qua l` ´ b` v` ang l` a V; qua. a a + v` a cây khê´ sinh ra qua l` a =. Khi d ai to´ an c´ o ¯o , ´˘n gon: thê viê´t lai . nga . V + V = V,

X + X = V,

V + X = X.

1.2. Ph´ at hiên ai to´ an . bâ´t biê´n trong b`

9

, , , `˘ng sô´ lu,o.,ng qua xanh hoa ˘. c l` T`u c´ ach viê´t trên ta thâ´y ra a không thay d¯ôi , , , ˜ lâ`n a ˜ lâ`n Qua. a ˘. c l` ˘n (môi ˘n hai qua). V`ı trên cây hoa a giam d ¯i 2 qua sau môi , , , , , ˜˘n, nên sô´ nh˜ung qua m` au xanh l` a le, c` on sô´ nh˜ung qua m` au v` ang l` a cha , , ˘n qua qua cuô´i c` ung trên cây s˜e l` a m` au xanh, không phu. thuôc ao c´ ach a . v` , cua Qua. . , , T´ınh bâ´t biê´n trong b` ai to´ an trên l` a g`ı? д o l` a sô´ nh˜ung qua xanh d` u , , , ´ , ´ ˜ ˘n qua nhu thê n` ˘. c nêu n´ Qua. c´ oa ao d¯i nua th`ı n´ o không thay d¯ôi hoa o thay , , , , ´ ` ´ d¯ôi th`ı thay d¯ôi môt ach cô d¯inh a giam d¯i hai qua. Ch´ınh d ¯iêu bât biê´n . l` . c´ , , , , , , , ´i qua xanh v` d¯ô´i vo a gia thiê´t b` ai to´ an d¯ua ta d¯ê´n l`oi giai. Nhu vây . viêc . , , ,, ´ ´ ´ ˜ ˜ cho cua b` t`ım ra t´ınh bât biên trong nhung d ai to´ an l` a rât quan ¯ai . luo. ng d¯a trong. . , , ,, Nh˜ung b` ai to´ an c´ o dang an thu`ong tô`n . nhu môt . quy tr`ınh hay thuât . to´ ,, , ,´, ,´, , tai th´ ai khoi d¯â`u v` a môt ay nh˜ung buo c d¯i ho. p lê. (buo c biê´n . môt . trang . . d˜ , , , , , , , , , d¯ôi). Kê´t luân ai to´ an loai ay thu`ong phai tra l`oi nh˜ung câu . cua nh˜ung b` . n` , hoi sau d¯ây : , ´,i d¯u,o.,c trang ˜ cho không ? 1. C´ o thê d ai cuô´i c` ung d ¯at ¯a . to . th´ , , ´,i ? 2. T`ım tâ´t ca trang ai cuô´i c` ung c´ o thê d ¯at . to . th´ ´,i han ´,i môt 3. C´ o tô`n tai ai cuô´i c` ung không ? . gio . tiê´n to . trang . th´ , , 4. T`ım tâ´t ca chu k`ı c´ o thê c´ o trong d˜ ay trang ai ? . th´ , ´,c t`ım t´ınh bâ´t biê´n trong môt Phâ`n sau d¯ây ta x´et nh˜ung c´ ach thu ai . b` , ´ to´ an nhu thê n` ao thông qua c´ ac v´ı du. .

´ hiên ` toan ´ 1.2. Phat . bâ´t biê´n trong bai , ,, , Thông qua v´ı du. sau d ¯ây, ban ¯oc ¯uo. c nh`ın t`u . d . s˜e thâ´y su. bâ´t biê´n d , ,, , nh˜ung kh´ıa canh kh´ ac nhau d¯ê`u giai d¯uo. c b` ai to´ an. . , `, tai V´ı du. 1.1. Trên bang ta viê´t 10 dâ´u công a 15 dâ´u tru ac vi. tr´ı bâ´t k`ı. . v` . c´

10

,, Chuong 1. Nguyên l´ı bâ´t biê´n

, ´ môt ´ v` oa hai dâ´u bâ´t k`ı trong d a viê´t v` ao d oa Ta thu. c hiên ¯o ¯o . x´ . dâ´u công . nê´u x´ , , , ` nê´u x´ hai dâ´u giô´ng nhau v` a dâ´u tru oa hai dâ´u kh´ ac nhau. Hoi trên bang , c` on lai ac trên 24 lâ`n? . dâ´u g`ı sau khi ta thu. c hiên . thao t´ , ˜ dâ´u công ˜ dâ´u tr`u, ba `˘ng sô´ 1, c` `˘ng sô´ `,i giai. C´ Lo ach 1: Ta thay môi on môi . ba , , −1. Thao t´ ac thu. c hiên oa hai sô´ v` a viê´t lai a t´ıch cua ch´ ung. . x´ . môt . sô´ s˜e l` , , , , ´ ´ ´ ´ V`ı thê t´ıch cua tât ca c´ ac sô viêt trên bang s˜e không thay d¯ôi. V`ı vây . ngay , , , ` ` ´ ´ ´ ˘ng −1, th`ı cuôi c` t`u d¯âu gia thiêt cho t´ıch c´ ac sô trên bang ba ung c˜ ung c` on , , ´ ´ lai a trên bang c` on lai . sô −1, ngh˜ıa l` . dâu tr`u. , ` ˜ dâ´u công `˘ng sô´ 0, c` ˘ng sô´ 1. C´ ach 2: Ta lai ba on dâ´u tr`u ba . thay môi . , , , ˜˘n th`ı ta viê´t lai Thao t´ ac thu. c hiên a tông cua hai sô´ x´ oa d¯i l` a sô´ cha . l` . sô´ , , , , ˘. c l` 0. Nhu vây ac sô´ trên bang sau khi thu. c hiên ac hoa a . tông c´ . môt . thao t´ , , , , ` ´ ˘. c l` không thay d¯ôi hoa a giam d¯i 2. Ðâu tiên tông c´ ac sô trên bang l` a môt . sô´ , , , `˘ng 15), th`ı sô´ cuô´i c` le (ba ung trên bang c` on lai a sô´ le, vây a sô´ 1. Ngh˜ıa . l` . l` , , l` a trên bang c` on dâ´u tr`u. , ,, , ˜ lâ`n thao t´ `˘ng c´ C´ ach 3: Ba ach thay nhu o c´ ach 1, bây gi`o sau môi ac, sô´ , , , ˘. c l` ˘. c l` −1 hoa a không thay d¯ôi hoa a giam d¯i 2. V`ı thê´ l´ uc ban d ¯â`u sô´ ch˜u sô´ , , , −1 l` a le, th`ı cuô´i c` ung chı c` on lai a c` on lai . môt . sô´ −1, ngh˜ıa l` . môt . dâ´u tr`u.

J

, , , , Phân t´ıch ba c´ ach giai ta thâ´y c´ ach 1 lo. i dung t´ınh không d¯ôi cua t´ıch . , , , , ,, ˜˘n c´ c´ ac sô´ viê´t trên bang; c´ ach 2 su dung t´ınh không d¯ôi cua tông cha ac sô´ . , , , , , , ˜ ˘n c´ ac dâ´u tr`u. Nhu vây ach giai v` a c´ ach 3 l` a su. không d ¯ôi cua sô´ cha . trong c´ , , , , ,, ,, ˜˘n le cua ˘. c sô´ luo. ng cha ˜ su dung ta d¯a t´ınh châ´t bâ´t biê´n cua t´ıch, tông hoa . , , `˘ng khi ga ´,p b` ˘. p nh˜ung lo c´ ac sô´. Qua c´ ach giai trên ta thâ´y ra ai to´ an m` a thao , , , , ´ ´ ´ ˘. p d¯i la ˘. p lai, t´ ac la a t`ım ra nh˜ung t´ınh bât biên cua thao . ta phai biên d¯ôi v` , , ` ˘ng c´ t´ ac ta thu. c hiên. u ´y ra ac thao t´ ac ta thu. c hiên ao . Ch´ . không phu. thuôc . v` , , , , , ´ ´ ` ´ ng minh n´ ˘t d thao t´ ac trên hai sô n` ao ba o tuong tu. nhu c´ ach ¯âu, viêc . chu , , ,, ´ ` ´ l` am trên, ban am b` ai tâp ay). Môt . d¯oc . l` . 1.18 (o cuôi phân n` . biên thê cua

1.2. Ph´ at hiên ai to´ an . bâ´t biê´n trong b`

11

,, ,´, , b` ai to´ an trên d¯uo. c cho duo i dang . nhu v´ı du. sau: , , ,, ,, on V´ı du. 1.2. Bô´n k´ı tu. X v` a n˘ am k´ı tu. O d ¯uo. c viê´t xung quanh môt ¯u`ong tr` . d , , , , ´ ´ ´ theo môt a nhu nhau th`ı ta viêt thêm . thu´ tu. bât k`ı. Nêu hai k´ı tu. canh . nhau l` , , , , , ´ ˜ a ch´ ˜,a ch´ ´ v` ao giu ung X m´ oi, nguo. c lai ao giu ung O m´ oi. Sau d ¯o . ta viêt thêm v` , , , ˜ ng k´ı tu. c˜ ta x´ oa nhu u X v` aOd ac trên la ˘. p lai ¯i. Ta thu. c hiên . thao t´ . môt . sô´ , , , , , lâ`n. Hoi sau môt a tr`ınh trên, ta c´ o nhân ¯uo. c ch´ın k´ı tu. . sô´ lâ`n thu. c hiên . qu´ . d ,, O quanh d on hay không? ¯u`ong tr` , , , , `,i giai. Nê´u ta d ˘. t X = 1 v` ´ ch´ Lo a O = −1, khi d¯o u ´y l` a gi˜ua nh˜ung k´ı tu. ¯a , , , ,, `˘ng t´ıch cua ch´ canh nhau d¯uo. c thay ba ung. Nê´u ta x´et t´ıch P cua tâ´t ca gi´ a . , ,´, , , `˘ng P mo `˘ng ´i ba tri. truoc v` a sau khi thu. c hiên ¯ôi, ta s˜e thâ´y ra . môt . lâ`n thay d , ,, , `˘ng 1 sau khi thu. c hiên ´ P luôn luôn ba b`ınh phuong cua P c˜ u. Do d¯o . thay , , , , , , `i hoi P = −1 không bao gi`o c´ d¯ôi. Nhung ch´ın k´ı hiêu o thê xay ra. . O d¯o

J

,, ˜ ô V´ı du. 1.3. Môt o canh anh 16 ô vuông, môi ¯uo. c chia th` . h`ınh vuông c´ . 4 cm d , ˜ ô vuông d ` môt ´nh ´nh dâ´u công vuông c´ o canh 1cm. Trong môi ¯a ¯a . . (+), tru . ôd , , ,, , , , ` (−). Nhu ˜ ng dâ´u o c´ dâ´u tru ac ô vuông c´ o thê thay d¯ôi d ang, ¯ô`ng th`oi theo h` , , ,`, , ˜ u han côt ˘. c d o kha n˘ ang sau hu ˘´c ¯uong ch´eo. C´ ¯ôi dâ´u theo nguyên ta . hoa . lâ`n d , ˜ d¯ê´n tâ´t ca c´ trên dân ac ô vuông d o dâ´u công ¯ê`u c´ . (+) không?. , , , ´, , ` `,i giai. Ta thay dâ´u công, ˘ng c´ Lo tr`u ba ac sô´ tuong u ng 1 v` a −1. Trang ai . . th´ , ,, ,, ,, , , ban d a h`ınh 1.1. Ðai a t´ıch c´ ac sô´ o c´ ac ô ¯â`u gia su l` . luo. ng bâ´t biê´n o d¯ây l` , , c´ o gach ac mô ta trong b` ai to´ an . ch´eo trong h`ınh 1.2, v`ı sau nh˜ung thao t´ ,, ,, d¯ai ay luôn luôn c´ o gi´ a tri. −1. Ngh˜ıa l` a trong c´ ac ô d¯uo. c gach . luo. ng n` . ch´eo , , , , luôn luôn tô`n tai o sô´ −1, suy ra không thê nhân ¯uo. c bang không . môt . ô c´ . d , , ´ a môt chu ao. . dâ´u tr`u n`

J

, , , V´ı du. 1.4. Trên bang ta viê´t tâp ac sô´ 0, 1 v` a 2. Ta thu. c hiên . ho. p sô´ gô`m c´ . , , ´ ` ´ ´ ˜ sô cua sô c` ´ chu x´ oa d ac nhau v` ad ao d¯o on lai a 2 thay ¯i hai sô kh´ ¯iên v` . (ngh˜ıa l` , ` cho 0 v` a 1; 1 thay cho 0 v` a 2; 0 thay cho 2 v` a 1). Chu´ng minh ra ˘ng nê´u sau

,, Chuong 1. Nguyên l´ı bâ´t biê´n

12

1

1

−1

1

1

1

1

1

1

1

1

1

1

1

1

1

H`ınh 1.1

H`ınh 1.2

, , , ˜, sô´ duy nhâ´t môt ac trên, trên bang chı c` on lai . sô´ lâ`n thu. c hiên . thao t´ . môt . chu , , , ˜, sô´ d ˜ c´ ´ không phu. thuôc o ao thu´ tu. thu. c hiên ac thao t´ ac c´ ac sô´ d th`ı chu ¯a ¯o . v` . c´ , trên bang. , ,, ˜ loai `,i giai. Ta thu.,c hiên Lo ac th`ı sô´ luo. ng môi . môt . lâ`n thao t´ . trong ba loai . , , , , ´ ˘. c giam d sô´ trên t˘ ang lên hoa ac loai sô thay d ô i t´ ınh ¯i 1, suy ra sô´ luo. ng c´ ¯ . , , , ˜˘n le. Khi trên bang chı c` ´ on lai m ôt sô , ngh˜ ıa l` a hai trong c´ ac sô´ 0, 1 v` a2 cha . . , , , , `˘ng không, c` `˘ng môt. ´ ba ba a ngay t`u d¯â`u sô´ on sô´ thu c´ o sô´ luo. ng ba . Ngh˜ıa l` , , , ,, ˜˘n le v` luo. ng hai sô´ trong ba sô´ trên bang phai c´ o c` ung t´ınh cha a môt . loai . sô´ , , , , ˜˘n le kh´ ´ tu. thu. c hiên c` on lai o t´ınh cha ac. V`ı thê´ không phu. thuôc ao thu . c´ . v` . , ´ ´ thao t´ ac, cuô´i c` ung chı c` on môt trong c´ a c sô 0, 1 v` a 2 c` o n l ai, sô n` a y c´ o sô´ . . , , , , ,, , , ˜˘n le cua hai sô´ kia. ˜˘n le kh´ ´i sô´ luo. ng cha luo. ng cha ac vo , , ,, ´,ng minh b` Trong chu ai to´ an trên, nê´u sô´ luo. ng ca ba loai . sô´ trên bang , , , ˜˘n le th`ı d` c´ o c` ung t´ınh cha u c´ o thu. c hiên ac thao t´ ac trên thê´ n` ao d ¯i n˜ua, . c´ , , , , cuô´i c` ung c˜ ung không thê c` on môt ach giai nh˜ung . sô´ duy nhâ´t trên bang. C´ , , , , ,, v´ı du. trên l` a râ´t d¯iên h`ınh, d¯ê cung cô´ phuong ph´ ap giai ta x´et môt ai b` ai . v`

J

to´ an sau :

1.2. Ph´ at hiên ai to´ an . bâ´t biê´n trong b`

13

, ´ ta x´ oa d a viê´t V´ı du. 1.5. Trên bang ta viê´t ba sô´ nguyên. Sau d ¯o ¯i môt . sô´ v` , , , `d ´ tông hai sô´ c` v` ao d on lai ac nhu vây ˘. p lai a ¯o ¯i 1. Thao t´ . tru . la . môt . sô´ lâ`n v` , , , , ˜ ng sô´ d cuô´i c` ung ta nhân ang nhu ¯â`u tiên ¯uo. c ba sô´ 17, 1967, 1983. Phai ch˘ . d , ,, ´ trên bang d a 2, 2, 2 ? ¯uo. c viêt l` , ,, `,i giai. B` ˜d Lo ai to´ an n` ay l` a môt ai to´ an kh´ o d¯a ¯uo. c ra trong môt . k`ı thi hoc . . b` , ,, , , ,, , , , , , , ` ´ sinh gioi o Nga. Ta tuong ch`ung nhu phai t`ım lai c´ a c bu o c th u c hi ên t u ba . . . , , , , ´,i c´ sô´ kê´t qua d ac sô´ ban d¯â`u. Nhung b` ai to´ an c´ o câu tra l`oi l` a: Vo ac ¯ê´n c´ , , , , ˜ ra, ta không thê thu. c hiên thao t´ ac d¯a ¯â`u d¯ê´n ba sô´ . biê´n d¯ôi t`u ba sô´ ban d , , ,, ´ c´ kê´t qua, khi d¯o ac c´ ach thu l` a vô vong. B` ai to´ an c´ o cho thao t´ ac biê´n d¯ôi . , ´˘t d¯â`u t`u, sô´ n` ´, tu., ra sao? Thê´ ba sô´ nhung không cho biê´t g`ı vê` ba ao v` a thu , th`ı c´ ai g`ı bâ´t biê´n trong b` ai to´ an n` ay ? Ta x´et cu. thê : ,´, ` , ,, Sau buoc d¯âu tiên t`u ba sô´ 2, 2, 2 ta nhân ay c´ o hai . d¯uo. c 2, 2, 3, ba sô´ n` , , ,, , , , , ˜ ´ ´ ´ ´ hai tro d¯i th`ı kêt qua luôn luôn c´ ´c thu ˘n v` sô cha a môt o hai . sô le. T`u buo , , , ´ ´, ´ , ˜ ´ ´ ´ ` ` ˜ ˘n v` ˘t d sô cha a môt u ta thu. c hiên ao (v`ı nhung sô´ ¯âu tu bât cu sô n` . sô le d` . ba , , , , , , , ˜˘n ba `˘ng tông cua môt ˜˘n v` cha a môt a tông cua hai . sô´ cha . sô´ le tr`u d¯i 1; sô´ le l` , , , ˜˘n tr`u, d¯i 1). Nhu,ng trong kê´t qua d¯a ´,i ˜ cho d sô´ cha a ba sô´ le ca, nên vo ¯ê`u l` , , , ˜ cho v` thao t´ ac d¯a a xuâ´t ph´ at t`u 2, 2, 2 không thê cho kê´t qua. , , , ,, , ˜˘n le cua ba sô´ không B` ai to´ an trên d¯uo. c giai nh`o ph´ at hiên . ra t´ınh cha , , , ,, thay d¯ôi, nên t`u trang th´ ai xuâ´t ph´ at không thê nhân th´ ai kê´t ¯uo. c trang . . d . , qua. , ,, ˜,ng sô´ 1, 2, 3, ..., 1974 d V´ı du. 1.6 (Kiev 1974). Nhu ¯uo. c viê´t trên môt . bang. , , ,, Ngu`oi ta thay hai sô´ bâ´t k`ı ba ˘`ng môt ˘. c l` a tông hoa ˘. c l` a hiêu . sô´ hoa . cua hai sô´ , , , ´. Chu´ng minh ra d ˘`ng sau 1973 lâ`n thu. c hiên ac trên, chı c` on môt ¯o . thao t´ . sô´ , , c` on lai a sô´ 0. . trên bang không thê l`

J

, , , ,´, ˜˘n le `,i giai. Vo ´,i kinh nghiêm Lo ai to´ an truo c, ta quan tâm d¯ê´n t´ınh cha . giai b` , , ,, ˜ lâ`n thao t´ ˜˘n le nhu, thê´ n` ˜ cho v` cua c´ ac sô´ d a sau môi ac d¯uo. c sô´ cha ao. Khi ¯a , , , , , , ˜ ´ ` ´ ` ´ ˘t d¯âu trên bang c´ ba o 987 sô le. Môi lân ta thu. c hiên . thay d¯ôi, sô cua nh˜ung

14

,, Chuong 1. Nguyên l´ı bâ´t biê´n

, , ˜˘n le kh´ ˘. c l` ˘. c sô´ le hoa a c` on nguyên (khi ta lâ´y hai sô´ c´ o t´ınh cha ac nhau hoa , ˜ ˘n) hoa ˘. c l` hai sô´ c` ung t´ınh cha a giam d ung t´ınh ¯i hai sô´ (khi ta lâ´y hai sô´ c` , , , , , , , ˜ le). Nhu vây on lai . sô´ cua nh˜ung sô´ le c` . sau môi lâ`n thu. c hiên . thay d¯ôi luôn , , , luôn l` a môt on lai ung trên bang th`ı n´ o phai . sô´ le. Vây . khi c` . môt . sô´ cuô´i c` , , ´ n´ l` a sô´ le, do d o không thê l` a 0. ¯o

J

, ,, ˜,ng sô´ 1, 0, 1, 0, V´ı du. 1.7. Môt on d anh s´ au re quat. ¯uo. c chia th` . h`ınh tr` . Nhu , ,, , , ,, 0, 0 d ao trong c´ ac re quat ay thu´ tu. theo nguo. c chiê`u kim d ¯uo. c viê´t v` ¯ô`ng hô`. . n` , , , , Ta thu. c hiên ac la ˘. p: T˘ ang sô´ cua hai re quat nhau lên 1 d ¯on vi. . thao t´ . canh . . , , , , ´ ´ ´ Khi thu. c hiên c´ a c thao t´ a c trên c´ o d u a d ê n kê t qu a c´ a c sô trong c´ a c r e qu at ¯ ¯ . . d ˘`ng nhau không ? ¯ê`u ba , , , , `,i giai. K´y hiêu Lo a nh˜ung sô´ trong c´ ac re quat . a1 , a2 , ..., a6 l` . hiên . th`oi. Khi , ,, ,, ´ sô´ S = a1 − a2 + a3 − a4 + a5 − a6 l` d¯o a môt . d¯ai . luo. ng không d¯ôi. Khoi , d¯â`u ta c´ o S = 2. дıch cuô´i c` ung ta muô´n l` a S = 0 s˜e không bao gi`o d ¯at . d¯ê´n.

J

, , V´ı du. 1.8. Ngo` ai biên d¯ông, trên môt on d ˘`n la ˘`n c´ o ¯ao sinh sô´ng giô´ng tha . h` , ba loai au: m` au x´ am c´ o 133 con; m` au nâu 155 con v` a m` au d o 177 con. ¯o c´ . m` , , Nê´u hai con tha ˘`n la ˘`n kh´ ac m` au ga ˘. p nhau, th`ı ch´ ung d au sang ¯ô`ng th`oi d ¯ôi m` , , m` au thu´ ba. (V´ı du. nê´u tha ˘`n la ˘`n m` au x´ am ga ˘. p tha ˘`n la ˘`n m` au nâu, th`ı ca hai , , ˜,ng tru,`o,ng ho.,p hai con tha con d au d ˘`n la ˘`n c` ung ¯ê`u d ¯ôi sang m` ¯o.) Trong nhu , , , ˜ nguyên không d m` au ga ˘. p nhau th`ı ch´ ung giu au. C´ o xay ra t`ınh trang a ¯ôi m` . l` , , ,, , , trên d ˘`n la ˘`n tro th` anh c` ung môt au d ¯ao tâ´t ca tha ¯uo. c không ? . m` , , , , ,´, , ˜˘n le o,, c´ `,i giai. T´ınh cha ac b` ai truo c l` a nh˜ung bâ´t biê´n râ´t tô´t d Lo ¯ê ta giai , ,, ,, ,, , ˜˘n le c˜ to´ an, t´ınh cha ung d¯uo. c x´ ac d ac sô´ cho 2. Tuong tu. ¯inh . boi ph´ep chia c´ , ,, , nhu vây a 0. . ba sô´ nguyên 133, 155, 177 chia cho 3 ta d¯uo. c bô. sô´ du 1, 2 v` ,, `˘n la `˘n x´ `˘n la `˘n nâu, th`ı ˘. p môt Ta thu x´et nê´u môt am ga . con tha . con tha , , , ´ ta c´ ch´ ung d¯ô`ng th`oi d anh m` au d¯o. Khi d¯o o 132 con x´ am, 154 con nâu ¯ôi th`

1.2. Ph´ at hiên ai to´ an . bâ´t biê´n trong b`

15

, , , , , , ´, v` a 179 con d ng l` a 0, 1 v` a a 179 cho 3 tuong u ¯o. Nh˜ung sô´ du cua 132, 154 v` , , ˘. p lai ˜ c´ 2, ngh˜ıa l` a lai ac sô´ du d¯a o. ¯â`y d ¯u c´ . ga . d , `˘n la `˘n x´ `˘n la `˘n m` ˘. p con tha Nê´u môt am ga au d¯o, th`ı ch´ ung d¯ô`ng . con tha , , `˘n la `˘n x´ `˘n la `˘n nâu ´ ta c´ th`oi d au th` anh nâu. Khi d o 132 tha am, 157 tha ¯ôi m` ¯o , , , , , `˘n la `˘n d¯o. Lâ´y nh˜ung sô´ trên chia cho 3 cho sô´ du tuong u ´,ng l` v` a 176 tha a , , , , ´ ˘. p ca ba kha n˘ 0, 1 v` a 2, ngh˜ıa l` a lai ang cua sô du. . ga , , `˘n la `˘n nâu v` `˘n la `˘n d¯o ga ˘. p nhau, th`ı ch´ Nê´u con tha a tha ung c` ung d¯ôi m` au `˘n la `˘n x´ `˘n la `˘n nâu v` `˘n la `˘n ´ c´ th` anh x´ am. Khi d¯o o 135 tha am, 154 tha a 176 tha , , , , , , , ˜ `˘n la `˘n trên chia cho 3 tuong u ´ ng l` d¯o. Sô´ du cua nh˜ung sô´ tha a 0, 1 v` a 2, vân , , c´ o d¯â`y d¯u c´ ac sô´ du khi chia cho 3. , ,, , , , Bâ´t biê´n o d a d` u thay d¯ôi m` au nhu thê´ n` ao th`ı sô´ du cua c´ ac sô´ ¯ây l` , ,, ` ` ˘n la ˘n chia cho 3 d¯ê`u c´ luo. ng tha o d¯â`y d¯u ba sô´ 0, 1, 2. , , , , `˘n la `˘n trên d¯ao l` Sô´ luo. ng tâ´t ca tha a 133 + 155 + 177 = 465 l` a môt . sô´ , , , `˘n la `˘n d chia hê´t cho 3. Nê´u tâ´t ca tha ung môt au th`ı sô´ du cua sô´ ¯ê`u c` . m` , ,, , , `˘n la `˘n m` ´,ng l` luo. ng tha au x´ am, nâu v` ad a 0, 0, 0. Ðiê`u ¯o chia cho 3 tuong u , , , , , n` ay vô l´ı v`ı c´ ac sô´ du phai c´ o d¯â`y d ac sô´ du n` ay khi chia cho 3. Nhu vây ¯u c´ . , , , ,, câu tra l`oi l` a không thê d ¯uo. c. , ,, ,, , Viêc ai to´ an . t`ım ra d¯ai . luo. ng bâ´t biê´n cua môt . d¯ô´i tuo. ng trong d˜u kiên . b` , , , , , thât ai to´ an loai ay. Su. d¯a dang cua c´ ac b` ai to´ an . lo. i hai . khi giai nh˜ung b` . n` . ,, ,´, n` ay d¯uo. c liêt . kê duoi d¯ây: , ,, ˜ ô vuông trong bang 8 × 8 d o V´ı du. 1.9. Tai ¯uo. c viê´t môt . sô´ nguyên. Ta c´ . môi , , , , , , ´ ´ ´ thê chon ˘. c 4 × 4 v` a t˘ ang tât ca sô trong bang nho . bât k`ı bang nho 3 × 3 hoa , , ,, , ˜,ng sô´ chia hê´t cho 3 lên 1. V´ oi c´ ach l` am nhu vây o thê nhân ¯uo. c nhu . liêu . c´ . d , , , , ˜,u han trong tâ´t ca ô vuông cua bang 8 × 8 sau môt . sô´ hu . lâ`n thu. c hiên . không

J

?

, , , , `,i giai. Không, không bao gi`o, c´ Lo o kê´t qua nhu vây. . Thât . vây, . ta t´ınh tông ,, ,, ˜ h`ınh vuông c´ ac sô´ trong c´ ac ô d¯uo. c gach . ch´eo trong h`ınh 1.3. Boi v`ı môi

,, Chuong 1. Nguyên l´ı bâ´t biê´n

16

, ˜ h`ınh vuông c˜o, 3 × 3 chu ´,a 12 ô gach ´,a 6 hoa ˘. c 9 c˜o 4 × 4 chu on môi . ch´eo, c`

ô,

, th`ı sau môt ac tông c´ ac sô´ . thao t´ trong c´ ac ô gach o sô´ . ch´eo chia cho 3 c´ , , , du không d ¯ôi. V`ı thê´ nê´u ngay t`u d¯â`u ,, , t´ınh d¯uo. c tông không chia chia hê´t cho , 3, th`ı trong nh˜ung ô gach ch´eo luôn . , , ´ ˜ ´,a ´ c´ luôn chua nhung ô m` a trong d¯o o chu , , c´ ac sô´ không phai bôi . cua 3. , , C` ung l´ı luân o thê ¯oc . nhu vây . ban . d . c´ , giai b` ai tâp . 1.19. H`ınh 1.3 , V´ı du. 1.10. Trên môt o 8 × 8 ô vuông bao gô`m 32 ô tra ˘´ng . bang ô vuông c´ , , , , , o thê thay tâ´t ca ô tra ˘´ng th` anh d aô v` a 32 ô d ¯en v` ¯en. Nê´u môt . ngu`oi choi c´ , ´ ´ d anh tra ˘ng c` ung môt uc trong môt ang hoa ˘. c môt o thê ¯en th` . l´ . h` . côt . bât k`ı, th`ı c´ , , , , , ,, , ˜,u han ´ng môt thu. c hiên on d ¯ôi nhu vây ¯ê trên bang chı c` ¯u . hu . buo´c thay d . d . ô

J

d ¯en hay không? , ,´, `,i giai. Không. Nê´u c´ ˘. c môt ´ ng k ô d¯en trong môt Lo od ang hoa c ¯u . h` . côt . truo , , , , ` ´ khi thu. c hiên . thay d¯ôi, th`ı sau khi thu. c hiên . môt . lân thay d¯ôi, sô ô d¯en , , ˘. c côt ´ hoa ´ s˜e l` trong h` ang d¯o a 8 − k, su. thay d¯ôi sô´ ô d¯en l` a (8 − k) − k = . d¯o , , , , ˜ ˜ ´ ˘n le cua sô´ nh˜ung ˘n, t´ınh cha 8 − 2k ô d¯en trên bang. V`ı 8 − 2k l` a môt . sô cha , , , ˜ gi˜u, nguyên tru,o ´˘t d ´,c c˜ ung nhu sau thu. c hiên ô d¯en vân ¯â`u . thay d¯ôi. Do ba , , , ,´, c´ o 32 ô d on lai c ¯en, nên không thê chı c` . môt . ô d¯en trên bang tai . môt . buo , ´ ´. biên d ao d ¯ôi n` ¯o

J

V´ı du. 1.11. , ,, Cho môt o canh 10 cm, d anh môt ¯uo. c chia ra th` . bang h`ınh vuông c´ . . 100 ô , , , , ˜ nhât ´ 25 h`ınh chu vuông nho v´ oi canh 1cm. Ngo` ai ra ta d¯a ˘. t lên d ¯o . . nhu nhau ,, ˜ h`ınh chu ˜, nhât c´ o chiê`u cao 4cm v` a chiê`u rông anh ¯uo. c chia ra th` . 1cm, môi . d

1.2. Ph´ at hiên ai to´ an . bâ´t biê´n trong b`

17

, , ˜,ng h`ınh chu ˜, nhât ˘´p d ˘. t nhu 4 ô vuông c´ o canh l` a 1cm. C´ o thê sa ¯a . trên bang . , , h`ınh vuông sao cho ch´ ung phu to` an bô. bang vuông hay không ? (Không châ´p , , , ˜, nhât nhân o h`ınh chu ao lô`i ra khoi canh cua bang). . c´ . n` . , , `˘ng m` `,i giai. Ta tô bang vuông ba Lo au , ´ ˘ng sao cho nhu h`ınh 1.4. Ta nhân d¯en tra . ,, ´˘ng. Ta ch´ a 75 ô tra d¯uo. c 25 ô d¯en v` u ´y , , , ˘. t nh˜ung h`ınh ch˜u nhât l` ad ¯a . trên bang , ˜ ô vuông cua vuông sao cho môi h`ınh , , ´i môt ch˜u nhât ung vo ao . tr` . ô vuông n` , , , ´ cua bang vuông. H`ınh ch˜u nhât d¯o ay . n` , ˘. c l` ˘. c l` s˜e phu lên hoa a 2 hoa a 0 ô vuông , , ˘. t tâ´t ca 25 h`ınh ´ suy ra khi d d¯en. T`u d¯o ¯a H`ınh 1.4 , , ch˜u nhât ung s˜e . trên bang vuông, ch´ , , ˜˘n nh˜u,ng ô vuông d¯en. Bo,,i v`ı sô´ lu,o.,ng cua ô vuông phu k´ın môt . sô´ cha , , , ˜˘n. Nhu, vây ˜ tô l` d¯en d¯a a 25, n´ o không phai l` a môt . sô´ cha . không thê phu `˘ng 25 h`ınh ch˜u, nhât ˜ cho. ba . trên h`ınh vuông d¯a , ,, ,, V´ı du. 1.12. Cho n l` a môt ac sô´ 1, 2, ..., 2n . sô´ nguyên duong le. Ngu`oi ta viê´t c´ , ,`, ´ nguoi ta lâ´y hai sô´ bâ´t k`ı a, b thuôc lên bang. Sau d ay trên, x´ oa ch´ ung d ¯o ¯i . d˜ , , , ´ |a − b|. Chu´ng minh ra v` a viê´t v` ao d ˘`ng sau môt ¯o . sô´ lâ`n thu. c hiên . nhu vây, . , ´ ´ môt sô l e s˜ e c` o n l ai cuô i c` u ng. . . , , , , , ,, , , `,i giai. K´ı hiêu Lo a tông cua tâ´t ca nh˜ung sô´ trên bang (chua x´ oa). Khoi . S l` , , ˜ bu,o ´,c S bi. giam d¯â`u S = 1 + 2 + · · · + 2n = n(2n + 1) l` a môt . sô´ le. Sau môi , ˜˘n. Nhu, vây ˜˘n le d¯i 2 min(a, b) = |a + b| − |a − b|, d¯ây l` a môt . sô´ cha . t´ınh cha , , , ,, cua S không d a tr`ınh giam dâ`n ta c´ o S ≡ 1 (mod 2). Khoi d¯â`u ¯ôi. Trong qu´ , , , S l` a môt uc s˜e c˜ ung l` a môt . sô´ le. Nhu vây . kê´t th´ . sô´ le. , , V´ı du. 1.13. Tai ac d¯ınh cua môt ac lô`i, ta ghi c´ ac sô´: 8, 3, 12, . c´ . h`ınh luc . gi´ , , ,, , ˜ lâ`n thu.,c hiên 1, 10 v` a 6. Môi o thê thêm hoa ˘. c b´ ot v` ao hai ¯ôi ngu`oi ta c´ . thay d

J

J

,, Chuong 1. Nguyên l´ı bâ´t biê´n

18

, , , , d uy ´y c` ung môt o thê ¯ınh liên tiê´p t` . sô´. Sau môt . sô´ lâ`n thu. c hiên . nhu vây, . ta c´ ,, ,, ,, , , d au sô´: 5, 2, 14, 6, 13 v` a4d ao s´ au sô´ trên ¯at ¯uo. c s´ ¯uo. c thay lâ`n luo. t thu´ tu. v` . d không? , , , , , ,, `,i giai. Gia su, tai ´, tai Lo ao d¯o ac d¯ınh cua luc ac c´ o c´ ac sô´: ¯i n` . môt . luo. t d . c´ . gi´ , a1 , a2 , a3 , a4 , a5 , a6 . X´et tông S = a1 − a2 + a3 − a4 + a5 − a6 . Khi thêm , ˜ gi˜u, nguyên gi´ v` ao hai sô´ canh nhau c` ung môt o r` ang l` a tông S vân a . . sô´ th`ı r˜ , , ,`, , ´ ´ tri. ai to´ an, tông S = 20 trong d˜ ay sô kêt qua v` a d˜ ay . Trong truong ho. p b` , , ,, ´ ` ` sô khoi d ung nhau. Vây ach chuyên tu 8, 3, 12, 1, 10, 6 ¯âu tr` . ta d¯i t`ım môt . c´ I−1,II−1 ´ ´, nhâ´t v` ´, hai tr`u, d th` anh 5, 2, 14, 6, 13, 4. K´ı hiêu −→ l` a sô thu a sô´ thu ¯i 1: . 8, 3, 12, 1, 10, 6 III+2,IV+2

−→

7, 2, 14, 3, 10, 6

−→

5, 2, 14, 6, 13, 4.

IV+3,V+3

I−1,II−1

−→

7, 2, 12, 1, 10, 6

−→

5, 2, 14, 3, 10, 4

I−2,VI−2

J

, ˜ m´ V´ı du. 1.14. Ta sa ˘´p d ˘. t ba m´ ay tu. d ay nhân ¯a ¯ông . trên môt . dây chuyê`n, môi . , , , , ´ ´ ´ d oc m ôt tâ m th e c´ o ghi hai sô nguyên v` a d u a ra m ôt tâ m th e m´ o i theo nguyên ¯. ¯ . . , , , ´ ta ˘´c sau: Sau khi d oc th e c´ o ghi c a ˘ p sô (a, b), m´ a y thu´ nhâ´t (I) in ra the c´ o ¯. . , , , ca ˘. p sô´ (a − b, b), m´ ay thu´ hai (II) in ra the c´ o ca ˘. p sô´ (a + b, b) v` a m´ ay thu´ , , ,, ba (III) in ra the c´ o ca ˘. p sô´ (b, a). Khoi d¯â`u ta cho the c´ o ca ˘. p sô´ (19, 87). C´ o , , , , , , , , , thê d` ung ba m´ ay tu. d oi ¯ông ¯ê nhân ¯uo. c the v´ . trên theo môt . thu´ tu. bâ´t k`ı d . d ca ˘. p sô´: a) (41, 14)?

b) (18, 81)?

, `˘ng c´ `,i giai. T`ım bâ´t biê´n trong b` Lo ai to´ an n` ay ba ach quan s´ at hê. thô´ng in , , , , `˘ng ca ba m´ ˘. p sô´ d¯a ˜ cho v` the v` a nh˜ung ca a kê´t qua. Ta thâ´y ra ay d¯ê`u thay , , , , ,´, ´ ´,n nhâ´t cua n´ ´, d¯ôi sô´ in ra nhung uo c sô chung lo o không thay d¯ôi do ta nho , `˘ng u,o ´,c d¯ê´n thuât an Euclid v` a c´ ach t´ınh sô´ in ra cua ba m´ ay. Ta thâ´y ra . to´ ´,n nhâ´t: (19, 87) = 1 v` sô´ chung lo a (18, 81) = 9 l` a hai sô´ kh´ ac nhau, d` u kê´t , , , , ,, , ho. p ba m´ ay nhu thê´ n` ao th`ı kê´t qua b) c˜ ung không thê xay ra. C` on tru`ong

1.2. Ph´ at hiên ai to´ an . bâ´t biê´n trong b`

19

, , , , ´i c´ ho. p a) t´ınh bâ´t biê´n thoa m˜ ac sô´ (14, 41) an v`ı (14, 41) = 1. Vây . the vo , , ,, ` , , , ˘ng c´ c´ o thê nhân ach kê´t ho. p c´ ac m´ ay tu. d trên. Nhung ta phai ¯ông . d¯uo. c ba . , , ,´, , ´,i l` chı c´ ac buo c thu. c hiên a c´ ach giai tron ai to´ an. Tâ´t nhiên c´ o râ´t . mo . ven . b` , , , , , , ` ´ ´ ` nhiêu c´ ach thu. c hiên ay d¯ê in ra kêt qua, ta chı cân chı ra môt . kêt ho. p ba m´ . , ,, , ´ ´n l` phuong a ad on c´ o tôi uu hay không d` anh cho ban K´ı hiêu ¯u (c` . d¯oc). . . sô´ , m´ ay trên m˜ ui tên l` a thu. c hiên ay n` ay: . in theo sô´ m´ I4

III

III

I

III

I

(19, 87) −→ (87, 19) −→ (11, 19) −→ (19, 11) −→ (8, 11) −→ (11, 8) −→ III

I

2

III

I

III

11

II

III

(3, 8) −→ (8, 3) −→ (2, 3) −→ (3, 2) −→ (1, 2) −→ (2, 1) −→ (13, 1) −→ II

III

II

2

J

(1, 13) −→ (14, 13) −→ (13, 14) −→ (41, 14). , , Nh˜ung b` ai to´ an vê` thay d¯ôi vi. tr´ı c´ ac sô´ trong môt ung c´ o c´ ach . bô. sô´ c˜ , , , , ´ ´ giai t`ım bât biên tuong tu. .

,, ´ ´ , , ˜,ng sô´ 1, 2, 3, ..., n d ´. Môt V´ı du. 1.15. Nhu ˘p xêp theo môt ao d¯o ¯uo. c sa . thu´ tu. n` . , , , ˜ ´ ´ ph´ep biê´n d ad nhau trong b ô sô c´ o s a ˘ n. Ch u ng ¯ôi l` ¯ôi chô˜ bâ´t k`ı hai sô´ canh . . , , , , ´ ` ´ minh ra ˘`ng nê´u ta thu. c hiên sô l e lâ n ph´ e p biê n d ô i nhu vây, ¯ . . th`ı luôn luôn , ,, , nhân ac v´ oi bô. sô´ ban d ac vi. tr´ı cua c´ ac sô´ 1, 2, ..., n. ¯uo. c môt ¯â`u vê` c´ . d . sô´ kh´ , , `,i giai. Ta k´ı hiêu Lo a môt an vi. cua bô. sô´ 1, 2, ..., n. Ta n´ oi . a1 , a2 , ..., an l` . ho´ , `˘ng hai sô´ ai v` a aj trong ho´ an vi. n` ay l` a nghich ra ¯ao nê´u i < j th`ı ai > aj . . d , Khi ta thay d nhau trong ho´ an vi, a ch´ ung ta t˘ ang ¯ôi hai sô´ canh . ngh˜ıa l` . , , , , , , , ˘. c giam sô´ luo. ng nghich hoa ac nhu ¯ao d¯i 1. Ta thu. c hiên . d . sô´ le lâ`n thao t´ , , , , ˜˘n le cua nh˜u,ng sô´ nghich ˜ biê´n d ´ ngh˜ıa vây, ¯a ¯ôi t´ınh cha ¯o . d¯ao, d¯iê`u d . th`ı ta d , ,, , , , , , `˘ng c´ ´ ng minh tuong tu. ta c´ ˜ thay d¯ôi ho´ ach chu o thê mo l` a ta d¯a an vi. . Ba , , rông b` ai to´ an n` ay nhu trong b` ai tâp ay). . . 1.20 (trong phâ`n cuô´i cua tiê´t n`

J

,, ˜,ng bê´n ô tô kh´ V´ı du. 1.16. Tai ac nhau trong môt ung . nhu . tuyê´n d¯u`ong ô tô c` ,`, ´ ` ´ xuât ph´ at môt uc 25 ô tô theo c` ung chiêu (môt a môt . l´ . tuyên d¯uong ô tô l` . con , ,, , ,`, ´ ´ ˜ ng ô tô n` duong duy nhât v` a kh´ep k´ın). Theo nguyên ta ˘c, nhu ay c´ o thê vuo. t ,, ,, ˜ nhau, nhu,ng không d ˜,ng ô tô n` qua lân a hai xe. Nhu ay c` ung kê´t ¯uo. c vuo. t qu´

,, Chuong 1. Nguyên l´ı bâ´t biê´n

20

, , ˜ xuâ´t ph´ at. Chu´ng minh ra ˘`ng th´ uc môt ong d¯ô`ng th`oi tai . bê´n ô tô m`ınh d¯a . v` ,, , trong to` an bô. th`oi gian sô´ lâ`n xe vuo. t nhau l` a môt ˘˜n. . sô´ cha , , , `˘ng m` `,i giai. Ta k´ı hiêu Lo au v` ang, nh˜ung . môt . trong nh˜ung chiê´c ô tô ba ,, , ´, tu., xuâ´t ph´ ´nh sô´ 1, 2, 3, ..., 24 theo thu chiê´c c` on lai at t`u vi. tr´ı xe . d¯uo. c d¯a ,, ˜ lâ`n c´ a n´ o l` a xe ghi sô´ v` a d¯i sau xe m` au m` au v` ang. Khi môi o xe vuo. t m` , , , , , , v` ang, th`ı d˜ ay sô´ thay d ay ¯ôi hai sô´ liê`n nhau trong tru`ong ho. p lâ`n xe vuo. t n` không tham gia xe m` au v` ang. ,, ,, , ´ vuo. t chiê´c ô tô m` Ta x´et tru`ong ho. p môt ao d¯o au v` ang. Nê´u . chiê´c xe n` , , , , ,´, , , truoc th`oi d¯iêm vuo. t nh˜ung sô´ trong bang l` a môt an vi. a1 , a2 , ..., a24 , th`ı . ho´ , ,, `˘ng sau khi vuo. t ch´ ung tao anh ho´ an vi. a2 , a3 , ..., a24 , a1 . Ta gia thiê´t ra . th` , , ,, an to` an liên tiê´p 23 lâ`n thay d ay sô´: a1 , a2 , ..., a24 → su. vuo. t nhau ho` ¯ôi d˜ a2 , a3 , ..., a24 , a1 →a3 , a4 , ..., a24 , a1 , a2 → ... → → a2 , a3 , ..., a24 , a1 . , ,, , Nê´u xe m` au v` ang thu. c hiên an vi. a1 , a2 , ..., a24 . môt . lâ`n vuo. t th`ı t`u ho´ , ,, , ta nhân an vi. a24 , a1 , a2 , ..., a23 , t`u ho´ an vi. n` ay c˜ ung biê´n d¯ôi 23 . d¯uo. c ho´ , lâ`n thay d¯ôi d˜ ay sô´. , , ,, , , ˜ lâ`n vu,o.,t d Nhu vây u trong tru`ong ho. p n` ao th`ı môi ¯ua d¯ê´n môt . d` . sô´ le , , , , ,, ,, , lâ`n chuyên d ay sô´. Nê´u tru`ong ho. p tông qu´ at sô´ lâ`n vuo. t l` a le, th`ı c´ o ¯ôi d˜ , , , , , , ´i bô. sô´ sô´ le lâ`n chuyên d¯ôi. Do b` ai to´ an 1.15 nhân ay sô´ kh´ ac vo . d¯uo. c d˜ ,, , , ´i vi. tr´ı c´ ban d¯â`u (kh´ ac vo ac xe l´ uc ban d¯â`u), nhung cuô´i c` ung c´ ac xe lai . tro ,, ˜˘n lâ`n c´ vê` vi. tr´ı ban d¯â`u, nên ta c´ o sô´ cha ac xe vuo. t nhau. ,, Râ´t nhiê`u dang ai to´ an t`ım d ¯ai . b` . luo. ng bâ´t biê´n, v´ı du. sau d¯ây t´ınh bâ´t , , ,, biê´n d¯uo. c cho ngay trong gia thiê´t cua d ai: ¯ê` b` , , , , V´ı du. 1.17. Cho d ˘. t pha ˘ng v´ oi 0 < b < a, ta du. ng ¯iêm S = (a, b) trên ma , d˜ ay c´ ac d ˘´c sau d¯ây: ¯iêm (xn , yn ) theo quy ta

J

x0 = a,

y0 = b,

xn+1 =

x n + yn , 2

, , ,, , T`ım gi´ oi han ac d ¯iêm sinh ra o trên. . cua c´

yn+1 =

2xn yn . x n + yn

1.2. Ph´ at hiên ai to´ an . bâ´t biê´n trong b`

21

, ,, , `,i giai. B` ´,c ai to´ an n` ay d˜ê t`ım thâ´y d¯ai Lo ac công thu . luo. ng bâ´t biê´n. T`u c´ , , ´,i moi trên suy ra xn+1 yn+1 = xn yn vo anh giam chı sô´ d¯ê´n . n. Ta tiê´n h` ,, , , , ´i moi xn yn = ab vo ad ¯ai . n. Ðây ch´ınh l` . luo. ng bâ´t biê´n. Khoi d¯â`u ta d¯ê`u ´,i moi c´ o y0 < x0 . Mô´i quan hê. n` ay c˜ ung bâ´t biê´n vo . n: yn < xn . Thât . vây, . , ,, , , , , ` ´ ng minh ba ´i môt ˘ng phuong ph´ ta chu ap quy nap, ao . gia su yn < xn vo . n n` , , , , ` ` ´ ng minh ra ˘ng yn+1 < xn+1 . V`ı trung b`ınh d¯iêu h` ´, ta phai chu d¯o oa thu. c su. , , nho hon trung b`ınh công, nên . xn − yn xn − yn xn − yn 0 < xn+1 − yn+1 = < · xn + yn 2 2 , , , , , ´i moi vo ay giam thu. c su. , nên ta c´ o lim xn = lim yn = x. Nhu . n. Do d˜ n→+∞ n→+∞ √ 2 vây, a x = ab. . x = ab, ngh˜ıa l` , ,, Ðai ai to´ an trên gi´ up ta râ´t nhiê`u, ta phai d` ung . luo. ng bâ´t biê´n trong b` , , , , , , , d¯ê´n hai d¯ai ai to´ an kiêu thê´ n` ay c´ o thê t`ım . luo. ng bâ´t biê´n d¯ê giai. Nh˜ung b` , , thâ´y râ´t nhiê`u trong chuong 2 cuô´n s´ ach [7].

J

` tâp Bai . , , , `˘ng nh˜u,ng c´ ´p dung Ba ach l` am cua c´ ac v´ı du. trên ta c´ o thê a v` a giai c´ ac . , , , ´ ´ b` ai tâp ai c´ o d¯ua ra go. i ´y, c` on nêu không c´ o go. i ´y, c´ ac . sau d¯ây. Môt . sô b` , ,, , , , , ˜ ` ´c v` ban ai mâu o phân truo a thu. c hiên ach giai ho` an to` an . xem lai . nh˜ung b` . c´ ,, , tuong tu. . , , . 1.18. Trên bang d (+) v` a môt ¯en viê´t môt . sô´ dâ´u công . . sô´ dâ´u tr`u (−). Cho ph´ep x´ oa d a nê´u ch´ ung c` ung dâ´u ta viê´t lai ¯i hai dâ´u bâ´t k`ı v` . dâ´u công . , , ` ´u ´ ` ´ ˘ a ng dâ (+), c` on nê´u ch´ ung kh´ ac dâ´u ta viê´t lai dâ u tr u (−). Ch u ng minh r . , , , ´c liên tiê´p thu. c hiên cuô´i c` ung không phu. thuôc ao c´ ac buo ac trên . v` . thao t´ , , , ´c). (Go. i ´y: Xem lai ai h´ ai khê´ phâ`n truo . b` , , , , , . 1.19. T`u bang trong gia thiê´t cua v´ı du. 1.9 v` a c´ ac thao t´ ac nhu vây o . c´ , ,, , , ˜ ´ ´ a môt ˘n n` nhân ao không? (Go. i ´y: Xem lai ach . d¯uo. c bang không chu . sô cha . c´ , giai v´ı du. 1.9).

22

,, Chuong 1. Nguyên l´ı bâ´t biê´n

, `˘ng kê´t luân ´,ng minh ra ´ ng, nê´u trong . 1.20. Chu on d ¯u . cua v´ı du. 1.15 c` , , , , ˜ ˜ cho. (Giai nhu v´ı ph´ep biê´n d an vi. d¯a ¯ôi ta d¯ôi chô hai sô´ bâ´t k`ı trong ho´ du. 1.15). , , ´, . 1.21. Ta x´e môt anh 10 manh, trong môt ¯o . miê´ng giâ´y th` . sô´ manh giâ´y d , , , , , , ˜ ´ ta lai anh 10 manh nho, v` a tiêp tuc o thê . x´e môi manh th` . nhu vây. . Hoi ta c´ , ,, nhân ach l` am trên 1975 manh giâ´y không? . d¯uo. c theo c´ , . 1.22. Trên bang ta viê´t c´ ac sô´ 1, 2, ..., 1975. Cho ph´ep x´ oa hai sô´ bâ´t k`ı , , , ´ sô´ du cua ph´ep chia tông hai sô´ n` v` a viê´t v` ao d¯o ay cho 13. Sau môt . sô´ lâ`n , , , , , , ´ c´ l` am nhu vây on lai o thê l` a nh˜ung sô´ ¯o . trên bang chı c` . môt . sô´, hoi sô´ d n` ao? , , , ˜ ô trong mang ˘. c dâ´u tr`u. Ta . 1.23. Trong môi c˜o 4 × 4 ta viê´t dâ´u công . hoa , , , ´,i dâ´u ngu,o.,c lai. ˘. c theo côt thu. c hiên ang hoa ¯ôi d¯ô`ng th`oi theo h` . biê´n d . vo . Sô´ , , , , ,, , , , , luo. ng dâ´u tr`u nho nhâ´t ta c´ o thê t´ınh d¯uo. c t`u mang ban d a c´ ac mang ¯â`u v` , , , , , , ˘. c trung n` ´ goi biê´n d¯ôi sau d¯o a sô´ d ˘. c trung cua bang n` ay. Sô´ d ay c´ o thê ¯a ¯a . l` ,, , nhân a tri. nhu thê´ n` ao? . d¯uo. c gi´ ,, ´˘ng v` a 20 kiê´n d¯en. . 1.24. Trên môt on c´ o 30 con kiê´n: 10 kiê´n tra ¯u`ong tr` . d , ˜ , Cho ph´ep d¯ôi chô hai con kiê´n m` a gi˜ua ch´ ung c´ o ba con kiê´n kh´ ac. Hai vi. , , , , , , , , , tr´ı cua hai con kiê´n bâ´t k`ı d¯uo. c goi a tuong d ung c´ o thê d ¯uong nê´u ch´ ¯ ôi . l` , , , , ˜ cho nhau sau môt ˜ nhu trên. Hoi tô`n tai chô . sô´ lâ`n thu. c hiên . ph´ep d¯ôi chô . , ,, ,, , bao nhiêu vi. tr´ı không tuong d¯uong cua nh˜ung con kiê´n? , ,, ´, tu., n` . 1.25. Nh˜ung sô´ 1, 2, ..., 2003 d¯uo. c viê´t theo thu ay. Môt . ph´ep biê´n , ˘. t lai ˜ chiê´m d¯ôi l` a chon ung v` ad ac vi. tr´ı ch´ ung d ¯a ¯a . bô´n sô´ bâ´t k`ı trong ch´ . c´ , , , , , , , , , ` ´ tu. c´ ˘ng ph´ep biê´n d nhung theo thu ac sô´ nguo. c lai. o thê ba ay d¯ê thu. c ¯ôi n` . C´ ,, ´˘p xê´p th` hiên anh 2003, 2002, ..., 2, 1 không ? ¯uo. c viêc . d . sa , ,, ˜ lâ`n thu.,c hiên . 1.26. Cho bô´n sô´ 4, 5, 6, 7. Môi ¯ôi ngu`oi ta viê´t 4 sô´ . thay d , ˜ sô´ mo `˘ng trung b`ınh công ´,i thay v` ´,i ba ˜ c´ mo ao bô´n sô´ c˜ u: Môi cua ba sô´ d o ¯a .

1.2. Ph´ at hiên ai to´ an . bâ´t biê´n trong b`

23

, ,´, ,, , `˘ng sau môt ´,ng minh ra truo c. Chu ¯ôi ta không bao gi`o d¯at ¯uo. c . sô´ lâ`n thay d . d , , , `˘ng tông cua bô´n sô´ a, b, c, d v` nh´ om 4 sô´: 5, 6, 7, 3 (Go. i ´y: Ch´ u ´y ra a bô´n sô´ a+b+c b+c+d c+d+a a+b+d `˘ng nhau). , , , luôn luôn ba 3 3 3 3 , , , ,, , ˘ng phân bô´ môt . 1.27. Trên môt ¯iêm. Gi˜ua moi ¯iêm . d¯u`ong tha . sô´ d . hai d , , , ´nh dâ´u thêm môt ´ gi˜ua moi liên tiê´p ta d¯a ¯o ¯iêm liên tiê´p . d¯iêm. Sau d . hai d , , ,, ´nh dâ´u thêm môt trong c´ ac d a vân vân. Sô´ ¯iêm nhân ¯uo. c ta lai . d . d¯a . d¯iêm v` , , , , , ,, , ˜ ˘n hay le sau khi ta thu. c hiên ach thêm luo. ng nh˜ung d¯iêm nhân . d¯uo. c cha . c´ , , , , d¯iêm trên 1000 lâ`n (Tra l`oi: Sô´ le).

, , ,, ˘. t hai m´ . 1.28. Ta d¯a ac thao t´ ac sau d ay ay tu. d o thê l` am d ¯ây: M´ ¯ông ¯uo. c c´ . c´ , , ˘. c A + 15 theo I cho the v` ao c´ o sô´ nguyên A in ra trên n´ o tông A + 10 hoa , , , , ` su. d ay II cho the v` ao c´ o sô´ nguyên A in ra trên n´ o hiêu ¯iêu khiên cua ta. M´ . , , , , ` `˘ng ˘. c A − 15 theo su. d¯iêu khiên cua ta. Cho the v` A − 10 hoa ao c´ o sô´ 0, ba , , , ,, ˘. c b) 123 d hai m´ ay tu. d trên ta c´ o thê in ra the c´ o sô´: a) 125 hoa ¯ông ¯uo. c . , , , , không? (Tra l`oi: a) C´ o thê; b) Không thê).

, , . 1.29. Chia môt anh nh˜ung ô vuông c´ o canh 1cm. N´ o . h`ınh ch˜u nhât . th` . , , ,, , d¯uo. c phu boi to` an bô, a không th` oi ra m´ep ngo` ai, . không chô`ng lên nhau v` , , , , , , ` ˘ng sô´ luo. ng d¯a ˜ biê´t nh˜ung manh c´ ba o hai dang: Nh˜ung manh h`ınh vuông . , , c´ o canh 2cm v` a manh h`ınh ch˜u nhât o chiê`u d` ai 4cm v` a rông 1cm. Nê´u . . c´ . , , , ` `˘ng c´ ˘ng môt ach ta thay môt a ba . manh h`ınh ch˜u nhât . v` . manh h`ınh vuông ba , , , , , , , ˜ chuyên chô nh˜ung manh c` on lai, o d¯uo. c phu . th`ı h`ınh ch˜u nhât . ban d¯â`u c´ , , , , k´ın không? (Go. i ´y: Tô h`ınh ch˜u nhât ai 1.11. Ch´ u ´y môt . nhu b` . manh h`ınh , , , ˜˘n ô vuông tra ´˘ng v` ˜˘n ô vuông d ch˜u nhât a cha on manh h`ınh ¯en, c` . phu sô´ cha , , ´˘ng. Tra l`o,i: Không d¯u,o.,c). vuông phu 1 ô vuông d¯en v` a ba ô vuông tra

24

,, Chuong 1. Nguyên l´ı bâ´t biê´n

, ,, ´ ba ˘` ng d¯ai 1.3. Giai toan . luo. ng bâ´t biê´n , ,, `˘ng c´ Ba ach ph´ at hiên ai to´ an ta . ra nh˜ung d¯ai . luo. ng bâ´t biê´n trong b` , , c´ o thê giai nhiê`u b` ai to´ an. Tuy nhiên nê´u không luyên o v` a . tâp . th`ı râ´t kh´ , , , , ,, không c´ o phuong ph´ ap s´ ang sua d¯ê giai. Tiê´t n` ay ta tiê´p tuc . t`ım hiêu thêm , , , nh˜ung c´ ach t`ım d¯ai ai to´ an. . luo. ng bâ´t biê´n trong b` , , , ,, ,, , ˜,ng viên soi nho c´ V´ı du. 1.30. C´ o ba d o sô´ luo. ng tuong u´ng ¯ô´ng soi gô`m nhu , , , ,, l` a 19, 8 v` a 9 (viên soi). Ta d a chuyên môt ¯uo. c ph´ep chon ¯ô´ng soi v` . hai d . viên , , , , , , ´ ´ ´ ` ˜ ng d ˜ chon soi cua nhu am ¯ông soi d ¯a ¯ông soi thu´ ba. Sau môt . sang d . sô lân l` , , , , ´ ` nhu vây o kha n˘ ang tao o 12 viên soi không ? ¯ông soi d ¯êu c´ . th`ı c´ . ra moi . d , , , ,, `,i giai. Không. Ða ´,ng l` ˘. t sô´ viên soi trong ba d Lo a a, b v` a c. ¯ô´ng soi tuong u , , , , Ta x´et sô´ du chia cho 3 cua nh˜ung sô´ n` ay. Khi xuâ´t ph´ at, nh˜ung sô´ d¯ô`ng , , , , du n` ay l` a 1, 2, 0. Sau môt ay l` a 0, 1, 2 v`ı . lâ`n chon . thay d¯ôi, nh˜ung sô´ du n` , , , , , , , ´ ba. Nhu vây hai d¯ô´ng soi c´ o su. chuyên môt . viên soi d¯ê´n d¯ô´ng thu . nh˜ung , ,, ´, tu., kh´ ´,i nh˜u,ng thu sô´ du luôn luôn l` a 0, 1, 2 vo ac nhau (d ¯ai . luo. ng bâ´t biê´n). , , , , ,, ´ tâ´t ca c´ ´ Do d ac d¯ô´ng soi d o 12 viên soi l` a không thê d ¯o ¯ê`u c´ ¯uo. c (v`ı khi d¯o , , , sô´ du cua ba d¯ô´ng soi l` a 0, 0, 0, vô l´ı).

J

,, , , , , V´ı du. 1.31. Hai ngu`oi choi môt o choi v´ oi hai d ¯ô´ng keo. . tr` . Ðô´ng keo . thu´ , ,, ˜ ngu,`o,i cho,i d nhâ´t c´ o 12 c´ ai v` ad o 13 c´ ai. Môi ¯ô´ng keo ¯uo. c lâ´y hai . thu´ hai c´ , , `, môt `, d c´ ai keo ˘. c chuyên môt ai keo ¯ô´ng keo ¯ô´ng thu´ nhâ´t . tu . trong hai d . hoa . c´ . tu , ,, , ,, , ˜,ng thao t´ ac trên sang d ao không thê l` am d ¯ô´ng thu´ hai. Ngu`oi choi n` ¯uo. c nhu , , , ,`, , , ´, , ` ´ coi nhu l` a thua. H˜ ay chung minh ra ˘ng nguoi choi d¯i luo. t thu hai không thê , ,, ´ c´ thua. Ngu`oi d o thê tha ˘´ng không? ¯o , , `,i giai. Ta k´ı hiêu ´, hai tr`u, Lo a gi´ a tri. tuyêt ¯ô´i cua sô´ keo . S l` . d . trong d¯ô´ng thu , , ˜ lâ`n cho,i S s˜e giam ´, nhâ´t. Kho,i d d¯i d¯ô´ng thu ¯â`u, S = |13 − 12| = 1. Sau môi , , , ˜ ˘. c t˘ hoa ang lên 2. Nhu vây, o dang: 1, 3, 1, 3, .... Môi . sô´ du cua S chia cho 4 c´ . , , , ´, , , lâ`n sau khi ngu`oi thu nhâ´t choi, sô´ du cua S chia cho 4 luôn luôn l` a 3. Ta

, ,, ˘`ng d¯ai 1.3. Giai to´ an ba . luo. ng bâ´t biê´n

25

, ,, `˘ng ngu,`o,i cho,i bi. thua khi v` thâ´y ra a chı khi không c` on c´ ai keo ao o d¯ô´ng . n` , ,, , ´, nhâ´t v` ´, hai, khi d¯o ´ S = |1 − 0| = 1. Nhu thu a chı c` on môt ai keo . c´ . o d¯ô´ng thu , ,, , ,, , ´, , ´ vây hai luôn luôn c´ o thê thu. c hiên ach choi, do d ¯o . ngu`oi choi thu . d¯uo. c c´ ,`, ´ không thua. nguoi d¯o , , ,, ,, `˘ng, hoa ˘. c l` ˘. c l` Ta thâ´y ra a tông sô´ keo a sô´ keo . o hai d¯ô´ng giam d¯i hoa . o , , , ,, , , ´, nhâ´t giam d ´ ngu`oi choi d¯ô´ng thu o choi phai c´ o kê´t th´ uc, do d ¯i, nhu vây ¯o . tr` , ´˘ng. ´, hai phai tha thu

J

, , ˜ th` V´ı du. 1.32. Môi anh viên cua môt o nhiê`u nhâ´t l` a ba d ¯ô´i thu . câu lac . bô. c´ , ,, ,, ˜ nhau). Chu´,ng minh ra trong câu lac a tuong t´ ac lân ˘`ng ¯ây l` . bô. (d¯ô´i thu o d , , , ˜ ˜ ng th` nhu anh viên cua câu lac o thê chia th` anh hai nh´ om sao cho môi . bô. c´ , ˜ nh´ th` anh viên trong môi om c´ o nhiê`u nhâ´t môt ung nh´ om. ¯ô´i thu trong c` . d , ˜ nhiên ta chia nh˜u,ng th` ´˘t d `,i giai. Khi ba Lo anh viên trong câu lac ¯â`u, ngâu . , ´ ´ ´ ˘. p d¯ôi thu trong c` bô. th` anh hai nh´ om. K´ı hiêu a sô c´ ac ca ung nh´ om. Nêu . S l` , ´ ´ môt anh viên c´ o ´ıt nhât hai d¯ôi thu trong c` ung nh´ om, th`ı th` anh viên . th` , ,, ` ´ ´ n` ay c´ o nhiêu nhât môt om kh´ ac. Th` anh viên n` ay d ¯uo. c di . d¯ôi thu trong nh´ , , chuyên sang nh´ om kh´ ac, ta s˜e giam S d¯i ´ıt nhâ´t 1. V`ı S l` a môt . sô´ nguyên , , ,, , , không âm, n´ o không thê giam m˜ ai d¯uo. c. Nhu vây . sau môt . sô´ h˜uu han . lâ`n , , , , ˜ th` chuyên d¯ôi, môi anh viên c´ o thê c´ o nhiê`u nhâ´t môt ung ¯ô´i thu trong c` . d

J

môt om. . nh´ ,, ,, ´,ng minh b` ´: Phuong ph´ Ch´ uy ap chu ai to´ an trên goi a phuong ph´ ap xuô´ng . l` , , , , , , `˘ng ta không thê giam m˜ dô´c vô han. o chı ra ra ai sô´ luo. ng khi n´ o chı c´ o . N´ , , , , h˜uu han a tri. (ban o thê xem k˜ı phuong ph´ ap n` ay trong cuô´n s´ ach . gi´ . d¯oc . c´ [9]). ,, , ˜,ng sô´ a, b, c, d. Ta thay ch´ V´ı du. 1.33. Ta ba ˘´t d oi nhu ung boi c´ ac sô´ ¯â`u v´ , a 0 = a − b, b 0 = b − c, c 0 = c − d, d 0 = d − a. Ta thu. c hiên a tr`ınh n` ay . qu´ , ` ´ ´ ´ 1996 lân. C´ o kha n˘ ang d ung A, B, C, D sao cho |BC − AD|, |AC − ¯ên sô cuôi c` , ´ ˜ ng sô nguyên tô´? BD|, |AB − CD| l` a nhu

,, Chuong 1. Nguyên l´ı bâ´t biê´n

26 , , `,i giai. Tra l`o,i: Không. Lo , ,´, ˘. p d Bô´n buo c la ¯â`u tiên cho kê´t qua a-b a-2b+c a-3b+3c-d 2a-4b+6c-4d

b-c c-d d-a b-2c+d c-2d+a d-2a+b b-3c+3d-a c-3d+3a-b d-3a+3b-c 2b-4c+6d-4a 2c-4d+6a-4b 2d-4a+6b-4c , ,´, ˜˘n. Nhu, vây ˘. p tâ´t ca c´ ´ sau 4 buo Do d c la ac sô´ d¯ê`u l` a sô´ cha ¯o . sau 1996 , , ´ ´ ´ ˘. p tât ca c´ ph´ep la ac sô l` a bôi . cua 2. V`ı thê |BC − AD|, |AC − BD|, |AB − CD| , , ´ tât ca l` a bôi o sô´ n` ao l` a sô´ nguyên tô´. . cua 4, suy ra không c´

J

, , , V´ı du. 1.34. Trong môt o n × n ô v´ oi n l` a môt . bang ô vuông c´ . sô´ le. Trong , , ˜ ô ta viê´t +1 hoa môi ˘. c −1. Goi a t´ıch tâ´t ca c´ ac sô´ thuôc ang thu´ i v` a bj . ai l` . h` , , , ` ´ ´ l` a t´ıch tât ca c´ ac sô thuôc ˘ng . côt . thu´ j (i, j = 1, 2, ..., n). Chu´ng minh ra a1 + a2 + · · · + an + b1 + b2 + · · · + bn 6= 0. , , , `˘m o,, h` `,i giai. Nê´u ta d¯ôi dâ´u cua sô´ na ´, p v` ´, q, nh˜u,ng sô´ Lo ang thu a côt . thu , , ˜ gi˜u, nguyên. Ta xem su., thay ap v` a bq c˜ ung d¯ôi dâ´u, c` on nh˜ung sô´ kh´ ac vân , , , ,´, d¯ôi cua ap + bq truo c v` a sau khi d¯ôi dâ´u: ,´, truo c

sau

,´, truo c

sau

,´, truo c

sau

,´, truo c

sau

ap

-1

+1

-1

+1

+1

-1

+1

-1

bq

-1

+1

+1

-1

-1

+1

+1

-1

a p + bq

-2

+2

0

0

0

0

+2

-2

, , `˘ng ap + bq biê´n d¯ôi ba `˘ng c´ Ta thâ´y ra ach thêm v` ao môt . sô´ bôi . cua 4. Ta x´et , , , ´,i bang n` bang chı c´ o sô´ +1, vo ay a1 + a2 + · · · + an + b1 + b2 + · · · + bn = 2n , , , ` v` a theo d a môt ay không chia hê´t cho 4. ¯iêu kiên . n l` . sô´ le, nhu vây . tông n` , , , ,, ,, , ˜ bang kh´ `˘ng c´ Boi v`ı môi ac nhân an sô´ +1 ba ach biê´n d¯ôi . d¯uo. c t`u bang to` , ,, `˘ng tông a1 + a2 + · · · + an + b1 + b2 + · · · + bn môt a ta thâ´y ra . sô´ phâ`n tu v` , , d¯ê`u không chia hê´t cho 4 v` a nhu vây ay luôn kh´ ac 0. . tông n`

J

, 1.4. Bâ´t biê´n d¯on d¯iêu .

27

` tâp Bai . , , ,, `˘ng tông c´ . 1.35. Moi ac sô´ t`u 1 d ac sô´ ¯ê´n 1 000 000 d¯uo. c thay ba . sô´ trong c´ , , , , , , ´i d˜ am nhu vây a tiê´p tuc. ch˜u sô´ cua n´ o. Vo ay sô´ ta nhân . l` . v` . Cuô´i . d¯uo. c, lai ,, , , , c` ung nhân o môt . d¯uo. c môt . triêu . sô´ c´ . ch˜u sô´. Nh˜ung sô´ 1 nhiê`u hon hay , , , , , , , nh˜ung sô´ 2 nhiê`u hon? (Go. i ´y: H˜ ay su dung sô´ du cua môt . . sô´ chia cho 9 , , , , , , , , ` ˘ng sô´ du cua tông c´ ba ac ch˜u sô´ chia cho 9. Tra l`oi: Nh˜ung sô´ 1 nhiê`u hon.) , , ˜ d¯ınh ˘. t môt . 1.36. (Hungari 1989) Tai cua môt ¯a . môi . h`ınh vuông, ta d . sô´ , , , ,, que diêm. Khoi d¯â`u chı c´ o môt a ba d ¯ınh v` ¯ınh kia . que diêm trên môt . d ,´, ´ ,`, ˘. t diêm: Nguoi ta cho ph´ep không c´ o môt ao. Buoc tiên h` anh d¯a . que diêm n` , , , ˘. t v` lâ´y môt ad ao hai d¯ınh bên canh ¯ınh v` ¯a . sô´ bâ´t k`ı nh˜ung que tai . môt . d . , , , ` ´ ` ´ ´ ´ ˘ng hai lân sô que d ˜ lây d sô que m` a tông cua ch´ ung ba ¯a ¯i. Sau môt . sô h˜uu , , ˘. t diêm, sô´ diêm tai han o thê l` a 1, 9, 8, 9 t´ınh ¯a ¯ınh h`ınh vuông c´ . lâ`n d . bô´n d ,, ,, ` ` ` ` ˘. c l` theo chiêu kim d a t´ınh theo nguo. c chiêu kim d ¯ông hô hoa ¯ô`ng hô` d¯uo. c không? , ,, ´ ´, tu., n` ˘p xê´p theo môt ´ trên môt . 1.37. Nh˜ung sô´ 1, 2, ..., n d¯uo. c sa ao d ¯o . thu . , , , , , ,`, ´ ´ ´ ng minh ˘ng. Ta c´ d¯uong tha o thê chuyên d¯ôi hai sô bât k`ı canh nhau. Chu . , , , , ` ´ ´ ´,i d˜ ˘ng môt ra ay sô luôn luôn kh´ ac vo ay . sô le nh˜ung chuyên d¯ôi tao . ra d˜ ,, ´ , , ´ ` ´ tu. ban d¯âu. ˘p xêp theo thu d¯uo. c sa

, 1.4. Bâ´t biê´n do ¯ n di ¯ êu . ,, ,´, ,, Theo d c th`ı d¯ai a môt ¯inh . ngh˜ıa bâ´t biê´n o phâ`n truo . luo. ng bâ´t biê´n l` . , , , , , ´ ´ t´ınh chât cua b` ai to´ an không thay d ac d¯ông biên d¯ôi cua hê. ¯ôi qua su. t´ . , , , ,, ´ ´ thông. Nhung khi ta thay d¯ôi hê. thông m` a c´ o d¯ai ¯ôi theo môt . luo. ng biê´n d . , , ´ ´ ´ th`ı sao ? V´ı du. nhu khi hê. thông biên d¯ôi c´ quy luât ao d¯o o môt ¯ai . n` . d . , ,, ,, , ´ ˘. c luôn luôn giam môt luo. ng luôn luôn t˘ ang hoa V´ı du. nhu ¯inh. . . luo. ng cô d , ,´, ˘. c câ´p sô´ nhân th`ı nh˜ung sô´ hang câ´p sô´ công hoa sau kh´ ac sô´ hang truo c . . .

28

,, Chuong 1. Nguyên l´ı bâ´t biê´n

,, ,, , ,, ˘. c công bôi. ´ ngu`oi ta t´ınh d¯uo. c môt a công sai hoa ¯o . l` . d¯ai . luo. ng cô´ d¯inh . T`u d , , , ´,c tông qu´ ˘. c nh˜ung tông c´ công thu at cho c´ ac sô´ hang hoa ac sô´ hang d ¯â`u . . , ,, ˘. c luôn luôn giam khi hê. thô´ng bi. tiên. Môt ang hoa . d¯ai . luo. ng luôn luôn t˘ , ,, ,, , ´ ´ t´ ac d¯ông d ad ay ta giai ¯ai ¯iêu. ¯uo. c goi . luo. ng bât biên d¯on d . Trong phâ`n n` . . l` , ,, , ,´, , nh˜ung b` ai to´ an c´ o d¯ai ung nhu phâ`n truo c ta ¯on d¯iêu. . luo. ng bâ´t biê´n d . C˜ , , , ,´, , ,, , ´ ´ ´ phai t`ım ra d¯ai ac biên cua b` ai ¯iêu . luo. ng bât biên d¯on d . duoi su. thay d¯ôi c´ , , , , , , , ˜ ´ ´ ´ to´ an. Ch´ınh nh˜ung d¯ai ai ¯on d ¯iêu ¯ên l`oi giai cua b` . luo. ng bât biên d . dân ta d to´ an. ,, ,, `˘ng môt ´,i môt Ngu`oi ta thu`ong thâ´y ra ¯at . hê. thô´ng câ`n d . to . câ´u h`ınh cu. , , , ,, , , ´ m` thê n` ao d a bâ´t biê´n d ai m˜ ai. Tru`ong ho. p ¯o ¯on d¯iêu ¯ôi m˜ . không thê thay d , , , , ,, ˘. c luôn t˘ d¯iên h`ınh l` a môt ai to´ an phai giam (hoa ang) v` a . d¯ai . luo. ng cua b` , , , , ´ ` không bao gi`o d`ung lai. Tr´ a i l ai, quy tr` ınh biê n d ô i trong d iê u ki ên c ua b` a i ¯ ¯ . . . , , , , , , ˘. c to´ an phai d`ung lai, a h˜uu han a tri. hoa . v`ı bâ´t biê´n d¯on d¯iêu . chı l` . nh˜ung gi´ , , , , , , , ´i han ´ suy ra su. vô l´ı v` l` a môt ac kha n˘ ang thay d a ¯ôi. T`u d¯o . gio . cua tâ´t ca c´ , ,, b` ai to´ an d¯uo. c giai. Ta x´et v´ı du: . , ,, , ˜ ô trong bang h`ınh chu ˜, nhât V´ı du. 1.38. Môi . m × n d¯uo. c viê´t môt . sô´ thu. c. , , Ta thu. c hiên ac sô´ trên môt ang hoa ˘. c môt ¯ôi dâ´u môt . biê´n d . lâ`n c´ . h` . côt . bâ´t k`ı. , , , , ` ` Chu´ng minh ra ˘ng ba ˘ng thao t´ ac nhu trên, ta c´ o thê d ang ¯u a d ¯ê´n trong moi . h` , , v` a trong moi ac sô´ cua n´ o l` a môt . côt . tông c´ . sô´ không âm. , , , , `,i giai. Ðê thuân ˘. c côt Lo ang hoa a môt . vêt. . Ta x´et tông tâ´t ca . tiên . ta goi . h` . l` , , , , ´,i su., thu.,c hiên c´ ac sô´ trong bang vo ay s˜e t˘ ang nê´u tông ¯ôi. Tông n` . biê´n d , , , , nh˜ung sô´ trong môt ac l` a môt ay s˜e giam khi tông . vêt . thao t´ . sô´ âm; tông n` , , , ,, c´ ac sô´ trong vêt ac l` a môt a tông không thay d¯ôi nê´u tông . thao t´ . sô´ duong v` , , , `˘ng không. Ngh˜ıa l` ´,i tông cua vêt ac ba a nê´u trong bang c´ o môt . thao t´ . vêt . vo , , `˘ng âm, th`ı ba `˘ng thao t´ c´ ac sô´ ba ac cho ph´ep trên ta t˘ ang tông tâ´t ca c´ ac sô´ , trong bang. , , , , , `˘ng thao t´ Liêu ac sô´ trong bang ba ac trên c´ o thê t˘ ang . tông cua tâ´t ca c´

, 1.4. Bâ´t biê´n d¯on d¯iêu .

29

, , ,, , lên vô han ac l` am t˘ ang . không? Không thê nhu vây . d¯uo. c v`ı nh˜ung thao t´ , , , , , , , tông c´ ac sô´ trong bang chı c´ o h˜uu han a chı c´ o h˜uu han . lâ`n, ngh˜ıa l` . nh˜ung , ˜ sô´ trong môt ´,i sô´ ban ˘. c l` bang sô´ kh´ ac nhau. Thât a tr` ung vo . vây, . môi . ô hoa , ,, , `˘ng môt ´,i sô´ ban d¯â`u ba ˘. c l` d¯â`u hoa a kh´ ac vo . dâ´u tr`u. V`ı thê´ sô´ luo. ng nh˜ung , , ,, , , , bang kh´ ac nhau không thê vuo. t qu´ a 2mn , ngh˜ıa l` a tông cua tâ´t ca c´ ac sô´ , , , , , trong bang chı c´ o thê nhân a tri. kh´ ac nhau. . h˜uu han . nh˜ung gi´ , , , Ta x´et bang sô´ ban d¯â`u. Ta chon o nh˜ung vêt o tông c´ ac sô´ l` a . trong n´ . c´ , , , , , ´ ˜ d¯uo. c giai). Ta thu. c hiên âm (nêu không c´ o vêt ao nhu vây ai to´ an d ¯a . n` . th`ı b` . , , , ,, ´ ´ ` ph´ep biên d¯ôi trên vêt ay. Trong bang vua nhân o . n` . d¯uo. c lai . t`ım thây vêt . c´ , , ,, , ´ ´ tông c´ ac sô l` a âm th`ı ta lai o v` a nhân . thu. c hiên . ph´ep biên d¯ôi trên n´ . d¯uo. c , , , , , , ˜ ´ ` ´ bang moi v` a tiêp tuc ¯ ôi . nhu vây. . Nhu vây . môi lân thu. c hiên . ph´ep biê´n d , , , , , , th`ı tông c´ ac sô´ trong bang t˘ ang lên. Tông n` ay chı c´ o thê nhân . h˜uu han . ,´, , ,, , `n ˘ ´ gi´ a tri, nên ho a c l` a t ai m ôt bu o c th u c hi ên n` a o d o ta nh ân d u o c b ang câ ¯ . . . . . . . ¯ . , , , , , , , , ´m hay muôn ´i kha n˘ ˘. c l` t`ım, hoa a so ung nhân ang tông cu. c ¯uo. c bang vo . ta c˜ . d , , , ´,i t´ınh châ´t nhu, vây d¯ai. on môt o tông âm . Bang vo . ta phai t`ım v`ı nê´u c` . vêt . c´ , , , ´ n` ao tô`n tai o th`ı ta lai . trong n´ . thu. c hiên . môt . lâ`n n˜ua ph´ep biê´n d¯ôi, khi d¯o , , , , , , , , ´i n` ´n hon tông cu. c d¯ai tông c´ ac sô´ trong bang mo ay t˘ ang lên v` a lo ac . cua c´ , sô´ trong bang, d ay vô l´ı. ¯iê`u n`

J

, , , V´ı du. 1.39. Trên ma ˘. t pha ˘ng cho n d o ba d ao na ˘`m trên ¯iêm, không c´ ¯iêm n` , , , ,`, ,`, ,`, môt ˘ng v` and ˘ng, không c´ o hai d¯uong tha ˘ng n` ao song ¯uong tha ¯uong tha . d , , , , , , , ˜,ng ` c´ song v´ oi nhau. Chu´ng minh ra ˘`ng tu ac d ay c´ o thê ha. d ¯iêm n` ¯uo. c nhu , ,, ,, ˜,ng d ˜ cho d oc d ˘ng d ¯u`ong vuông g´ ¯ôi môt ¯u`ong tha ¯a . không giao nhau lên nhu , , , , , , , , ˜ d oc. sao cho trên môi ˘ng d¯uo. c ha. chı môt ¯u`ong vuông g´ ¯u`ong tha . d , , , ,, , ,, `,i giai. Ta ha. c´ ˘ng Lo ac d¯u`ong vuông g´ oc t`u c´ ac d¯iêm xuô´ng c´ ac d¯u`ong tha , ,, ˜ d ˜ cho môt d¯a ach bâ´t k`ı (môi oc xuô´ng môt ¯iêm ha. vuông g´ ¯u`ong riêng). . c´ . d ,, ,, ˜ d¯uo. c Nê´u không c´ o hai d¯u`ong vuông g´ oc n` ao giao nhau th`ı b` ai to´ an d ¯a , giai.

30

,, Chuong 1. Nguyên l´ı bâ´t biê´n

,, ,, ,, , Trong tru`ong ho. p nguo. c lai, oc giao nhau AA1 ¯u`ong vuông g´ . ta x´et hai d , , , , ,, , , ´, , ˘ng tuong u oc t`u c´ ac d¯iêm A v` a B lên d v` a BB1 , d ng ¯uo. c ha. vuông g´ ¯u`ong tha , , , ,`, ˘ng vuông g´ α v` a β (h`ınh 1.5). Cho P l` a giao d oc n` ay. ¯iêm cua hai d¯uong tha , , , ` ˘ng AA2 v` Bây gi`o ta thay hai d oc AA1 v` a BB1 ba a BB2 l` a ¯u`ong vuông g´ , , , , , , , ´ ´ ng. Ta s˜e chu ´ ng minh nh˜ung d oc ha. xuông β v` a α tuong u ¯u`ong vuông g´ , , , , , , , ` ´i su. thay d ˘ng vo ra ai c´ ac d oc giam d¯i. ¯ôi trên th`ı tông d¯ô. d` ¯u`ong vuông g´ Thât o AA2 < AB1 < AP + PB1 v` a BB2 < BA1 < BP + PA1 , công . vây, . ta c´ . , , , ´ ´ ´ ´ c trên cho kêt qua AA2 + BB2 < AA1 + ˘ng thu theo vê hai bât d ¯a B BB1 . , , A Bây gi`o ta tiê´n h` anh l´ı luân . nhu ,´, , ´ c tranh gô`m n b` ai truoc, ta x´et bu , ,`, ,, P ˘ng v` d¯uong tha a n d¯u`ong vuông g´ oc. B2 , , A2 Ta lâ´y hai d¯u`ong vuông g´ oc giao A1 B1 , , nhau v` a thu. c hiên ¯ôi: Thay . biê´n d ,, `˘ng hai hai d¯u`ong vuông g´ oc n` ay ba , β ,`, α d¯uong vuông g´ oc c´ o tông d¯ô. d` ai , , , ´,c tranh cua ch´ ung nho hon. Trong bu ,, ,, nhân . d¯uo. c, ta lai . t`ım hai d¯u`ong vuông H`ınh 1.5 , g´ oc giao nhau v` a lai th u c hi ên thao . . . , , ,´, ˘. c l` ´ hoa t´ ac biê´n d¯ôi trên v` a tiê´p tuc a d¯ê´n môt c n` ao ¯o . nhu vây. . Khi d . buo , , , , ´, , , ´i tâ´t ca c´ ´ ta nhân d¯o ac d¯u`ong vuông g´ oc không . d¯uo. c buc tranh câ`n t`ım vo , , , , , ´ c tranh c´ ˘. c l` giao nhau; hoa a cuô´i c` ung ta nhân o tông d ai tâ´t ca ¯uo. c bu ¯ô. d` . d , , , , ,, , ,, c´ ac d¯u`ong vuông g´ oc nho nhâ´t (boi v`ı tông n` ay chı c´ o thê nhân . h˜uu han . , ,`, ` ´ ´ c´ ac gi´ a tri!). Ðây l` a b u c tranh m` a ta câ n t` ım. Th ât v ây nê u c` o n hai d u o ng ¯ . . . , ´˘t nhau th`ı ta lai ´ ca ´ a p d ung thao t´ ac thay d vuông g´ oc n` ao d¯o ¯ôi môt . . . lâ`n , , , , , , , , , , ´ c tranh vo ´i tông d n˜ua v` a nhân ai c´ ac d¯u`ong vuông g´ oc nho hon ¯uo. c bu ¯ô. d` . d , n˜ua, vô l´ı!. , , ,, `˘ng viêc Phân t´ıch c´ ach giai hai b` ai to´ an trên ta thâ´y ra ¯uo. c tiê´n . giai d , , ,, ,´, , ` ´ (b` h` anh theo môt ao môt ao d ai truo c l` a tông ¯ô: Ta d¯ua v` ¯o . so d . d¯ai . luo. ng n`

J

, 1.4. Bâ´t biê´n d¯on d¯iêu .

31

, , ,, c´ ac sô´ trong bang, b` oc) v` a môt ai n` ay l` a tông d¯ô. d` ai c´ ac d ¯u`ong vuông g´ . , , , , , , , , ao thao t´ ac biê´n d a kê´t qua cua su. biê´n d am d¯ai ¯ôi m` ¯ôi l` ¯ua v` . luo. ng ta d , , ,, , , ´c l` d¯uo. c biê´n d ac d¯inh ai truo a tông t˘ ang, ¯ôi theo môt . (trong b` . quy luât . x´ , , , , ,, , , , , , , trong b` ai sau l` a tông giam). L`oi giai du. a trên co so d¯ai ao chı . luo. ng d¯ua v` , , , , , nhân a tri. kh´ ac nhau. Suy ra thao t´ ac chı c´ o thê thu. c hiên . h˜uu han . gi´ . h˜uu , , ,, ˘ng d han a ta kha ai to´ an vê` trang ai câ`n t`ım. ¯inh ¯uo. c b` . d¯ua d . lâ`n v` . th´ , , , , `i hoi su. s´ Nhu vây, ai to´ an n` ay d¯o ang tao a nh`ın b` ai to´ an . giai loai . b` . cao v` , , ,´, , ,, , ´ ´ ´ ˜ ´ duoi g´ oc d ¯ô. biên d¯ôi voi viêc ¯on d ¯iêu . . t`ım ra nhung d¯ai . luo. ng bât biên d , , , , ` ˜ ˜ ˜ ˘. c t˘ ˘. c giam huu han (hoa ang, hoa a nhung b` ai . lân). Nhung v´ı du. sau d¯ây l` , , ,, , ` ` ˘ng so d¯ô ta d ˜ vach to´ an kh´ o nhung d¯uo. c giai ba ¯a . ra: , , , V´ı du. 1.40. Chu´ng minh ra ˘`ng 2n d ˘. t pha ˘ng l` ad ut ¯iêm bâ´t k`ı trên ma ¯â`u m´ , , cua n d ˘ng không giao nhau. ¯oan . tha , , , `,i giai. Ta ke n d ´,i c´ ˘ng vo Lo ac ¯oan . tha , , , ˜ d D ˜ cho (môi d¯iêm m´ ut l` a c´ ac d¯iêm d¯a ¯iêm A , , ,, ˘ng). chı d ad ut môt ¯uo. c l` ¯â`u m´ ¯oan . d . tha , , ˘ng Nê´u tâ´t ca c´ ac d tha ¯oan . , , , ´˘t nhau ta c´ không ca o l`oi giai cua b` ai , ,, ˘ng ˘. p d¯oan to´ an. Nguo. c lai, . ta x´et ca . tha ´˘t nhau, cho d¯o ´ l` a AB v` a CD (h`ınh ca C B , ,, ´ 1.6). Môt ph´ e p biê n d ô i d u o c x´ a c d inh ¯ ¯ ¯ . . . , , , ´ H`ınh 1.6 ˘ ˘ a ng c a t nhau nhu sau: Ta d th ¯ôi hai d ¯oan . , ˘ng không AB v` a CD th` anh hai d ¯oan . tha ´˘t nhau AC v` ca a BD. , , ,, , , Ta d ai c´ ac ¯i t`ım d¯ai ¯on d¯iêu . luo. ng bâ´t biê´n d . giam nhu sau: V`ı tông d¯ô. d` , , ,`, , , , ` ´ gi´ ´n hon tông d¯ô. d` d a CD cua tu ac lôi ABCD lo ai hai canh ¯uong ch´eo AB v` . , , , ´ ´ ´ ˘ng l` d a BD, nhu vây ai n d¯oan am bât biê´n ¯ôi diên ¯ô. d` . AC v` . ta lây tông d . tha , , , , , , , d ay chı nhân a tri. (v`ı ta chı ¯on d¯iêu. . Hiên nhiên tông n` . h˜uu han . nh˜ung gi´

32

,, Chuong 1. Nguyên l´ı bâ´t biê´n

, , ,, , , ˘ng). L´ı luân c´ o h˜uu han ach tao an to` an tuong tu. nhu c´ ac ¯oan . c´ . nd . tha . ho` , , , ,´, , , ´i tông d ˘ng vo b` ai truoc d¯ua d ai nho nhâ´t, ¯ê´n môt ¯oan ¯ô. d` . hê. thô´ng n d . tha , , ˘ng giao nhau. trong hê. thô´ng n` ay không thê c´ o hai d¯oan . tha

J

, , ,, on ta d ˘. t n sô´. Nê´u thu´ tu. c´ ac sô´ a, b, c, d V´ı du. 1.41. Trên môt ¯a ¯u`ong tr` . d , , , thoa m˜ an (a − d)(b − c) > 0, th`ı hai sô´ b v` acd ¯ôi chô˜ cho nhau. Chu´ng , ,, ,, minh ra ˘`ng sau môt on không c` on bô. tu´ n` ao sa ˘´p xê´p ¯u`ong tr` . sô´ buo´c th`ı trên d , nhu vây. . , , , ˜ ,, , `,i giai. Ph´ep biê´n d ˜d Lo ai to´ an d¯a a d¯ôi chô ¯ôi trong b` ¯uo. c cho. Thu. c châ´t l` , ,, ˜ gi˜u, nguyên. Ta phai hai sô´ trên d¯u`ong tr` on, c` on c´ ac sô´ kh´ ac vân t`ım d ¯ai . , , , ,, ´, luo. ng bâ´t biê´n c´ o liên quan d ay. Nhu vây, ¯ê´n su. thay d ¯ôi hai sô´ n` . cho thu , tu. a, b, c, d sao cho (a − d)(b − c) > 0, ngh˜ıa l` a ab + cd > ac + bd. Nê´u , , , , , ´ ´ a, b, c, d chuyên th` ´, ˜ cho th`ı t`u bô. tu thu. c hiên anh bô. tu ¯ôi d¯a . ph´ep biên d , , , , , `˘ng tông cua t´ıch c´ a, c, b, d. Ta thâ´y ra ac sô´ canh nhau bi. giam d ¯i thu. c su. : . ab + bc + cd > ac + cb + bd. , , , ,, , Ho` an to` an tu. nhiên, d¯ai a tông cua t´ıch c´ ac sô´ ¯on d¯iêu . luo. ng bâ´t biê´n d . l` , , ,, , , ´i ph´ep biê´n d ˜ cho th`ı d¯ai kê` nhau liên tiê´p trên d¯u`ong tr` on. Vo ¯ôi d¯a . luo. ng , , , , , , bâ´t biê´n d¯on d¯iêu a v`ı n´ o chı c´ o h˜uu han a tri. (tai . giam thu. c su. , v` . gi´ . sao?), , , , , , , ´ ` ´ ˜ nên ph´ep biên d¯ôi cua ta chı c´ o thê thu. c hiên ach . huu han . lân. Tiêp tuc . c´ , , ,´, giai nhu c´ ac b` ai truoc d` anh cho ban ¯oc. . d . , , , , Nh˜ung b` ai to´ an vê` h`ınh hoc o dang ung c´ o thê giai ¯ôi câ´u h`ınh c˜ . c´ . thay d ,, `˘ng phu,o,ng ph´ ap d¯ai ba . luo. ng bâ´t biê´n.

J

, `, môt V´ı du. 1.42. Tu ac l˜ om ta tiê´n h` anh thao t´ ac nhu sau: Nê´u d ac ¯a gi´ ¯a gi´ . d , , , , ,`, , ` ` ´ na ˘m vê môt oi d ˘ng AB, o d a B hai d ¯ôi v´ ¯uong tha ¯ây A v` ¯ınh không liên . ph´ıa d , , , ,, ,, ´ ` tiêp cua d ac, th`ı môt ac canh ac d¯uo. c chia ra boi hai d ¯a gi´ ¯a gi´ ¯iêm . phân c´ . cua d , , , , ,, , A v` a B, d a trung d ˘ng AB. Chu´ng ¯uo. c lâ´y d ¯ô´i xu´ng qua tâm l` ¯iêm cua d ¯oan . tha , , ˜,u han ˜ cho minh ra ˘`ng sau môt ac nhu vây ac d ¯a gi´ ¯a . sô´ hu . lâ`n thu. c hiên . thao t´ . d ` s˜e th` anh d ac lôi. ¯a gi´

, 1.4. Bâ´t biê´n d¯on d¯iêu .

33

, , `,i giai. Nhu, vây Lo . ph´ep biê´n d¯ôi , ˜ cho d¯o ´ l` d¯a a viêc ¯ôi d¯a . biê´n d X′ Y′ ⃗ ′ = Y⃗B ,, , AY gi´ ac. Ðai ¯on d¯iêu . . luo. ng bâ´t biê´n d A B , ⃗ Y ′⃗X ′ = XY ´,i ph´ep biê´n d¯ôi n` vo ay l` a diên . ⃗ , X⃗′ B = AX , t´ıch cua d ac t˘ ang d¯on d¯iêu. ¯a gi´ . , Y X `˘ng diên D˜ê thâ´y ra . t´ıch cua d¯a , , gi´ ac trong qu´ a tr`ınh biê´n d ¯ôi cua , , , d¯a gi´ ac chı c´ o thê nhân . h˜uu han . gi´ a tri. . H`ınh 1.7 , Thât . vây, . ta x´et bô. vecto doc . theo , , , , ´,i môt c´ ac canh ac. Vo ac bô. vecto không thay d¯ôi, m` a chı ¯a gi´ . cua d . lâ`n thao t´ , , , ´, tu., liên tiê´p cua c´ thay d ac vecto canh nhau (h`ınh 1.7 minh hoa ¯ôi thu . . môt . , , , ,`, ,, , , ´ ac nhân truong ho. p cu. thê). Ngh˜ıa l` a sô luo. ng nh˜ung d ¯a gi´ . d¯uo. c thông qua , , , , , ,, ´ suy ra d¯ai thu. c hiên ac thao trên chı c´ o thê l` a h˜uu han. ¯o . c´ . T`u d . luo. ng bâ´t , , ,, , , biê´n d a tri. ¯on d ¯iêu . . cua ta chı nhân . d¯uo. c h˜uu han . gi´

J

, ,, ,, ao c´ ac ph` ong cua môt oa nh` a 115 V´ı du. 1.43. 2000 ngu`oi d ¯uo. c chia v` . t` , , , ,, , , ˜ ph´ buô`ng. Môi ut không phai moi ong, m` a môt ¯ê`u o trong ph` . ngu`oi d . sô´ ngu`oi ,, , , ,, , , `, ph` d ong ´ıt ngu`oi t´ oi ph` ong c´ o nhiê`u ngu`oi hon. Chu´ng minh ra ˘`ng t´ oi môt ¯i t u . , ,`, ` ´ ´ l´ uc n` ao d o tâ t c a m oi ngu o i d ê u t âp trung t ai m ôt ph` o ng. ¯ ¯ . . . . , , , ,, , , ,, `,i giai. Kê´t luân Lo ai to´ an tuong ch`ung nhu không tin d¯uo. c. B` ai to´ an . cua b` , ,, , , , , , ˜ ´ ´ ´i môi ph` d ong ta x´et b`ınh phuong ¯uo. c giai nh`o t`ım ra bât biên d¯on d¯iêu. . Vo , , , ,`, ,, ´ ´ cua sô nguoi trong ph` ong. K´ı hiêu ac sô b`ınh phuong l` a S. Ta chı ra . tông c´ , , ,, ,, ,, , ˜ ngu,`o,i d `˘ng S t˘ ´,i môi ra ang vo ¯uo. c chuyên d ¯i. Thât . vây, . gia su môt . ngu`oi d¯i t`u ,, , , , ´, ´ nh˜ung sô´ b`ınh ph` ong c´ o n ngu`oi to i ph` ong c´ o m ngu`oi, m` a m > n. Khi d ¯o , , ,, ,, , phuong cua sô´ ngu`oi trong c´ ac ph` ong biê´n d¯ôi t`u n2 v` a m2 th` anh (n − 1)2 , , , ´, , v` a (m + 1)2 tuong u ng v` a c´ ac ph` ong kh´ ac th`ı không d¯ôi. Nhu vây . S thay

,, Chuong 1. Nguyên l´ı bâ´t biê´n

34 , , d¯ôi nhu sau: ((n−1)2 + (m + 1)2 ) − (n2 + m2 ) =

= (n2 + m2 − 2n + 2m + 2) − (n2 + m2 ) = 2(m − n) + 2 > 2 > 0. Vây ang. . S luôn luôn t˘ , `˘ng sô´ ngu,`o,i l` Ta biê´t ra a 2000 (môt . bâ´t biê´n). Tô`n tai . môt . sô´ h˜uu han . ,`, , , ` c´ ach chia 2000 nguoi v` ao c´ ac ph` ong kh´ ac nhau, nhu vây . tôn tai . h˜uu han . , , , , ´ kha n˘ ang gi´ a tri. cua S. Ngh˜ıa l` a S không thê t˘ ang m˜ ai m˜ ai. Nhung d¯ên môt . , ,`, ,, ´, quy tr`ınh b` a l´ uc hai ph` ong c` on nguoi trong d ai to´ an d¯uo. c thu. c hiên ¯o . v` , , , , ´ ´ ` ´ tât ca moi S s˜e t˘ ang v` a qu´ a tr`ınh n` ay d uc d¯o ¯ên khi d`ung lai, . l´ . ngu`oi d¯êu trong môt ong. . ph`

J

, , ,, V´ı du. 1.44. Cho bô´n sô´ a, b, c, d không phai tâ´t ca ba ˘`ng nhau. Khoi d ¯â`u ba ˘`ng bô. (a, b, c, d) v` a la ˘. p lai ˘`ng . viêc . thay thê´ (a, b, c, d) ba , (a − b, b − c, c − d, d − a). Chu´ng minh ra ˘`ng khi la ˘. p lai . nhiê`u lâ`n viêc . ,, , ´ ´ thay thê´ trên th`ı ´ıt nhâ´t môt trong bô n sô s˜ e tr o th` a nh vô c` u ng l´ o n. . , `,i giai. Ða ˘. t Pn = (an , bn , cn , dn ) l` ˘. p. Khi d ´ Lo a bô. bô´n sô´ sau n ph´ep la ¯o ,, , ´ ´ ´ ´ ta c´ o an + bn + cn + dn = 0 voi n ≥ 2. Ta không biêt su dung bât biên . , , ´ , , ,, , ´ n` ay nhu thê n` ao. Nhung ta liên tuong toi trong h`ınh hoc, am khoang . h` , , , , ,, ´,c ´,i biêu thu c´ ach t`u d ¯iêm Pn d¯ê´n d ¯iêm gô´c toa ¯ô. (0, 0, 0, 0) d¯uo. c liên hê. vo . d , ,, ´,ng minh d ´,c trên không bi. cha ˘. n a2n + b2n + c2n + d2n . Nê´u ta chu ¯uo. c biêu thu , ,, trên th`ı b` ai to´ an d ¯uo. c giai. , ,´, Ta d¯i t`ım mô´i liên hê. gi˜ua hai buo c liên tiê´p Pn+1 v` a Pn : a2n+1 + b2n+1 + c2n+1 + d2n+1 = = (an − bn )2 + (bn − cn )2 + (cn − dn )2 + (dn − an )2 = = 2(a2n + b2n + c2n + d2n ) − 2an bn − 2bn cn − 2cn dn − 2dn an .

(1.1)

, 1.4. Bâ´t biê´n d¯on d¯iêu .

35

, , Bây gi`o ta c´ o thê d` ung an + bn + cn + dn = 0: 0 = (an + bn + cn + dn )2 = (an + cn )2 + (bn + dn )2 + + 2an bn + 2an dn + 2bn cn + 2cn dn .

(1.2)

´,i (1.2): Công . (1.1) vo a2n+1 + b2n+1 + c2n+1 + d2n+1 = = 2(a2n + b2n + c2n + d2n ) + (an + cn )2 + (bn + dn )2 ≥ 2(a2n + b2n + c2n + d2n ).

, , ´,i n ≥ 2, T`u mô´i quan hê. bâ´t biê´n n` ay ta d¯ua ra kê´t luân . vo a2n + b2n + c2n + d2n ≥ 2n−1 (a21 + b21 + c21 + d21 ). , , , , Khoang c´ ach t`u d¯iêm Pn d¯ê´n d¯iêm d ang vô han, a ´ıt nhâ´t c´ o ¯â`u t˘ . ngh˜ıa l` , ,, , ´n bâ´t k`ı. môt anh phâ`n phai tro lên lo . th` , , , , H` am khoang c´ ach t`u môt ¯iêm d ¯â`u toa ¯ô. râ´t quan trong, . d¯iêm d¯ê´n d . d . , , , ´ khi n` ao c´ o môt d˜ a y d iê m ta ph ai ngh˜ ı ngay t o i h` a m n` a y. ¯ .

J

, ,, ac d V´ı du. 1.45. Môt an d ¯inh ¯uo. c x´ . nhu sau: . thuât . to´ ,, , Buo´c xuâ´t ph´ at: (x0 , y0 ) v´ oi 0 < x0 < y0 , x n + yn √ ,, Buo´c tiê´p theo: xn+1 = , yn+1 = xn+1 yn . 2 , , T`ım gi´ oi han ay (xn , yn ). . cua d˜ , , , `,i giai. T`u, nh˜u,ng gia thiê´t d ˜ cho v` Lo a t´ınh châ´t cua trung b`ınh công v` a ¯a . , trung b`ınh nhân cua hai sô´ ta c´ o: yn − x n xn < yn =⇒ xn+1 < yn+1 , yn+1 − xn+1 < 4 , , , ,, , , ´i moi ´i han, vo ac d˜ ay c´ o gio ay sô´ duong v` a giam . n. Nhu vây . c´ . v`ı nh˜ung d˜ , , , ´,i han ´,i han ´ gio thu. c su. . Khi d ac d˜ ay n` ay tr` ung nhau v` a ta t`ım gio ¯o . cua c´ . chung lim xn = x = lim yn = y. n→∞

n→∞

,, Chuong 1. Nguyên l´ı bâ´t biê´n

36

, , , T´ınh bâ´t biê´n c˜ ung gi´ up ´ıch cho viêc o hoi kh´ ac ai n` ay. Nhung c´ . giai b` , ,, , , ´ ng gi˜ua c´ l` a không c´ o phuong ph´ ap d¯ô´i xu ac biê´n d¯ê t`ım bâ´t biê´n. Tô`n tai . , , , xn , , , ´ c biê´n d¯ôi cua ˘. c yn − xn khi môt ap x´ ac d hoa ¯inh . biêu thu . sô´ phuong ph´ yn , , ,, ´c n d biê´n d ¯ôi t`u buo ¯ê´n n + 1. ,, ,, , a) Tru`ong ho. p x´et thuong: s r 1 + yxnn xn+1 xn+1 xn+1 . (1.3) =√ = = yn+1 xn+1 yn yn 2 r ,, 1 + cos α α , , , , , ´ c n` ´ c cos = ´i công thu . V`ı ta T`u công thu ay ta liên tuong to 2 2 , ,, xn xn ˘. t ´ (1.3) tro th` luôn c´ o0< < 1, ta c´ o thê d = cos αn . Khi d anh ¯a ¯o yn yn αn α0 cos αn+1 = cos ⇒ αn = n ⇒ 2n αn = α0 . 2 2 ,, ,, , ` ´i Ðiêu n` ay tuong d ¯uong vo xn x0 2n arccos = arccos . (1.4) yn y0 ,, d¯ây l` a d¯ai . luo. ng bâ´t biê´n. , ,, , 2 2 b) Tru`ong ho. p x´et hiêu: anh c˘ an bâc . Ðê tr´ . hai, ta x´et yn − xn thay v`ı ,, yn − xn v` a ta nhân . d¯uo. c q q y2 − x2n ⇒ 2 y2n+1 − x2n+1 = y2n − x2n y2n+1 − x2n+1 = n 4 ngh˜ıa l` a q q 2n y2n − x2n = ,, ´, hai. d¯ây l` a d¯ai . luo. ng bâ´t biê´n thu , ,, T`u (1.4) v` a (1.5), ta nhân . d¯uo. c

y20 − x20

(1.5)

q

y20 − x20 − x0 xn n n n arccos = 2 arccos = 2 arcsin = 2 arcsin . y0 yn yn 2n yn q y20 − x20 , , , ´,c trên tiê´n to ´,i ˘ng thu Vê´ phai cua d khi n → ∞. Cuô´i c` ung ta ¯a y p

y2n

x2n

, 1.4. Bâ´t biê´n d¯on d¯iêu .

37

,, nhân . d¯uo. c

q

y20 − x20 x0 . arccos y0 , ` ´ ˘ Ðây l` a môt b` a i to´ a n kh´ o nê u ta gi ai b a ng c´ ach kh´ ac. . x=y=

J

˜ sô´ trong nhu ˜,ng sô´ a1 , a2 , ..., an l` V´ı du. 1.46. Môi a +1 hoa ˘. c −1 v` a ta c´ o S = a1 a2 a3 a4 + a2 a3 a4 a5 + · · · + an a1 a2 a3 = 0.

, Chu´ng minh ra ˘`ng n chia hê´t cho 4.

, , , , `˘ng `,i giai. Ðây l` Lo a b` ai to´ an vê` l´ı thuyê´t sô´, nhung ta c˜ ung c´ o thê giai n´ o ba , , `˘ng −ai , th`ı S không thay d bâ´t biê´n. Nê´u ta thay ai ba ¯ôi d¯ô`ng du theo 4 , , v`ı bô´n sô´ hang canh nhau theo v` ong tr` on thay d¯ôi dâ´u cua ch´ ung (v´ı du. . . , ´ ´ ´ ta thay dâu cua a1 th`ı bôn sô hang canh nhau l` a a1 a2 a3 a4 , an a1 a2 a3 , . . , ´ an−1 an a1 a2 , an−2 an−1 an a1 ). Thât . vây, . nêu hai trong sô´ nh˜ung sô´ hang . , , ,, ´ ´ l` a duong v` a hai l` a âm th`ı tông S không thay d¯ôi. Nêu ba sô hang . trong bô´n , ,, ,, sô´ hang o c` ung dâ´u th`ı S s˜e thay d¯ôi boi ±4 (nê´u c´ o ba sô´ hang a . c´ . duong v` ,, , ´ ´ ´ ` ˜ môt âm th`ı khi thay dâu d o duong, m` a lai on tru 2 ¯a mât 2 khi n´ . sô hang . . c` , ,, , ´ ´ ´ khi n´ o âm, vây mât d¯i 4. Tuong tu. , nêu ba sô hang âm v` a môt . tông công . . . , , ,, , ´ ´ ´ ´ ´ ` sô´ hang du o ng th` ı tô ng t˘ a ng lên 4). Cuô i c` u ng tâ t c a bô n th u a sô trong sô . , ˜ lâ`n thay dâ´u nê´u S d¯a ˜ mâ´t 4 lai hang c` ung dâ´u, th`ı S thay d ¯ôi ±8 (v`ı môi . . , , , , , , , ` ´ ´ ` ` mâ´t thêm 4 khi ca bô´n sô´ hang d ê u du o ng, tru o ng h o p c a bô n sô h ang d ê ¯ ¯ u . . . ,, âm th`ı nguo. c lai). . ,, , Khoi d¯â`u ta c´ o S = 0 d¯iê`u n` ay k´eo theo S ≡ 0 (mod 4). Bây gi`o ta tiê´n , ,´, ,´, ˜ dâ´u âm thay ba `˘ng dâ´u du,o,ng, c´ ac buo c n` ay không h` anh t`ung buo c môi , , ˜ c` thay d ac dâ´u tr`u th`ı vân on S ≡ 0 (mod 4), ¯ôi S (mod 4). Khi ta thay hê´t c´ , ´ S = n, ngh˜ıa l` nhung khi d¯o a n chia hê´t cho 4.

J

,, , , ˜ vi. kh´ V´ı du. 1.47. Tai o m`oi 2n quan kh´ ach. Môi ach d¯uo. c m`oi . môt . hôi . nghi. c´ , , , c´ o nhiê`u nhâ´t l` a n − 1 ke th` u. Chu´ng minh ra ˘`ng c´ ac vi. kh´ ach c´ o thê ngô`i

,, Chuong 1. Nguyên l´ı bâ´t biê´n

38

, , u cua m`ınh. quanh môt an tr` on sao cho không ai ngô`i canh ke th` . b` .

A′ B

B



B′ A′

B A

H`ınh 1.8

A

H`ınh 1.9

, `,i giai. Tru,o ´,c tiên ta xê´p c´ ˘. t H l` Lo ac vi. kh´ ach ngô`i v` ao vi. tr´ı bâ´t k`ı. Ða a sô´ , , , , nh˜ung d ud nhau. Ta phai t`ım d¯uo. c thuât an m` a n´ o ¯ôi th` ¯ich . ngô`i canh . . to´ , , , ´i B ngô`i bên phai ˘. p th` l` am giam sô´ H khi H > 0. Cho (A, B) l` a ca ud ¯ich . vo , ,, , ,, ˘. p n` A (h`ınh 1.8). Ta phai t´ ach d¯uo. c ca ay ra. Ðiê`u n` ay thu. c hiên . d¯uo. c nê´u , , ˜ B, A 0 (h`ınh 1.9) v` ˘. p kh´ c´ o môt ac (A 0 , B 0 ), khi ta d a H s˜e giam nê´u ¯ôi chô . ca , , , `˘ng ´,ng minh ra ˘. p ke th` (A, A 0 ) v` a (B, B 0 ) không l` a nh˜ung ca u. Ta chı c` on chu , , , 0 0 0 0 ´i B ngô`i bên phai cua A m` ˘. p (A , B ) luôn luôn tô`n tai môt a A 0 l` a . ca . vo , , , , , , ` `, 0 ban a B l` a ban ad ¯i theo chiê`u nguo. c . cua A v` . cua B. Ta khoi d¯âu tu A v` , ,, ´˘t ga ˘. p ´ıt nhâ´t n ngu`oi kim d¯ô`ng hô` (d an. Ta s˜e ba ¯i vê` ph´ıa phai) quanh b` , , , , , , ˜ ngu,`o,i ban o ban u) cua A. Vê` ph´ıa phai cua môi . (không phai ke th` . cua A c´ , , , ,, , ˜ ˜ ´ıt nhâ´t n chô. Nh˜ung chô n` ay không thê bi. chiê´m hê´t boi c´ ac ke th` u cua B , , , , , , 0 v`ı B chı c´ o nhiê`u nhâ´t n − 1 ke th` u. Nhu vây . tô`n tai . ngu`oi ban . A cua A , , ,, ,, m` a vê` ph´ıa phai ngu`oi n` ay l` a B 0 , m` a B 0 l` a ngu`oi ban . cua B. , , Môt ai to´ an ta phai d` ung d¯ê´n hai bâ´t biê´n d¯on d¯iêu a hai bâ´t biê´n . sô´ b` . v` ´,i nhau. n` ay c´ o mô´i liên hê. vo , ,, ˜,ng ô vuông, môt V´ı du. 1.48. Trên môt ˘. t pha ˘ng chia luo´i vô han . ma . nhu . sô´ ô ,, ,, ,, vuông d au theo nguyên ta ˘´c sau ¯uo. c tô d¯en. Môt ¯uo. c tô m` . ô vuông trong luo´i d

J

, 1.4. Bâ´t biê´n d¯on d¯iêu .

39

, ,, ˜ ô vuông d a chı khi ´ıt nhâ´t c´ o ba ô vuông bên canh d ¯en khi v` ¯ây: Môi ¯uo. c tô d . ,`, ,´, ,´, , ´ ´ v` ac th`ı tô tra ˘ng. Quy tr`ınh a truong ho. p kh´ n´ o l` ad ¯en trong buoc tô truoc d ¯o ,, , ` n` ay d¯uo. c la ˘. p lai. ˘ng cuô´i c` ung không c` on môt ao trên ¯en n` . Chu´ng minh ra . ôd ,´, luoi ô vuông. , , , , `,i giai. Ta x´et tâ´t ca h` ´,a sô´ ô d¯en. Ta xem x´et c˜o, h` Lo ang c´ o chu ang cua nh˜ung , , h` ang n` ay l` a khoang c´ ach chiê`u doc ang cao nhâ´t v` a h` ang thâ´p nhâ´t. . gi˜ua h` , , , `˘ng c˜o, h` ´,ng Ta chı ra ra ang n` ay không bao gi`o t˘ ang. Thât o thê chu . vây, . ta c´ , , minh mênh d¯ê` manh hon: H` ang bâ´t k`ı không chu´a ô vuông d ¯en s˜e không . . , ,, ,´, ˜ ˜ ô vuông ´ ˜ cho. Boi v`ı tai thay d ˘c tô m` au d ¯ôi theo nguyên ta ¯a . môi buoc, môi ,´, ´ c´ trong h` ang d o nhiê`u nhâ´t hai ô d a duo i (v`ı ô bên canh bên ¯o ¯en trên v` . , , , ´ ´ ` ˘ng), nhu vây ˘ng cho phai v` a bên tr´ ai n´ od a tra o s˜e gi˜u nguyên l` a tra ¯êu l` . n´ ,´, buoc sau. , , , , ,, , ´˘ng không thê t˘ Nhu vây ang cua tâ´t ca c´ ac h` ang không tra ang d ¯uo. c. . c˜o h` , , ,, , , Tru`ong ho. p g`ı s˜e xay ra khi c˜o h` ang cua c´ ac h` ang ta d¯ang x´et l` a môt . sô´ , , , , , , , ´n hon 0 ? Trong tru`ong ho. p nhu vây lo ang ph´ıa trên . ta x´et c˜o ô d¯en trong h` , , , , ´i ô vuông d¯en phai nhâ´t (ngh˜ıa l` a khoang c´ ach t`u ô vuông d ai nhâ´t to ¯en tr´ , , , nhâ´t). Khoang c´ ach n` ay không thê t˘ ang m` a n´ o luôn luôn giam: Ô vuông ´˘ng (hu,o ´,ng ph´ıa trên d¯en bên tr´ ai nhâ´t s˜e c´ o ´ıt nhâ´t hai ô vuông lân cân . tra , ,, , ,, , ´˘ng. L´ı luân v` a bên tr´ ai ô n` ay) v` a nhu vây o phai tro th` anh tra . tuong tu. . n´ , , , , , ´ ng cho ô d¯en bên ph´ıa phai nhâ´t. Nhu vây d¯u ang n` ay giam ¯en cua h` . c˜o ô d , ,, , ´˘ng, ´ h` d¯ê´n khi n´ o tro th` anh không, tai ang n` ay to` an bô. l` a ô tra . th`oi d¯iêm d¯o , , , , , ´˘ng c˜ ung giam. nhu vây ang cua h` ang tâ´t ca không tra . c˜o h` , `˘ng d¯ê´n môt ´,i môt uc tô`n tai ang vo ai Ðiê`u trên chı ra ra . l´ . ´ıt nhâ´t môt . h` . v` , , , , , , , ´, c˜o ô d¯en cua nh˜ung h` ô d¯en trong d¯o ang nhu vây ang . s˜e giam. Giam c˜o h` , , , ´i trang n` ay không phai m˜ ai m˜ ai, nhu vây ung ta d¯at ai không c´ o . cuô´i c` . to . th´ , , , ´˘ng. ´ a ô vuông d ´i tra môt ang n` ao chu a to` an bô. luo ¯en, ngh˜ıa l` . h` , , ,, ,, o tr`ınh b` ay phuong ph´ ap giai phuong Trong cuô´n s´ ach [9] t´ ac gia c´

J

40

,, Chuong 1. Nguyên l´ı bâ´t biê´n

, , ,, ,, ,, `˘ng phu,o,ng ph´ tr`ınh ba ap giam d ung. Ngu`oi ta thu`ong chı ra phuong ¯ê´n vô c` ,, ,, tr`ınh vô d o nghiêm ap sau: Ngu`oi ta ¯inh . không c´ . nguyên theo phuong ph´ , , , , , xây du. ng môt am cua nghiêm ac biê´n) m` a gi´ a tri. cua n´ o . h` . (v´ı du. nhu tông c´ ,, ,`, , ,, ´ ´ cho môt l` a sô nguyên duong; khi d nguoi ta lai ¯o ¯uo. c . nghiêm, . . xây du. ng d , , , ´ ´i, vo ´i nghiêm ˜ xây du. ng nhân nghiêm ay h` am sô d¯a a tri. nguyên . mo . n` . gi´ , , , ,, ,`, ,, ˘. p lai duong nho hon. Nguoi ta la a tr`ınh n` ay, nhân ay giam vô ¯uo. c d˜ . qu´ . d , ,, ,, ´,ng ˜ chu han ac gi´ a tri. nguyên duong; d ay không thê d¯uo. c, ngh˜ıa l` ad ¯iê`u n` ¯a . c´ ,, , ˜ xây du. ng không tô`n tai, minh d˜ ay sô´ nguyên duong d ai to´ an vô nghiêm. ¯a . b` . , ,, V´ı du. 1.49. Chu´ng minh ra ˘`ng phuong tr`ınh vô d o nghiêm ¯inh . sau không c´ . nguyên kh´ ac không 8x4 + 4y4 + 2z4 = u4 . , , , `,i giai. Gia su, phu,o,ng tr`ınh c´ Lo o nghiêm . nguyên (x0 , y0 , z0 , u0 ) 6= (0, 0, 0, 0). . .. ˘. t u0 = 2u1 . Ta c´ o 8x40 + 4y40 + 2z40 = u40 , suy ra u40 .. 2, vây . u0 . 2. Ða . .. ,, ˘. t z0 = 2z1 . Ta d¯uo. c 4x40 + 2y40 + z40 = 8u41 , suy ra z40 .. 2, vây . z0 . 2. Ða . . ,, . ˘. t y0 = 2y1 . Ta d¯uo. c 2x40 + y40 + 8z41 = 4u41 , suy ra y40 .. 2, vây . y0 . 2. Ða . .. ,, ˘. t x0 = 2x1 . Ta d¯uo. c x40 + 8y41 + 4z41 = 2u41 , suy ra x40 .. 2, vây . x0 . 2. Ða , ,, Ta d¯uo. c 8x41 + 4y41 + 2z41 = u41 . Vây ung l` a nghiêm . (x1 , y1 , z1 , u1 ) c˜ . cua , x0 y0 z0 u0 ,, ˘. p ˜ cho v` phuong tr`ınh d¯a a c´ o dang ay c´ o thê la . ( 2 , 2 , 2 , 2 ), quy tr`ınh n` x0 y0 z0 u0 , , ´c k: ( k , k , k , k ). lai ai d ¯ê´n buo . m˜ 2 2 2 2 , , x0 y0 z0 u0 ´,i moi C´ ac sô´ k , k , k , k l` a c´ ac sô´ nguyên vo k ∈ N. Ðiê`u n` ay chı xay . 2, 2 2 2 ,, `˘ng 0. Vây ˜ cho không c´ o nghiêm ra khi tâ´t ca c´ ac biê´n ba . phuong tr`ınh d¯a . nguyên kh´ ac 0. , , , , ´˘t c´ Ta c´ o thê t´ om ta ach giai theo bâ´t biê´n d¯on d¯iêu . nhu sau: , , , ,, ,, , ´i t´ 1. T`ım d¯ai a chı ra n´ o phai thay d¯ôi duo ac ¯ on d ¯iêu . luo. ng bâ´t biê´n d . v` , ´. d ac n` ao d¯o ¯ông . cua môt . thao t´

J

, 1.4. Bâ´t biê´n d¯on d¯iêu .

41

, , , , , `˘ng n´ ´,ng ´ chu 2. H˜ ay chı ra ra o chı c´ o thê thay d ¯ôi h˜uu han . lâ`n; khi d¯o , , , , , , , `˘ng n´ ´. minh ra o c´ o thê chı d`ung su. thay d¯ôi tai ao d¯o . môt . th`oi d¯iêm n` , , , , , , , , , ´c, th`ı ta phai xây du. ng n´ o môt ach Nê´u su. chuyên d ¯ôi không d ¯uo. c cho truo . c´ , , , , , ´ ` ´ ´ th´ıch ho. p. C´ o rât nhiêu bât biên d ac nhau c´ o thê xây du. ng nhu: ¯on d¯iêu . kh´ , , , , ,, c´ ac tông, c´ ac t´ıch, gi´ a tri. cu. c d¯ai, a tri. cu. c tiêu v` a nhiê`u d¯ai . gi´ . luo. ng th´ıch , , ˜ o,, ´,i hu,o ´,ng dân ho. p kh´ ac. Ban oi trong muc ai tâp ay vo ¯oc . d . t`ım t` . b` . cua tiê´t n` trên. , ,´, , , Truo c khi chuyên sang b` ai tâp, n˜ua cua bâ´t biê´n . ta x´et môt . kh´ıa canh . , , , d¯on d¯iêu a d` ung bâ´t biê´n d o câ´u h`ınh ¯ on d ¯iêu . l` . chı ra hê. thô´ng ta d¯ang x´et c´ , , , , , nhu mong muô´n. Trong tru`ong ho. p n` ay bâ´t biê´n d¯on d ¯iêu . không câ`n thiê´t , , , ˘. t d¯iê`u kiên ung vây . thay d¯ôi h˜uu han . lâ`n; c˜ . bâ´t biê´n d¯on d¯iêu . không câ`n cha , , , , , , , ´˘m, n´ `˘ng sô´ trong môt ´c chuyên d¯ôi. Ta chı câ`n chı ch˜e la o c´ o thê l` a ha . sô´ buo , , , , ,´, , ra bâ´t biê´n d o thê chı thay d¯ôi theo môt ng nê´u n´ o thay d¯ôi ¯on d ¯iêu . c´ . huo , , , , , ,´, ´,i môt tâ´t ca v` a n´ o d¯at ng kh´ ac sinh ra. ¯ôi cua huo . to . câ´u h`ınh t`u su. thay d , , ,, ˜ d V´ı du. 1.50. (IMO 1986). Tai u gi´ ac ngu`oi ta g´ an môt ¯ınh cua môt . môi . ng˜ . , , , ,, ,, ´ ´ ´ ´ sô nguyên sao cho tông cua n˘ am sô l` a duong. Nêu ba d an ¯ınh liên tiêp d ¯uo. c g´ ,, , , , ´ ´ ˜ ng sô x, y, z c´ ac sô x, y, z v´ oi y < 0, th`ı thao t´ ac sau d ¯ây d ¯uo. c thu. c hiên: . Nhu ,, , , ´, , ,, ,, , ac nhu vây ˘. p lai oi d ¯uo. c la ¯uo. c thay boi x + y, −y, z + y tuong ung. Thao t´ . t´ . d khi ´ıt nhâ´t môt am sô´ l` a sô´ âm. H˜ ay x´ ac d o môt ¯inh . trong n˘ . c´ . quy tr`ınh câ`n , , , , ˜ u han thiê´t d uc trong hu ¯ê kê´t th´ . buo´c không ? , , `,i giai. Thao t´ ˘. p lai Lo ac la ai to´ an luôn luôn kê´t th´ uc. Ch`ıa kh´ oa cua . trong b` , ,, ´,ng minh l` chu a t`ım ra môt am gi´ a tri. nguyên duong f(x1 , x2 , ..., x5 ) cua . h` , , , , nh˜ung nh˜ an cua b´ at gi´ ac, m` a gi´ a tri. h` am n` ay giam khi ta thu. c hiên ac . c´ , , , ` ˘ng c´ ˜ cho ta c´ thao t´ ac. Ba ac d˜u kiên ai to´ an d o thê lâ´y h` am ¯a . cua b` f(x1 , x2 , x3 , x4 , x5 ) =

5 X i=1

,, o d¯ây x6 = x1 ,

(xi − xi+2 )2 ,

, ,, ,´, x7 = x2 . Gia su d¯ê´n buo c y = x4 < 0, f(x1 , x2 , x3 , x4 , x5 )

42

,, Chuong 1. Nguyên l´ı bâ´t biê´n

,´, c´ o gi´ a tri. fc v` a sau khi thao t´ ac tai c n` ay f(x1 , x2 , x3 , x4 , x5 ) c´ o gi´ a tri. . buo , , ´ fm − fc = 2(x1 + x2 + x3 + x4 + x5 )x4 < 0, v`ı tông cua c´ fm . Khi d¯o ac , , , sô´ x1 , x2 , x3 , x4 , x5 luôn luôn l` a sô´ duong. Nê´u thao t´ ac trên không d`ung, , ,, , ,, ta t`ım d ay vô han ¯uo. c d˜ . giam f0 > f1 > f2 > ... nh˜ung sô´ duong không , , , , âm. D˜ ay sô´ nhu vây a tr`ınh phai d`ung lai . không tô`n tai, . nên qu´ . tai . môt . th`oi , d¯iêm.

J

` tâp Bai . , ,, ´ ˘p . 1.51. (M˜y 1997). Cho p1 , p2 , p3 , ... l` a nh˜ung sô´ nguyên tô´, d ¯uo. c sa , , , , ´ tu. t˘ theo thu ang dâ`n v` a cho x0 l` a môt a 1. Cho sô´ nguyên . sô´ thu. c gi˜ua 0 v` ,, duong k, d¯inh . ngh˜ıa  ´ 0 

nêu xk−1 = 0, xk = pk  nê´u xk−1 6= 0, xk−1 , ,, `˘ng vo ´,ng minh ra ´,i od a phâ`n thâp ay t`ım v` a chu ¯ây {x} k´ı hiêu . l` . phân cua x. H˜ , , ´,i 0. tâ´t ca x0 thoa m˜ an 0 < x0 < 1 th`ı d˜ ay sô´ x0 , x1 , x2 , ... tiê´n to , , ,, . 1.52. (M˜y 1993). Cho a, b l` a nh˜ung sô´ duong le. Ðinh ay (fn ) . ngh˜ıa d˜ , , , , , , ` ´ ´ ´i n ≥ 3 l` ´c sô le lo ´n nhâ´t cua ˘ng c´ ˘. t f1 = a, f2 = b v` ba ach d¯a a lây fn vo a uo , , `˘ng sô´ vo `˘ng fn l` ´,ng minh ra ´,i n d¯u lo ´n v` fn−1 + fn−2 . Chu a ha a x´ ac d a ¯inh . gi´ , , tri. n` ay nhu h` am cua a v` a b. , , ,, ,, 2 2 . 1.53. T`ım tâ´t ca nghiêm . nguyên duong cua phuong tr`ınh x + y = 3(z2 + w2 ). , `˘ng c´ ´,ng minh ra . 1.54. (Paul Zeitz). Cho môt ac ¯ô` thi. h˜uu han . d . bâ´t k`ı. Chu , , , ,, , ˜ ´˘ng sao cho, vo ´i môi d d¯ınh c´ o thê tô mâ`u d a tra ¯en v` ¯ınh, ´ıt nhâ´t môt . nua , , , ,, ,, ´,i mâ`u n´ nh˜ung d od o c´ o. ¯ınh lân cân ¯uo. c tô mâ`u nguo. c lai . cua n´ . vo , ,, . 1.55. Môt on d¯uo. c chia ra l` am 2000 re quat. o 2001 con ê´ch . h`ınh tr` . C´ , , trong c´ ac re quat ay. Luôn luôn tô`n tai ung môt . n` . hai con ê´ch trong c` . re

˜,ng b` 1.5. Nhu ai to´ an nâng cao

43

, , ´, hai con ê´ch nhu, vây ´,i hai re quat quat; (theo chiê`u . cu . nhay to . bên canh . , , , ,, , ` ´ ng minh ra ˘ng tai ´ ´ıt nhâ´t 1001 re nguo. c nhau). Chu ao d ¯o . môt . th`oi d¯iêm n` quat o ê´ch. . d¯ê`u c´

, ` toan ´ nâng cao 1.5. Nh˜ung bai , , ,, , V´ı du. 1.56. Môt anh n˘ am manh; môt ¯uo. c x´e th` . t`o giâ´y d . sô´ trong sô´ 5 manh , , , , , , ,, ˜ a, v` nho n` ay lai anh 5 manh nho nu a môt ac manh nho ¯uo. c x´e th` . d . sô´ trong c´ , , , , n` ay lai anh 5 manh, ... Vây, o ¯uo. c x´e tiê´p th` . d . nê´u cu´ tiê´p tuc . x´e nhu vây . th`ı c´ , , ,, ,, ´ ´ khi n` ao ta d¯uo. c 2002 manh giây hay không? Ðuo. c 2005 manh giây không? , , , `,i giai. Khi ta chia t`o, giâ´y l` Lo am 5 manh v` a sau n` ay chia c´ ac manh giâ´y ra , , , , ˜ lâ`n sô´ manh ´, môi l` am 5 manh nho th`ı cu giâ´y t˘ ang thêm 4. Vây . sô´ manh , ˜ lâ`n x´e th`ı c´ ´,c n` giâ´y, sau môi o dang 4k + 1(k ∈ N∗ ), biêu thu ay l` a bâ´t biê´n . trong qu´ a tr`ınh x´e giâ´y.

, , ,, V`ı 2002 6= 4k + 1, nên không thê x´e d¯uo. c 2002 manh. , , ´, 501. 2005 = 501.4 + 1, vây o thê x´e th` anh 2005 manh sau lâ`n thu . c´ , ,, ˜ ôd V´ı du. 1.57. Trong môt o 8 × 8 ô. Trong môi ¯uo. c viê´t môt . . bang ô vuông c´ , , ,`, , ´ sô tu. nhiên. Nguoi ta cho ph´ep thu. c hiên ac: T˘ ang môt ¯on vi. d ¯ô´i . môt . thao t´ . d , , , ´ , ˜ ng bang ô vuông c´ v´ oi tât ca c´ ac sô´ na ˘`m trong nhu o chiê`u 3 × 3 hoa ˘. c 4 × 4 , , , , ´ ˜ cho. Hoi sau khi thu. c hiên thuôc ac nhu trên ¯a . bang ô vuông d . môt . sô thao t´ , ˜,ng sô´ trong bang c´ th`ı to` an bô. nhu o chia hê´t cho 10 không?

J

, , , , , , , `,i giai. Hiên nhiên tô`n tai a c´ o thê thu. c hiên Lo . nh˜ung bang m` . nh˜ung thao , , , , , , t´ ac trên. Nhung ta chı ra c´ o nh˜ung bang khi thu. c hiên ac trên . nh˜ung thao t´ , , , ´ ´ ´ không thê cho kêt qua. V`ı t´ınh chât chia hêt cho 10, ta x´et nh˜ung sô´ theo , , ,, , , modulo 10. Hai bang sô´ ta coi nhu nhau khi v` a chı khi nh˜ung phâ`n tu , , , ´, , ´,i d¯ô`ng nhâ´t nhu, vây tuong u ng cua n´ od o ¯ô`ng du theo modulo 10. Vo . ta c´ , , , , ´ 64 10 bang kh´ ac nhau. Ta s˜e chı ra thao t´ ac cua ch´ ung ta l` a thuân . lo. i nêu

44

,, Chuong 1. Nguyên l´ı bâ´t biê´n

, , , ,, , , ,, ´, hai c˜ môt ung c´ o thê nhân ac, th`ı bang thu ¯uo. c t`u bang kh´ ¯uo. c . bang nhân . d . d , , , ´p dung ˜ l` t`u bang ban d¯â`u. Ðiê`u kiên aa thêm 9 lâ`n thao t´ ac ta d am. ¯u l` ¯a . d . , , , , ,, , ´ ng minh tô`n tai Theo phuong ph´ ap nhu vây, a n´ o không ¯u d¯ê chu . d . bang, m` , , , , `, , ,, , , ´ ´ thê nhân an sô không. Sô luo. ng cua nh˜ung h`ınh vuông c˜o . d¯uo. c tu bang to` , ˜ h`ınh vuông nhu, 3 × 3 l` a 36, c` on h`ınh vuông c˜o 4 × 4 l` a 25. V`ı trong môi , ´, 9 a ´p dung ´p dung vây o thê a thao t´ ac nhiê`u nhâ´t 9 lâ`n (sau lâ`n thu s˜e . ta c´ . . , , , , ,, , ,, ´ ´ at), sô luo. ng chung cua nh˜ung bang m` a ta c´ o thê nhân . d¯uo. c bang xuât ph´ , ,, , , ,, , ˘ng d ˜ nhân a 1061 . Nhu vây ¯inh . d¯a . d¯uo. c t`u bang không, không vuo. t qu´ . kha ,, , ´ ng minh. d¯uo. c chu

J

, ,, V´ı du. 1.58. Trong môt o 100 × 100 ô d ¯uo. c d ¯iê`n dâ´u công . bang ô vuông c´ . , ,, ,´, , , ` ˜ ng dâ´u o môt (+). Môt ˘ng c´ ach d an bô. nhu ang hoa ˘. c ¯ôi to` . buoc thu. c hiên . ba . h` , , , , , , , ˜ u han ´ sang dâ´u nguo. c lai. môt ao d o kha n˘ ang sau hu ¯o . côt . n` . C´ . buo´c nhu trên, , ,, , ` (−) ? ´ng 1970 dâ´u tru bang ô vuông nhân od ¯uo. c s˜e c´ ¯u . d , , , , , ,´, ,, , `,i giai. Gia su, c´ Lo o kha n˘ ang sau môt c nhân o . sô´ h˜uu han . buo . d¯uo. c bang c´ , , , , ´ ´ ` ´ i ta d ´ j ta ˜d 1970 dâu tr`u. Cho tai ang thu on tai ¯a ¯ôi dâu xi lân, c` . h` . côt . thu , , ´ ` ´ ` ˜ d¯ôi dâu yj lân. Khi d¯o ´ dâu tai ˜ thay d¯ôi xi + yj lân. Suy ra tai d¯a . ô (i, j) d¯a . , , , , , ´ ´ ´ ô n` ay c´ o dâu tr`u (−) khi v` a chı khi xi + yj l` a sô le. Cho p l` a sô luo. ng sô´ , , , , , , , ´ sô´ le gi˜ua c´ ac sô´ xi , c` on q l` a sô´ luo. ng nh˜ung sô´ le gi˜ua c´ ac sô´ yj . Khi d ¯o , ,, , , luo. ng chung nh˜ung dâ´u tr`u trong bang s˜e l` a p(100 − q) + (100 − p).q = 100p + 100q − 2pq, ,, ,, , ´,c ˘ng thu od ¯ây ta nhân ¯a . d¯uo. c d 100p + 100q − 2pq = 1970 hay l` a (p − 50)(q − 50) = 1515 = 15.101. ,, , ´ Boi v`ı 101 l` a môt . sô nguyên tô´, ´ıt nhâ´t môt . trong nh˜ung sô´ p−50, q−50 chia , ´ l` hê´t cho 101. Ta cho d a p−50 chia hê´t cho 101. Nhung −50 ≤ p−50 ≤ 50, ¯o

˜,ng b` 1.5. Nhu ai to´ an nâng cao

45

, , , ´,i nên su. chia hê´t p − 50 cho 101 chı xay ra khi p − 50 = 0, d¯iê`u n` ay tr´ ai vo , ´,c (p − 50)(q − 50) 6= 0. ˘ng thu bâ´t d¯a

J

, , , `, (−), ´nh dâ´u tru V´ı du. 1.59. Tai ac d ¯ınh A1 cua môt ¯a gi´ ¯ê`u 12 d¯ınh d ¯a . d . d , , , , ,, ´nh dâ´u công ad c` on tâ´t ca c´ ac d on lai (+). Môt ¯ô i ¯ınh c` ¯a . l` . d . . buo´c thu. c hiên , ,, , , d anh dâ´u nguo. c lai. ˘`ng ¯ô`ng th`oi ba dâ´u tai ¯ınh liên tiê´p th` . ba d . Chu´ng minh ra , , , , , , ˜ u han không c´ o kha n˘ ang sau môt . sô´ hu . buo´c thu. c hiên . nhu trên s˜e nhân . , , ,, , ´ ´ ´ ` d u o c A c´ o dâ u tr u (−), c` o n tâ t c a c´ a c d ınh c` o n l ai mang dâ u c ông (+). ¯ . ¯ 2 . . , , , `,i giai. Ta chia c´ Lo ac d¯ınh cua d ac d¯ê`u 12 canh ra l` am 3 nh´ om: ¯a gi´ . {A1 , A4 , A7 , A10 }, {A2 , A5 , A8 , A11 } v` a {A3 , A6 , A9 , A12 }. , , ˜ d¯ınh ro,i v` D˜ê thâ´y khi chon ao môt om. Suy ra ¯ınh liên tiê´p th`ı môi . ba d . nh´ , , , , , , , ˜ lâ`n d¯ôi dâ´u o ba d ˜ nh´ sau môi om ¯ınh liên tiê´p, sô´ luo. ng dâ´u tr`u trong môi , , , , , ˘. c giam d t˘ ang lên hoa om hai ¯i 1. T`u d ¯ây suy ra sô´ luo. ng dâ´u tr`u trong nh´ , , ˜˘n le. Thât ´˘t d¯â`u choi th`ı sô´ v` a nh´ om ba luôn luôn c` ung t´ınh cha . vây, . khi ba , , , ,, , , `˘ng 0. Sau lâ`n d ´ nhâ´t c´ ´, luo. ng dâ´u tr`u ba o 1 dâ´u tr`u, sau lâ`n d¯ôi thu ¯ôi thu , , ˜ nh´ ´, ba l` ´, ˘. c 2, sau lâ`n d¯ôi thu ˘. c 3, sau lâ`n d¯ôi thu hai môi om c´ o 0 hoa a 1 hoa , ˘. c 4 v` tu l` a 0, 2 hoa a vân vân. , , ,, , , Tru`ong ho. p riêng không thê c´ o su. phân bô trong nh´ om hai c´ o môt . dâ´u , tr`u m` a c´ ac nh´ om kh´ ac không c´ o.

J

, , , ˜,ng phâ`n V´ı du. 1.60. Cho bang sô´ (h`ınh 1.10) c´ o t´ınh châ´t sau: Tông cua nhu ,, ,, ˜ h` ˜ côt tu trong môi ang, môi ˘. c d ac cho ¯u`ong ch´eo chia hê´t cho 2. Môt . hoa . thao t´ , , ,, , ph´ep chuyên môt d o n v i o m ôt ô sang ô bên c anh (ô bên c anh c ua m ôt a . ¯ . . . . . ô l` , , , , , ` h`ınh 1.10 nhân ô c´ o chung canh). C´ o thê tu ¯uo. c h`ınh 1.11 sao cho tâ´t ca . . d ,, ,, phâ`n tu o c´ ac ô xung quanh l` a sô´ cha ˘˜n không? , , `,i giai. T`u, gia thiê´t b` ˘. p sô´ a2 , a7 v` Lo ai to´ an suy ra ca a a4 , a5 l` a c` ung t´ınh , ,, ˜ ´ ˘n le. Boi v`ı a1 + a4 + a6 v` cha a a6 + a7 + a8 chia hêt cho 2 suy ra a1 + a4 + 2a6 + a7 + a8 chia hê´t cho 2, ngh˜ıa l` a a1 + a4 + a7 + a8 chia hê´t cho 2.

,, Chuong 1. Nguyên l´ı bâ´t biê´n

46 a1

a2

a3

b1

b2

b3

a4

2

a5

b4

1

b5

a6

a7

a8

b6

b7

b8

H`ınh 1.10

H`ınh 1.11

, ˜ cho ta c´ C˜ ung t`u d¯iê`u kiên o a1 + 2 + a8 chia hê´t cho 2, ngh˜ıa l` a a1 + a8 . d¯a ´ ´ ´ a4 + a7 chia hêt cho 2 v` chia hêt cho 2. Khi d a suy ra a4 v` a a7 c` ung t´ınh ¯o , , , ˜ ˜˘n le. ´ ´ ˘n le. Nhu vây cha a a2 , a4 , a5 v` a a7 c` ung t´ınh cha ¯i d ¯ên kêt luân . ta d . l` , , ˜˘n, th`ı a1 , a3 , a6 v` ˘. c tâ´t ca Nê´u a2 , a4 , a5 v` a a7 l` a nh˜ung sô´ cha a a8 hoa , , , , ,, ˜˘n hoa ˜˘n l` ˘. c tâ´t ca d¯ê`u le. Khi d ˘. c ´ sô´ luo. ng cua nh˜ung sô´ cha d¯ê`u cha a 5 hoa ¯o l` a 9. , , , ˘. c tâ´t ca d¯ê`u Nê´u a2 , a4 , a5 v` a a7 l` a nh˜ung sô´ le, th`ı a1 , a3 , a6 v` a a8 hoa , , , ,, , ˜˘n hoa ˜˘n l` ˘. c tâ´t ca d ˘. c l` ´ sô´ luo. ng cua nh˜ung sô´ cha cha a 1 hoa a 5. ¯ê`u le. Khi d ¯o , , , ,, , ˜˘n trong bang l` ˘. c 9). Nhu vây, a môt . sô´ luo. ng nh˜ung sô´ cha . sô´ le (1, 5, hoa , , ´ , , Ta x´et su. biên d¯ôi trên nh˜ung ô bên canh môt a y l` a nh˜ung . . ô. Nê´u x v` ,, sô´ o hai ô bên canh nhau, th`ı ta c´ o thao t´ ac x, y → x − 1, y + 1. Ta x´et tâ´t . , , , , , ,`, , ˜˘n, cha ˜˘n), (cha ˜˘n, le), (le, le). Sau ˜˘n le cho x v` ca c´ ac truong ho. p cha a y: (cha , , , , , , , , ´, ˜˘n), (cha ˜˘n, khi thu. c hiên ac ta nhân ng (le, le), (le, cha ¯uo. c tuong u . thao t´ . d ˜˘n). cha , , , , ˜˘n cua sô´ lu,o.,ng sô´ Nhu vây, ac biê´n d ¯ôi không thay d¯ôi t´ınh cha . thao t´ , , , ˜˘n (v`ı thay d¯ôi sô´ lu,o.,ng 0 hoa ˘. c 2). Trong h`ınh 1.10 c´ o sô´ le nh˜ung sô´ cha , ˜˘n, c` ˜˘n, ngh˜ıa l` ˜˘n. Suy ra bang nhu, a sô´ cha on trong h`ınh 1.11 c´ o 8 sô´ cha cha ,, , vây ac thao t´ ac trên. . không nhân . d¯uo. c khi thu. c hiên . c´

J

, , ˜ ô ta ghi V´ı du. 1.61. Cho môt a trong môi . bang h`ınh vuông ke ô 10 × 10 v` , , , , `, sô´ 1 d `, 1 theo thu´ tu. môt ang thu´ nhâ´t ghi tu ¯ê´n sô´ 100: H` . sô´ tu. nhiên gô`m tu , , , , `, 11 d d ang thu´ hai ghi tu ˘`ng tông S cua 10 sô´ ¯ê´n 10; h` ¯ê´n 20; ... Chu´ng minh ra

˜,ng b` 1.5. Nhu ai to´ an nâng cao

47

, , ´ không c´ bâ´t k`ı cua bang, trong d o bâ´t k`ı hai sô´ n` ao thuôc ung môt ang ¯o . c` . h` , v` a không c´ o bâ´t k`ı hai sô´ n` ao thuôc ung môt a môt ¯ôi. T`ım . c` . côt, . l` . sô´ không d sô´ S. , , , `,i giai. Ta k´ı hiêu Lo . sô´ hang . cua tông S: - Thuôc ang 1 l` a a1 ; . h` - Thuôc ang 2 l` a 10 + a2 ; . h` - ... - Thuôc ang 10 l` a 90 + a10 . . h` , , , ´, c´ Trong d¯o ac sô´ tu. nhiên a1 , a2 , ..., a10 bao gô`m gi˜ua 1 v` a 10, v` a nh˜ung sô´ n` ay d¯ôi môt ac nhau, v`ı nê´u ta c´ o a1 = a2 th`ı hai sô´ a1 v` a 10 + a2 . kh´ , , , `˘m trong c` phai na ung môt o: . côt . cua bang. Ta c´ S = a1 + (10 + a2 ) + (20 + a3 ) + · · · + (90 + a10 ) = (10 + 20 + · · · + 90) + (a1 + a2 + · · · + a10 ) = 450 + (a1 + a2 + · · · + a10 ).

,, , Boi v`ı c´ ac sô´ a1 , a2 , ..., a10 d¯ôi môt ac nhau v` a nhân a tri. nguyên t`u . kh´ . gi´ , , , ˜ môt ˘. t trong tông 1d ac sô´ tu. nhiên t`u 1 d¯ê´n 10 c´ o ma ¯ê´n 10, môi . sô´ trong c´ , , , ´i tu c´ ´ a1 + a2 + · · · + a10 vo ach l` a môt c˜ ung chı c´ o môt ¯o . sô´ hang, . . lâ`n, do d a1 + a2 + · · · + a10 = 1 + 2 + 3 + 4 + 5 + 6 + 7 + 8 + 9 + 10 = 55.

, ,, ´,i moi Nhu vây ad ach chon ¯ai ¯ô´i vo . S = 450 + 55 = 505 l` . luo. ng bâ´t biê´n d . c´ . , , tông c´ ac sô´ trong bang.

J

, ,, ,, ,, ˘`ng nhau. V´ı du. 1.62. Cho khô´i lâp . phuong tao . boi 27 khô´i lâp . phuong nho ba , , ,, , ˜ khô´i lâp Trong môi o chu´a môt ˘. c −1. (Hai khô´i nho . phuong nho c´ . sô´ +1 hoa , goi a canh nhau nê´u ch´ ung c´ o chung môt ˘. t). Ta goi ˘. t ca ˘´t" cua khô´i . l` . . ma . "ma , , ,, ,, ˜,ng khô´i lâp lâp a tâ´t ca nhu nhau v` a na ˘`m trong . phuong l` . phuong nho canh . , , , , môt ˘. t pha ˘ng". C´ o thê thay d ˘. t ca ˘´t c´ o chung ¯ôi dâ´u d ¯ô`ng th`oi trong hai ma . "ma

48

,, Chuong 1. Nguyên l´ı bâ´t biê´n

, , , ,, , ,, ˜,ng khô´i lâp khô´i lâp ¯ôi dâ´u nhu . phuong nho . phuong nho, nhung không thay d chung. , , , , ,, Ban d ac khô´i nho c´ o chu´a sô´ −1. C´ o thê sau môt ¯â`u trong tâ´t ca c´ . sô´ buo´c , , ,, , , , ,, thay d ac khô´i nho o d ¯ôi dâ´u nhu trên th`ı c´ ¯ınh cua khô´i lâp . phuong mang −1, , ,, ˜,ng khô´i nho c` c` on nhu on lai ¯ê`u mang +1 d¯uo. c không? . d , , , ,, , ,, `,i giai. Ta d¯i x´ Lo ac d¯inh ¯ôi sô´ luo. ng cua −1 trong khô´i lâp . su. thay d . phuong , , ,, ,´, ´˘t ´,i môt ˘. t ca nho trong khô´i lâp c biê´n d ¯ôi bâ´t k`ı. Trong hai ma . phuong vo . buo , , , , , , nh˜ung dâ´u d o sô´ luo. ng 12 sô´. Nê´u n trong ch´ ung l` a −1, c` on ¯uo. c thay d¯ôi c´ , ´ ´ ´ ´ sau khi biên d¯ôi ta c´ 12 − n l` a +1, khi d o n sô +1 v` a (12 − n) sô −1, ngh˜ıa ¯o , , ,, ,, ´ l` a sô luo. ng cua −1 d a 12 − n − n = 2(6 − n), d a môt ¯uo. c thay d¯ôi l` ¯ây l` . sô´ , , , , , , , , ˜ buo ˜˘n. Suy ra tai ˜˘n le cua sô´ luo. ng cua −1 gi˜u, ´c biê´n d¯ôi t´ınh cha cha . môi , ,, nguyên. Ban d a 27, c` on ta muô´n c` on 8 (trên c´ ac d ¯â`u sô´ luo. ng sô´ −1 l` ¯ınh , , , , ,, cua khô´i lâp ay không thê xay ra. . phuong). Hiên nhiên d¯iê`u n`

J

, ,, ,, V´ı du. 1.63. Cho khô´i lâp ˘`ng . phuong bao gô`m 27 khô´i lâp . phuong nho ba , , , , , , , , , ˜ ng khô´i lâp ˜ ng sô´ nhau. Trong nhu . phuong nho cua khô´i lâp . phuong chu´a nhu , , ,, ,´, ´ +1. Môt o thê thêm c` ung môt ao hai khô´i lâp ¯ôi ta c´ . buoc biên d . sô´ v` . phuong , , ,, `, khô´i lâp ˜ cho nho bên canh o chung ma ˘. t). Tu ¯a . nhau (hai khô´i nho c´ . phuong d , ,, ,, ´ c´ o thê nhân ¯uo. c khôi lâp . d . phuong sao cho , , , ,, ,, a) Trong tâ´t ca khô´i lâp o 0, c` on khô´i nho o trong o −1; . phuong nho c´ . tâm c´ , , , , ,, , b) Trong tâ´t ca khô´i lâp o 0, c` on khô´i nho o trong . phuong nho c´ . tâm +1 ? , , ,, , ,, `,i giai. a) C´ ´n. Lo o thê nhân ¯uo. c khô´i nhu vây, . d . ban . d¯oc . t`ım môt . phuong a , , ´˘ng, nh˜u,ng khô´i nho b) Ta cô´ d¯inh a goi o l` a khô´i tra . môt . khô´i nho v` . n´ , , , canh n´ o l` a nh˜ung khô´i d¯en, bên canh nh˜ung khô´i d a nh˜ung khô´i ¯en s˜e l` . . , , ˜ khô´i nho s˜e hoa ´˘ng v` ´˘ng hoa ˘. c l` ˘. c l` ´ môi nho tra a vân vân. Khi d a tra a d¯en, ¯o , , ,, ˜ ´ ´ ´ ˘ng c´ môi khôi tra o khôi nho bên canh l` a d¯en v` a nguo. c lai. a tông . . Cho Pt l` , , , , ´˘ng, Pd l` nh˜ung sô´ trong c´ ac khô´i nho tra a tông c´ ac sô´ trong c´ ac khô´i nho

˜,ng b` 1.5. Nhu ai to´ an nâng cao

49

, , , , ´,c P = Pt − Pd không thay d¯ôi vo ´i thao t´ ˜ cho, d¯en. Biêu thu ac thay d¯ôi d¯a , v`ı ta công thêm c` ung môt ao hai khô´i nho canh nhau (ngh˜ıa l` a Pt v` a . . sô´ v` . , , , , ˜ Pd d¯ê`u gia t˘ ang nhu nhau). Dê t´ınh d a tri. ban d a 1, nê´u ¯uo. c gi´ ¯â`u cua P l` , ,, , ,, , ´ ´ `i hoi b) lai khôi nho o tâm l` a d¯en (P = 14 − 13). Khôi lâp o . phuong nhu d¯o . c´ , ,, ` ´ P = 0 − 1 = −1. Ðây l` a d¯iêu vô l´ı, ngh˜ıa l` a không thê c´ o khôi lâp . phuong , ` nhu d¯iêu kiên . b). , , , ,´, , Nh˜ung b` ai to´ an t`ım sô´ buo c thu. c hiên a râ´t kh´ o v` a ¯at . d¯ê d . kê´t qua l` , ,, , không c´ o phuong ph´ ap chung n` ao. Ta x´et môt ¯on gian sau d ¯ây: . v´ı du. d

J

, , , ˜ sô´ V´ı du. 1.64. Trên bang viê´t môt ay gô`m N sô´ theo môt . d˜ . thu´ tu. bâ´t k`ı, môi , , ´ hoa d ˘. c l` a −1 hoa ˘. c l` a +1. Môt ac sô´ ¯o ¯ôi cho ph´ep ta thay d ¯ôi dâ´u c´ . ph´ep biê´n d , , , , , , , o thê d¯ê chuyên trong môt ay. T`ım sô´ buo´c thu. c hiên . ´ıt nhâ´t c´ . d¯oan . con cua d˜ , `, d˜ tu ay ban d ay chı to` an sô´ +1. ¯â`u vê` d˜ , , ,, ,, , `,i giai. Ðai ˘. p sô´ Lo ai n` ay l` a sô´ luo. ng c´ ac ca . luo. ng bâ´t biê´n d¯on d¯iêu . cua b` , , trong d˜ ay canh nhau kh´ ac dâ´u. Khi ta thu. c hiên . . ph´ep biê´n d¯ôi thay dâ´u , , , ,, ´,n ho,n 2. môt o thê thay d¯ôi không lo . d¯oan . th`ı d¯ai . luo. ng bâ´t biê´n chı c´ , ,, `˘ng t`u, bô. sô´ −1, +1, −1, +1, ... d¯ê nhân ´,ng minh ra Ta s˜e chu . d¯uo. c bô. , ,´, ,, , ´,i [ N+1 a phâ`n sô´ +1, +1, +1, ... phai thu. c hiên ¯ây [x] k´ı hiêu . l` . vo 2 ] buoc (o d , , ,`, , ´ ˜˘n v` nguyên cua x). Ta x´et hai truong ho. p sô N cha a sô´ N le. , ,, ,, , ´d Gia su N = 2k, ngh˜ıa l` a [ N+1 ¯o ¯ai ¯iêu . luo. ng bâ´t biê´n d¯on d . 2 ] = k. Khi d , , , , `˘ng không. Nhung vo ´i k − 1 buo ´c thu.,c ban d a 2k − 1, c` on cuô´i c` ung ba ¯â`u l` , hiên . s˜e không d¯at . kê´t qua. , ,, ,`, , Gia su N = 2k + 1, ngh˜ıa l` a [ N+1 ay d ¯ai . p n` . 2 ] = k + 1. Trong truong ho ,, ,´, , , ´ ´ ` ´ luo. ng bât biên d¯on d a 2k, v`ı thê sau k − 1 buoc thu. c hiên ¯iêu . ban d¯âu l` . , , , ` ´ ´ ng minh th`ı t`u trang th´ ai ban d o d˜ ay to` an sô 1. Ta s˜e chu ¯âu không thê c´ . , ,´, , ,, ` ´ ˘ng sau k buoc thu. c hiên ra ai mong muôn. Thât ¯uo. c trang . phai d¯at . d . th´ . vây, . , , ,, , ` ` ´ ´ ´ ˘ng sau môt ta ch´ u ´y ra ¯ ôi d ¯ai ¯on . lân thu. c hiên . ph´ep biên d . luo. ng bât biên d , , d¯iêu a môt . . thay d¯ôi không qu´ . d¯on vi.

50

,, Chuong 1. Nguyên l´ı bâ´t biê´n

, ,´, , , `˘ng vo ´,ng minh ra ´,i [ N+1 Chı c` on chu . c hiên . th`ı t`u bô. sô´ ban 2 ] buoc thu , , , ,, o 1. Ta d¯ua v` ao trong bô. sô´ ban d ac d¯â`u s˜e nhân ¯â`u tâ´t ca c´ . d¯uo. c bô. sô´ chı c´ , , , , , , N+1 ´ a −1 sô´ luo. ng không lo ´n hon [ 2 ]. Tiê´p tuc nh´ om c´ o chu . ta thay d¯ôi dâ´u ,, , ´ o c´ ac sô trong nh˜ung nh´ om n` ay.

J

` tâp Bai . , , , , ´,i nhau th` ˘. t pha ˘ng cho N d¯iêm, t`u ch´ . 1.65. Trên ma ung c´ o thê nô´i vo anh , , , , `˘ng t`u môt ˘ng. Biê´t ra nh˜ung d¯oan at qu´ a 11 . tha . d¯iêm bâ´t k`ı không xuâ´t ph´ , , , , , `˘ng nh˜ung d `˘ng 4 m` ´ ng minh ra ˘ng. Chu d¯oan ay c´ o thê tô ba au sao ¯iêm n` . tha , , , , ´n hon N. ˘ng c´ cho nh˜ung d¯oan o hai d ut c` ung m` au không lo ¯â`u m´ . tha , , , . 1.66. Cho môt au d a môt au xanh. Môt ¯o v` . sô´ d¯iêm m` . sô´ d¯iêm m` . sô´ trong , , , ` ´i nhau th` ˘ng. Ta n´ ˘ng môt ch´ ung nô´i vo anh d oi ra a d¯a ˘. c biêt, ¯oan ¯iêm l` . . tha . d , , ,, , , , ´i n´ ´i m` nê´u hon môt ac d¯iêm c` on lai o c´ o m` au kh´ ac vo au cua n´ o. . nua c´ . nô´i vo , , ˘. c biêt ˘. c biêt Nê´u tô`n tai ay v` a tô lên n´ o m` au ¯iêm d¯a . d . th`ı ta chon . d¯iêm d¯a . n` , , , , , ` ´ ng minh ra ´c không c` ˘ng sau môt ˘. c kh´ ac. Chu on môt . sô´ h˜uu han . buo . d¯iêm d¯a biêt ao. . n` , , ,, . 1.67. Trên môt on ta viê´t n sô´ tu. nhiên. Gi˜ua hai sô´ canh nhau . d¯u`ong tr` . ,´, ´ , , , ´ ´ ´ ˜ ˜ ´ ´ ta x´ ta viêt uoc sô chung lon nhât. Sau d¯o oa nhung sô c˜ u d¯i, nhung sô´ c` on , , , ` ´ ˜ ´ ˘ng sau môt lai ac trên. Chung minh ra . ta lai . thu. c hiên . thao t´ . sô huu han . lâ`n , , , , `˘ng nhau. thu. c hiên ac th`ı tâ´t ca c´ ac sô´ trên d on d¯ê`u ba ¯u`ong tr` . thao t´ , , . 1.68. Trên bang ta viê´t 10 sô´: Môt a ch´ın sô´ kh´ ac 0. Ta thu. c hiên . sô´ 1 v` . , ` ´ ´ ˘ chon hai sô bâ t k` ı v` a thay v` a o v i tr´ ı c ua ch´ u ng b a ng trung b` ınh c ông hai sô´ . . . , , , , , n` ay. Sô´ n` ao l` a sô´ nho nhâ´t c´ o thê d¯at ay . d¯uo. c tai . vi. tr´ı cua sô´ 1 sau môt . d˜ , ph´ep thu. c hiên . trên. ´,ng th` ˘. t v` . 1.69. Môt anh môt ong tr` on v` a quay ma ao . sô´ em thiê´u nhi d¯u . v` , , ˜ ˜ ` ´ ` trong, môi em d o môt d¯ông th`oi, môi em d¯ua ¯êu c´ . sô keo. . Theo hiêu . lênh . , , , ,, ,, ´ l` nua sô´ keo ao d¯o a . cua m`ınh cho ban . o bên phai (nê´u sô´ keo . cua môt . em n`

1.6. Chuyên d am bâ´t biê´n ¯ê` vê` h`

51

, ,, , , ´ môt le, th`ı ngu`oi phu. tr´ ach d¯ua thêm cho em d ai keo). Hiêu choi ¯o . c´ . . lênh . , , `˘ng d¯ê´n môt ´,ng minh ra ˘. p d ˘. p lai nhu vây uc, tâ´t ca c´ ac ¯ i la . la . nhiê`u lâ`n. Chu . l´ , ` ˘ng nhau. em trong cuôc o sô´ keo . choi c´ . ba , ´,u tiê´ng Anh. . 1.70. Trong môt a s´ ach c´ o N tâp ach tra cu . thu viên . trên gi´ . s´ ,, , ˜ môt Môt ay môi ut l` am nh˜ung công viêc . ngu`oi m´ . ph´ . sau: Chon . môt . tâp . bâ´t ,, ˘. t n´ ´ ng vi. tr´ı v` ´ ng vi. tr´ı (ngh˜ıa l` k`ı, m` a n´ o không o d a d¯a o v` ao d¯u a nê´u sô´ ¯u ,`, , ´ ng minh ˘. t tâp tâp a k, th`ı nguoi m´ ay d¯a ay v` ao vi. tr´ı k trên gi´ a s´ ach). Chu . l` . n` , , , , `˘ng sau môt ´˘p d¯u ´ ng vi. tr´ı. ra ac tâp ach s˜e d¯uo. c sa . th`oi gian tâ´t ca c´ . s´ , , ,, ˜ ma ˘. t cua môt ´ v` . 1.71. Môi o viê´t sô´ trên d¯o a tâ´t ca . khô´i lâp . phuong c´ , ,, ˜ sô´ d `˘ng trung b`ınh công c´ ac sô´ d¯ê`u không giô´ng nhau. Môi ¯uo. c thay ba . cua , , ,, ˘. t bên canh. nh˜ung sô´ trong bô´n ma C´ o kha n˘ ang nhân ac sô´ ban d ¯â`u . . d¯uo. c c´ , , ˘. t d¯ô´i diên trên ma . sau ´ıt nhâ´t môt . lâ`n thay d¯ôi nhu trên không ? , , , ˜ ô cua . 1.72. Trên môi bang ke ô vuông 8 × 8 c´ o ghi môt ay l` a t´ıch . sô´, sô´ n` , , , , cua chı sô´ h` ang v` a chı sô´ côt a trong c´ ac ô â´y . cua ô â´y. Lâ´y ra 8 ô bâ´t k`ı v` ` ´,ng ˘m trong c` ung môt ang hay c` ung môt không c´ o hai ô n` ao na . h` . côt. . Chu , , `˘m trong c´ `˘ng t´ıch cua c´ ´ ac ô n` ay l` a không d minh ra ac sô´ na ¯ôi. T´ınh t´ıch d ¯o 2 (д ap sô´: (8!) = 1625702400). ,, , k ´ ta nhân . 1.73. Cho môt ac sô´ +1 v` a −1. T`u d ¯o . bô. sô´ luo. ng 2 c´ . bô. sô´ ˜ sô´ nhân vo `˘ng c´ ´,i sô´ tiê´p theo, sô´ cuô´i c` ´,i sô´ d ´,i ba ach: Môi ung nhân vo mo ¯â`u , , , `˘ng ´ ng minh ra ´i bô. sô´ mo ´i lai ˘. p lai tiên. Vo ac trên v` a tiê´p tuc. . la . thao t´ . Chu , , , cuô´i c` ung ta nhân o sô´ +1. ¯uo. c chı c´ . d

` bâ´t biê´n 1.6. Chuyên dê ¯ ` vê` ham , , , , `˘ng c´ Ta tông qu´ at h´ oa nh˜ung c´ ach giai loai ai to´ an n` ay ba ach d¯ua ra . b` , , ,, ,, , nh˜ung kh´ ai niêm ai tuong d¯uong v` a nh˜ung h` am bâ´t biê´n. . nh˜ung trang . th´

52

,, Chuong 1. Nguyên l´ı bâ´t biê´n

` bâ´t biê´n trên trang ´ 1.6.1. Ð.inh ngh˜ıa ham thai . , , ,´, , ,, `˘ng c´ ˜ x´et nh˜ung b` Nh˜ung phâ`n truo c ta d¯a ai to´ an giai ba ac d¯ai . luo. ng bâ´t , , , , , , biê´n c´ o dang: Cho môt o l` a nh˜ung trang . . tâp . ho. p M (nh˜ung phâ`n tu cua n´ . , , , ´ ´ ` ˘ th´ ai) v` a môt quy t a c biê n d ô i cho ph´ e p ta chuyê n t u tr ang th´ a i n` a y sang ¯ . . , , ,`, `˘ng ˘ trang th´ a i kh´ a c; câu h oi thu o ng d a t ra l` a cho m ôt tr ang th´ ai α c´ o thê ba ¯ . . . . , , , , ´˘c biê´n d¯ôi d ´c thu. c hiên ˜ cho β không môt ai d¯a ¯ê´n môt . sô´ buo . quy ta . trang . th´ , , , ? N´ oi c´ ach kh´ ac, cho hai trang ai α v` a β, c´ o thê biê´n d¯ôi t`u trang ai α . th´ . th´ , , ´˘c d ˜ cho không? d¯ê´n trang ai β sau môt ¯a . th´ . sô´ h˜uu han . lâ`n thu. c hiên . quy ta , , Hiên nhiên, ta c´ o t´ınh châ´t sau d¯ây: Nê´u môt th´ ai α c´ o thê tiê´n . trang . , , , ´,i trang ´,i trang to th´ ai β v` a t`u trang th´ ai β c´ o thê tiê´n to th´ ai γ, th`ı t`u α c´ o . . . , , ´ ´ ´ ` ´i γ, t´ınh chât n` thê tiên to ay goi a t´ınh ba ˘c câu. . l` , ,, , , ´,i trang Nê´u t`u môt th´ ai α c´ o thê tiê´n to th´ ai β v` a nguo. c lai . trang . . . t`u , ´,i trang trang th´ ai β c´ o thê tiê´n to th´ ai α, th`ı t´ınh châ´t n` ay goi a t´ınh d ¯ô´i . . . l` , , , ˜ ca ´ ng, v`ı vây ˘. p hai trang xu´ng. Không phai môi ai n` ao c˜ ung c´ o t´ınh d ¯ô´i xu . th´ . , , ´ ´ ´ ta han chê ch ı x´ e t c´ a c t âp tr ang th´ a i c´ o t´ ınh d ô i x u ng. ¯ . . . , , `˘ng t`u trang ´,i ch´ınh m`ınh, t´ınh châ´t Ta công nhân ai α c´ o thê tiê´n to . ra . th´ , n` ay goi a t´ınh phan xa. . . l` ,, ,, `˘ng hai trang Ðinh ngh˜ıa 1.1. Ta n´ oi ra th´ ai α v` a β l` a tuong d ¯uong, nê´u . . , , ,, ´˘c d¯a ´,i β v` ˜ cho t`u trang theo quy ta ai α c´ o thê tiê´n to a nguo. c lai. . th´ . Hai trang . ,, ,, th´ ai tuong d a β, k´ı hiêu a : α ∼ β. ¯uong α v` . l` , , `˘ng su., tu,o,ng d¯u,o,ng cua nh˜u,ng trang ai c´ o t´ınh phan xa, D˜ê thâ´y ra ¯ô´i . th´ . d , , , , , , , , ´ ` ´ ng v` ´p trang ˘c câu. Nh˜ung lo th´ ai tuong d xu a ba ¯uong s˜e phân chia tâp . . ho. p ˜ tâp trang ai M th` anh c´ ac tâp a trong môi . th´ . con M = M1 ∪ M2 ∪ ..., m` . con , ,, ,, ,, , ` ` ´ ´i nhau: Nêu α ∈ Mi v` moi ai cua n´ o d¯êu tuong d¯uong vo a . phân tu trang . th´ ,, ´ β ∈ Mi , th`ı α ∼ β. Nêu α ∈ Mi v` a β ∈ Mj (i 6= j) th`ı α v` a β không tuong ,, , ˜ d¯uong. Môi ho. p con Mi ta goi a môt th´ ai ¯ao. . qu˜y d . Môt . l` . không gian trang . , , , cua b` ai to´ an l` a ho. p nh˜ung qu˜y d ac nhau. ¯ao . kh´

1.6. Chuyên d am bâ´t biê´n ¯ê` vê` h`

53

Ðinh ngh˜ıa 1.2. Môt am sô´ f x´ ac d ai M goi a h` am ¯inh . trên tâp . h` . trang . th´ . l` . , , , , , , , ˘. c d bâ´t biê´n hoa th´ ai tuong d ¯uong ¯ai . . luo. ng bâ´t biê´n, nê´u trên nh˜ung trang n´ o nhân ung môt a tri, a . ngh˜ıa l` . c` . gi´ nê´u α ∼ β th`ı f(α) = f(β).

(1.6)

, , , , ,, ,, T`u f(α) = f(β) không thê n´ oi g`ı vê` su. tuong d¯uong cua hai trang th´ ai . , , ´ ´ ´i: α v` a β. Ta d ao môt am bât biên mo ¯ua v` . loai . h` ,, Ðinh ngh˜ıa 1.3. Môt am bâ´t biê´n f d¯uo. c goi a bâ´t biê´n van ang, nê´u . n˘ . h` . l` . ,, ,, , hai trang th´ ai không tuong d o nhân a tri. kh´ ac ¯uong nhau th`ı n´ . . nh˜ung gi´ nhau: nê´u α 6∼ β th`ı f(α) 6= f(β). , ´,i bâ´t Bâ´t biê´n van ang trên môt a tri. . V`ı thê´ vo . n˘ . qu˜y d¯ao . nhân . chı môt . gi´ biê´n van ang . n˘ α ∼ β ⇔ f(α) = f(β). , , ´ ´ ´ ˘. p trang Nhu vây, ang, cho môt th´ ai bâ´t k`ı luôn cho . voi bât biên van . n˘ . ca . , , , , ph´ep ta kê´t luân ung c´ o tuong d¯uong hay l` a không. . ch´ , , , Nhung l` am thê´ n` ao d¯ê kiêm tra môt am bâ´t biê´n l` a van ang? Không . h` . n˘ , , ,, , , c´ o phuong ph´ ap chung nhung c´ o môt . tiêu chuân d¯on gian sau d¯ây: Ðinh l´ı 1.1. Nê´u . ,, a) tô`n tai ai δ1 , δ2 , ..., δ` sao cho moi ai α ∈ M tuong . ` trang . th´ . trang . th´ ,, , d¯uong v´ oi môt ung v` a . trong ch´ ˜,u han b) h` am bâ´t biê´n f nhân a tri. kh´ ac nhau, . hu . ` gi´ , ˜ ng trang th`ı f l` a h` am bâ´t biê´n van ang v` a nhu th´ ai δi , δj (i 6= j) d ¯ôi môt . n˘ . . ,, ,, , không tuong d oi nhau. ¯uong v´ , ` , ` T`u d a ` qu˜y d ¯iêu kiên ¯ao. ¯iêu kiên . a) cho ta tô`n tai . không qu´ . T`u d . b) cho , , , ,, ` ` ta tôn tai . không nho hon ` qu˜y d¯ao. . Suy ra tôn tai . ` qu˜y d¯ao. . Trên co so

,, Chuong 1. Nguyên l´ı bâ´t biê´n

54

, , cua b) lai a tri. kh´ ac nhau, ngh˜ıa l` a f l` a bâ´t . cho ta bâ´t biê´n f chı nhân . ` gi´ , , `˘m o,, biê´n van ang. Cuô´i c` ung t`u a) cho ta nh˜ung trang ai δ1 , δ2 , ..., δ` na . n˘ . th´ ,, ,, ´,i nhau. ´ d¯ôi môt c´ ac qu˜y d ac nhau, do d ¯ao ¯o ¯uong vo . kh´ . không tuong d , Ta lâ´y v´ı du. cu. thê: ,, on. L´ uc d V´ı du. 1.74. Ngu`oi ta trô`ng n cây xung quanh môt ¯â`u . hô` h`ınh tr` , , , ,`, ˜ ` ´ ´ ba ˘ng cu´ nhu ai d ˘t nhip: môi cây d o, nhung nguoi ta thâ´y ra ¯o ¯âu . . môt . con c` ˜ ´ c` Môi lâ`n c´ o hai ch´ u c` o trên hai cây n` ao d ung bay lên v` a d¯âu ¯o . sang cây bên , , canh, môt on con kia lai ¯ô`ng hô` cua b`o hô`, c` . . con bay theo chiê`u kim d . bay ,, ,, , ˜ ` trang nguo. c chiê`u kim d th´ ai ban d o o môt ¯ô`ng hô`. Tu ¯â`u µ: Môi con c` . . cây, , , , ˜ ´ theo nhip ac ch´ u c` o n´ oi trên, d uc n` ao d ¯ôi chuyên chô cua c´ ¯ê´n môt ¯o . thay d . l´ , , , , ´d t´ oi trang ai ν: Tâ´t ca c´ ac ch´ u c` od ao d ¯âu ¯o ¯uo. c không? . th´ . dô`n trên môt . cây n` , , , `˘ng c´ `,i giai. Ta c´ Lo o thê giai b` ai to´ an ba ach t`ım c´ ac h` am bâ´t biê´n theo c´ ach kh´ ac nhau:

(a)

(b)

(c)

H`ınh 1.12 ´˘ng. Khi C´ ach 1: Nê´u n = 2m. Ta tô m` au môt ach môt . cây xanh c´ . cây tra , , , ˜ lâ`n d¯ôi chô ˜ sô´ c` ´˘ng hoa ˘. c l` ´ môi d¯o o trong cây tra a không thay d¯ôi nhu h`ınh

1.6. Chuyên d am bâ´t biê´n ¯ê` vê` h`

55

, , , ˘. c l` ˘. c l` 1.12(a), hoa a t˘ ang lên 2 nhu h`ınh 1.12(b), hoa a giam d¯i 2 nhu h`ınh , , ´,i môt 1.12(c). Vo ai bâ´t k`ı α cua nh˜ung con c` o ta k´ı hiêu a sô´ . trang . th´ . δ(α) l` ,, , , ´ ˘ng. Ta x´et h` con c` od au tra am p(α) nhu sau: ¯âu . o nh˜ung cây m`  ˜˘n , 0 nê´u δ(α) l` a sô´ cha , p(α) = 1 nê´u δ(α) l` a sô´ le . ,, , ˜˘n nh˜u,ng con c` Theo nh˜ung nhân am sô´ p c´ o sô´ cha o trên . x´et o trên th`ı h` , ´ ˘ng, l` cây tra a không d¯ôi v` a p(ν) = 0. , n , ´,i trang Nê´u m = 2k + 1, th`ı l` a môt a, vo th´ ai ngay t`u . sô´ le. Ngh˜ıa l` . 2 , d¯â`u µ ta c´ o p(µ) = 1. T`u p(µ) 6= p(ν) suy ra hai trang th´ ai µ v` a ν không . ,, ,, , ,`, , , tuong d ¯uong. Nhu vây . trong truong ho. p (n = 2m, m = 2k + 1) t`u trang . , ´,i trang th´ ai µ không thê tiê´n to th´ a i ν. . n , ˜˘n v` ´ l` Nhung c` on nê´u m = 2k th`ı sao? Khi d a sô´ cha a p(µ) = p(ν) = ¯o 2 ,`, , ,, , 0. Trong truong ho. p n` ay h` am bâ´t biê´n p không d a c´ o ¯ua ra kê´t luân . d¯uo. c l` ,, , ,, tuong d¯uong gi˜ua hai trang ai µ v` a ν hay không. . th´ , , ´,i h` Nhu vây am n` ay không d¯ua d¯ê´n kê´t luân. u ´y l` a nê´u f l` a . vo . Môt . ch´ , , ´ ´ ` ´ α v` h` am bât biên th`ı t`u f(α) = f(β) không suy ra d¯iêu g`ı ca: Khi d¯o a β c´ o , ,, , ,, ,, ,, ´ thê tuong d¯uong v` a c˜ ung c´ o thê không tuong d ¯uong. Nêu f(α) 6= f(β) th`ı ,, ,, , ` trang th´ ai α v` a β không tuong d¯uong (d¯iêu n` ay suy ra t`u d ¯inh . ngh˜ıa). Ta . , , ´ ´ ´ c´ o thê thiêt lâp am bât biên theo kiêu kh´ ac. . h`

i+1

j−1 j

H`ınh 1.13

n

i

i

i−1

1

H`ınh 1.14

, ˜ trên ´,i trang ´nh sô´ c´ C´ ach 2: Ta d ac cây t`u 1 d ai α c´ ac con c` o d¯ô ¯a ¯ê´n n. Vo . th´

,, Chuong 1. Nguyên l´ı bâ´t biê´n

56

, ´, k vo ´,i trang cây bâ´t k`ı, ta k´ı hiêu a sô´ c` o tai ai n` ay. Bây gi`o . ak (α) l` . cây thu . th´ ta x´et h` am q(α) = 1.a1 (α) + 2.a2 (α) + 3.a3 (α) + · · · + n.an (α).

(1.7)

, H` am q(α) c´ o phai l` a h` am bâ´t biê´n? , , , ,, Môt ai thay d¯ôi bâ´t k`ı d¯uo. c mô ta trên h`ınh 1.13 v` a tông (1.7) . trang . th´ ,, , ,, , chı anh huong d¯ê´n 4 th`ua sô´ sau: · · · + i.ai (α) + (i + 1).ai+1 (α) + (j − 1).aj−1 (α) + j.aj (α) + · · ·

(1.8)

, , ,, ´,i viêc Vo ai trong h`ınh 1.13 th`ı tông (1.8) tro th` anh . thay d¯ôi trang . th´ · · ·+i.[ai (α)−1]+(i+1).[ai+1 (α)+1]+(j−1).[aj−1 (α)+1]+j.[aj (α)−1]+· · ·

, , , , , ,, ,´, `˘ng tông v`u,a thay d D˜ê kiêm tra thâ´y ra a tông truo c khi thay d ¯ôi v` ¯ôi o trên , ´ ng, v`ı l` a tr` ung nhau. Phai ch˘ ang q(α) l` a h` am bâ´t biê´n? Ðiê`u n` ay không d¯u , , , , `˘ng c´ c` on ba kha n˘ ang n˜ua nhu c´ ac h`ınh 1.14, 1.15, 1.16. Ba ach x´et tông , , nhu trên th`ı h`ınh 1.14, q(α) giam d¯i n, c` on h`ınh 1.15, q(α) t˘ ang lên n. , , ,`, , , Truong ho. p cuô´i c` ung h`ınh 1.16, q(α) không thay d a tri. cua ¯ôi. Nhu vây, . gi´ , , , q(α) c´ o thay d a sô´ n. Suy ra, d am r(α) gi´ a ¯ôi nhung chı l` ¯inh . ngh˜ıa môt . h` , , , , tri. cua n´ o l` a c´ ac sô´ du cua ph´ep chia q(α) cho n, l` a môt am bâ´t biê´n. T`u . h` , trang ai ν tâ´t ca con c` od ¯âu . th´ . trên môt . cây ` th`ı  a1 (ν) = a2 (ν) = ... = a`−1 (ν) = a`+1 (ν) = ... = an (ν) = 0, a` (ν) = n. ´,i bâ´t k`ı ` v` ˘. t kh´ Ngh˜ıa l` a q(ν) = `.n v` a r(ν) = 0 (vo a n). Ma ac, a1 (µ) = a2 (µ) = ... = an (µ) = 1. Vây . q(µ) = 1 + 2 + · · · + n =

n(n + 1) . 2

˜˘n ´,i n l` Nê´u n = 2m, th`ı q(µ) = n.m + m v` a r(µ) = m 6= 0. Suy ra vo a sô´ cha ,, ,, ,, ´ µ v` th`ı ta nhân a ν không tuong d ¯uong. . d¯uo. c r(ν) 6= r(µ), do d¯o

1.6. Chuyên d am bâ´t biê´n ¯ê` vê` h`

i+1

57

n

n

i 1

1

H`ınh 1.15

H`ınh 1.16

, Nê´u n = 2m + 1, th`ı q(µ) = n(m + 1) v` a r(µ) = 0. Nhu vây . trong , , ,`, , , , truong ho. p n l` a sô´ le ta lai o r(ν) = r(µ). Tru`ong ho. p n` ay ta lai o . c´ . không c´ , , , , , , kê´t luân ai ν v` a µ. . g`ı vê` su. tuong d¯uong cua hai trang . th´ , , `˘ng r(α) l` ´ ng minh ra Ta d` ung d¯inh a bâ´t biê´n van ang. Thât . l´ı 1.1 d¯ê chu . n˘ . ,, , , ´ ˜ vây, k´ ı hi êu δ l` a tr ang th´ a i sau: M ôt con c` o o trên cây th u i, c` o n nh u ng i . . . . , , ,, , ,, ´ con c` on lai d âu o cây th u n. V ây δ d a trang ai tâ´t ca c´ ac ch´ u ¯ ¯uo. c hiêu l` n . . . . th´ , ´ n. c` o d¯ê`u trên cây thu , ,, ,, `˘ng môt ´,i môt Ta s˜e chı ra ra ai bâ´t k`ı d¯ê`u tuong d¯uong vo . trang . th´ . trong , , ´ nh˜ung trang ai δ1 , δ2 , ..., δn . Thât a trang ai bât k`ı cua c´ ac . th´ . vây, . cho α l` . th´ , , ´ ´ ` ´ ` ´ n. Theo ph´ep ˘ng dôn tât ca c´ ac ch´ u c` o vê cây thu ch´ u c` o trên cây. Ta cô ga , ,, , , ´ ´ ˜ ra ta tiên h` biên d ai d¯a anh nhu sau: Con c` o o cây sô´ 1 bay theo ¯ôi nhu b` ,´, ,, , ´ con sô´ 2 bay theo chiê`u nguo. c lai. huong vê` cây sô´ n, d uc d¯o ¯ô`ng th`oi l´ . , ` ´ ` ´ ´ ´ lai Sau d a con d ¯o ¯âu . dôn con sô hai bay vê cây thu n v` . trên cây sô 3 bay ,, ,, , ´ , ` ´ ´ ´, theo chiêu nguo. c lai, od ¯âu . ... cu tiêp tuc . nhu vây . cho d¯ên con c` . o cây thu ´, (n − 1) vê` cây thu ´, n th`ı con c` ´, n (n − 1). Khi dô`n ch´ u c` o thu o trên cây thu ,, ´, i n` ´ (i = 1, 2, ..., n − 1). Ðiê`u n` s˜e bay nguo. c lai ao d¯o ay c´ o ¯ê´n môt . d . cây thu , ´ ngh˜ıa l` a α ∼ δi . Ta t´ınh r(δi ). Voi i 6= n:    a1 (δi ) = a2 (δi ) = ... = ai−1 (δi ) = ai+1 (δi ) = ... = an−1 (δi ) = 0, ai (δi ) = 1,   an (δi ) = n − 1. Suy ra, q(δi ) = i.1 + n.(n − 1) v` a r(δi ) = i. Ngo` ai ra q(δn ) = n.n v` a

,, Chuong 1. Nguyên l´ı bâ´t biê´n

58

, , r(δn ) = 0. Nhu vây a tri. kh´ ac nhau. . r nhân . n h˜uu han . gi´ Theo d¯inh a van ang v` a c´ ac trang th´ ai δ1 , δ2 , ..., δn . l´ı 1.1 bâ´t biê´n r l` . n˘ . ,, ,, d¯ôi môt a van ang nên . không tuong d¯uong. V`ı r l` . n˘ α ∼ β ⇔ r(α) = r(β). ,´, ,, ´,ng minh d ˜ chu Phâ`n truo c ta d ¯a ¯uo. c , r(µ) = r(ν) ⇔ n l` a sô´ le. , , , , ,, an Suy ra µ ∼ ν khi v` a chı khi n l` a sô´ le. Nhu vây ai to´ an d ¯uo. c giai ho` . b` to` an. , , , , Vê` nh˜ung h` am bâ´t biê´n, ban o thê giai nh˜ung b` ai tâp . d¯oc . c´ . sau: `˘ng nê´u f l` ´,ng minh ra 1. Chu a h` am bâ´t biê´n v` a g l` a môt am sô´ bâ´t k`ı, . h` th`ı h` am h(α) = g(f(α)) c˜ ung l` a h` am bâ´t biê´n. , ` ´ ng minh ra ˘ng nê´u h l` 2. Chu a bâ´t biê´n, f l` a h` am van ang, th`ı tô`n tai . n˘ . , , ´ c sau d¯u ˘ng thu ´ ng h(α) = g(f(α)). môt am sô´ g sao cho d ¯a . h` , , , , 3. T`u h` am van ang r cua l`oi giai trong v´ı du. 1.74 ta d¯inh . ngh˜ıa hai . n˘ , 2 2 ´,ng minh h` am bâ´t biê´n n˜ua: f(α) = [r(α)] v` a g(α) = [r(α) − 2] . Chu

J

`˘ng h` am bâ´t biê´n f l` a van ang, c` on h` am g không van ang. ra . n˘ . n˘ , , , ´ ´ ` ´i d 4. Cho h` am f bât biên van ang. Vo am sô´ g d¯ê cho ¯iêu kiên . n˘ . g`ı cua h` ,, ´,c h(α) = g(f(α)) l` h` am h d ac d¯inh a môt am bâ´t ¯uo. c x´ . theo công thu . h` biê´n van ang ? . n˘ , 1.6.2. Hê. thô´ng bâ´t biê´n dâ ¯ `y d¯u ,, ,, ´ng l˜e d¯i t`ım v` Nhiê`u khi d¯a a su dung h` am bâ´t biê´n van ang, ngu`oi ta . . n˘ , ,, , d¯i t`ım hê. bâ´t biê´n d¯â`y d¯u d¯uo. c d ¯inh . ngh˜ıa nhu sau: , , ,, Ðinh ngh˜ıa 1.4. Môt ad ¯â`y d ¯u, ¯uo. c goi . hê. nh˜ung bâ´t biê´n {f1 , f2 , ..., fk } d . l` .

1.6. Chuyên d am bâ´t biê´n ¯ê` vê` h` , , ´,c ˘ng thu nê´u nh˜ung d¯a

 f1 (α)    f (α) 2     fk (α)

59

= f1 (β), = f2 (β), ... = fk (β).

(1.9)

, ,, ,, , ´ ng khi v` d¯ô`ng th`oi d¯u a chı khi trang ai α v` a β tuong d¯uong. . th´ , , , , T`u d¯inh a hê. nh˜ung bâ´t biê´n d a tông qu´ at ¯â`y d ¯u l` . ngh˜ıa ta thâ´y ngay l` h´ oa kh´ ai niêm ang: Nê´u f l` a bâ´t biê´n van ang, th`ı hê. {f} . bâ´t biê´n van . n˘ . n˘ , gô`m môt a hê. d¯â`y d¯u. . bâ´t biê´n l` , , , ˜,ng sô´ nguyên. Ðu,o.,c ph´ep thay V´ı du. 1.75. Trên bang ke ô c˜o 2 × 2 ta viê´t nhu , d ¯ôi: ,, ˜,ng sô´ c` 1. Trên môt od on ¯uo. c công . côt . bâ´t k`ı, môt . sô´ trong n´ . thêm 2, nhu , ` d¯i 2; trong côt . tru ,, ˜,ng sô´ c` 2. Trên môt ang bâ´t k`ı, môt od on ¯uo. c công . h` . sô´ trong n´ . thêm 3, nhu , ` d¯i 3; trong h` ang tru , , , ,, , ˜ ng bang tu,o,ng d Nhu o l` a nhu thê´ n` ao? ¯uong cua n´ , , `,i giai. Ta x´et ba h` ´,i môt Lo am: vo . bang bâ´t k`ı α =

 a c

 b , k´ı hiêu . d

p(α) = a + b + c + d, , , q(α) = sô´ du cua sô´ a + b chia cho 2, , , r(α) = sô´ du cua sô´ a + c chia cho 3. , , , Nh˜ung h` am p, q, r trên l` a h` am bâ´t biê´n. Không kh´ o kiêm tra môt . bang α , ,, ,, ´,i bang sau bâ´t k`ı tuong d¯uong vo   0 q(α) β= . r(α) p(α) − q(α) − r(α)

,, Chuong 1. Nguyên l´ı bâ´t biê´n

60 , , ´,c ˘ng thu Thât ac d ¯a . vây, . t`u c´

  p(α) = p(β), q(α) = q(β),   r(α) = r(β),

(1.10)

, , , , ,, ,, ´,i môt ˘ng d¯inh kha a β c` ung tuong d¯uong vo a . nh˜ung bang α v` . bang, ngh˜ıa l` ,, ,, , ´i nhau. ch´ ung tuong d¯uong vo , , , ,, , ,, ,, ´,c ˘ng thu Nguo. c lai, a β k´eo theo c´ ac d¯a . su. tuong d¯uong cua hai bang α v` , trong (1.10), v`ı thê´ p, q, r l` a bâ´t biê´n. Nhu vây a hê. bâ´t biê´n d ¯â`y . {p, q, r} l` , d¯u. , , , Ta c´ o thê d˜ê d` ang giai nh˜ung b` ai tâp . sau: , `˘ng nê´u f1 , ..., fk l` ´,ng minh ra 1. Chu a nh˜ung h` am bâ´t biê´n v` a g l` a môt .

J

h` am sô´ k d am ¯ô´i sô´, th`ı h` h(α) = g(f1 (α), f2 (α), ..., fk (α))

(1.11)

l` a môt am bâ´t biê´n. . h` , `˘ng nê´u h l` ´ ng minh ra 2. Chu a môt am bâ´t biê´n v` a {f1 , ..., fk } l` a hê. bâ´t . h` , , biê´n d am g c´ okd an ¯â`y d¯u, th`ı tô`n tai ¯ô´i sô´ sao cho thoa m˜ . môt . h` (1.11).

, , , , , ˘. p sô´ thu. c (x, y). X´et ph´ep chuyên d 3. Cho M l` a tâp ¯ôi duy . ho. p nh˜ung ca ˘. t nhâ´t (x, y) → (y, x). Ða  f1 (x, y) = xy, f2 (x, y) = x + y. , `˘ng hê. {f1 , f2 } l` ´,ng minh ra Chu a môt ¯â`y d ¯u. . hê. bâ´t biê´n d , , , , , ˘. p sô´ thu. c (x, y, z). X´et ph´ep chuyên d¯ôi 4. Cho M l` a tâp . ho. p nh˜ung ca ˘. t (x, y, z) → (y, x, z) v` a (x, y, z) → (x, z, y). Ða   f1 (x, y, z) = xyz, f2 (x, y, z) = xy + yz + zx,   f2 (x, y, z) = x + y + z.

1.6. Chuyên d am bâ´t biê´n ¯ê` vê` h`

61

, `˘ng hê. {f1 , f2 , f3 } l` ´,ng minh ra Chu a môt . hê. bâ´t biê´n d¯â`y d¯u. , , , Ban o thê tông qu´ at h´ oa b` ai tâp ung n` ay. Nh˜ung b` ai tâp ¯oc . d . c´ . cuô´i c` . , , , , ´ ´ ` ´ ´ c. Nh˜ung h` ˘c trong d n` ay l` a nh˜ung vân d am n` ay ¯ê sâu sa ¯ai . sô nh˜ung d¯a thu , , , ´ ´ ´ ` ˜ ˜ không nhung c´ o nhung t´ınh chât bât biên m` a c` on nhiêu ´ıch lo. i kh´ ac trong ,, , , ,, ` , ´ ` ´ ˜ ´ ´ nhung lop d ay d¯uo. c d ¯a thuc, vân d¯ê n` ¯ê câp . trong chuong tiêp theo d¯ây. , ,, , , , , , Ðê kê´t th´ uc chuong n` ay c´ ac ban ai tâp ac ¯ây nh`o c´ . thu giai nh˜ung b` . sau d , , , ´ c ta d kiê´n thu ¯a thu nhân . d¯uo. c trong phâ`n chuyên d¯ê`.

` tâp Bai . , ´˘c m´ . 1.76. Môt ay vi t´ınh gô`m hai phâ`n nhu h`ınh 1.17, môt . công ta . bên , , , ˜ ´˘m v` ´˘m. C´ ´nh sô´ t`u 1 d¯ê´n n nh˜ung c´ o n lô ca a môt o thê d¯a . bên gô`m n d¯â`u ca , ˜ ca ˜ lâ`n nô´i công ta ´˘m sao cho vo ´˘c môt ´˘m ro,i v` ´,i môi lô ao . trong nh˜ung d¯â`u ca , , , ˜ ´ ng sô´ cua m`ınh d¯uo. c không? lô c´ o d¯u

2 3 4 1 n n−1

H`ınh 1.17

1

2

3

4

1

2

3

4

5

6

7

8

5

6

7

8

9

10 11 12

9

10 11 12

13 14 15 (a)

13 15 14 (b) H`ınh 1.18

, , , ´,i môt . 1.77. Tr` o choi d ac sô´ trong bang 16 ô vo a 15 ô c´ o ¯ây c´ . ô trô´ng v` , , , ,, ´ ` ´ c´ ac sô nhu h`ınh v˜e 1.18. Ðuo. c ph´ep môt ¯ây sô cua môt . lân d . ô sang bên , , , , ´ ´ canh ô c` on trông. C´ o thê thu. c hiên ac ph´ep d¯ây t`u trang th´ ai . . liên tiêp c´ .

,, Chuong 1. Nguyên l´ı bâ´t biê´n

62

,, h`ınh 1.18(a) sang trang th´ ai 1.18(b) d¯uo. c không? H˜ ay t`ım h` am bâ´t biê´n . , , van ang cua tr` o choi n` ay. . n˘ , ,, , , ˜ . 1.78. Môt o nhiê`u ô c˜o 11 × 11 d¯uo. c bo d ¯i 22 ô sao cho trên môi . bang c´ , , , ,, ,, ˜ h` ´ ng 2 ô. Hai vi. tr´ı nh˜ung ô d¯uo. c bo d¯i côt a trên môi ang d¯uo. c bo d¯i d ¯u . v` , , ,, ,, , tuong d ac côt ung v` a ¯uong nhau nê´u ta chuyên d¯ôi môt . sô´ bâ´t k`ı c´ . gi˜ua ch´ , , , , , nh˜ung h` ang gi˜ua ch´ ung, th`ı t`u vi. tr´ı n` ay nhân o bao nhiêu . d¯uo. c vi. tr´ı kia. C´ , , ,, ,, , , , vi. tr´ı không tuong d ¯uong cua nh˜ung ô d¯uo. c bo d¯i? , , ,, . 1.79. Tâ´t ca c´ ac sô´ nguyên t`u 1 d¯ê´n 2n d¯uo. c viê´t th` anh môt ang. Ta . h` , , , , , ˜ `˘ng ´ ng minh ra ´i môi sô´ công ´i sô´ l` ´. Chu thu. c hiên v` ao vo a vi. tr´ı cua sô´ d ¯o . vo . , , ,, ` ˘ng c´ trong nh˜ung tông nhân ach trên c´ o hai sô´ chia 2n cho c` ung . d¯uo. c ba , môt . sô´ du. ,, , ´,i nh˜u,ng d . 1.80. Vo u c` o c` ung bay lên ¯iê`u kiên . o v´ı du. 1.74, nhung hai ch´ , , ˘. c c` d¯ê chuyên sang cây kh´ ac theo hai chiê`u kh´ ac nhau hoa ung môt . chiê`u , , , d¯ê`u d¯uo. c. H˜ ay t´ınh h` am bâ´t biê´n van ang cua b` ai to´ an n` ay. . n˘

H`ınh 1.19 , ,, , ˘. t v` ˘. c −1. . 1.81. Trong bang c˜o 3 × 3 ô, ngu`oi ta d¯a ao c´ ac ô c´ ac sô´ +1 hoa , ,, ˘. c môt Môt a thay c´ ac dâ´u o c´ ac ô theo môt ang hoa ¯ôi l` . ph´ep biê´n d . h` . côt. . , `˘ng ´ ng minh ra Chu a) Sô´ qu˜y d a 16; ¯ao . l`

1.6. Chuyên d am bâ´t biê´n ¯ê` vê` h`

63

, ˜ qu˜y d ´,a 32 phâ`n tu,; b) Môi ¯ao . d¯ê`u chu , , , , c) T´ıch cua tâ´t ca c´ ac sô´ trong h`ınh vuông bâ´t k`ı c˜o 2 × 2 trong bang l` a ´ ´ bât biên; , , ,, , ´nh dâ´u trong bô´n h`ınh vuông nhu d) T´ıch cua tâ´t ca c´ ac sô´ d¯uo. c d ¯a , h`ınh 1.19, tao anh môt . th` . hê. bâ´t biê´n d¯â`y d¯u. , ´, tu., bâ´t k`ı; d˜ê H˜ ay giai b` ai to´ an n` ay trong c´ ac phâ`n trên theo môt . thu , ˜ nhau. thâ´y c´ ac phâ`n d up lân ¯ê`u tro. gi´ , ,, , ,, , . 1.82. Vecto (a, b), o d a nh˜ung sô´ nguyên, d¯uo. c biê´n d¯ôi th` anh ¯ây a, b l` , , môt ay t`ım h` am bâ´t biê´n . trong nh˜ung vecto (a + b, b), (a − b, b), (b, a). H˜ , van ang cua b` ai to´ an n` ay. . n˘ ,, , ,, , ˘. p vecto (a, b), (c, d), o d . 1.83. Ca a nh˜ung sô´ nguyên, d¯uo. c ¯ây a, b, c, d l` , , ˘. p vecto (a+b, b), (c+d, d); (a−b, b), (c−d, d); ph´ep biê´n d anh c´ ac ca ¯ôi th` , , (b, a), (d, c). H˜ ay t´ınh hê. bâ´t biê´n d ai to´ an n` ay. ¯â`y d ¯u cua b`

, , CHUONG

2

, , ´ c dô ´ ng hai biê´n Ða thu ¯ ´i xu

` t´ınh châ´t . . . . . . . . . . . . . . . . . . . . . . . . . . . . . . . . . . . . . . . . . . . . . . . . . . 65 2.1. Ð.inh ngh˜ıa va , , , 2.2. Ð.inh l´ı co ban cho da uc hai biê´n . . . . . . . . . . . . . . . . . . . . . . . . . . . . . . . . . . . . 67 ¯ th´ , ,, , 2.3. Giai hê. phuong tr`ınh dô ung . . . . . . . . . . . . . . . . . . . . . . . . . . . . . . . . . . . . . . . . . . 71 ¯ ´i x´ , , , , 2.4. Ðua vê` hê. phuong tr`ınh dang dô ung . . . . . . . . . . . . . . . . . . . . . . . . . . . . . . . . 77 ¯ ´i x´ . , , , , ˘ ng th´ 2.5. Ch´ ung minh bâ´t d¯a uc dô ung . . . . . . . . . . . . . . . . . . . . . . . . . . . . . . . . . . . 82 ¯ ´i x´ , ` toan ´ vê` tam th´ 2.6. Bai uc bâc . hai . . . . . . . . . . . . . . . . . . . . . . . . . . . . . . . . . . . . . . . . . . . 87 , , , 2.7. Phân t´ıch da uc dô ung ra th`ua sô´ . . . . . . . . . . . . . . . . . . . . . . . . . . . . . . . . . . 91 ¯ th´ ¯ ´i x´ , ` toan ´ khac ´ . . . . . . . . . . . . . . . . . . . . . . . . . . . . . . . . . . . . . . . . . . . . . . . . . . . . 95 2.8. Nh˜ung bai ,, , ` ` 2.9. Chuyên dê ung . . . . . . . . . . . . . . . . . . . . . . . . . . . 101 ¯ vê phuong tr`ınh hê. sô´ dô ¯ ´i x´

, , , ´,c hê. sô´ d¯ô´i xu ´,ng . . . . . . . . . 2.9.1. Ðinh . l´ı co ban cua d¯a thu , , ,, 2.9.2. Nh˜ung v´ı du. giai phuong tr`ınh bâc . cao . . . . . . . . . . . .

, , ,, , ` tra l`oi bai ` tâp 2.10. Go. i y´ va . chuong 2 . . . . . . . . . . . . . . . . . . . . . . . . . . . . . . . . . . . . .

101 105 110

,, , ´,c hai biê´n, x´et môt ´,c Chuong n` ay d¯ua ra kh´ ai niêm ¯a thu . vê` d . loai . d¯a thu , , ˘. c biêt ´ d¯a a c´ o gi´ a tri. không d¯ôi khi ta thay d¯ôi c´ ac biê´n cho nhau. Sau d ¯o . l` , , , , , , , ` ´ ´c d ´ ng. Ðinh ´ ng minh chi l` a x´et môt ¯inh ¯ôi xu . l´ı d¯uo. c chu . l´ı co ban vê d¯a thu . d , , , ´ ´ ng dung tiêt v` a du. a trên d¯inh ay ta u giai h` ang loat ai tâp . l´ı n` . . b` . trong d¯ai . sô´ , , ,, ´,c ra th`u,a sô´, chu ´,ng minh bâ´t nhu: Giai hê. phuong tr`ınh, phân t´ıch d ¯a thu , ´,c, ... ˘ng thu d¯a

2.1. Ðinh a t´ınh châ´t . ngh˜ıa v`

65

` t´ınh châ´t 2.1. Ð.inh ngh˜ıa va , , ´,c). Môt ˘. c sô´ phu Cho b l` a môt o thê l` a sô´ thu. c hoa am sô´ . sô´ (c´ . h` , ,, , , , ` k ´i `, k l` a môt a nh˜ung sô´ ϕ(x, y) = ax y d ¯on thu´c hai biê´n x, y vo ¯uo. c goi . d . l` , ,, , ´,c v` nguyên không âm. Sô´ a d¯uo. c goi a hê. sô´ cua d¯on thu a sô´ ` + k goi a . l` . l` , , , , , ` k bâc a d¯uo. c k´ı hiêu a deg(ax y ) = ` + k. ¯on thu´c v` . cua d . l` , , ´,c d¯u,o.,c goi Hai d ad nê´u ch´ ung chı kh´ ac nhau vê` hê. ¯ô`ng dang, ¯on thu . . l` , ´,c ϕ = ax`1 yk1 v` a d¯ô`ng dang, nê´u ch´ ung phu. a ψ = bx`2 yk2 l` sô´. Ðon thu . ´ ´ thuôc ao c` ung sô biên v` a `1 = `2 , k1 = k2 . . v` , , , `˘ng tông cua hai d¯o,n thu ´,c d¯ô`ng dang ´,c, v`ı khi D˜ê thâ´y ra a môt . l` . d¯on thu , , ´,c d¯ô`ng dang ´ ax` yk + bx` yk = (a + b)x` yk . T´ıch cua hai d d¯o c˜ ung ¯on thu . , , , ` k ` k , , , 2` 2k ´ tao ¯on . ra môt . d¯on thuc nhu ax y .bx y = (a.b)x y . Nhung tông hai d , , , ´,c không d ´,c. thu o thê không phai l` a môt ¯ô`ng dang ¯on thu . c´ . d , Ðinh ngh˜ıa 2.1. Môt am hai biê´n P(x, y) goi ad o ¯a thu´c hai biê´n, nê´u n´ . h` . l` . , , , , , , , , ´ c hai biê´n. c´ o thê biêu di˜ên nhu môt . tông h˜uu han . nh˜ung d¯on thu , , ´˘p xê´p bâc ´,c hai biê´n c´ ´ Nhu vây o thê sa ao d¯o ¯a thu . môt . d . theo môt . biê´n n` , , , 4 3 2 ´ ´n d¯ê´n nho goi t`u lo a sa ˘p xê´p theo bâc . biê´n. V´ı du. P(x, y) = x + x y + . l` , ,, ´ 2 4 2 ˘p xê´p theo bâc x y + 2xy d¯uo. c sa . cua biê´n x. , , , , , ´,c hai biê´n tông qu´ ´i Ta s˜e không x´et tâ´t ca nh˜ung d at m` a chı gio ¯a thu , , , , ´ ´ ´ c c´ ´ ng theo bâc ´p d¯a thu han o t´ınh d ac biên. ¯ôi xu . cho môt . lo . cua c´ , ´,c hai biê´n P(x, y) goi Ðinh ngh˜ıa 2.2. Môt a d ¯ô´i xu´ng, nê´u d¯a . d¯a thu . l` . , , , `˘ng y v` `˘ng x, ngh˜ıa l` ´,c n` a thu ay không thay d a y ba ¯ôi khi chuyên d¯ôi x ba P(x, y) = P(y, x). , , ´,c d¯ô´i xu ´,ng hai biê´n nhu, : Ta c´ o thê chı ra h` ang loat ac v´ı du. vê` d ¯a thu . c´ `˘ng y v` ´,c d¯ô´i xu ´,ng v`ı khi ta thay x ba 1) P(x, y) = x2 y + xy2 l` a d¯a thu ay , , ˜ ` ´ c vân nhu c˜ ˘ng x d¯a thu ba u.

66

,, ´,c d ´,ng hai biê´n Chuong 2. Ða thu ¯ô´i xu

´,c d ´,ng. 2) P(x, y) = (x + y)3 + x5 + y5 l` a d¯a thu ¯ô´i xu , `˘ng y ´,c d¯ô´i xu ´,ng, v`ı thay x ba 3) P(x, y) = x3 − 5y2 không phai l` a d¯a thu `˘ng x, P(y, x) = y3 − 5x2 6= P(x, y) = x3 − 5y2 . v` a y ba

, ´,c d ´,ng hai biê´n ta suy ra ph´ep công, T`u d ngh˜ıa d ph´ep ¯inh ¯a thu ¯ô´i xu . . , , , , , ´ c d¯ô´i xu ´ ng c˜ ´ c d¯ô´i xu ´ ng. Ph´ep lâ´y tr`u, t´ıch hai d¯a thu ung l` a môt . d¯a thu , , , , , ´c d ´ ng c˜ ´ c d¯ô´i xu ´,ng. Ngh˜ıa l˜ uy th`ua cua môt ung l` a môt ¯a thu ¯ô´i xu . d . d¯a thu ´,c d¯ô´i xu ´,ng, th`ı P(x, y) ± Q(x, y), l` a nê´u P(x, y) v` a Q(x, y) l` a hai d¯a thu , ´,c d ´,ng, o, d¯ây n l` P(x, y).Q(x, y) v` a (P(x, y))n c˜ ung l` ad a môt ¯a thu ¯ô´i xu . sô´ , ,, tu. nhiên duong. , , , ´ ´, , ˜,ng d Mênh d¯ê` 2.1. Môt a y l` a tông cua nhu ¯a thu´c d ¯ôi xung hai biê´n x v` ¯on . d . , , ´ ´, , thu´c dang axk yk v` ad b(xn ym + xm yn ) v´ oi a, b l` a ¯a thu´c d ¯ôi xung dang . . , , , ´ ´ ˜ ng hê. sô v` ˜ ng sô tu. nhiên 0 ≤ m < n, 0 ≤ k. nhu a nhu ´,ng minh. Cho P(x, y) l` ´,c d¯ô´i xu ´,ng, theo d Chu a d¯a thu ¯inh . ngh˜ıa P(x, y) = , , , , ` ´ c. Nh˜u,ng d¯o,n thu ´,c n` P(y, x) v` a P(x, y) bao gôm h˜uu han ay . nh˜ung d¯on thu ,`, , c´ o bâc ac nhau. Ta x´et hai truong ho. p: . kh´ , , ´,c cua d ´,c c´ ´,i k 1) Ðon thu o c´ ac biê´n d¯ô`ng bâc a axk yk vo ¯a thu . ngh˜ıa l` , , ,, , ´,c n` ´ ta chuyên d¯ôi biê´n sô´ th`ı d¯on thu l` a môt ay . sô´ nguyên duong. Khi d¯o , không d¯ôi. ,, , n m ´, ´,c c´ 2) Ðon thu o dang i n, m l` a sô´ nguyên duong v` an>m≥ . bx y vo , , , ´,c trên 0, b l` a môt ¯ôi hai biê´n sô´ cho nhau th`ı d ¯on thu . hê. sô´. Khi ta chuyên d , , , ,, , ´,c n` tro th` anh bxm yn , hai d¯on thu ay ho` an to` an kh´ ac nhau. Ðê d ¯am bao , , , , ´,c n` ´,c, ˘. t trong tông cua d P(x, y) = P(y, x) th`ı hai d ay phai c´ o ma ¯on thu ¯a thu , ´,c (bxn ym + bxm yn ) l` ´,c. Nê´u tr´ ngh˜ıa l` ad a môt cua d¯a thu ai ¯a thu . sô´ hang . , , , ´c d ´ ng, vô l´ı. ˜ cho không phai l` lai a d¯ô´i xu ¯a . th`ı d¯a thu , , ´,c d ´,ng bao gô`m tông nh˜u,ng d ´,c vo ´,i c´ T´ om lai, ac ¯a thu ¯ô´i xu ¯on thu . môt . d , , , , , , , ´ ´ ´ c c` ´i c´ biên c` ung bâc a tông cua hai d¯on thu ung hê. sô vo ac bâc . v` . o biê´n , , chuyên d¯ôi cho nhau.

J

, , ´,c hai biê´n 2.2. Ðinh . l´ı co ban cho d¯a thu

67

, 5 3 2 3 3 2 3 5 ´,c Ta c´ o thê lâ´y v´ı du: a d¯a thu . P(x, y) = x + 3x y − x y + 3x y + y l` , , ´,ng hai biê´n x, y, o, d¯ây ta ch´ ´,c x5 = x5 y0 v` d¯ô´i xu u ´y hai d¯on thu a y5 = x0 y5 , v` a tông 3(x3 y2 + x2 y3 ). , , ´,c d ´,ng hai biê´n d Ta x´et hai d a ¯a thu ¯ô´i xu ¯on gian nhâ´t P(x, y) = x + y v` , , ,, , , ´ ´, , ,, ˜ ´ ˜ ´ Q(x, y) = xy. Nhung d¯a thuc n` ay d¯uo. c goi a nhung d ¯a thuc d ¯ôi xung co so . l` ,, ´ ˘. c biêt hai biên v` a d¯uo. c k´ı hiêu ¯a . d .

δ1 = x + y,

δ2 = xy.

, , , `˘ng d¯a thu ´,c d¯ô´i xu ´,ng hai biê´n d¯ê`u c´ Phâ`n sau d o thê biêu ¯ây ta s˜e chı ra ra , , , ´,c hai biê´n vo ´,c co, so, trên. ´,i c´ di˜ên nhu môt ac biê´n l` a nh˜ung d¯a thu . d¯a thu

, , , 2.2. Ð.inh l´ı co ban cho da uc hai biê´n ¯ th´ , , , , ´,c d¯ô´i xu ´,ng d ˜ biê´t tông, hiêu, Nhu ta d uy th`ua nh˜ung d¯a thu ¯a ¯ê`u . t´ıch, l˜ ,, , , , , , , , ´c d ´ ng. Nhu vây ´ c d¯ô´i xu ´ ng co so ta c´ cho ta môt o ¯a thu ¯ô´i xu . d . t`u hai d¯a thu , , , , ´ c d¯ô´i xu ´ ng t` thê tao uy ´y. V´ı du: . ra nh˜ung d¯a thu . 1) Q(δ1 , δ2 ) = δ31 −δ1 δ2 = (x+y)3 −(x+y)xy = x3 +2x2 y+2xy2 +y3 = ´,c d¯ô´i xu ´,ng hai biê´n theo x v` P(x, y) l` a môt a y. . d¯a thu ´,c d a môt 2) Q(δ1 , δ2 ) = 2δ22 − δ21 δ2 = −x3 y − xy3 = P(x, y) l` ¯a thu ¯ô´i . d , ´ ng hai biê´n theo x v` xu a y. , , `˘ng trong môt ´,c hai biê´n δ1 v` a δ2 , khi Nhu vây o thê n´ oi ra . d¯a thu . ta c´ , , , ´ c n` ´,c d thay δ1 = x + y v` a δ2 = xy v` ao d ay s˜e nhân ¯a thu ¯uo. c môt ¯a thu ¯ô´i . d . d , ´ ng hai biê´n x v` xu a y. , , ,, ´,c d¯ô´i xu ´,ng hai biê´n x v` Câu hoi nguo. c lai a: Môt a y c´ o biêu di˜ên . l` . d¯a thu , ,, , ,, ´,c cua hai biê´n δ1 = x + y v` d¯uo. c nhu môt a δ2 = xy d¯uo. c không ? V´ı ¯a thu . d

,, ´,c d ´,ng hai biê´n Chuong 2. Ða thu ¯ô´i xu

68

, ´,c d ´,ng bâ´t k`ı nhu, x4 y + xy4 , ta biê´n d du: ¯ô´i xu ¯ôi . X´et d¯a thu x4 y + xy4 = xy(x3 + y3 ) = xy(x + y)(x2 − xy + y2 ) = = xy(x + y)((x + y)2 − 3xy) = δ2 δ1 (δ21 − 3δ2 ). , , , , ´,c l` ´,i d¯a thu V´ı du. kh´ ac: Vo a tông l˜ uy th`ua cua t`ung biê´n Sn = xn + yn , n = 1, 2, ... D˜ê thâ´y S1 = x + y = δ1 ; S2 = x2 + y2 = (x + y)2 − 2xy = δ21 − 2δ2 ; S3 = x3 + y3 = (x + y)(x2 − xy + y2 ) = = (x + y)((x + y)2 − 3xy) = δ1 (δ21 − 3δ2 ); S4 = x4 + y4 = (x2 + y2 )2 − 2x2 y2 = (δ21 − 2δ2 )2 − 2δ22 . , ˜ , , , ´,c ta x´et mênh Ðê dân d¯ê´n d d¯ê`. ¯inh . l´ı co ban cua d¯a thu . , `,a hai biê´n Sn = xn + yn , n = 1, 2, 3, ... c´ Mênh d¯ê` 2.2. Moi uy thu o . tông l˜ . , công thu´c truy hô`i Sn = δ1 Sn−1 − δ2 Sn−2 , n = 3, 4, ... , , , , ˜ du,o´,i dang v` a Sn c´ o thê biêu diên a δ2 . ¯a thu´c cua hai biê´n δ1 v` . d

(2.1)

, ,, ´,ng minh. 1) Ta c´ ´,c (2.1) nhu, sau: Ta x´et hai Chu o thê t`ım d ¯uo. c công thu , , ´,c theo ´,c Sn−1 = xn−1 + yn−1 v` ˘ng thu ˘ng thu d a δ1 = x + y. Ta nhân hai d¯a ¯a ,, vê´ v` a nhân ¯uo. c . d δ1 Sn−1 = (x + y)(xn−1 + yn−1 ) = xn + xyn−1 + xn−1 y + yn = = xn + yn + xy(xn−2 + yn−2 ) = Sn + δ2 Sn−2 . , , , `˘ng quy nap ´,ng minh ba ´,c cua δ1 v` 2) Ta chu a δ2 . ¯a thu . Sn biêu di˜ên nhu d , ,, , ,´, 2 ` ` Thât an o phân truoc th`ı S1 = δ1 v` a S2 = δ1 − 2δ2 d¯êu biêu . vây, . nhu t´ınh to´ , ,, , , ´,c qua hai biê´n δ1 v` di˜ên nhu d¯a thu a δ2 . Gia su Sn−1 v` a Sn−2 c˜ ung biêu , , , `˘ng Sn c˜ ´,c cua hai biê´n δ1 v` ´,ng minh ra di˜ên nhu d¯a thu a δ2 . Ta phai chu ung

, , ´,c hai biê´n 2.2. Ðinh . l´ı co ban cho d¯a thu

69

, , , ´,c hai biê´n trên. Ðiê`u n` biêu di˜ên nhu d ay l` a hiên nhiên v`ı theo công ¯a thu , ´,c (2.1) v` ´,ng minh. Nhu, vây thu a gia thiê´t quy nap ¯iê`u câ`n chu ¯iê`u . suy ra d . d , , , , , ´i moi ˘ng d ´ ng vo kha ¯inh ¯u . d . sô´ tu. nhiên duong. , ´ c (2.1) không nh˜u,ng râ´t quan trong Công thu trong mênh d a ¯ê` trên m` . . , , , , ˜ ´ ` ´ c` on l` a môt a δ2 . Tu công thuc (2.1) . công thuc t´ınh Sn biêu diên theo δ1 v` , ,, ` ta c´ o thê t´ınh lân luo. t c´ ac Sn :

J

S3 = δ1 S2 − δ2 S1 = δ1 (δ21 − 2δ2 ) − δ2 δ1 = δ31 − 3δ1 δ2 ; S4 = δ1 S3 − δ2 S2 = δ1 (δ31 − 3δ1 δ2 ) − δ2 (δ21 − 2δ2 ) = δ41 − 4δ21 δ2 + 2δ22 ; S5 = δ1 S4 − δ2 S3 = δ1 (δ41 − 4δ21 δ2 + 2δ22 ) − δ2 (δ31 − 3δ1 δ2 ) = = δ51 − 5δ31 δ2 + 5δ1 δ22 . , , , , ,, uy th`ua tiê´p theo. Ban o v` a ta c´ o thê tiê´p tuc ¯oc . d . c´ . t`ım d¯uo. c nh˜ung tông l˜ , , , , , , ´ c tiê´p theo. V`ı t´ınh quan trong ´ ng dung thê tu. t´ınh lâ´y nh˜ung biêu thu . cho u . , , n n ` ` ´ ` ˘ sau n` ay, ch´ ung tôi liêt kê 10 tô ng l˜ u y th u a d â u tiên: Biê t r a ng S = x +y ¯ n . v` a δ1 = x + y, δ2 = xy. S1 = δ1 ; S2 = δ21 − 2δ2 ; S3 = δ31 − 3δ1 δ2 ; S4 = δ41 − 4δ21 δ2 + 2δ22 ; S5 = δ51 − 5δ31 δ2 + 5δ1 δ22 ; S6 = δ61 − 6δ41 δ2 + 9δ21 δ22 − 2δ32 ;

(2.2)

S7 = δ71 − 7δ51 δ2 + 14δ31 δ22 − 7δ1 δ32 ; S8 = δ81 − 8δ61 δ2 + 20δ41 δ22 − 16δ21 δ32 + 2δ42 ; S9 = δ91 − 9δ71 δ2 + 27δ51 δ22 − 30δ31 δ32 + 9δ1 δ42 ; 8 6 2 4 3 2 4 5 S10 = δ10 1 − 10δ1 δ2 + 35δ1 δ2 − 50δ1 δ2 + 25δ1 δ2 − 2δ2 ;

, , , , ´ ´, ˜ Ðinh ac biê´n x v` a y c´ o thê biêu diên ¯a thu´c d ¯ôi xung bâ´t k`ı cua c´ . d . l´ı 2.1. Môt

70

,, ´,c d ´,ng hai biê´n Chuong 2. Ða thu ¯ô´i xu

,, , , a δ2 = xy. duo´i dang ¯a thu´c cua δ1 = x + y v` . môt . d , , ´,ng minh. Môt ´,c d¯ô´i xu ´,ng hai biê´n bâ´t k`ı d Chu a tông cua hai ¯a thu ¯ê`u l` . d , ´,c nhu, mênh dang a d¯a thu d¯ê` 2.1. . d¯on v` . , ,, , , , ´,c l` ´,c d¯ô`ng bâc ´,i c´ 1) Tru`ong ho. p nh˜ung sô´ hang ad ac ¯a thu ¯on thu . cua d . vo , , k k ´ c c´ biê´n: Ðon thu o dang . ax y . D˜ê thâ´y axk yk = a(xy)k = aδk2 . , , Nhu vây, ay biêu di˜ên theo δ1 v` a δ2 . . sô´ hang . n` , ,, , , ´,c c´ 2) Tru`ong ho. p nh˜ung sô´ hang cua d¯a thu o dang b(xn ym + xm yn ). . . , `˘ng n > m, khi d¯o ´ Ta gia thiê´t ra b(xn ym + xm yn ) = bxm ym (yn−m + xn−m ) = bδm 2 Sn−m . , , , , ´,c cua δ1 v` theo mênh d o thê biêu di˜ên nhu môt a δ2 . ¯ê` 2.2, Sn−m c´ ¯a thu . . d , , , , ˜ ´ c cua δ1 v` Suy ra sô´ hang ung biêu diên nhu môt a δ2 . . trên c˜ . d¯a thu , , , , , ´c d ´,c cua δ1 v` T´ om lai, a ¯a thu ¯ê`u biêu di˜ên nhu d ¯a thu . moi . sô´ hang . cua d , , , , , , ˜ ´ ´c d ´ ng biêu diên nhu d¯a thu ´ c cua δ1 v` δ2 , suy ra d a δ2 . ¯a thu ¯ôi xu , , , , ´ ng minh trên ta c´ ´,c d Thông qua c´ ach chu o thê chuyên d¯ôi moi ¯a thu ¯ô´i . d , , ´,ng th` ´,c cua δ1 v` ´,c xu anh d a δ2 nhu v´ı du. sau: X´et d¯a thu ¯a thu

J

P(x, y) = x6 + 3x3 y2 − x5 y5 − 2xy4 + y6 + 3x2 y3 − 2x4 y. , , , , ´˘p xê´p lai ´,c v` ´,c nhu, trong a hai d¯on thu Ta sa . theo tông nh˜ung môt . d¯on thu ´,ng minh: chu P(x, y) = −x5 y5 + (x6 + y6 ) + 3(x3 y2 + x2 y3 ) − 2(xy4 + x4 y) hay P(x, y) = −x5 y5 + (x6 + y6 ) + 3x2 y2 (x + y) − 2xy(y3 + x3 ) = −δ52 + S6 + 3δ22 δ1 − 2δ2 S3 ,

, ,, ´,ng 2.3. Giai hê. phuong tr`ınh d ¯ô´i xu

71

´,c (2.2) ta c´ v` a theo công thu o P(x, y) = −δ52 + S6 + 3δ22 δ1 − 2δ2 S3 = −δ52 + (δ61 − 6δ41 δ2 + 9δ21 δ22 − 2δ32 ) + 3δ22 δ1 − 2δ2 (δ31 − 3δ1 δ2 ) = δ61 − 6δ41 δ2 − 2δ31 δ2 + 9δ21 δ22 + 9δ1 δ22 − 2δ32 − δ52 .

, ,, , 2.3. Giai hê. phuong tr`ınh dô ung ¯ ´i x´ , , ,, ,, , ,, ˘. p hê. phuong tr`ınh hai ân m` Ta thu`ong ga a nh˜ung vê´ tr´ ai cua phuong , , ,, , ´,c d¯ô´i xu ´,ng hai ân x v` tr`ınh l` a nh˜ung d a y. Trong tru`ong ho. p n` ay ta ¯a thu , ,, , , , chuyên hê. phuong tr`ınh th` anh hê. nh˜ung phuong tr`ınh phu. thuôc ao δ1 . v` , ,, , , , , , , ´i n` v` a δ2 v` a giai hê. phuong tr`ınh mo ay, thu`ong l` a nh˜ung hê. phuong tr`ınh , , , , , , ´ , ´ nh`o nh˜ung gi´ d¯on gian hon rât nhiê`u. Sau d¯o a tri. cua δ1 v` a δ2 ta d¯i t`ım ân sô´ x v` a y. , ,´, , Truo c khi x´et nh˜ung v´ı du. cu. thê, ta x´et d ¯inh . l´ı: ,, Ðinh l´ı 2.2. Cho δ1 v` a δ2 l` a hai sô´ bâ´t k`ı. Phuong tr`ınh bâc . hai . z2 − δ1 z + δ2 = 0 ,, v` a hê. phuong tr`ınh



(2.3)

x + y = δ1 , (2.4) xy = δ2 ,, ,, c´ o mô´i liên hê. tuong d ¯uong theo ngh˜ıa sau: , ,, ,, Nê´u z1 , z2 l` a nghiêm . cua phuong tr`ınh bâc . hai (2.3), th`ı hê. phuong tr`ınh (2.4) c´ o hai nghiêm .   x1 = z1 , x2 = z2 , y1 = z2 , y2 = z1 , ,, v` a không c´ o nghiêm ac; nguo. c lai, a nghiêm . kh´ . nê´u x = a, y = b l` . cua hê. , ,, ,, ˜,ng sô´ a v` phuong tr`ınh (2.4), th`ı nhu a b c˜ ung l` a nghiêm . cua phuong tr`ınh bâc . hai (2.3).

72

,, ´,c d ´,ng hai biê´n Chuong 2. Ða thu ¯ô´i xu

, ,, ´,ng minh. Nê´u z1 v` Chu a z2 l` a nghiêm . hai (2.3), th`ı . cua phuong tr`ınh bâc , ´ c Vi´et theo công thu    z1 + z2 = δ1 , x1 = z1 , x2 = z2 , ngh˜ıa l` a z1 z2 = δ2 , y1 = z2 , y2 = z1 , ,, , , l` a nghiêm on không c´ o nghiêm ac n˜ua cua . cua hê. phuong tr`ınh (2.4). C` . kh´ ,, ,, , , ˘ng d hê. d¯uo. c suy ra t`u kha ¯inh . nguo. c lai. . , ,, Nê´u x = a, y = b l` a nghiêm a . cua hê. phuong tr`ınh (2.4), ngh˜ıa l`  a + b = δ1 , ab = δ2 . ´ ta c´ Khi d¯o o z2 − δ1 z + δ2 = z2 − (a + b)z + ab = (z − a)(z − b). , , ,, Ðiê`u n` ay c´ o ngh˜ıa l` a nh˜ung sô´ a, b l` a nghiêm . cua phuong tr`ınh (2.3). , ,, V´ı du. 2.1. Giai hê. phuong tr`ınh  x3 + y3 = 2, xy(x + y) = 2.

J

, , , ,, , `,i giai. Vê´ tr´ ´,c d¯ô´i xu ´,ng ˜ cho l` Lo ai cua nh˜ung phuong tr`ınh d¯a a nh˜ung d ¯a thu , ,, , ´,c ´,i x v` ˘ng thu ˘. t δ1 = x + y v` d¯ô´i vo a y. Ta d a δ2 = xy. Ta su dung nh˜ung d¯a ¯a . ,, ,, ,, trong (2.2), t`ım d a hê. phuong tr`ınh tro th` anh ¯uo. c x3 + y3 = δ31 − 3δ1 δ2 v`     δ31 − 3δ1 δ2 = 2, δ31 − 6 = 2, δ31 = 8, δ1 = 2, ⇔ ⇔ ⇔ δ1 δ2 = 2. δ1 δ2 = 2. δ1 δ2 = 2. δ2 = 1. , ,, , , , , , ´i Nhu vây . hê. phuong tr`ınh ban d¯â`u tuong d¯uong vo  x + y = 2, xy = 1. , , ,, ,, Hê. phuong tr`ınh d˜ê d` ang giai d¯uo. c, v`ı theo d ay ¯inh . l´ı 2.2, viêc . giai hê. n` , ` , ,, ,, 2 d¯ua vê giai phuong tr`ınh bâc o . hai z − 2z + 1 = 0. Phuong tr`ınh bâc . hai c´ , nghiêm o nghiêm a y = 1. . bôi . z = 1 nên hê. chı c´ . x = 1 v`

J

, ,, ´,ng 2.3. Giai hê. phuong tr`ınh d ¯ô´i xu

73

, ,, V´ı du. 2.2. Giai hê. phuong tr`ınh  x5 + y5 x+y

= 33, = 3.

, , , , ,, `,i giai. Ta su, dung ´,c (2.2) d Lo ¯ê chuyên hê. phuong tr`ınh trên vê` hê. . công thu theo c´ ac biê´n δ1 v` a δ2 :  δ51 − 5δ31 δ2 + 5δ1 δ22 = 33, δ1 = 3. , , , ´ t`ım δ2 trong phuong tr`ınh bâc T`u d¯o . hai 15δ22 − 135δ2 + 210 = 0, hay l` a δ22 − 9δ2 + 14 = 0. , , , ,, ,, T`u phuong tr`ınh n` ay ta t`ım d a tri. cua δ2 : δ2 = 2 v` a δ2 = 7. Ðê ¯uo. c hai gi´ ,, ,, t`ım nghiêm . cho hê. phuong tr`ınh ban d¯â`u ta x´et hai hê. phuong tr`ınh   x + y = 3, x + y = 3, v` a xy = 2. xy = 7. , , , , ´,c) a sô´ phu Giai nh˜ung hê. n` ay ta nhân . l` . d¯uo. c (không t´ınh nghiêm   x1 = 2, x2 = 1, y1 = 1; y2 = 2. , ,, V´ı du. 2.3. Giai hê. phuong tr`ınh  x3 − y3 = 19(x − y), x3 + y3 = 7(x + y).

J

, , ,, `,i giai. Ta thâ´y d¯ây l` ´,ng v` a hê. phuong tr`ınh hai biê´n d¯ô´i xu a ta c˜ ung c´ o thê Lo , ,, , , ,, ´c d¯u,o.,c nh˜u,ng sô´ hang gian uo chung, nhung d ac nghiêm ¯ê t`ım d ¯uo. c hê´t c´ . . ta x´et: , ,, ,, 1) Nê´u x = y, th`ı hê. phuong tr`ınh chı c` on 2x3 = 14x (v`ı phuong tr`ınh , ,, ,, ´, nhâ´t luôn l` thu a d¯ô`ng nhâ´t), t`u phuong tr`ınh n` ay ta t`ım d¯uo. c ba nghiêm . sau

 x1 y1

= 0, = 0;

 x2 y2

√ = 7, √ = 7;



x3 y3

√ = − 7, √ = − 7.

,, ´,c d ´,ng hai biê´n Chuong 2. Ða thu ¯ô´i xu

74

,, ´, hai trong hê. d¯ô`ng nhâ´t l` 2) Nê´u x = −y, th`ı phuong tr`ınh thu a 0, c` on ,, , , , , 3 ´ nhâ´t l` ´ ta t`ım thêm d¯uo. c hai nghiêm phuong tr`ınh thu a 2x = 38x. T`u d ¯o . , n˜ua   √ √ x4 = 19, x5 = − 19, √ √ y4 = − 19; y5 = 19. , , ,, , ´, ´c d¯u,o.,c x − y o, phu,o,ng tr`ınh thu 3) Nê´u x 6= ±y, th`ı ta c´ o thê gian uo ,, ,, ,, ´, hai v` nhâ´t, x + y o phuong tr`ınh thu a nhân . d¯uo. c hê.  x2 + xy + y2 = 19, x2 − xy + y2 = 7. , ´,c (2.2) Ta biê´n d ao biê´n δ1 v` a δ2 theo công thu ¯ôi vê` hê. phu. thuôc . v`  δ21 − δ2 = 19, 2 δ1 − 3δ2 = 7. , ,, , T`u d¯ây ta t`ım d o hai nghiêm ¯uo. c δ21 = 25, δ2 = 6, nhu vây . ta c´ .   δ1 = 5, δ1 = −5, δ2 = 6; δ2 = 6. , , ,, 2 T`u hai hê. trên d ac phuong tr`ınh bâc a ¯ua vê` c´ . hai z + 5z + 6 = 0 v` , ,, ,, 2 z − 5z + 6 = 0. Giai c´ ac phuong tr`ınh bâc ay ta nhân ac . hai n` . d¯uo. c c´ nghiêm .

 x6 y6

= 2, = 3;

 x7 y7

= 3, = 2;

 x8 y8

= −2, = −3;



x9 y9

= −3, = −2.

J

, ,, , V´ı du. 2.4. Giai hê. phuong tr`ınh v´ oi x 6= 0, y 6= 0:  2 2  x + y = 12,  y x  1 1 1    + = . x y 3 , ˜ sô´ `,i giai. Hê. phu,o,ng tr`ınh d¯a ´,ng vo ´,i x, y. Ta quy d ˜ cho l` Lo ad ¯ô´i xu ¯ô`ng mâu

, ,, ´,ng 2.3. Giai hê. phuong tr`ınh d ¯ô´i xu

75

, ,, hai phuong tr`ınh v` a chuyên sang hê. c´ o hai biê´n δ1 v` a δ2 :   x3 + y3 = 12xy, δ31 − 3δ1 δ2 = 12δ2 , ⇔ 3(x + y) = xy. 3δ1 = δ2 . , ,, ,, ´, hai thay v` ´, nhâ´t, ta Lâ´y δ2 cua phuong tr`ınh thu ao phuong tr`ınh thu , ,, ,, nhân a δ1 (δ21 − 9δ1 − 36) = 0. Giai phuong ¯uo. c δ31 − 9δ21 − 36δ1 = 0, ngh˜ıa l` . d , ,, ´,i δ1 cho kê´t qua δ1 = 0, δ1 = −3, δ1 = 12. Ta nhân tr`ınh n` ay d¯ô´i vo ¯uo. c . d ba hê.



  = 0, δ1 = 12, δ1 = −3, = 0; δ2 = 36; δ2 = −9. , , , , , ´ ´ nhât suy ra x = y = 0 v` T`u hê. phuong tr`ınh thu a d¯ây không phai l` a , , , , , , , , ` ´ hai tuong u ´ ng vo ´i phuong tr`ınh bâc nghiêm ¯âu. Hê. thu . cua hê. ban d . hai ,, 2 ˜ cho c´ z − 12z + 36 = 0, phuong tr`ınh n` ay c´ o nghiêm o ¯a . k´ep nên hê. d δ1 δ2

nghiêm .

 x1 = 6, y1 = 6. ´, ba tu,o,ng tu ´,ng vo ´,i z2 + 3z − 9 = 0 c´ ´ C` on hê. thu o hai nghiêm a khi d ¯o . v`

˜ cho c´ hê. d¯a o thêm hai nghiêm . √ √  −3 − 3 5 −3 + 3 5  , x3 = , 2 √ 2 √  y = −3 + 3 5 . y = −3 − 3 5 ;  2 3 2 2 , ,, V´ı du. 2.5. Giai hê. phuong tr`ınh   = 7, x + y − z x2 + y2 − z2 = 37,   3 x + y3 − z3 = 1.   x2

=

J

, , ,, `,i giai. Ta coi biê´n thu ´, ba z nhu, l` Lo ad ¯ôc . lai), . lâp . (cô´ d¯inh . th`ı nh˜ung phuong , ´,c d ´,ng theo biê´n x, y. ˜ cho c´ tr`ınh trong hê. d¯a o vê´ tr´ ai l` a nh˜ung d ¯a thu ¯ô´i xu

,, ´,c d ´,ng hai biê´n Chuong 2. Ða thu ¯ô´i xu

76

, ,, , ´,i c´ Theo phuong ph´ ap giai c´ ac biê´n δ1 v` a δ2 : ac b` ai trên ta d¯ua vê` hê. vo   = 7, δ1 − z 2 2 δ1 − 2δ2 − z = 37,   3 δ1 − 3δ1 δ2 − z3 = 1. ,, ,, ,, ,, `˘ng c´ ´, nhâ´t v` ´, hai Ba ach lâ´y z o phuong tr`ınh thu a δ2 o phuong tr`ınh thu , ,, ,, ´, ba. Sau khi t´ınh to´ thay v` ao phuong tr`ınh thu an ta d¯uo. c 18δ1 = 342. T`u , ,, d ¯ây ta t`ım d¯uo. c nghiêm . cua hê. trên δ1 = 19, δ2 = 90, z = 12. , , , ,, ,, T`u d a t`ım d¯uo. c c´ ac nghiêm: ¯ây d ¯ua vê` giai phuong tr`ınh bâc . hai v` .     x2 = 10, x1 = 9, y2 = 9, y1 = 10,     z2 = 12. z1 = 12;

J

,

, , ` tâp Bai (Môt a tra l`oi c´ ac b` ai tâp . sô´ go. i ´y v` . sau tai . trang 110) . , , ,, H˜ ay giai nh˜ung hê. phuong tr`ınh sau  . 2.6.  . 2.7.  . 2.8. 

x + y = 5, x2 − xy + y2 = 7; x + y = 5, x3 + y3 = 65; 4(x + y) = 3xy, x + y + x2 + y2 = 26;

x2 + y2 + x + y = 32, 12(x + y) = 7xy;  xy = 15, . 2.10. x + y + x2 + y2 = 42;  2 y2 x + = 18, . 2.11. y x  x + y = 12; . 2.9.

 x2 y + y2 x = 30, . 2.12. 1 1 5  + = ; y x 6  x + y = a, . 2.13. x3 + y3 = b(x2 + y2 );  x2 + y2 + 2(x + y) = 23, . 2.14. x2 + y2 + xy = 19;  xy(x + y) = 20, . 2.15. 1 1 5  + = ; x y 4  x4 − x2 y2 + y4 = 1153, . 2.16. x2 − xy + y2 = 33.

, ,, ´,ng 2.4. Ðua vê` hê. phuong tr`ınh dang . d¯ô´i xu

77

, ,, , 2.4. Ðua vê` hê. phuong tr`ınh dang dô ung ¯ ´i x´ . ,, , ,´, ,, ´,ng râ´t thuân O phâ`n truo c, ta thâ´y viêc ¯ô´i xu . giai môt . hê. phuong tr`ınh d . , , , , ,, ´ ˘ ˜ tiên v` a c´ o quy t a c chung d ê gi ai. Nhu ng không ph ai h ê phu o ng tr` ınh d a cho ¯ ¯ . . ,, , ,, ´,ng. Nhu,ng c´ n` ao c˜ ung l` a hê. phuong tr`ınh d¯ô´i xu o môt . sô´ nh˜ung hê. phuong , ,, , ,, ´,ng hoa ˘. c phuong tr`ınh ta c´ tr`ınh không d¯ô´i xu o thê d¯ua vê` hê. phuong tr`ınh , , , , , ´,ng d¯ê giai. T` ˘. t c´ ˜ cho, ta s˜e d d¯ô´i xu uy v` ao nh˜ung b` ai to´ an cu. thê d¯a ac ân ¯a , , , ,, ´,ng d¯ô´i vo ´,i ân sô´ mo ´,i. Ta sô´ phu. d ai to´ an vê` hê. phuong tr`ınh d ¯ê d ¯ua b` ¯ô´i xu x´et môt . sô´ v´ı du. , ,, V´ı du. 2.17. Giai hê. phuong tr`ınh   2 2  x + y   x − y

=

5 xy, 2

=

1 xy. 4

, , `,i giai. Ta thâ´y phu,o,ng tr`ınh thu ´, hai vi pham ´,ng cua hê. khi Lo . t´ınh d¯ô´i xu , ,, ,, ˘. t z = −y, hê. phuong tr`ınh tro th` thay d anh ¯ôi x, y cho nhau. Ta d¯a  5  2 2  x + z = − 2 xz,  1  x + z = − xz. 4 ,, , ´ ´ ´ ´,i x v` ˘. t δ1 = x + z ´d Ðây l` a môt h ê phu o ng tr` ınh d ô i x u ng d ô i v o a z. Khi d¯o ¯ ¯ ¯a . . ,, ,, , , ´ c (2.2)): v` a δ2 = xz, ta nhân . d¯uo. c hê. phuong tr`ınh (t`u công thu  5  2    δ1 − 2δ2 = − 2 δ2 , δ1 = 0, δ1 = 2, Hê. n` ay c´ o hai nghiêm .  δ2 = 0; δ2 = −8. 1  δ1 = − δ2 . 4 ˜ hê. phu,o,ng tr`ınh trên cho ta nghiêm ´,i x v` Môi a z: ¯ô´i vo . d    x1 = x2 = 0, x3 = 4, x4 = −2, z1 = z2 = 0; z3 = −2; z4 = 4.

78

,, ´,c d ´,ng hai biê´n Chuong 2. Ða thu ¯ô´i xu

, ,, ,, Cuô´i c` ung ta nhân ¯uo. cnghiêm ¯â`u: . d . cua hê. phuong tr`ınh ban d    x1 = x2 = 0, x3 = 4, x4 = −2, y1 = y2 = 0; y3 = 2; y4 = −4.

J

, ,, V´ı du. 2.18. Giai hê. phuong tr`ınh p √ 4 y3 − 1 + x = 3, x2 + y3 = 82. , , ,, `,i giai. Hê. phu,o,ng tr`ınh d¯a ´,ng ˜ cho không phai l` Lo a hê. phuong tr`ınh d ¯ô´i xu p √ ,, , ´,i x v` ˘. t u = x v` d¯ô´i vo a y. Ta d¯a a v = 4 y3 − 1. Hê. phuong tr`ınh d¯ua vê`   u + v = 3, u + v = 3, ⇔ u4 + (v4 + 1) = 82; u4 + v4 = 81. ,, ´,ng d¯ô´i vo ´,i hai biê´n u, v. Vo ´,i δ1 = u + v Hê. phuong tr`ınh trên l` a hê. d¯ô´i xu ,, ,, ´,c (2.2) ta nhân ´,i công thu v` a δ2 = uv, c` ung vo ¯uo. c hê. phuong tr`ınh . d  δ1 = 3, δ41 − 4δ21 δ2 + 2δ22 = 81. ,, ,, 2 ,, Thay δ1 = 3 v` ao phuong tr`ınh hai, ta nhân . d¯uo. c δ2 − 18δ2 = 0. Phuong ,, tr`ınh n` ay c´ o hai nghiêm a δ2 = 18. Ta nhân . δ2 = 0 v` . d¯uo. c hai nghiêm .   δ1 = 3, δ1 = 3, δ2 = 0; δ2 = 18. , ,, ,, 2 ´, nhâ´t d¯u,a vê` viêc Hê. phuong tr`ınh thu . giai phuong tr`ınh bâc . hai z −3z = 0 , ´i u v` v` a theo d¯inh o nghiêm a v: . l´ı 2.2 ta c´ . d¯ô´i vo     u1 = 3, u2 = 0, x1 = 9, x2 = 0, √ ´ Khi d ¯o v1 = 0; v2 = 3. y1 = 1; y2 = 3 82. , ,, , ,, 2 ´, hai d Hê. phuong tr`ınh thu ¯ua vê` giai phuong tr`ınh bâc . hai z − 3z + 18 = 0. ,, , , , ´ nghiêm Phuong tr`ınh n` ay không c´ o nghiêm . thu. c. Do d¯o . thu. c cua hê. ban , ,, d¯â`u chı c´ o hai bô. sô´ o trên.

J

, ,, ´,ng 2.4. Ðua vê` hê. phuong tr`ınh dang . d¯ô´i xu

79

, ,, V´ı du. 2.19. Giai hê. phuong tr`ınh r r  x + y = √7 + 1, y x xy p p 3 x y + y3 x = 78. , , `˘ng x, y phai kh´ `,i giai. T`u, hê. phu,o,ng tr`ınh ta thâ´y ra Lo ac không v` a c´ o c` ung √ √ , , ˘. t u = x, v = y. Nê´u x, y l` dâ´u. Nê´u x, y duong th`ı ta d¯a a sô´ âm th`ı ta √ √ ,`, , , ˘. t u = −x, v = −y. Trong moi ˜ cho d¯ua vê`: d¯a ¯a . truong ho. p hê. d  v 7 u + = + 1, v u uv u3 v + v3 u = 78. `˘ng c´ ´,c (2.2) ta nhân ˘. t δ1 = u + v v` Ba ach d a δ2 = uv v` a theo công thu ¯a . ,, d¯uo. c  δ21 − 3δ2 = 7 δ2 (δ21 − 2δ2 ) = 78. ,, ,, 2 `, ´, nhâ´t v` ´, hai, nhân Ta thay δ1 tu phuong tr`ınh thu ao phuong tr`ınh thu . ,, ,, d¯uo. c δ22 + 7δ2 − 78 = 0. Ta nhân d u o c hai nghi êm δ = 6 v` a δ = −13. Do ¯ 2 2 . . . ,, ´ hê. phuong tr`ınh hai biê´n δ1 v` d¯o a δ2 c´ o nghiêm: .   δ1 = 5 δ1 = −5 δ2 = 6; δ2 = 6. ,, , ,, , Boi v`ı u, v l` a nh˜ung sô´ duong, nên δ1 > 0 v` a δ2 > 0, ngh˜ıa l` a trong nh˜ung , , , ,, ´, nhâ´t l` ´ ta hê. nghiêm a δ2 o trên chı c´ o thê. thu a thoa m˜ an. Do d ¯o . cua δ1 v` ,, nhân . d¯uo. c   u1 = 2, u2 = 3, v1 = 3; v2 = 2. √ ,, , , , ´,i c´ ˘. t u = x, ˜ cho vo T`u d ach d¯a ¯ây ta t`ım x, y o hê. phuong tr`ınh d¯a √ √ √ ,, ˘. c u = −x, v = −y, ta nhân v = y hoa ¯uo. c bô´n nghiêm . d .     x1 = 4, x2 = 9, x3 = −4, x4 = −9, y1 = 9; y2 = 4; y3 = −9; y4 = −4.

J

,, ´,c d ´,ng hai biê´n Chuong 2. Ða thu ¯ô´i xu

80

, ,, V´ı du. 2.20. Giai phuong tr`ınh √ √ 4 97 − x + 4 x = 5. √ , √ ,, , `,i giai. Ta d ˘. t u = 4 x v` ´ phuong tr`ınh d¯ua vê` dang Lo a v = 4 97 − x. Khi d ¯a ¯o . ˘. t kh´ u + v = 5. Ma ac u4 + v4 = x + (97 − x) = 97. ,, , ,, Nhu vây ¯uo. c hê. phuong tr`ınh . ta nhân . d  u + v = 5, u4 + v4 = 97. , ,, `˘ng c´ ˘. t δ1 = u + v v` Ba ach d a δ2 = uv, ta d ¯a ¯ua hê. phuong tr`ınh trên vê` dang .  δ1 = 5, δ41 − 4δ21 δ2 + 2δ22 = 97. , ,, T`u d¯ây ta t`ım δ2 trong phuong tr`ınh bâc . hai δ22 − 50δ2 + 264 = 0. , ,, Giai phuong tr`ınh trên c´ o nghiêm a δ2 = 44. Ta c´ o hai hê. . δ2 = 6 v`   δ1 = 5 δ1 = 5 δ2 = 6; δ2 = 44. , ,, ´, nhâ´t ta nhân T`u hê. thu . d¯uo. c   u1 = 2 u2 = 3 v1 = 3; v2 = 2. , √ ,, V`ı u = 4 x nên nghiêm a x1 = 16 v` a x2 = 81. . cua phuong tr`ınh ban d¯â`u l` ,, , ´, hai không c´ ´,i u v` Hê. phuong tr`ınh thu o nghiêm a v. . thu. c d¯ô´i vo

J

, ,, V´ı du. 2.21. Giai phuong tr`ınh x+ √

x x2

−1

=

35 . 12

, ,, ´,ng 2.4. Ðua vê` hê. phuong tr`ınh dang . d¯ô´i xu

81

, , ,, `˘ng x = 0 không l` `,i giai. Ta thâ´y ra ˜ cho. Ta a nghiêm Lo . cua phuong tr`ınh d¯a √ x2 − 1 1 ,, ´,ng ˘. t u = , v = ´ ta c´ d¯a . Khi d¯o o hê. phuong tr`ınh d¯ô´i xu x x  u2 + v2 = 1 1 1 35  + = . u v 12 , , ,´, ,, `˘ng c´ Ba u v` a v : ach giai hê. nhu muc c ta t`ım d¯uo. c nghiêm . . truo , 4 3 , , , a u1 = , u2 = . T`u d ¯ây suy ra nghiêm . cua phuong tr`ınh ban d¯â`u l` 5 5 5 5 x1 = v` a x2 = . 4 3 , , , ` tâp Bai (Môt a tra l`oi c´ ac b` ai tâp . sô´ go. i ´y v` . sau tai . trang 114) . , , ,, H˜ ay giai nh˜ung hê. phuong tr`ınh sau

J

 . 2.22.  . 2.23.

. 2.24.

x − y = 2, x3 − y3 = 8;

 a b   + = 4; x y

, , ,, Giai nh˜ung phuong tr`ınh sau: r r 1 5 1 + x + 5 − x = 1; . 2.27. 2 2 . 2.28. sin3 x + cos3 x = 1; . 2.29.

√ 4

629 − x +

. 2.30.

√ 3

8+x+

√ 3

√ 4

√ √ x − y = 2 xy, x + y = 20;  x 43 + y 35 = 35, . 2.26. x 14 + y 15 = 5.

. 2.25.

x2 + y = 5, x6 + y3 = 65; x y    a + b = 1,

77 + x = 8;

8 − x = 1;

√

,, ´,c d ´,ng hai biê´n Chuong 2. Ða thu ¯ô´i xu

82

1 35 1 +√ = ; 2 x 12 1−x √ √ . 2.32. 3 10 − x − 3 3 − x = 1;   19 − x 19 − x . 2.33. x x+ = 84; x+1 x+1 √ √ . 2.34. x + 17 − x2 + x 17 − x2 = 9; . 2.31.

√ √ . 2.35. x 3 35 − x3 (x + 3 35 − x3 ) = 30.

, , , , ˘ ng th´ 2.5. Ch´ ung minh bâ´t d¯a uc dô ung ¯ ´i x´ , , , ,, ´,c d¯ô´i xu ´,ng râ´t hiêu ´ ng minh nh˜u,ng Phuong ph´ ap d` ung d¯a thu . qua d¯ê chu , , ´,c. Nh˜u,ng bâ´t d ´,c câ`n chu ´,ng minh thu,`o,ng c´ ˘ng thu ˘ng thu bâ´t d o dang ¯a ¯a . , , , , , ´ ´ ´ ´ ´ ´i P(x, y) l` ˘ P(x, y) ≥ 0, vo a môt d a th u c d ô i x u ng. M ôt bâ t d a ng th u c không ¯ ¯ . ¯ . , , ´,c th`ı n´ ˘ng thu thay d¯ôi khi ta thay vai tr` o c´ ac biê´n cho nhau trong bâ´t d¯a o , , , , , ,, , , ´ ng minh bâ´t d¯a ´c ˘ng thu d¯uo. c goi a bâ´t d¯a ˘ng thu´c d ¯ô´i xu´ng. Công cu. d¯ê chu . l` , , ´ ng l` ´ c (2.2) v` d¯ô´i xu a công thu ad ¯inh ¯ây: . l´ı sau d , ˜,ng sô´ x, y x´ ´ nhu Ðinh l´ı 2.3. Cho hai sô´ thu. c δ1 v` a δ2 . Khi d ac d ˘`ng ¯o ¯inh . ba . ,, hê. phuong tr`ınh  x + y = δ1 , xy = δ2 , , , , , , ´ ˜ ng sô thu. c khi v` l` a nhu a chı khi δ1 v` a δ2 thoa m˜ an bâ´t d¯a ˘ng thu´c , , , ,, δ21 − 4δ2 ≥ 0. Ða ˘ng thu´c δ21 = 4δ2 chı d¯at . d¯uo. c khi x = y. , ,, ,, , ˜,ng sô´ x, y x´ `, hê. Tru`ong ho. p riêng hay su dung: Ðê cho nhu ac d tu ¯inh . . , ,, ˜,ng sô´ thu.,c, không âm, d ˜,ng phuong tr`ınh trên l` a nhu ad ¯iê`u kiên ¯u nhu . câ`n v` , , , ˜,ng bâ´t d¯a sô´ δ1 v` a δ2 thoa m˜ an nhu ˘ng thu´c sau δ21 − 4δ2 ≥ 0,

δ1 ≥ 0,

δ2 ≥ 0.

, ´,ng minh bâ´t d¯a ´,c d¯ô´i xu ´,ng ˘ng thu 2.5. Chu

83

, , ,, ´,ng minh. Nh˜u,ng sô´ x, y thoa m˜ Chu an d a l` a nghiêm ¯inh . l´ı 2.2 v` . cua phuong tr`ınh bâc . hai ´,i c´ ngh˜ıa l` a tr` ung vo ac sô´

z2 − δ1 z + δ2 = 0, δ1 ±

q δ21 − 4δ2

. z1,2 = 2 , , , , , ´,c du,o ´,i dâ´u V`ı thê´ d¯ê x, y l` a nh˜ung sô´ thu. c, d a d¯u l` a biêu thu ¯iê`u kiên . câ`n v` , , , , ´,c δ21 − 4δ2 ≥ 0. Ða ˘ng thu ˘ng c˘ an không âm, ngh˜ıa l` a phai thoa m˜ an bâ´t d¯a , ,, ´,c δ21 − 4δ2 = 0 ngh˜ıa l` thu a nghiêm ung nhau . cua phuong tr`ınh bâc . hai tr` x = y. , Nê´u nh˜ung sô´ x, y không âm, th`ı d˜ê thâ´y δ21 − 4δ2 ≥ 0, ngo` ai ra c` on c´ o , ,, , , 2 ´ ˜ ´ ˘ng thuc δ1 − 4δ2 ≥ 0, δ1 ≥ 0 δ1 ≥ 0 v` a δ2 ≥ 0. Nguo. c lai, . cho nhung bât d¯a , ,, , , ´, ` ` ´,c thu ´, nhâ´t suy ˘ng thu v` a δ2 ≥ 0. Nhu chung minh o phân trên, tu bâ´t d¯a , , , ra x, y l` a nh˜ung sô´ thu. c. T`u δ2 ≥ 0, suy ra x, y c´ o c` ung dâ´u; v` a cuô´i c` ung

J

δ1 ≥ 0 suy ra x, y không âm. , , , ´ ng dung ´,ng minh bâ´t d¯a ´,c d ´,ng ta ˘ng thu Ðê u d¯inh ¯ô´i xu . l´ı trên trong chu . , tiê´n h` anh nhu sau: , , `˘ng cho môt ´,c d¯ô´i xu ´,ng P(x, y) v` ´,ng minh Gia thiê´t ra a phai chu ¯a thu . d , , `˘ng vo ´,i nh˜u,ng gi´ ´,i gi´ ˘. c l` ra a tri. thu. c bâ´t k`ı cua x, y (hoa a vo a tri. không âm bâ´t , , ´ c n` ´i x + y ≥ a, ...) d ˘. c vo k`ı hoa ay nhân a tri. không âm: P(x, y) ≥ 0. ¯a thu . gi´ , , , , , ´ ng minh, ta chuyên d¯a thu ´ c P(x, y) th` ´,c phu. thuôc Ðê chu anh biêu thu ao . v` , , , , , ´ c n` δ1 v` a δ2 . Trong d¯a thu ay ta thay δ2 boi δ1 v` a d ¯ai . luo. ng không âm , 1 ,, z = δ21 − 4δ2 , ngh˜ıa l` a thay δ2 = (δ21 − z). Trong kê´t qua, ta nhân ¯uo. c . d 4 `˘ng vo ´,c hai biê´n δ1 v` ´,ng minh ra ´,i nh˜u,ng gi´ d¯a thu a z v` a câ`n chu a tri. không , , , , ´ ´ ´ c câ`n chu ´,ng ´i han ´i δ1 d¯a ˘t buôc ˜ cho, th`ı d¯a thu âm z v` a nh˜ung gio ¯ ôi v o . ba . d , , ´,c ban d¯â`u d ˘ng thu ´ ng. minh nhân a tri. không âm. T`u d ¯ây suy ra bâ´t d¯a ¯u . gi´ ´ Ta x´et môt . sô v´ı du: . , , ˜,ng sô´ thu.,c, thoa m˜ an d V´ı du. 2.36. Chu´ng minh ra ˘`ng nê´u a v` a b l` a nhu ¯iê`u

,, ´,c d ´,ng hai biê´n Chuong 2. Ða thu ¯ô´i xu

84

, , ˜,ng bâ´t d ´ng ˘ng thu´c sau d¯ây d kiên ¯a ¯u . a + b ≥ c, th`ı nhu

c4 c8 c2 ; a4 + b4 ≥ ; a8 + b8 ≥ . 2 8 128 , `,i giai. Trong tru,`o,ng ho.,p b` Lo ai n` ay δ1 = a + b v` a δ2 = ab. Ta c´ o 1 1 1 S2 = a2 + b2 = δ21 − 2δ2 = δ21 − 2. (δ21 − z) = δ21 + z. 4 2 2 , 1 2 ´ S2 ≥ c . Theo d¯inh ¯o . l´ı 2.3 th`ı z ≥ 0, theo gia thiê´t δ1 ≥ c, do d 2 ,, , ´, `˘ng c´ ´,c (2.2) v` Ho` an to` an tuong tu. vo i S4 = a4 + b4 ba ach tra công thu a ,, t`ım d ¯uo. c  2 1 1 2 1 a4 + b4 ≥ c = c4 . 2 2 8 , ,, , ´,c c` ˘ng thu Bâ´t d on lai ung l` am tuong tu. . ¯a . c˜ , , ˜,ng sô´ thu.,c, th`ı bâ´t d V´ı du. 2.37. Chu´ng minh ra ˘`ng nê´u x v` a y l` a nhu ˘ng ¯a , ´ng thu´c sau d ¯ây d ¯u x6 + y6 ≥ x5 y + xy5 . a2 + b2 ≥

J

, , `,i giai. Ða thu ´,c d¯ô´i xu ´,ng co, so, l` ´,c Lo a δ1 = x + y v` a δ2 = xy v` a công thu 1 2 (2.2) cho S4 , S6 v` a δ2 = (δ1 − z). Ta c´ o: 4 x6 + y6 − x5 y − xy5 = S6 − δ2 S4 = = (δ61 − 6δ41 δ2 + 9δ21 δ22 − 2δ32 ) − δ2 (δ41 − 4δ21 δ2 + 2δ22 ) =

= δ61 − 7δ41 δ2 + 13δ21 δ22 − 4δ32 = 1 1 1 = δ61 − 7δ41 . (δ21 − z) + 13δ21 . (δ21 − z)2 − 4. (δ21 − z)3 = 4 16 64 5 4 5 1 = δ z + δ21 z2 + z3 ≥ 0. 16 1 8 16 , ˜,ng sô´ thu.,c du,o,ng, th`ı bâ´t V´ı du. 2.38. Chu´ng minh ra ˘`ng nê´u x v` a y l` a nhu , , ´ng d ˘ng thu´c sau d ¯a ¯ây d ¯u s r √ √ x2 y2 + ≥ x + y. y x

J

, ´,ng minh bâ´t d¯a ´,c d¯ô´i xu ´,ng ˘ng thu 2.5. Chu

85

, , √ √ ,, , `,i giai. Ta d ´,c d ˘ng thu ˘. t u = x, v = y. Khi d¯o ´ bâ´t d Lo ¯a ¯a ¯uo. c d¯ua vê` dang . u2 v 2 + ≥ u + v, hay u3 + v3 ≥ uv(u + v). v u , , ´,ng minh bâ´t d ´,c sau c` ´,i u > 0 v` ˘ng thu Ta phai chu ung vo a v > 0. D` ung ¯a S3 = δ31 − 3δ1 δ2 . Ta c´ o u3 + v3 − uv(u + v) = δ31 − 3δ1 δ2 − δ1 δ2 = δ31 − 4δ1 δ2 = δ1 (δ21 − 4δ2 ). , 2 ´,c sau c` Theo d ung không âm, ¯inh . l´ı 2.3 δ1 ≥ 0, δ1 − 4δ2 ≥ 0, nên biêu thu , , ´ ng minh. suy ra kê´t qua câ`n chu

J

, , , , , ` V´ı du. 2.39. T`ım gi´ a tri. nho nhâ´t cua biêu thu´c xy(x − y)2 v´ oi d ¯iêu kiên . x + y = a. , 1 `,i giai. Vo ´,i δ1 = x + y v` o Lo a δ2 = xy v` a δ2 = (δ21 − z) , ta c´ 4 1 xy(x − y)2 = δ2 (δ21 − 4δ2 ) = δ2 z = (δ21 − z)z = 4 1 1 2 = (a − z)z = (−z2 + a2 z) 4 4 !   2 2 1 a a4 = − z− + . 4 2 4 , , a4 a4 , ´,c trên không thê vu,o.,t qu´ ´ suy ra biêu thu T`u d¯o a v` a nhân l` a gi´ a . 16 16 2 , a ´,n nhâ´t khi z − ´,i δ1 = a (gia thiê´t d¯a ˜ cho) v` tri. lo = 0. Ngh˜ıa l` a vo a 2 2 , a , ,, ,, δ21 − 4δ2 = , t`u d¯ây d˜ê t`ım d¯uo. c x, y l` a nghiêm . cua phuong tr`ınh bâc . 2 2 a hai z2 − az + = 0. 8 , , ˜,ng sô´ du,o,ng thoa m˜ V´ı du. 2.40. Chu´ng minh ra ˘`ng nê´u x v` a y l` a nhu an

J

x + y = 1, th`ı  x+

1 x

2

 2 1 25 + y+ ≥ . y 2

,, ´,c d ´,ng hai biê´n Chuong 2. Ða thu ¯ô´i xu

86

, , , , , `,i giai. Ta biê´n d¯ôi biêu thu ´,c vê` dang ´,c cua δ1 v` Lo a δ2 (trong biê´n . biêu thu , , , ´ c ta ch´ d¯ôi biêu thu u ´y δ1 = 1):  2  2 1 1 25 x2 + y2 17 x+ + y+ − = x2 + y2 + 2 2 − = x y 2 x y 2 = (δ21 − 2δ2 ) +

δ21 − 2δ2 17 1 − 2δ2 17 − = 1 − 2δ2 + − = 2 2 δ22 δ22

1 (−4δ32 − 15δ22 − 4δ2 + 2). 2δ22 , ´,ng minh biêu thu ´,c trong dâ´u ngoa ˘. c không âm, ngh˜ıa l` Ta câ`n chu a =

4δ32 + 15δ22 + 4δ2 ≤ 2.

(2.5)

V`ı x, y > 0, nên δ2 > 0; ngo` ai ra z = δ21 − 4δ2 ≥ 0 (ngh˜ıa l` a 1 − 4δ2 ≥ 0), 1 1 , , , ´ c 4δ32 + 15δ22 + 4δ2 c´ ´ suy ra δ2 ≤ . Nhu vây o tâ´t t`u d¯o . 0 < δ2 ≤ 4 . Ða thu 4 , , 1 ,, ´,n nhâ´t trên khoang 0 < δ2 ≤ l` ca hê. sô´ d¯ê`u duong, n´ a2 o nhân a tri. lo . gi´ 4 1 , ´,ng minh d¯u,o.,c (2.5). ˜ chu tai ¯a . ta d . δ2 = 4 . Nhu vây

J

` tâp Bai . , , , Môt a tra l`oi c´ ac b` ai tâp . sô´ go. i ´y v` . sau tai . trang 119. , , , , `˘ng vo ´ ng minh ra ´i nh˜ung sô´ thu. c bâ´t k`ı x v` Chu a y: . 2.41. 5x2 − 6xy + 5y2 ≥ 0;

. 2.43. x4 + y4 ≥ x3 y + xy3 ;

. 2.42. 8(x4 + y4 ) ≥ (x + y)4 ;

. 2.44. x2 + y2 + 1 ≥ xy + x + y.

`˘ng vo ´,ng minh ra ´,i nh˜u,ng sô´ không âm bâ´t k`ı x v` Chu a y: √ √ . 2.45. ( x + y)8 ≥ 64xy(x + y)2 ; . 2.46.

x3 + y3 ≥ 2



x+y 2

3 ;

´,c bâc 2.6. B` ai to´ an vê` tam thu . hai

87

. 2.47. x4 + 2x3 y + 2xy3 + y4 ≥ 6x2 y2 . `˘ng vo ´,ng minh ra ´,i nh˜u,ng sô´ du,o,ng bâ´t k`ı Chu . 2.48.

x y + ≥ 2. y x

, ` toan ´ vê` tam th´ 2.6. Bai uc bâc . hai , , ´,c c´ ´ câ`n phai t´ınh to´ Râ´t nhiê`u b` ai to´ an trong d an môt o ¯o . sô´ biêu thu , , , , , , , ´ a nghiêm ˜ cho. Ta c´ chu o thê giai nh˜ung b` ai . cua phuong tr`ınh bâc . hai d¯a , , ` ´c d ´ ng. ˘ng c´ to´ an n` ay ba ac d¯a thu ¯ô´i xu ,, ,, 2 V´ı du. 2.49. Cho phuong tr`ınh bâc ay lâp . hai x + 6x + 10 = 0. H˜ . phuong , ,, ,, , ˜,ng nghiêm tr`ınh bâc oi c´ o nghiêm ˘`ng b`ınh phuong nhu . hai m´ . ba . cua phuong ˜ cho. tr`ınh d ¯a , , ,, `,i giai. K´ı hiêu ˜ cho; y1 v` Lo a x2 l` a nghiêm a y2 l` a ¯a . x1 v` . cua phuong tr`ınh d , , , ,, ,, , nghiêm a gia su b v` a c l` a nh˜ung hê. . cua phuong tr`ınh bâc . hai ta phai t`ım v` , ,, , 2 ´ ´ c Vi`ete1 sô cua phuong tr`ınh bâc . hai: y + by + c = 0. Theo công thu δ1 = x1 + x2 = −6,

δ2 = x1 x2 = 10

˜ c´ v` a ta d o ¯a y1 + y2 = −b, y1 y2 = c. , , ´ Nhung theo gia thiê´t y1 = x21 , y2 = x22 , do d ¯o b = −(y1 + y2 ) = −(x21 + x22 ) = −S2 = −(δ21 − 2δ2 ) = −16, c = y1 y2 = x21 x22 = δ22 = 100. , , ,, Nhu vây o dang . phuong tr`ınh phai t`ım c´ . y2 − 16y + 100 = 0. 1 Francois

Vi`ete (1540-1603): Nh` a to´ an hoc ap . Ph´

J

,, ´,c d ´,ng hai biê´n Chuong 2. Ða thu ¯ô´i xu

88

,, 2 V´ı du. 2.50. H˜ ay lâp . hai z + pz + q = 0 sao cho nghiêm . . phuong tr`ınh bâc , , ˜ ng sô´ cua n´ o l` a nhu z1 = x61 − 2x22 , z2 = x62 − 2x21 , , ,, ,, 2 od a nghiêm ¯ây x1 , x2 l` . cua phuong tr`ınh bâc . hai x − x − 3 = 0. , `,i giai. Theo công thu ´,c Vi`ete Lo δ1 = x1 + x2 = 1,

δ2 = x1 x2 = −3.

˘. t kh´ Ma ac ta lai o . c´ −p = z1 + z2 = (x61 − 2x22 ) + (x62 − 2x21 ), q = z1 z2 = (x61 − 2x22 )(x62 − 2x21 ). , ´,c (2.2) d˜ê d` Ta d` ung công thu ang biêu di˜ên p v` a q theo c´ ac biê´n δ1 v` a δ2 , ,, , ´ thay gi´ sau d a tri. δ1 = 1 v` a δ2 = −3, t´ınh d a q. ¯o ¯uo. c nh˜ung hê. sô´ p v` Ta c´ o −p = (x61 + x62 ) − 2(x21 + x22 ) = S6 − 2S2 = = (δ61 − 6δ41 δ2 + 9δ21 δ22 − 2δ32 ) − 2(δ21 − 2δ2 ) = = (16 − 6.14 (−3) + 9.12 (−3)2 − 2(−3)3 ) − 2(12 − 2(−3)) = 140; q = (x61 − 2x22 )(x62 − 2x21 ) = x61 x62 − 2(x81 + x82 ) + 4x21 x22 = =δ62 − 2S8 + 4δ22 = δ62 − 2(δ81 − 8δ61 δ2 + 20δ41 δ22 − 16δ21 δ32 + 2δ42 ) + 4δ22 =(−3)6 − 2(18 − 8.16 (−3) + 20.14 .(−3)2 − 16.12 .(−3)3 + 2(−3)4 ) + 4(−3)2 = − 833. , , ,, Nhu vây a phuong tr`ınh bâc a . p = −140, q = −833 v` . hai phai t`ım l` 2 z − 140z − 833 = 0. , ,, V´ı du. 2.51. Cho x1 v` a x2 l` a nghiêm cua phuong tr`ınh bâc . . hai , , , , , 2 k k ˜ ng gi´ x + px + q = 0. H˜ ay t´ınh nhu a tri. cua biêu thu´c x1 + x2 v´ oi k =

J

±1, ±2, ±3, ±4, ±5.

´,c bâc 2.6. B` ai to´ an vê` tam thu . hai

89

, `,i giai. Theo công thu ´,c Vi`ete ta c´ Lo o δ1 = x1 + x2 = −p, δ2 = x1 x2 = q. , , , ´ c o (2.2) c´ Theo c´ ac công thu o x1 + x2 = S1 = −p; x21 + x22 = S2 = δ21 − 2δ2 = p2 − 2q; x31 + x32 = S3 = δ31 − 3δ1 δ2 = −p3 + 3pq; x41 + x42 = S4 = δ41 − 4δ21 δ2 + 2δ22 = p4 − 4p2 q + 2q2 ; x51 + x52 = S5 = δ51 − 5δ31 δ2 + 5δ1 δ22 = −p5 + 5p3 q − 5pq2 . v` a −1 x−1 1 + x2 = −2 x−2 1 + x2 = −3 x−3 1 + x2 = −4 x−4 1 + x2 = −5 x−5 1 + x2 =

1 (x1 + x2 ) = x1 x2 1 (x2 + x22 ) = x21 x22 1 1 (x3 + x32 ) = x31 x32 1 1 (x4 + x42 ) = x41 x42 1 1 (x5 + x52 ) = x51 x52 1

1 S1 δ2 1 S2 δ22 1 S3 δ32 1 S4 δ42 1 S5 δ52

1 = − p; q 1 = − 2 (p2 − 2q); q 1 = 3 (−p3 + 3pq); q 1 = 4 (p4 − 4p2 q + 2q2 ); q 1 = 5 (−p5 + 5p3 q − 5pq2 ). q

J

, , ,, ˜,ng nghiêm V´ı du. 2.52. Chu´ng minh ra ˘`ng nê´u x1 v` a x2 l` a nhu . cua phuong , , , 2 tr`ınh bâc oi c´ ac hê. sô´ nguyên p v` a q, th`ı v´ oi sô´ tu. nhiên . hai x + px + q = 0 v´ n kh´ ac 0 bâ´t k`ı n, sô´ xn a sô´ nguyên. 1 + x2 l` , `,i giai. Ta c´ ´,c trong Lo o δ1 = x1 + x2 = −p, δ2 = x1 x2 = q . Theo công thu mênh d ¯ê` 2.2 ta viê´t lai: . . n n−1 xn + xn−1 ) − q(xn−2 + xn−2 ). 1 + x2 = −p(x1 2 1 2

90

,, ´,c d ´,ng hai biê´n Chuong 2. Ða thu ¯ô´i xu

, , , `˘ng ´,c n` ´,ng minh kha ˘ng d¯inh Ta d` ung công thu ay d¯ê chu cua b` ai to´ an ba . ,, ´,i n = 1, 2 ta c´ phuong ph´ ap quy nap. o . Vo x11 + x12 = x1 + x2 = δ1 = −p ∈ Z, x21 + x22 = S2 = δ21 − 2δ2 = p2 − 2q ∈ Z.

, ,, , , ´,c o, Gia su Sn−1 v` a Sn−2 l` a nh˜ung sô´ nguyên (n ≥ 3), th`ı theo công thu n ´,i trên ta suy ra Sn = xn ung l` a sô´ nguyên. Suy ra Sn l` a sô´ nguyên vo 1 + x2 c˜

J

moi . n.

, , , ˜,ng sô´ x, y, z thoa m˜ V´ı du. 2.53. Chu´ng minh ra ˘`ng tâ´t ca nhu an x + y + z = 5   , 7 v` a xy + yz + zx = 8, na ˘`m trong khoang 1, . 3   , , 7 , , ` `i giai. Ta s˜e chu ´ ng minh sô´ z na ˘m trong khoang 1, , c` Lo on c´ ac sô´ kh´ ac 3 , ,, , , ´,c ta t´ınh d¯u,o.,c δ1 = x + y = 5 − z, ˘ng thu ho` an to` an tuong tu. . T`u hai d¯a δ2 = xy = 8 − z(x + y) = 8 − z(5 − z) = z2 − 5z + 8. Suy ra x, y l` a nghiêm . , ,, cua phuong tr`ınh u2 − (5 − z)u + z2 − 5z + 8 = 0. , , , , ,, V`ı x, y l` a nh˜ung sô´ thu. c nên biêt . sô´ cua phuong tr`ınh phai không âm: (5 − z)2 − 4(z2 − 5z + 8) ≥ 0.

, ,, ,´, Ta c´ o thê viê´t phuong tr`ınh cuô´i c` ung duo i dang a t`ım . (z − 1)(3z − 7) ≤ 0 v` 7 ,, d¯uo. c 1 ≤ z ≤ . 3 , , , ` tâp Bai (Môt a tra l`oi b` ai tâp . sô´ go. i ´y v` . sau tai . trang 120) .

J

, ,, , . 2.54. Lâp a c´ ac nghiêm o l` a l˜ uy th`ua bâc . phuong tr`ınh bâc . hai m` . cua n´ . , , ,, 2 ba cua c´ ac nghiêm c ua phu o ng tr` ınh x + 6x + 10 = 0. . , ,, , . 2.55. Lâp a c´ ac nghiêm o l` a l˜ uy th`ua bâc . phuong tr`ınh bâc . hai m` . cua n´ . , , ,`, ,, 2 muoi cua c´ ac nghiêm . cua phuong tr`ınh x + x − 3 = 0.

´,c d¯ô´i xu ´,ng ra thu `,a sô´ 2.7. Phân t´ıch d¯a thu

91

, ,, . 2.56. Lâp a c´ ac nghiêm an x51 +x52 = . phuong tr`ınh bâc . hai m` . x1 , x2 thoa m˜ 31, x1 + x2 = 1. , , , , ,, ´,i gi´ . 2.57. Vo a tri. thu. c n` ao cua a th`ı tông b`ınh phuong c´ ac nghiêm . cua , ,, phuong tr`ınh x2 − (a − 2)x − a − 1 = 0 nhân a tri. nho nhâ´t? . gi´ , ,, `˘ng nê´u x1 , x2 l` ´,ng minh ra . 2.58. Chu a nghiêm . cua phuong tr`ınh bâc . hai , , 2 n n ´, ´ ´ x − 6x + 1 = 0, th`ı tông x1 + x2 voi n l` a sô tu. nhiên bât k`ı không chia hê´t cho 5. , ,, . 2.59. Cho x2 + px + q = 0 l` a phuong tr`ınh bâc o . hai, nghiêm . α, β cua n´ , , √ √ , ,, , ,, l` a nh˜ung sô´ duong. H˜ ay biêu di˜ên 4 α + 4 β theo nh˜ung hê. sô´ cua phuong tr`ınh.

, , , 2.7. Phân t´ıch da uc dô ung ra th`ua sô´ ¯ th´ ¯ ´i x´ , , , `˘ng ´,c d ´,ng bâc Ta c´ o thê phân t´ıch ra th`ua sô´ nh˜ung d¯a thu ¯ô´i xu . cao ba , ,´, ,, ´,c c´ c´ ach chuyên th` anh c´ ac d ac biê´n δ1 v` a δ2 . C´ ac buo cd anh: ¯a thu ¯uo. c tiê´n h` , , , , , ´ c d¯ô´i xu ´ ng th` ´ c cua δ1 v` 1. Chuyên d anh c´ ac d a δ2 v` a ta phân ¯a thu ¯a thu , , ´ ´ c n` t´ıch d ay ra th`ua sô. ¯a thu , , , , ,, ´,c theo bâc a nho hon bâc 2. Ta phân t´ıch d ¯a thu . cua d¯a . cua δ2 , thu`ong l` ´,c ban d¯â`u. thu , , , , ´,c v` 3. Thay gi´ a tri. cua δ1 = x + y v` a δ2 = xy v` ao biêu thu a biê´n d ¯ôi d¯ua , , , , ´,c bâc ˘. c ba theo x, y v` vê` nh˜ung t´ıch cua nh˜ung d¯a thu a t`u d ¯ây . hai hoa , , , ´ ´ ` ` ta t´ınh nghiêm a phân t´ıch tiêp. . cua tung thua sô v` ´,c bâc Ta lâ´y v´ı du. c´ ac d¯a thu . bô´n: , `,a sô´: V´ı du. 2.60. H˜ ay phân t´ıch d ¯a thu´c ra thu P(x, y) = 10x4 − 27x3 y − 110x2 y2 − 27xy3 + 10y4 .

,, ´,c d ´,ng hai biê´n Chuong 2. Ða thu ¯ô´i xu

92 , `,i giai. Ta c´ o Lo

P(x, y) = 10(x4 + y4 ) − 27xy(x2 + y2 ) − 110x2 y2 = 10S4 − 27δ2 S2 − 110δ22 . ,, ´,c (2.2) ta t`ım d Theo công thu ¯uo. c P(x, y) = 10δ41 − 67δ21 δ2 − 36δ22 . ´,c n` ´,c bâc ´,c tam ´,i kiê´n thu Ta coi d¯a thu ay l` a d¯a thu . hai theo biê´n δ2 , th`ı vo , , ,, ´,c bâc ´,c thu ang phân t´ıch ra th`ua sô´. Giai phuong tr`ınh d ¯a thu . hai ta d˜ê d` 5 2 ´,i δ2 ta c´ d¯ô´i vo o c´ ac nghiêm: δ2 = −2δ21 v` δ , vây a δ2 = . 36 1 . 5 P(x, y) = −36(δ2 + 2δ21 )(δ2 − δ21 ) = (2δ21 + δ2 )(5δ21 − 36δ2 ). 36 , Thay c´ ac gi´ a tri. cua δ1 v` a δ2 ta c´ o P(x, y) = (2(x + y)2 + xy)(5(x + y)2 − 36xy) = = (2x2 + 5xy + 2y2 )(5x2 − 26xy + 5y2 ). ,, ˜ th`u,a sô´ lai ´,c bâc ´,c ac d Môi a tam thu ¯a thu . l` . hai nên ta lai . lâ`n luo. t phân t´ıch c´ , , ´,c bâc n` ay. V´ı du. xem 2x2 + 5xy + 2y2 nhu tam thu o . hai cua biê´n x, ta s˜e c´ 1 nghiêm a v`ı thê´ . x = − 2 y, x = −2y v` 1 2x2 + 5xy + 2y2 = 2(x + y)(x + 2y) = (2x + y)(x + 2y). 2 ,, , Tuong tu. ta c˜ ung c´ o 5x2 − 26xy + 5y2 = (x − 5y)(5x − y). ,, Cuô´i c` ung ta nhân ¯uo. c . d P(x, y) = (2x + y)(x + 2y)(x − 5y)(5x − y). , `,a sô´: V´ı du. 2.61. H˜ ay phân t´ıch d¯a thu´c ra thu P(x, y) = 6x4 − 11x3 y − 18x2 y2 − 11xy3 + 6y4 .

J

´,c d¯ô´i xu ´,ng ra thu `,a sô´ 2.7. Phân t´ıch d¯a thu

93

, , `,i giai. Ta biêu di˜ên d ´,c d¯ô´i xu ´,ng P(x, y) qua δ1 v` Lo a δ2 : ¯a thu P(x, y) = 6(x4 + y4 ) − 11xy(x2 + y2 ) − 18x2 y2 = 6S4 − 11δ2 S2 − 18δ22 = 6δ41 − 35δ21 δ2 + 16δ22 . , ´,c n` ´,c bâc ´,i biê´n δ2 . Giai ra ta c´ Ta coi d¯a thu ay l` a tam thu o nghiêm . hai d¯ô´i vo . 3 , , , , , 2 2 ´c d δ v` a d¯a thu δ2 = 2δ1 v` a δ2 = ¯uo. c phân t´ıch ra th`ua sô´ nhu sau: 16 1 3 P(x, y) = 16(δ2 − 2δ21 )(δ2 − δ21 ) = (2δ21 − δ2 )(3δ21 − 16δ2 ). 16 ,, , , ` ` ´ c ban d T`u d¯ây tro vê d ¯a thu ¯ âu P(x, y) = (2(x + y)2 − xy)(3(x + y)2 − 16xy) = = (2x2 + 3xy + 2y2 )(3x2 − 10xy + 3y2 ). , ,, ´, nhâ´t c´ ´,c nên không phân t´ıch d Th`ua sô´ thu o nghiêm a sô´ phu anh ¯uo. c th` . l` , , , , , ´ c c´ ´ hai d˜ê t´ıch c´ ac d¯a thu o hê. sô´ l` a thu. c, nên ta gi˜u nguyên. Th`ua sô´ thu , , d` ang phân t´ıch th` anh (x − 3y)(3x − y). Cuô´i c` ung ta nhân . d¯uo. c P(x, y) = (2x2 + 3xy + 2y2 )(x − 3y)3x − y). , `,a sô´: V´ı du. 2.62. H˜ ay phân t´ıch d ¯a thu´c ra thu P(x, y) = 2x4 − x3 y + x2 y2 − xy3 + 2y4 . , `,i giai. Tu,o,ng tu., nhu, hai b` ai trên ta c´ o Lo P(x, y) = 2(x4 + y4 ) − xy(x2 + y2 ) + x2 y2 = 2S4 − δ2 S2 + δ22 = = 2(δ41 − 4δ21 δ2 + 2δ22 ) − δ2 (δ21 − 2δ2 ) + δ22 = = 2δ41 − 9δ21 δ2 + 7δ22 = (δ21 − δ2 )(2δ21 − 7δ2 ) = = ((x + y)2 − xy)(2(x + y)2 − 7xy) = = (x2 + xy + y2 )(2x2 − 3xy + 2y2 ).

J

94

,, ´,c d ´,ng hai biê´n Chuong 2. Ða thu ¯ô´i xu

, ,, , , ´,c c´ Hai th`ua sô´ trên không phân t´ıch d¯uo. c ra t´ıch nh˜ung d o hê. sô´ thu. c. ¯a thu

J

, ,, , ´,c d¯ô´i xu ´,ng chuyên sang d¯a thu ´,c phu. C´ o nhiê`u tru`ong ho. p khi d ¯a thu , , , ,, ´,i ân δ2 th`ı phu,o,ng thuôc ao δ1 v` a δ2 , nhung khi giai phuong tr`ınh d ¯ô´i vo . v` , , , tr`ınh không c´ o nghiêm ach phân t´ıch theo kiêu n` ay không . thu. c. Nhu vây . c´ , , , , , , , ´ ˜ ´ ´ cho kêt qua. Voi nhung d¯a thuc nhu vây, o thê phân t´ıch ra th`ua sô´ . ta c´ , ,, , ,, , `˘ng c´ ˘ng han ba ach d` ung phuong ph´ ap hê. sô´ bâ´t d¯inh, cha . . nhu tru`ong ho. p , bâc o phân t´ıch ¯ê c´ . bô´n, d P(x, y) = (ax2 + bxy + cy2 )(cx2 + bxy + ay2 ) , , , ,, , , od a nh˜ung ân sô´ câ`n phai t`ım. Ta c´ o thê l` am nhu v´ı du. sau: ¯ây a, b, c l` , `,a sô´: V´ı du. 2.63. H˜ ay phân t´ıch d ¯a thu´c ra thu P(x, y) = 2x4 + 3x3 y + 6x2 y2 + 3xy3 + 2y4 . , , , `,i giai. Ta c´ ´,c vê` dang Lo o thê chuyên d ac biê´n δ1 v` a δ2 : ¯a thu . c´ P(x, y) = 2δ41 − 5δ21 δ2 + 4δ22 . , ,, , ´,i δ2 không c´ Phuong tr`ınh n` ay d o nghiêm ung c´ ach ¯ô´i vo . thu. c. Vây . ta phai d` kh´ ac. Ta c´ o 2x4 + 3x3 y + 6x2 y2 + 3xy3 + 2y4 = (ax2 + bxy + cy2 )(cx2 + bxy + ay2 ). , , , , ´,c d¯u ´,i moi ˘ng thu ´ ng vo a tri. x, y. V`ı thê´ ta c´ o thê Ta phai t`ım a, b, c d¯ê d¯a . gi´ , ,, ´p dung a phuong ph´ ap gi´ a tri. riêng cua x, y, ngh˜ıa l` a cho môt a tri. cu. . . sô´ gi´ , , , 2 ´ ´ thê d ac hê. sô. Lây x = y = 1 th`ı 16 = (a + b + c) , t`u d¯ây c´ o ¯ê t`ım c´ , , ` a + b + c = ±4. Tuy c´ o hai gi´ a tri. nhung ta chı cân chon . môt . a + b + c = 4. ,, Cho x = 1, y = −1 ta nhân a a−b+c = ±2, ta ¯uo. c 4 = (a−b+c)2 , ngh˜ıa l` . d , , ,, ´ ´ ´ c˜ ung chı lây dâu công a d¯u. Cuôi c` ung x = 0, y = 1, ta nhân ¯uo. c ac = 2. . l` . d

˜,ng b` 2.8. Nhu ai to´ an kh´ ac

95

, , , ,, Nhu vây ac d ¯ê x´ ¯inh . a, b, c ta giai hê. phuong tr`ınh . d   a + b + c = 4, a − b + c = 2,   ac = 2. ,, , , , , ´, T`u hai phuong tr`ınh d¯â`u ta t`ım d a a + c = 3. Kê´t ho. p vo i ¯uo. c b = 1 v` ,, , , , ´ ba ta t`ım d¯uo. c a = 1, c = 2 (hoa ˘. c c = 1, a = 2). Cuô´i phuong tr`ınh thu ,, c` ung ta nhân . d¯uo. c 2x4 + 3x3 y + 6x2 y2 + 3xy3 + 2y4 = (x2 + xy + 2y2 )(2x2 + xy + y2 ). , , , `˘ng c´ Ta c´ o thê kiêm tra lai ach phân t´ıch n` ay ba ach khai triên v` a nh´ om . c´ , , ` ´ ´ ´ c ban d¯âu. Môt lai anh biêu thu u ´y l` a nêu lây a − b + c = −2 th`ı hê. . th` . ch´ ,, , , , ´ c v` phuong tr`ınh s˜e c´ o nghiêm a nhu vây . phu . không d¯ua d¯ê´n kê´t luân. .

J

,

, , ` tâp (Môt a tra l`oi b` ai tâp Bai . sô´ go. i ´y v` . sau tai . trang 123) . , ´,c sau ra th`u,a sô´ Phân t´ıch nh˜ung d¯a thu . 2.64. 2x4 + 7x3 y + 9x2 y2 + 7xy3 + 2y4 ; . 2.65. 18x4 − 21x3 y − 94x2 y2 − 21xy3 + 18y4 ; . 2.66. 3x4 − 8x3 y + 14x2 y2 − 8xy3 + 3y4 ; . 2.67. 2x4 − x3 y + 3x2 y2 − xy3 + 2y4 ; . 2.68. (x + y)5 − x5 − y5 .

, ` toan ´ khac ´ 2.8. Nh˜ung bai ´,c d ´,ng c` C´ ac dang ai to´ an liên quan d¯ê´n d on râ´t nhiê`u. Khi ra ¯a thu ¯ô´i xu . b` , , , , , ´ c hoa ˘. c nh˜ung kê´t luân ˜ chon d ¯ê`, ngu`oi ra d¯ê` d ¯a ¯ep, . nh˜ung công thu . d . phâ`n , , ´ ng cua c´ ´. Muc n` ao c´ o phâ`n d ac biê´n trong d¯o ay ta x´et môt ¯ô´i xu . n` . sô´ loai . , , , , , ´ ´ ´ ´p dung b` ai to´ an m` a c´ o kha n˘ ang a d a th u c d ô i x u ng d u o c. ¯ ¯ ¯ . . , , ´ c. Ta x´et v´ı du. 1. R´ ut gon . biêu thu

,, ´,c d ´,ng hai biê´n Chuong 2. Ða thu ¯ô´i xu

96

, , V´ı du. 2.69. R´ ut gon . biêu thu´c sau:       1 1 1 1 1 3 1 6 1 P(p, q) = + + + . + + (p + q)3 p3 q3 (p + q)4 p2 q2 (p + q)5 p q , ,, `,i giai. Ta d ´,c (2.2), ta c´ ˘. t δ1 = p + q v` Lo a δ2 = pq. Su dung công thu o ¯a . 1 S3 3 S2 6 δ1 P(p, q) = 3 3 + 4 2 + 5 = δ1 δ2 δ1 δ2 δ1 δ2 =

δ31 − 3δ1 δ2 3(δ21 − 2δ2 ) 6 + + 4 = δ31 δ32 δ41 δ22 δ1 δ2

=

δ4 1 1 δ1 (δ31 − 3δ1 δ2 ) + 3δ2 (δ21 − 2δ2 ) + 6δ22 = 413 = 3 = 3 3 . 4 3 p q δ1 δ2 δ1 δ2 δ2

J

, , ´,ng minh d ´,c (hoa ´,ng minh bâ´t d¯a ´,c). D` ˘ng thu ˘. c chu ˘ng thu 2. Chu ung d¯a ¯a , , , , , , , , ´ ´ ´c d ´ ng ta c´ ´ ng minh d ´ c (hoa ´ c) môt ˘. c bât d ˘ng thu ˘ng thu thu o thê chu ¯ôi xu ¯a ¯a . , , , ´ ´ c´ ach rât tu. nhiên v` a chı thông qua biên d¯ôi. , , , V´ı du. 2.70. Chu´ng minh d ˘ng thu´c sau ¯a (x + y)3 + 3xy(1 − x − y) − 1 = (x + y − 1)(x2 + y2 − xy + x + y + 1); , , , `,i giai. Ta biê´n d¯ôi vê´ tr´ Lo ai cua mô´i liên hê. (x + y)3 + 3xy(1 − x − y) − 1 = δ31 + 3δ2 (1 − δ1 ) − 1 = δ31 + 3δ2 − 3δ1 δ2 − 1. , , , , ´,c ˘ng thu Ta biê´n d ¯ôi vê´ phai cua d ¯a (x + y − 1)(x2 + y2 − xy + x + y + 1) = (δ1 − 1)(δ21 − 3δ2 + δ1 + 1) = δ31 − 3δ1 δ2 + δ21 + δ1 − δ21 + 3δ2 − δ1 − 1 = = δ31 − 3δ1 δ2 + 3δ2 − 1. , , ´,c. ˘ng thu Hai vê´ d¯ê`u c´ o kê´t qua tr` ung nhau, ta c´ o d¯a , ,, 3. Giai phuong tr`ınh vô d¯inh. . , ,, ,, V´ı du. 2.71. H˜ ay t`ım nghiêm . nguyên duong cua phuong tr`ınh sau x3 + y3 + 1 = 3xy.

J

˜,ng b` 2.8. Nhu ai to´ an kh´ ac

97

, , , ,, `,i giai. Ta chuyên c´ Lo ac sô´ hang ad ¯ua phuong tr`ınh vê` dang . sang môt . vê´ v` . δ31 − 3δ1 δ2 + 1 − 3δ2 = 0, suy ra (δ1 + 1)(δ21 − δ1 + 1 − 3δ2 ) = 0. , , , , T`u x > 0, y > 0 suy ra δ1 > 0, nhu vây a nghiêm . δ1 = −1 không phai l` . cua , , ,, ,, phuong tr`ınh trên. Ta chı c` on giai phuong tr`ınh δ21 − δ1 + 1 − 3δ2 = 0. ,, , Suy ra x v` ayd ac d¯inh ¯uo. c x´ . t`u hê.  x + y = δ1 1 2 (δ − δ1 + 1). 3 1 , , ,, ,, ,, ´,i viêc Theo d¯inh . l´ı 2.2, giai hê. trên tuong d¯uong vo . giai phuong tr`ınh sau 1 z2 − δ1 z + (δ21 − δ1 + 1) = 0. 3 , ,, Nghiêm a . cua phuong tr`ınh bâc . hai l` r δ1 δ21 1 2 z1,2 = ± − (δ1 − δ1 + 1) = 2 3 r4 r δ1 δ1 1 2 1 = ± − (δ1 − 4δ1 + 4) = ± − (δ1 − 2)2 . 2 12 2 12 , , , , ,, , ´,c du,o ´,i dâ´u c˘ Ðê nghiêm a sô´ thu. c, th`ı biêu thu an phai . cua phuong tr`ınh l` , , ´,i x, y nguyên không âm, d ay chı xay ra khi δ1 = 2 hay x + y = 2 vo ¯iê`u n` , , , ,, , ,, duong. Ðiê`u n` ay chı xay ra khi x = y = 1. Ta thu lai . phuong tr`ınh, th`ı , , ,, x=y=1d a nghiêm ¯´ıch thu. c l` . cua phuong tr`ınh. ´,c cho môt ´,c kh´ ´,c ´,i nh˜u,ng d 4. B` ai to´ an chia hê´t môt ac vo ¯a thu ¯a thu . d . d¯a thu ,, ,, , ´,ng theo c´ ´,ng minh gon d¯ô´i xu ac biê´n d¯uo. c chu ang hon c´ ac phuong ph´ ap . g` ,`, thông thuong. xy = δ2 =

J

, , ´ ´, V´ı du. 2.72. Chu´ng minh ra ˘`ng môt o ¯ôi xung hai biê´n x, y không c´ . d¯a thu´c d , , , , , , 2 2 ˜ ´ ´ hê. sô tu. do chia hêt cho x + xy + y khi v` a chı khi d ¯a thu´c d ¯uo. c biêu diên , , , ` ´ ´ theo δ1 v` a δ2 nhu môt o tông c´ ac hê. sô ba ˘ng không. ¯a thu´c hai biên c´ . d

,, ´,c d ´,ng hai biê´n Chuong 2. Ða thu ¯ô´i xu

98

, , , `,i giai. Gia su, d¯a thu ´,c d¯ô´i xu ´,ng P(x, y) chia hê´t cho x2 + xy + y2 , ngh˜ıa Lo ,, ´,c d¯ô´i xu ´,ng. Ta l` a P(x, y) = (x2 + xy + y2 )Q(x, y), o d a d¯a thu ¯ây Q(x, y) l` , , , , c´ o x2 + xy + y2 = δ21 − δ2 . Gia su Q(x, y) = ϕ(δ1 , δ2 ) biêu di˜ên qua δ1 v` a ´ P(x, y) c´ δ2 . Khi d¯o o dang . P(x, y) = (x2 + xy + y2 )Q(x, y) = (δ21 − δ2 )ϕ(δ1 , δ2 ). , ,, , ´,c trên ta nhân Thay δ1 = δ2 = 1 v` ao biêu thu ¯uo. c (1 − 1)ϕ(1, 1) = 0. Nhu . d , vây a δ2 v` a P(x, y) = (δ21 − δ2 )ϕ(δ1 , δ2 ) nhân a . P(x, y) biêu di˜ên qua δ1 v` . gi´ , , 2 tri. 0 khi δ1 = δ2 = 1. Ðiê`u n` ay ngh˜ıa l` a tông hê. sô´ cua (δ1 − δ2 )ϕ(δ1 , δ2 ) ` ˘ng không. ba ,, Nguo. c lai, . cho n−2 P(x, y) = δn δ2 + b2 δn−4 δ22 + b3 δn−6 δ32 + · · · 1 + b1 δ 1 1 1

, , , ,, , , , l` a biêu di˜ên cua P(x, y) qua δ1 v` a δ2 v` a gia su tông hê. sô´ cua c´ ach biêu ,, `˘ng không. Ðu,a δn ra ngo` di˜ên trên ba ai, th`ı P(x, y) d¯uo. c viê´t lai . 1

P(x, y) =

δn 1

 1 + b1

δ2 δ21



 + b2

δ2 δ21

2

 + b3

δ2 δ21

!

3 + ···

.

, δ2 , ,, , ´,c trong dâ´u ngoa ˘. c (theo biê´n z = 2 ) không vuo. t Hon n˜ua bâc . cua d¯a thu δ1 n qu´ a v` a k´ı hiêu o l` a k. . n´ 2 , , , , `˘ng 0. Ngh˜ıa ´,c trong dâ´u ngoa ˘. c ba Tông cua nh˜ung hê. sô´ trong biêu thu , , 2 3 `˘ng 0 vo ´ c 1 + b1 z + b2 z + b3 z + · · · b a ´i z = 1. Suy ra theo d¯inh l` ad ¯a thu . , , 2 ´ c n` l´ı B´ezout d ay chia hê´t cho 1 − z. Nhu vây ¯a thu . 1 + b1 z + b2 z2 + b3 z3 + · · · = (1 − z)H(z), 2 Etienne

,, B´ezout (1730-1783): Nh` a to´ an hoc ap. . ngu`oi Ph´

˜,ng b` 2.8. Nhu ai to´ an kh´ ac

99

, ,, `˘ng k − 1. V`ı thê´ od ¯ây bâc . cua H(z) ba !  2  3   δ2 δ2 δ2 n + b2 + b3 + ··· = P(x, y) = δ1 1 + b1 δ21 δ21 δ21       δ2 δ2 δ2 n 2 n−2 = δ1 1 − 2 H = (δ1 − δ2 )δ1 H . 2 δ1 δ1 δ21 , ´,c H(z) không vu,o.,t Nê´u n = 2m + 1, th`ı k ≤ m, ngh˜ıa l` a bâc . cua d¯a thu , , , ´,c qu´ a m − 1, c` on sô´ n − 2 = 2m − 1 không nho hon hai lâ`n bâc ¯a thu . cua d H(z). , ,, Nê´u n = 2m, th`ı k ≤ m, ngh˜ıa l` a bâc am−1 . xua H(z) không vuo. t qu´ , , , , v` a v`ı thê´ sô´ n − 2 = 2m − 2 không nho hon hai lâ`n bâc . cua H(z). Nhu vây, . , , ,`, , , ` ´ trong moi tru o ng h o p n − 2 không nh o ho n hai lâ n b âc c ua H(z), v` ı thê .  . . , , δ 2 ˜ ngh˜ıa l` ´,a δ1 o, mâu, ´,c cua δ1 .δ2 . không chu a n´ o l` a môt δn−2 H . d¯a thu 1 δ21 , Ta c´ o thê viê´t P(x, y) =(δ21 − δ2 )ψ(δ1 , δ2 ), ,, δ2 n−2 ´,c n` ´,c ´. Suy ra d¯a thu o d¯ây ψ(δ1 , δ2 ) = δ1 H l` a môt ao d ¯a thu ¯o . d δ21 P(x, y) chia hê´t cho δ21 − δ2 = x2 + xy + y2 .

J

, , ,, , ˘. c nh˜ung thông sô´ 5. T`ım nh˜ung d o ngh˜ıa hoa ¯iê`u kiên ¯ê hê. phuong tr`ınh c´ . d , , ,, ,, trong phuong tr`ınh thoa m˜ an d¯iê`u kiên o nghiêm. ¯ê hê. phuong tr`ınh c´ . d . , , V´ı du. 2.73. T`ım d a nghiêm ¯iê`u kiên ¯ê (x, y) l` . cho a, b, c d . cua hê.   = a, x + y x2 + y2 = b,   3 x + y3 = c. , , , , , , `,i giai. B` ai to´ an d ai to´ an loai on lai Lo ¯ua vê` b` ¯ê chı c` . tr`u x, y d . su. phu. thuôc . , , ´,i δ1 = x + y v` a, b, c. Vo a δ2 = xy ta c´ o thê chuyên hê. trên vê` dang .   = a, δ1 δ21 − 2δ2 = b,   3 δ1 − 3δ1 δ2 = c.

,, ´,c d ´,ng hai biê´n Chuong 2. Ða thu ¯ô´i xu

100

1 ,, , ,, ao phuong tr`ınh T`u hai phuong tr`ınh: δ1 = a, δ2 = (a2 − b), thay v` 2 3 `˘ng ´, ba ta c´ thu o a3 − a(a2 − b) = c, ngh˜ıa l` a a3 − 3ab + 2c = 0. D˜ê thâ´y ra 2 3 3 nê´u a − 3ab + 2c = 0 th`ı hê. c´ o nghiêm. vây a . . a − 3ab + 2c = 0 ch´ınh l` , , `i hoi mô´i liên quan gi˜ua a, b, c. d¯iê`u kiên . d¯o

J

,

, , ` tâp Bai (Môt a tra l`oi c´ ac b` ai tâp . sô´ go. i ´y v` . sau tai . trang 125) . , , ´,c sau R´ ut gon . nh˜ung biêu thu (x + y)7 − x7 − y7 ; (x + y)5 − x5 − y5     1 1 1 1 2 1 . 2.75. + + . + (a + b)2 a2 b2 (a + b)3 a b . 2.74.

`˘ng ´,ng minh ra Chu . 2.76. (x + y)4 + x4 + y4 = 2(x2 + xy + y2 )2 ; . 2.77. (x + y)7 − x7 − y7 = 7xy(x + y)(x2 + xy + y2 )2 . `˘ng vo ´,ng minh ra ´,c (x + y)n − xn − yn chia ´,i n = 6k ± 1, d . 2.78. Chu ¯a thu hê´t cho x2 + xy + y2 . ´,c x2n + xn + 1 chia hê´t cho ´,i nh˜u,ng d¯iê`u kiên . 2.79. Vo ao th`ı d ¯a thu . n` x2 + x + 1? ´,c (x + 1)n + xn + 1 chia hê´t cho ´,i nh˜u,ng d¯iê`u kiên . 2.80. Vo ao th`ı d¯a thu . n` x2 + x + 1? , `˘ng nê´u nh˜u,ng sô´ u, v, x, y thoa m˜ ´,ng minh ra . 2.81. Chu an u + v = x + , , , , 2 2 2 2 ´ ´ c sau d ´i moi ˘ng thu ´ ng y, u + v = x + y , th`ı vo ¯u . sô tu. nhiên n d¯a un + vn = xn + yn . , ,, . 2.82. Giai phuong tr`ınh trong tâp . sô´ nguyên x + y = x2 − xy + y2 .

,, ´,ng 2.9. Chuyên d ¯ê` vê` phuong tr`ınh hê. sô´ d¯ô´i xu

101

, , , , `˘ng nê´u n l` ´,ng minh ra ´,c . 2.83. Chu a nh˜ung sô´ le, l` a bôi . cua 3, th`ı biêu thu (a + b)n − an − bn − 3(ab)

n−1 2

(a + b)

chia hê´t cho a2 + ab + b2 .

,, , 2.9. Chuyên dê ung ¯ ` vê` phuong tr`ınh hê. sô´ dô ¯ ´i x´ , , , ,, ´,c d¯ô´i xu ´,ng c´ ´ ng dung Ða thu o thê u giai môt . . sô´ phuong tr`ınh bâc . cao. , , , ´ ˘. c biêt ´. Trong tiê´t Tât nhiên nh˜ung phuong tr`ınh bâc o dang ao d¯o ¯a . cao c´ . d . n` , , , , ´ ng: ´i phuong tr`ınh hê. sô´ d n` ay ta quan tâm to ¯ô´i xu ´,c môt Ðinh ngh˜ıa 2.3. Môt . d¯a thu . biê´n . P(z) = a0 zn + a1 zn−1 + · · · + an (a0 6= 0) (2.6) , , , an a0 = an , a1 = an−1 , a2 = goi a hê. sô´ d¯ô´i xu´ng, nê´u nh˜ung hê. sô´ thoa m˜ . l` an−2 , .... , , ´,c n` ˘ng han V´ı du. vê` loai ay râ´t nhiê`u cha ¯a thu . d . nhu: ˘. c zn + 1 d¯ê`u l` z5 − 3z4 + 2z3 + 2z2 − 3z + 1; 2z8 − 6z5 − 6z3 + 2; hoa a , , , ´ ´ ´ c hê. sô d ´ ng. nh˜ung d¯a thu ¯ôi xu ,, ´,c hê. sô´ d¯ô´i xu ´,ng goi Môt ai l` a môt a . phuong tr`ınh P(z) = 0, vê´ tr´ . d¯a thu . l` ,, , phuong tr`ınh hê. sô´ d o d¯inh ¯ô´i xu´ng. Ta c´ . l´ı , , , , , 2.9.1. Ð.inh l´ı co ban cua da uc hê. sô´ dô ung ¯ th´ ¯ ´i x´ , , Ðinh l´ı 2.4. Moi ˘˜n 2k ¯a thu´c hê. sô´ d¯ô´i xu´ng bâc . d . cha . P(z) = a0 z2k + a1 z2k−1 + · · · + a2k

, , ˜ du,o´,i dang c´ o thê biêu diên .

P(z) = zk Q(δ), , ,, 1 , od a Q(δ) l` a môt d k cua δ. ¯a thu´c bâc ¯ây δ = z + v` . . z

,, ´,c d ´,ng hai biê´n Chuong 2. Ða thu ¯ô´i xu

102

, , , a cho Moi ¯ê`u chia hê´t cho (z + 1) v` ¯ô´i xu´ng P(z) bâc ¯a thu´c hê. sô´ d . le d . d ,, , , ˜ thuong l` a môt ˘n. ¯ô´i xu´ng bâc . d¯a thu´c hê. sô´ d . cha ˜˘n 2k: P(z). Ta d ´,ng minh. 1) Ta x´et d ´,c bâc ˘. t sô´ hang Chu chung ¯a thu ¯a . cha . zk ra ngo` ai   1 1 P(z) = zk a0 zk + a1 zk−1 + · · · + a2k−1 k−1 + a2k k . z z `˘ng a0 = a2k , a1 = a2k−1 , ... Ta ch´ u ´y ra       1 1 P(z) = zk a0 zk + k + a1 zk−1 + k−1 + · · · + ak . z z , , , 1 , ´,ng minh zi + i , i = 1, 2, ...k biêu di˜ên thông Nhu vây on phai chu . ta chı c` z , 1 , , ´,c (2.2) vo ´,i qua δ = z + . Nhung b` ai to´ an n` ay l` a hê. cua nh˜ung công thu z , , tông Sk = xk + yk biêu di˜ên thông qua δ1 = x + y v` a δ2 = xy. Trong , 1 1 ,`, , k k truong ho. p cu. thê n` ay x = z, y = th`ı Sk = x + y ch´ınh l` a zk + k v` a z z 1 ´,c (2.2) vo ´,i δ1 = δ, δ2 = 1 d¯u,o.,c viê´t lai δ1 = z + = δ, δ2 = 1. Công thu . z , nhu sau: z2 + z−2 = δ2 − 2, z3 + z−3 = δ3 − 3δ, z4 + z−4 = δ4 − 4δ2 + 2, z5 + z−5 = δ5 − 5δ3 + 5δ, z6 + z−6 = δ6 − 6δ4 + 9δ2 − 2, z7 + z−7 = δ7 − 7δ5 + 14δ3 − 7δ, z8 + z−8 = δ8 − 8δ6 + 20δ4 − 16δ2 + 2, z9 + z−9 = δ9 − 9δ7 + 27δ5 − 30δ3 + 9δ, z10 + z−10 = δ10 − 10δ8 + 35δ6 − 50δ4 + 25δ2 − 2, ······

,, ´,ng 2.9. Chuyên d ¯ê` vê` phuong tr`ınh hê. sô´ d¯ô´i xu

103

, ´,c hê. sô´ d ´,ng bâc 2) Ta x´et d¯a thu ¯ô´i xu . le 2k + 1: P(z) = a0 z2k+1 + a1 z2k + · · · + a2k z + a2k+1 .

, ,´, ´ ta c´ Ta c´ o a0 = a2k+1 , a1 = a2k , a2 = a2k−1 , ... khi d¯o o thê viê´t duo i dang . P(z) = a0 (z2k+1 + 1) + a1 (z2k + z) + a2 (z2k−1 + z2 ) + · · · + ak (zk+1 + zk ) = a0 (z2k+1 + 1) + a1 z(z2k−1 + 1) + a2 z2 (z2k−3 + 1) + · · · + ak zk (z + 1).

, ˜ sô´ hang, `˘ng c´ ´,c trong dâ´u ngoa ˘. c c´ Trong môi biêu thu o chung z + 1, ba ach . , ´ c sau ´p dung a công thu . z2m+1 + 1 = (z + 1)(z2m − z2m−1 + z2m−2 − · · · + z2 − z + 1). ,, Ta nhân . d¯uo. c a0 (z2k+1 + 1) = a0 (z + 1)(z2k − z2k−1 + z2k−2 + · · · + z2 − z + 1), a1 z(z2k−1 + 1) = a1 z(z + 1)(z2k−2 − z2k−3 + · · · − z + 1) = = a1 (z + 1)(z2k−1 − z2k−2 + · · · − z2 + z), a2 z2 (z2k−3 + 1) = a2 z2 (z + 1)(z2k−4 − · · · + 1) = = a2 (z + 1)(z2k−2 − · · · + z2 ), ······ ak zk (z + 1) = ak (z + 1)zk .

,, ,, `˘ng c´ Ba ach nh´ om z + 1 ra ngo` ai ta d¯uo. c P(z) = (z + 1)Q(z), o d¯ây Q(z) l` a , , , , , , , , , , ´ ˜ ´ ˜ d a ta c´ o thê kiêm tra tông cua nhung d ¯a thuc tông cua nhung d¯a thuc v` ¯a , , , ´ ´ ´ ´ ´ thuc n` ay l` a môt ¯ôi xung. . d¯a thuc hê. sô d

J

, , , , Mênh d¯ê` 2.1. Ða thu´c P(z) bâc ad a chı khi ¯a thu´c hê. sô´ d ¯ô´i xu´ng khi v` . n l` . , n´ o thoa m˜ an   1 = P(z). zn P z

,, ´,c d ´,ng hai biê´n Chuong 2. Ða thu ¯ô´i xu

104

´,ng minh. Cho P(z) = a0 zn + a1 zn−1 + · · · + an (a0 6= 0). Ta c´ Chu o !    n  n−1 1 1 1 1 zn P = zn a0 + a1 + · · · + an−1 + an = z z z z = a0 + a1 z + · · · + an−1 zn−1 + an zn = = an zn + an−1 zn−1 + · · · + a1 z + a0 .  , , , , ´, ´,c P(z) = zn P 1z d¯u ˘ng thu ´ ng khi v` Ða a chı khi nê´u c´ ac hê. sô´ tuong u ng , , ` ´ c sau ba ˘ng nhau, ngh˜ıa l` cua hai d¯a thu a a0 = an , a1 = an−1 , ....  , , , , ´,c ´,c P(z) = zn P 1z chı xay ra khi v` ˘ng thu a chı khi d¯a thu N´ oi c´ ach kh´ ac d¯a ´,c hê. sô´ d ´,ng. P(z) l` a d¯a thu ¯ô´i xu

J

, , ˜,ng d a P(z) Mênh d a Q(z) l` a nhu ¯ô´i xu´ng v` ¯a thu´c hê. sô´ d ¯ê` 2.2. Nê´u P(z) v` . P(z) , , chia hê´t cho Q(z), th`ı H(z) = c˜ ung l` ad ¯a thu´c hê. sô´ d ¯ô´i xu´ng. Q(z) , , , ,, ´,ng minh. Ta k´ı hiêu ´,c P(z) v` Chu a Q(z) tuong . bâc . cua cua nh˜ung d¯a thu ,, , ´,ng l` ´,c H(z) c´ u a m v` a n th`ı d¯a thu o bâc a Q(z) l` a nh˜ung . m − n. Boi v`ı P(z) v` , ´,c hê. sô´ d ´,ng nên theo kê´t qua mênh d¯a thu d¯ê` 2.1 th`ı ¯ô´i xu .     1 1 n m , Q(z) = z Q . P(z) = z P z z , ,, ´,c trên, ta nhân ˘ng thu Chia theo vê´ hai d¯a . d¯uo. c   1 zm P 1z P(z) m−n P z  =z . = n Q(z) z Q 1z Q 1z , ,, ,´, Nhu vây i dang . H(z) viê´t d¯uo. c duo .   1 . z ´,c hê. sô´ d¯ô´i xu ´,ng. Ðiê`u n` ay ngh˜ıa l` a H(z) l` ad ¯a thu H(z) = zm−n H

J

,, ´,ng 2.9. Chuyên d ¯ê` vê` phuong tr`ınh hê. sô´ d¯ô´i xu

105

, , ,, 2.9.2. Nh˜ung v´ı du. giai phuong tr`ınh bâc . cao ´,ng dung Ta x´et môt d¯inh . l´ı trên: . sô´ v´ı du. u . , ,, V´ı du. 2.84. Giai phuong tr`ınh 6z4 − 13z3 + 12z2 − 13z + 6 = 0. , ,, ˜˘n) o,, trên l` `,i giai. Phu,o,ng tr`ınh bâc Lo a phuong tr`ınh hê. sô´ d ¯ô´i . bô´n (bâc . cha , , , , , , , ´ ng. Theo d¯inh xu ai cua phuong tr`ınh d¯uo. c biê´n d a ¯ôi l` . l´ı 2.4 vê´ tr´   1 1 4 3 2 2 2 6z − 13z + 12z − 13z + 6 = z 6z − 13z + 12 − 13 + 6 2 = z z       1 1 = z2 6 z2 + 2 − 13 z + + 12 = z z = z2 (6(δ2 − 2) − 13δ + 12) = z2 (6δ2 − 13δ) , , , , ,, ˜ cho, ta chı c` V`ı z = 0 không phai l` a nghiêm on giai ¯a . cua phuong tr`ınh d ,, phuong tr`ınh 6δ2 − 13δ = 0. , 13 , ,, ˜ cho D˜ê thâ´y δ = 0 v` aδ= . Nhu vây t`ım nghiêm cua phuong tr`ınh d¯a . . 6 , ,, ,, , , , ´i giai hai phuong tr`ınh tuong d¯uong vo 1 13 1 . z + = 0, z + = z z 6 ,, , ,, ´, nhâ´t không c´ ´, hai c´ Phuong tr`ınh thu o nghiêm o hai . thu. c, phuong tr`ınh thu 3 2 nghiêm a z1 = v` a z2 = . . l` 2 3 , ,, V´ı du. 2.85. Giai phuong tr`ınh

J

4z11 +4z10 −21z9 −21z8 +17z7 +17z6 +17z5 +17z4 −21z3 −21z2 +4z+4 = 0 , , ,, `,i giai. Ðây l` ´,ng bâc a phuong tr`ınh hê. sô´ d Lo ¯ô´i xu ¯inh . l´ı 2.4 vê´ . le. Theo d , , ,, tr´ ai cua phuong tr`ınh chia hê´t cho z + 1 v` a cho kê´t qua 4z11 +4z10 −21z9 −21z8 +17z7 +17z6 +17z5 +17z4 −21z3 −21z2 +4z+4 = = (z + 1)(4z10 − 21z8 + 17z6 + 17z4 − 21z2 + 4).

,, ´,c d ´,ng hai biê´n Chuong 2. Ða thu ¯ô´i xu

106

, , , ,, ,, ,, ´,i giai hai phu,o,ng tr`ınh Nhu vây, ¯â`u tuong d ¯uong vo . giai phuong tr`ınh ban d 4z10 − 21z8 + 17z6 + 17z4 − 21z2 + 4 = 0. , ,, ,, ´, nhâ´t cho ta nghiêm Phuong tr`ınh thu ai cua phuong tr`ınh . z1 = −1. Vê´ tr´ , ˜˘n. Ta lai ´, hai lai ´,c hê. sô´ d¯ô´i xu ´,ng bâc thu ad ¯a thu . l` . cha . biê´n d¯ôi z + 1 = 0;

4z10 − 21z8 + 17z6 + 17z4 − 21z2 + 4 =   1 1 1 5 5 3 = z 4z − 21z + 17z + 17 − 21 3 + 4 5 = z z z        1 1 1 5 5 3 = = z 4 z + 5 − 21 z + 3 + 17 z + z z z = z5 (4(δ5 − 5δ3 + 5δ) − 21(δ3 − 3δ) + 17δ) = z5 (4δ5 − 41δ3 + 100δ). , , , ,, ,, ˜ cho, nên ta chı c` Boi v`ı z = 0 không phai l` a nghiêm on ¯a . cua phuong tr`ınh d , , ,, phai giai phuong tr`ınh (4δ4 − 41δ2 + 100)δ = 0. ,, Suy ra ta c´ o nghiêm a thêm c´ ac nghiêm ung . δ = 0 v` . trong phuong tr`ınh tr` ,, phuong 4δ4 − 41δ2 + 100 = 0. , , Cuô´i c` ung ta nhân am nghiêm ¯uo. c n˘ . d . 5 5 δ = 0, δ = − , δ = , δ = 2, δ = −2. 2 2 , , , , ,, ,, ,, Ðê t`ım d am phuong ¯uo. c nghiêm . cua phuong tr`ınh ban d¯â`u ta phai giai n˘ tr`ınh 1 1 5 1 5 1 1 = 0, z + = − , z + = , z + = 2, z + = −2. z z 2 z 2 z z ,, ,, ,´, ´,i môt ˜ C` ung vo nghi êm d a t` ım d u o c tru o c d ây z = −1, ta t` ım thêm d ¯ . ¯ ¯uo. c 6 1 . . ¯ , , nghiêm o 2 nghiêm a . n˜ua (c´ . k´ep), tông công . l` 1 z1 = −1, z2 = −2, z3 = − , 2 1 z4 = 2, z5 = , z6 = z7 = −1, z8 = z9 = 1. 2 z+

J

,, ´,ng 2.9. Chuyên d ¯ê` vê` phuong tr`ınh hê. sô´ d¯ô´i xu

107

, ,, V´ı du. 2.86. Giai phuong tr`ınh 9z6 − 18z5 − 73z4 + 164z3 − 73z2 − 18z + 9 = 0. , ,, ˜˘n, ta c´ `,i giai. Ta nhân ´,ng bâc Lo a phuong tr`ınh hê. sô´ d¯ô´i xu o . thâ´y d¯ây l` . cha 9z6 − 18z5 − 73z4 + 164z3 − 73z2 − 18z + 9 =         1 1 1 3 3 2 = z 9 z + 3 − 18 z + 2 − 73 z + + 164 = z z z = z3 (9(δ3 − 3δ) − 18(δ2 − 2) − 73δ + 164) = = z3 (9δ3 − 18δ2 − 100δ + 200). , , , , ,, ˜ cho, ta chı c` V`ı z = 0 không phai l` a nghiêm on giai ¯a . cua phuong tr`ınh d ,, ´,i δ: phuong tr`ınh bâc ¯ô´i vo . ba d 9δ3 − 18δ2 − 100δ + 200 = 0. , , , ,, , ´p Vê´ tr´ ai cua phuong tr`ınh c´ o thê phân t´ıch th` anh c´ ac th`ua sô´ (c´ o thê a dung d ¯inh . l´ı B´ezout) . (δ − 2)(9δ2 − 100) = 0. ,, Ta t`ım d ¯uo. c ba nghiêm . δ = 2,

δ=

10 , 3

δ=−

10 . 3

, ,, , ,, `˘ng c´ Nhu vây, ach giai ba phuong ¯â`u ba . ta t`ım nghiêm . phuong tr`ınh ban d tr`ınh: 1 = 2, z , ,, Giai ra ta d ¯uo. c 6 nghiêm: . z+

z1 = z2 = 1,

z+

z3 = 3,

1 10 = , z 3

z4 =

1 , 3

z+

1 10 =− . z 3

z5 = −3,

1 z6 = − . 3

, ,, V´ı du. 2.87. Giai phuong tr`ınh z7 + 2z6 − 5z5 − 13z4 − 13z3 − 5z2 + 2z + 1 = 0.

J

,, ´,c d ´,ng hai biê´n Chuong 2. Ða thu ¯ô´i xu

108

, , ,, `˘ng c´ `,i giai. Ðây l` ´,ng bâc a phuong tr`ınh hê. sô´ d¯ô´i xu Lo ach chia vê´ tr´ ai . le. Ba , , ` 3 cho z + 1 (c´ o thê d` ung so d¯ô Horner ) P(z) = (z + 1)(z6 + z5 − 6z4 − 7z3 − 6z2 + z + 1). , ,, ,, Phuong tr`ınh ban d¯â`u c´ o môt a ta giai phuong tr`ınh . nghiêm . z = −1 v` Q(z) = z6 + z5 − 6z4 − 7z3 − 6z2 + z + 1 = 0. Ta c´ o Q(z) = z3



z3 +

1 z3



     1 1 + z2 + 2 − 6 z + −7 = z z

= z3 ((δ3 − 3δ) + (δ2 − 2) − 6δ − 7) = z3 (δ3 + δ2 − 9δ − 9). , , , , ,, V`ı z = 0 không phai l` a nghiêm on giai . cua phuong tr`ınh ban d¯â`u. Ta chı c` ,, phuong tr`ınh δ3 + δ2 − 9δ − 9 = 0. , ,, Phuong tr`ınh n` ay c´ o thê viê´t δ3 + δ2 − 9δ − 9 = δ2 (δ + 1) − 9(δ + 1) = (δ + 1)(δ2 − 9) = 0. , ´ nghiêm Ta c´ o ba nghiêm a ¯o . δ1 = −1, δ2 = 3, δ3 = −3. Khi d . cua Q(z) l` , ,, nghiêm . cua ba phuong tr`ınh z+

1 = −1, z

z+

1 = 3, z

z+

1 = −3. z

, , , ,, ,, Giai phuong tr`ınh trên cho ta nghiêm a tâ´t ca sô´ nghiêm . v` . cua phuong tr`ınh ban d a ¯â`u l` √ √ 3± 5 −3 ± 5 z1 = −1, z2,3 = , z4,5 = . 2 2 , ,, V´ı du. 2.88. Giai phuong tr`ınh

J

P(z) = 12z5 − 56z4 + 107z3 − 107z2 + 56z − 12 = 0. 3 William

,, George Horner (1786-1837): Nh` a to´ an hoc . ngu`oi Anh.

,, ´,ng 2.9. Chuyên d ¯ê` vê` phuong tr`ınh hê. sô´ d¯ô´i xu

109

, , ,, `,i giai. Ðây không phai l` ´,ng, nhu,ng ta nhân Lo a phuong tr`ınh hê. sô´ d¯ô´i xu . , , ´ ng v` thâ´y c´ ac hê. sô´ gâ`n d a c´ o nghiêm o thê phân t´ıch ra ¯ô´i xu . z = 1. Ta c´ , th`ua sô´ P(z) = (z − 1)(12z4 − 44z3 + 63z2 − 44z + 12). , ,, ,, Phuong tr`ınh ban d¯â`u c´ o môt a z = 1 v` a ta giai phuong tr`ınh . nghiêm . l` Q(z) = 12z4 − 44z3 + 63z2 − 44z + 12 = 0. , , , ,, ,, ´,ng, ta c´ Nhung phuong tr`ınh n` ay l` a phuong tr`ınh hê. sô´ d o thê biê´n d¯ôi ¯ô´i xu       1 1 2 2 + 63 = Q(z) = z 12 z + 2 − 44 z + z z = z2 (12(δ2 − 2) − 44δ + 63) = z2 (12δ2 − 44δ + 39). , , , , , z = 0 không phai l` a nghiêm on phai giai . cua Q(z) = 0. Ta chı c` 12δ2 − 44δ + 39 = 0. , 13 3 ,, ,, a δ2 = . Giai hai phuong tr`ınh Phuong tr`ınh n` ay c´ o hai nghiêm . δ1 = 2 v` 6 1 3 1 13 z+ = ; z+ = z 2 z 6 , ,, , ´, ´ ta d¯uo. c 2 nghiêm. Kê t h o p v o i nghi êm ban d ¯â`u z = 1, suy ra tông sô´ . . . , ,, ˜ cho l` nghiêm a: . cua phuong tr`ınh d¯a 3 2 z1 = 1, z2 = , z3 = . 3 2

J

` tâp Bai . , , , (Môt a tra l`oi b` ai tâp . sô´ go. i ´y v` . sau tai . trang 129). , ,, . 2.89. Giai phuong tr`ınh: 10z6 + z5 − 47z4 − 47z3 + z2 + 10z = 0. , , ,, `˘ng tâ´t ca nh˜u,ng nghiêm ´,ng minh ra . 2.90. Chu . cua phuong tr`ınh hê. sô´ d¯ô´i ´,ng bâc xu . bô´n az4 + bz3 + cz2 + bz + a = 0,

(a 6= 0) , ,, , ´,c. c´ o thê t´ınh d¯uo. c nh`o bô´n ph´ep t´ınh sô´ hoc a c˘ an thu . v`

,, ´,c d ´,ng hai biê´n Chuong 2. Ða thu ¯ô´i xu

110

, , ,, `˘ng tâ´t ca nh˜u,ng nghiêm ´,ng minh ra . 2.91. Chu . cua phuong tr`ınh hê. sô´ d¯ô´i ´,ng bâc xu am . n˘ az5 + bz4 + cz3 + cz2 + bz + a = 0,

(a 6= 0)

, ,, , ´,c. c´ o thê t´ınh d¯uo. c nh`o bô´n ph´ep t´ınh sô´ hoc a c˘ an thu . v`

, ,, `˘ng nê´u biê´t môt ´,ng minh ra . 2.92. Chu . nghiêm . cua phuong tr`ınh hê. sô´ d¯ô´i , ,, ` ´,ng bâc ˘ng bô´n ph´ep t´ınh sô´ xu au, th`ı nh˜ung nghiêm on lai . s´ . c` . t`ım d¯uo. c ba , ´ c. hoc a c˘ an thu . v` , , ´,ng minh kh´ . 2.93. H˜ ay d` ung d l´ı B´ezout d ach chu ac cua kê´t ¯inh ¯ua ra c´ . , , ´,c hê. sô´ d¯ô´i xu ´,ng bâc luân: ¯a thu . Nh˜ung d . le chia hê´t cho z + 1.

, , ,, , ` tra l`oi bai ` tâp 2.10. Go. i y ´ va . chuong 2 , ,, ˘. t δ1 = x+ y, δ2 = xy v` ˜ cho th` . 2.6. Ða a biê´n d¯ôi hê. phuong tr`ınh d anh ¯a δ1 = 5, ,, , hê. phuong tr`ınh Hê. n` ay c´ o nghiêm . δ1 = 5, δ2 = 6. T`u 2 δ1 − 3δ2 = 7. ´ suy ra hê. d¯a ˜ cho c´ d¯o o nghiêm .  x1 = 2, x2 = 3, y1 = 3; y2 = 2.  δ1 = 5, ,, Hê. phuong tr`ınh δ31 − 3δ1 δ2 = 65. ,, ˜ cho c´ n` ay c´ o nghiêm o nghiêm . δ1 = 5, δ2 = 4. Hê. phuong tr`ınh d¯a .   x1 = 1, x2 = 4, y1 = 4; y2 = 1. ,, , . 2.7. Hê. phuong tr`ınh tro. gi´ up l` a

 4δ1 = 3δ2 , δ1 + δ21 − 2δ2 = 26. , , ,, ,, 2 ´, hai, ta nhân Loai . δ2 t`u phuong tr`ınh thu . d¯uo. c 3δ1 − 5δ1 − 78 = 0. Giai ,, , . 2.8. Hê. phuong tr`ınh tro. gi´ up l` a

, , ,, , `i b` 2.10. Go. i ´y v` a tra lo ai tâp . chuong 2

111

,, ,, phuong tr`ınh bâc ¯uo. c . hai trên ta t`ım d   δ1 = 6, δ1 = − 13 3 , δ2 = 8; δ2 = − 52 9 . , , ,, T`u d¯ây cho nghiêm ¯â`u . cua hê. phuong tr`ınh ban d     √ √ x1 = 2, x2 = 4, x3 = −13+6√377 , x4 = −13−6√377 , y1 = 4; y2 = 2; y3 = −13−6 377 ; y4 = −13+6 377 .  δ21 − 2δ2 + δ1 = 32, ,, , . 2.9. Hê. phuong tr`ınh tro. gi´ up l` a 12δ1 = 7δ2 . , , , ,, ,, , 2 ´ nhâ´t, ta nhân Loai . δ2 t`u phuong tr`ınh thu . d¯uo. c phuong tr`ınh 7δ1 − 17δ1 − , ,, ,, 224 = 0. Giai phuong tr`ınh bâc . hai trên, ta t`ım d¯uo. c   δ1 = 7, δ1 = − 32 7 , δ2 = − 384 δ2 = 12; 49 . , , ,, T`u d¯ây cho nghiêm ¯â`u . cua hê. phuong tr`ınh ban d     √ √ 10 10 x3 = −16+8 x2 = 4, x1 = 3, x4 = −16−8 7 √ , 7 √ , 10 10 −16−8 −16+8 y2 = 3; y1 = 4; y3 = ; y4 = . 7 7  δ2 = 15, ,, , . 2.10. Hê. phuong tr`ınh tro. gi´ up l` a δ1 + δ21 − 2δ2 = 42. , , , ,, ,, , ´ hai, ta nhân Loai ¯uo. c phuong tr`ınh δ21 +δ1 −72 = 0. . δ2 t`u phuong tr`ınh thu . d , ,, ,, Giai phuong tr`ınh bâc . hai trên, ta t`ım d¯uo. c   δ1 = 8, δ1 = −9, δ2 = 15; δ2 = 15. , , ,, T`u d¯ây cho nghiêm ¯â`u . cua hê. phuong tr`ınh ban d     √ √ x1 = 3, x2 = 5, x3 = −9+2√21 , x3 = −9−2√21 , y1 = 5; y2 = 3; y3 = −9−2 21 ; y3 = −9+2 21 . , , ,, ˜ sô´ v` ´,i c´ . 2.11. Quy d ad ac ân δ1 , δ2 ta nhân ¯ô`ng mâu ¯ua vê` hê. vo . d¯uo. c:  δ31 − 3δ1 δ2 = 18δ2 , δ1 = 12.

112

,, ´,c d ´,ng hai biê´n Chuong 2. Ða thu ¯ô´i xu

, , ,, , ,, T`u d¯ây t`ım d¯uo. c δ1 = 12, δ2 = 32. T`u d¯ây cho hai nghiêm . cua hê. phuong tr`ınh ban d ¯â`u   x1 = 4, x2 = 8, y1 = 8; y2 = 4. , , ,, ˜ sô´ v` ´,i c´ . 2.12. Quy d ad ac ân δ1 , δ2 ta nhân ¯ô`ng mâu ¯ua vê` hê. vo . d¯uo. c:  δ1 δ2 = 30, 6δ1 = 5δ2 . ,, Hê. phuong tr`ınh n` ay c´ o nghiêm: .   δ1 = 5, δ1 = −5, δ2 = 6; δ2 = −6. , , ,, T`u d¯ây cho bô´n nghiêm ¯â`u . cua hê. phuong tr`ınh ban d     x1 = 2, x2 = 3, x3 = −6, x4 = 1, y1 = 3; y2 = 2; y3 = 1; y4 = −6. , , ,, ´,i c´ . 2.13. Ðua vê` hê. vo ac ân δ1 , δ2 ta nhân . d¯uo. c:  δ1 = a, δ31 − 3δ1 δ2 = b(δ21 − 2δ2 ). , ,, ,, Biê´n d¯ôi hê. phuong tr`ınh n` ay, ta d¯uo. c (3a − 2b)δ2 = a3 − a2 b. Nê´u sô´ a3 − a2 b ,, ,, 3a − 2b 6= 0, ta nhân . d¯uo. c nghiêm . duy nhâ´t δ2 = 3a − 2b . Trong tru`ong −a3 + 2a2 b ,, , ˜ cho c´ ho. p n` ay hê. phuong tr`ınh d¯a o nghiêm . (nê´u 4(3a − 2b) ≥ 0)   s s 3 2   a −a + 2a b a −a3 + 2a2 b     x1 = + x1 = − , ,   2 4(3a − 2b) 2 4(3a − 2b) s s     a −a3 + 2a2 b a −a3 + 2a2 b     ; y1 = − . y1 = 2 −  4(3a − 2b) 2 4(3a − 2b) , Nê´u 3a − 2b = 0, nhung a3 − a2 b 6= 0, th`ı hê. không c´ o nghiêm. ung . Cuô´i c` , 3 2 ´ ´i δ1 = 0 v` nêu 3a − 2b = 0 v` a a − a b = 0, ngh˜ıa l` a a = b = 0, th`ı vo a , ,`, , , ´ ´ δ2 l` a môt a nghiêm up. Suy ra trong truong ho. p n` ay . sô bât k`ı l` . cua hê. tro. gi´

, , ,, , `i b` 2.10. Go. i ´y v` a tra lo ai tâp . chuong 2

113

, , ,, ˘. p sô´ x, y thoa m˜ moi an x + y = 0 d¯ê`u l` a nghiêm . ca . cua hê. phuong tr`ınh ban d ¯â`u.  δ21 − 2δ2 + 2δ1 = 23, ,, , . 2.14. Hê. phuong tr`ınh tro. gi´ up l` a δ21 − δ2 = 19. , ,, ,, ,, ´, nhâ´t, ta nhân Loai . δ2 t`u phuong tr`ınh thu . d¯uo. c phuong tr`ınh , ,, , , ´ t`ım d¯uo. c hai hê. nghiêm δ21 − 2δ1 − 15 = 0. T`u d up: ¯o . cua hê. tro. gi´   δ1 = −3, δ1 = 5, δ2 = −10; δ2 = 6. , , , ˜ cho c´ ´ hê. phuong tr`ınh d¯a T`u d o nghiêm ¯o .     x1 = −5, x2 = 2, x3 = 2, x4 = 3, y1 = 2; y2 = −5; y3 = 3; y4 = 2. ,, , . 2.15. Hê. phuong tr`ınh tro. gi´ up   δ1 δ2 = 20, δ1 = 5, c´ o nghiêm . 4δ1 = 5δ2 δ2 = 4; , , ,, ´, nghiêm T`u d a ¯o . cua hê. phuong tr`ınh ban d¯â`u l`    √ x1 = 1, x2 = 4, x3 = −5+2√41 , y1 = 4; y2 = 1; y3 = −5−2 41 ; 



δ1 = −5, δ2 = −4.



x4 = y4 =

√ −5− 41 2√ , −5+ 41 . 2

δ41 − 4δ21 δ2 + δ22 = 1153, δ21 − 3δ2 = 33. , ,, ,, , ´, nhâ´t, ta nhân ´ Loai ¯uo. c δ22 − 33δ2 + 32 = 0. T`u d ¯o . δ1 t`u phuong tr`ınh thu . d , ,, , t`ım d¯uo. c hai hê. nghiêm up: . cua hê. tro. gi´   √ δ1 = ±6, δ1 = ± 129, δ2 = 32. δ2 = 1; , ,, ´ hê. phuong tr`ınh d ˜ cho c´ T`u d¯o o nghiêm ¯a .    √ √ √ x2 = 3 − 8, x3 = −3 + 8, x1 = 3 + 8, √ √ √ y1 = 3 − 8; y2 = 3 + 8; y3 = −3 − 8; ,, , . 2.16. Hê. phuong tr`ınh tro. gi´ up l` a

114

,, ´,c d ´,ng hai biê´n Chuong 2. Ða thu ¯ô´i xu

  √ √   129 + 1 129 − 1    √   x = x = , ,  5  6 x4 = −3 − 8, 2 2 √ √ √   y4 = −3 + 8;   129 − 1   y5 = y6 = 129 + 1 ; ; 2 2   √ √   − 129 + 1 x = − 129 − 1 ,    ,  8 x7 = 2 2 √ √       y8 = − 129 + 1 . y7 = − 129 − 1 ; 2 2 x + z = 2, , , ´,ng ˘. t y = −z d¯ua hê. d¯a ˜ cho vê` hê. d . 2.22. Ða Ðua vê` ¯ô´i xu x3 + z3 = 8.  δ1 = 2, , , ´ hê. tro. gi´ up Hê. n` ay c´ o nghiêm ¯o . δ1 = 2, δ2 = 0. T`u d 3 δ1 − 3δ1 δ2 = 8. ,, ta nhân ¯uo. c hai nghiêm . . d   x1 = 2, x2 = 0, z1 = 0; z2 = 2. , Vây a ¯â`u l` . nghiêm . cua hê. ban d   x1 = 2, x2 = 0, y1 = 0; y2 = −2. , ,, ´,ng ˘. t x2 = z v` ˜ cho th` . a biê´n d¯ôi hê. d¯a anh hê. phuong tr`ınh d ¯ô´i xu 2.23. Ða , y + z = 5, `˘ng phu,o,ng ph´ ´,ng v` Giai hê. n` ay ba ap hê. d¯ô´i xu a cuô´i c` ung 3 3 y + z = 65. , ta c´ o nghiêm ¯â`u . cua hê. ban d     x1 = 1, x2 = −1, x3 = 2, x4 = −2, y1 = 4; y2 = 4. y3 = 1; y4 = 1. , x y ,, ˘. t ˜ cho th` . 2.24. Ða = u, = v v` a biê´n d anh hê. phuong tr`ınh ¯ôi hê. d¯a b  a u + v = 1, , ,, ´,ng 1 ´,i δ1 = u + v, δ2 = uv, ta nhân ˘. t ân mo d¯ô´i xu Ða ¯uo. c 1 . d  + = 4. u v

, , ,, , `i b` 2.10. Go. i ´y v` a tra lo ai tâp 115 . chuong 2  δ1 = 1, 1 , hê. δ1 suy ra δ1 = 1, δ2 = . T`u d¯ây cho ta nghiêm . duy nhâ´t  4 = 4, δ2 a b 1 ,, hê. phuong tr`ınh ban d o nghiêm x = ,y = . u = v = . Vây ¯â`u c´ . . 2 2 2 , √ √ ,, ˘. t x = u, y = −v v` ˜ cho th` . 2.25. Ða a biê´n d anh hê. phuong ¯ôi hê. d¯a , u + v = −2uv, ´,ng ´,i δ1 = u + v, δ2 = uv, ta nhân ˘. t ân mo tr`ınh d¯ô´i xu Ða . 2 2 u + v = 20.  , δ1 = −2δ2 , ,, d¯uo. c hê. giai hê. n` ay cho nghiêm . δ21 − 2δ2 = 20,   δ1 = 4, δ1 = −5, δ2 = −2; δ2 = 52 . , √ √ ´, V`ı 0 ≤ x = u, 0 ≥ − y = v, nên phai c´ o δ2 = uv ≤ 0, v`ı thê´ nghiêm . thu , , ´, nhâ´t cho hai nghiêm ´,ng hai không thoa m˜ an. Nghiêm . thu . cua hê. d¯ô´i xu   √ √ u2 = 2 − 6, u1 = 2 + 6, √ √ v1 = 2 − 6; v2 = 2 + 6. , , , ´, hai không phai l` Hê. thu a nghiêm, v`ı n´ o không thoa m˜ an u ≥ 0, v ≤ 0. Chı . , , ´, nhâ´t v` ´ cho ta nghiêm c` on nghiêm a t`u d¯o . thu . cua hê. ban d¯â`u:  √ √ x = u21 = (2 + 6)2 = 10 + 4 6, √ √ y = u22 = (2 − 6)2 = 10 − 4 6. , 1 1 ,, ˘ ˜ cho th` . 2.26. Ða a biê´n d anh hê. phuong tr`ınh ¯ôi hê. d ¯a . t x 4 = u, y 5 = v v` , u3 + v3 = 35, ,, ´,ng ˜ giai trong b` d¯ô´i xu Ðây l` a hê. phuong tr`ınh d¯a ai 2.7 u + v = 5. v` a c´ o nghiêm .   u1 = 2, u2 = 3, v1 = 3; v2 = 2. , T`u d¯ây suy ra nghiêm .   x1 = 16, x2 = 81, y1 = 243; y2 = 32.

,, ´,c d ´,ng hai biê´n Chuong 2. Ða thu ¯ô´i xu

116 r

r 1 1 1 1 ´ u5 = + x, v5 = − x v` + x = u, 5 − x = v. Khi d a ¯o 2 2 2 2  u + v = 1, ,, ´,ng Hê. n` ay c´ o nghiêm ta nhân ¯uo. c hê. d¯ô´i xu . . d u5 + v5 = 1.   u1 = 1, u2 = 0, v1 = 0; v2 = 1. ˘. t . 2.27. Ða

5

, 1 ,, , cua phuong tr`ınh ban d¯â`u T`u u5 = + x cho ta hai nghiêm . 2 1 1 x1 = , x2 = − . 2 2  u2 + v2 = 1, ,, , ´ ng ˘. t sin x = u, cos x = v ta c´ . 2.28. Ða o hê. phuong tr`ınh d¯ô´i xu u3 + v3 = 1.  δ21 − 2δ2 = 1, ,, 3 , Ta c´ o hê. tro. gi´ up Loai . δ2 ta nhân . d¯uo. c δ1 −3δ1 +2 = 0. 3 δ1 − 3δ1 δ2 = 1. ,, ,, ,, Phuong tr`ınh n` ay c´ o nghiêm d ac ¯inh ¯uo. c c´ . l´ı Bêzu t´ınh d . δ1 = 1. Su dung . ,, ˘. p nghiêm nghiêm on lai. ¯uo. c hai ca . c` . Ta nhân . d .   δ1 = −2, δ1 = 1, 3 δ2 = .; δ2 = 0; 2 , ´,i δ1 = 1, δ2 = 0 ta c´ ˘. p cuô´i c` ˘. p Ca ung không cho nghiêm o ca . (u, v) thu. c. Vo nghiêm .   u1 = 1, u2 = 0, v1 = 0; v2 = 1. ,, ,, Ta nhân . d¯uo. c hai hê. phuong tr`ınh   sin x = 1, sin x = 0, cos x = 0; cos x = 1. ,, ´ phuong tr`ınh d¯a ˜ cho c´ Do d o nghiêm ¯o . x=

π + 2kπ, 2

x = 2kπ

(k = 0, ±1, ±2, ...).

, , ,, , `i b` 2.10. Go. i ´y v` a tra lo ai tâp . chuong 2

117

√ √ ´,ng sau ˘. t 4 629 − x = u, 4 77 + x = v. Khi d ´ ta c´ . 2.29. Ða o hê. d ¯o ¯ô´i xu  , u + v = 8, ,, ,, Giai hê. phuong tr`ınh n` ay ta nhân . d¯uo. c 4 4 u + v = 706.  u1 = 3, v1 = 5;



u2 = 5, v2 = 3.

,√ ,, T`u 4 629 − x = u ta nhân ¯uo. c hai nghiêm . d . x1 = 548, x2 = 4. √ √ ´,ng sau ˘. t 3 8 + x = u, 3 8 − x = v. Khi d¯ ´ ta c´ o o hê. d . 2.30. Ða ¯ô´i xu  , u + v = 1, δ1 = 1, , Biê´n d¯ôi vê` hê. tro. gi´ up Hê. n` ay c´ o u3 + v3 = 16. δ31 − 3δ1 δ2 = 16.  δ1 = 1, , ,, ´ ta nhân nghiêm T`u d¯o ¯uo. c hai nghiêm . . d . δ2 = −5.   √ √   1 1 21 21      u1 = 2 + 2 ,  u2 = 2 − 2 , √ √       v1 = 1 − 21 ; v2 = 1 + 21 . 2 2 2 2 √ , , , ,, , T`u 3 8 + x = u ta nhân cua phuong tr`ınh ban d ¯uo. c nghiêm ¯â`u . d . √ √ x1 = 3 21, x2 = −3 21. √ ´,ng ˘. t 1 − x2 = y. Khi d ´ ta c´ . 2.31. Ða o hê. d¯ô´i xu ¯o  x2 + y2 = 1, 1 35 1  + = . x y 12  , δ21 − 2δ2 = 1, , Biê´n d¯ôi vê` hê. tro. gi´ up Hê. n` ay c´ o nghiêm . 12δ1 = 35δ2 .  7   δ1 = 5 ,   δ2 = 12 . 25

 5   δ1 = − 7 ,   δ2 = − 12 . 49

118

,, ´,c d ´,ng hai biê´n Chuong 2. Ða thu ¯ô´i xu

, , , , T`u d¯ây ta t`ım ra bô´n nghiêm, nhung chı c´ o ba nghiêm an y ≥ 0: . . thoa m˜  √    5 4 3 73    x3 = − − x1 = , x2 = , ,    5 5 14 14 √    3 4       y1 = ; y2 = ; y3 = − 5 + 73 . 5 5 14 14 , ,, Nhu vây o nghiêm ¯â`u c´ . phuong tr`ınh ban d . √ 4 3 5 73 x1 = , x2 = , x3 = − − . 5 5 14 14 √ √ ´,ng sau ˘. t 3 10 − x = u, 3 3 − x = −v. Khi d¯o ´ ta c´ . 2.32. Ða o hê. d ¯ô´i xu  , u + v = 1, δ1 = 1, , Biê´n d¯ôi vê` hê. tro. gi´ up Hê. n` ay c´ o 3 3 u + v = 7. δ31 − 3δ1 δ2 = 7.  δ1 = 1, , ,, ´ ta nhân nghiêm T`u d ¯o . . d¯uo. c hai nghiêm . δ2 = −2.   u2 = −1, u1 = 2, v2 = 2. v1 = −1; √ , , ,, ,, T`u 3 10 − x = u ta nhân cua phuong tr`ınh ban d ¯uo. c nghiêm ¯â`u . d . x1 = 2, x2 = 11. 19 − x ,, ˘. t . 2.33. Ða = y, ngh˜ıa l` a 19 − x = xy + y, ta nhân ¯uo. c hê. . d x+1   , x + y + xy = 19, δ1 + δ2 = 19, , Biê´n d¯ôi vê` hê. tro. gi´ up Hê. n` ay c´ o xy(x + y) = 84. δ1 δ2 = 84.   δ1 = 7, δ1 = 12, ˜ hê. cho hai ca ˘. p nghiêm nghiêm Môi . . δ2 = 12; δ2 = 7.     √ √ x1 = 3, x2 = 4, x3 = 6 + 29, x4 = 6 − 29, √ √ y1 = 4; y2 = 3; y3 = 6 − 29; y4 = 6 + 29. ,, Phuong tr`ınh ban d¯â`u c´ o bô´n nghiêm . √ √ x1 = 3, x2 = 4, x3 = 6 + 29, x4 = 6 − 29.  √ x2 + y2 = 17, , 2 ´ ng ˘. t y = 17 − x . Khi d¯o ´ ta c´ . 2.34. Ða o hê. d ¯ô´i xu x + y + xy = 9.

, , ,, , `i b` 2.10. Go. i ´y v` a tra lo ai tâp 119 . chuong 2  , δ21 − 2δ2 = 17, , ´ ` Biên d¯ôi vê hê. tro. gi´ up Hê. n` ay c´ o nghiêm . δ1 + δ2 = 9.   δ1 = 5, δ1 = −7, δ2 = 4; δ2 = 16. , ´ nhâ´t cho nghiêm ´, hai không cho c´ Hê. thu ac nghiêm . x1 = 1, x2 = 4. Hê. thu . , thu. c x, y.  √ x3 + y3 = 35, , 3 3 ´ ´ ng ˘. t y = 35 − x . Khi d¯o ´ ta c´ o hê. d¯ôi xu . 2.35. Ða (x + y)xy = 30. , ,, , , ´ ´ t`ım ra hai nghiêm Giai hê. phuong tr`ınh voi hê. tro. gi´ up v` a sau d ¯o . x1 = 2, x2 = 3. . 2.41. Ta c´ o 5x2 − 6xy + 5y2 = 5S2 − 6δ2 = 5(δ21 − 2δ2 ) − 6δ2 = 5δ21 − 16δ2 = 1 = 5δ21 − 16. (δ21 − z) = δ21 + 4z ≥ 0. 4 . 2.42. Ta c´ o 8(x4 + y4 ) − (x + y)4 = 8S4 − δ41 = 8(δ41 − 4δ21 δ2 + 2δ22 ) − δ41 = 1 1 = 7δ41 − 32δ21 (δ21 − z) + 16 (δ21 − z)2 = 6δ21 z + z2 ≥ 0. 4 16 . 2.43. Ta c´ o x4 + y4 − x3 y − xy3 = x4 + y4 − xy(x2 + y2 ) = S4 − δ2 S2 = = δ41 − 4δ21 δ2 + 2δ22 − δ2 (δ21 − 2δ2 ) = δ41 − 5δ21 δ2 + 4δ22 = 1 1 3 1 = δ41 − 5δ21 . (δ21 − z) + 4. (δ21 − z)2 = δ21 z + z2 ≥ 0. 4 16 4 4 . 2.44. Ta c´ o x2 + y2 + 1 − xy − x − y = δ21 − 3δ2 − δ1 + 1 = 1 = δ21 − 3. (δ21 − z) − δ1 + 1 = 4 1 2 3 1 3 = δ1 − δ1 + 1 + z = ( δ1 − 1)2 + z ≥ 0. 4 4 2 4

,, ´,c d ´,ng hai biê´n Chuong 2. Ða thu ¯ô´i xu

120 ˘. t . 2.45. Ða



x = u,



, ´,c c´ ˘ng thu ´ bâ´t d y = v. Khi d¯o o dang ¯a .

(u + v)8 ≥ 64u2 v2 (u2 + v2 )2 . , , ´,ng minh V`ı u, v l` a nh˜ung sô´ không âm nên ta chı câ`n chu (u + v)4 ≥ 8uv(u2 + v2 ). Thât . vây, . (u + v)4 −8uv(u2 + v2 ) = = δ41 − 8δ2 (δ21 − 2δ2 ) = δ41 − 8δ21 δ2 + 16δ22 = 1 1 = δ41 − 8δ21 . (δ21 − z) + 16 (δ21 − z)2 = z2 ≥ 0. 4 16 . 2.46. Ta c´ o x 3 + y3 − 2



x+y 2

3

1 1 3 (δ − 3δ1 δ2 ) − δ31 = 2 1 8 3 3 3 = δ31 − δ1 δ2 = δ1 z ≥ 0. 8 2 8 =

. 2.47. Ta c´ o x4 + 2x3 y + 2xy3 + y4 − 6x2 y2 = S4 + 2δ2 S2 − 6δ22 = = δ41 − 4δ21 δ2 + 2δ22 + 2δ2 (δ21 − 2δ2 ) − 6δ22 = δ41 − 2δ21 δ2 − 8δ22 = = (z + 4δ2 )2 − 2(z + 4δ2 )δ2 − 8δ22 = z2 + 6δ2 z. , , ´,c hiên nhiên d¯u ˘ng thu ´ ng. V`ı x, y ≥ 0, nên δ2 ≥ 0 v` a z ≥ 0. Bâ´t d ¯a , , , 2 2 . 2.48. Bâ´t phuong tr`ınh d a δ21 − 2δ2 ≥ ¯ua vê` dang . x + y ≥ 2xy, ngh˜ıa l` , , , , , ´ ng minh δ21 ≥ 4δ2 , nhung d ´ ng hiên nhiên 2δ2 . T`u d ay d¯u ¯ây phai chu ¯iê`u n` ´,c d ˜ t´ınh trong v´ı du. theo công thu ¯a . ,, . 2.54. Ta c´ o δ1 = x1 + x2 = −6, δ2 = x1 x2 = 10. Phuong tr`ınh muô´n lâp . 3 3 ´ z2 + pz + q = 0 c´ o c´ ac nghiêm z = x , z = x . V` ı thê 1 . 1 2 2 −p = z1 + z2 = x31 + x32 = δ31 − 3δ1 δ2 = (−6)3 − 3(−6)10 = −36, q = z1 z2 = x31 x32 = δ32 = 1000.

, , ,, , `i b` 2.10. Go. i ´y v` a tra lo ai tâp . chuong 2

121

,, ´ phuong tr`ınh câ`n lâp Do d a z2 + 36z + 1000 = 0. ¯o . l` ,, . 2.55. Ta c´ o δ1 = x1 + x2 = −1, δ2 = x1 x2 = −3. Phuong tr`ınh muô´n lâp . 2 10 10 ´ z + pz + q = 0 c´ o c´ ac nghiêm . z1 = x1 , z2 = x2 . V`ı thê 10 −p = z1 + z2 = x10 1 + x2 = 8 6 2 4 3 2 4 5 = δ10 1 − 10δ1 δ2 + 35δ1 δ2 − 50δ1 δ2 + 25δ1 δ2 − 2δ2 = 4207, 10 10 q = z1 z2 = x10 1 x2 = δ2 = 59409.

,, ´ phuong tr`ınh câ`n lâp Do d a z2 − 4207z + 59049 = 0. ¯o . l` , ,, ´ ta c´ . 2.56. Phuong tr`ınh phai t`ım c´ o dang x2 + px + q = 0. Khi d¯o o . , 5 5 ˜ cho x1 +x2 = 31, x1 +x2 = 1 −p = x1 +x2 = δ1 , q = x1 x2 = δ2 . Gia thiê´t d ¯a , v` a biê´n d ung vê` dang ¯ôi ch´ .   δ51 − 5δ31 δ2 + 5δ1 δ22 = 31, −p5 + 5p3 q − 5pq2 = 31, ngh˜ıa l` a δ1 = 1. p = −1. , ,, ,, ,, 2 ´ ta nhân T`u d¯o a phuong tr`ınh n` ay . d¯uo. c phuong tr`ınh 5q − 5q − 30 = 0 v` , , ,, c´ o nghiêm o hai phuong tr`ınh thoa m˜ an d¯iê`u . q1 = 3, q2 = −2. Nhu vây . ta c´ kiên ai. . d¯â`u b` x2 − x + 3 = 0,

x2 − x − 2 = 0.

, . 2.57. Ta c´ o δ1 = x1 + x2 = a − 2, δ2 = x1 x2 = −(a + 1). Tông b`ınh , ,, phuong cua c´ ac nghiêm o dang . c´ . x21 + x22 = S2 = δ21 − 2δ2 = (a − 2)2 + 2(a + 1) = = a2 − 2a + 6 = (a − 1)2 + 5. , , , , `˘ng tông b`ınh phu,o,ng cua c´ T`u d¯ây thâ´y ra ac nghiêm o gi´ a tri. nho nhâ´t khi . c´ a = 1. , , `˘ng xn + xn l` ´,i moi . 2.58. Ta chı ra ra . sô´ nguyên vo . sô´ tu. nhiên n. 1 2 a môt Thât . vây, . v`ı δ1 = x1 + x2 = 6, δ2 = x1 x2 = 1,

122

,, ´,c d ´,ng hai biê´n Chuong 2. Ða thu ¯ô´i xu

´,c (2.1) c´ th`ı công thu o dang a Sn−1 = 6Sn−2 − Sn−3 . . Sn = 6Sn−1 − Sn−2 v` ,, Ta t´ınh d ¯uo. c Sn : Sn = 35Sn−2 − 6Sn−3 = −Sn−3 + 5(7Sn−2 − Sn−3 ). , `˘ng Sn v` ˘. c không chia hê´t Ta thâ´y ra a Sn−3 d¯ô`ng th`oi chia hê´t cho 5 hoa , cho 5. Nhu vây ac sô´ Sn−3 , Sn−6 , Sn−9 , ... d¯ê`u . nê´u Sn chia hê´t cho 5 th`ı c´ , ˘. c S3 chia hê´t cho 5. Cuô´i c` ung cua d˜ ay trên l` a môt . trong ba sô´ S1 , S2 hoa , , , ˜ chia hê´t cho 5. Nhung ta dê t´ınh d¯uo. c S1 = 6, S2 = 34, S3 = 198, ngh˜ıa l` a , , n n ´i moi không môt ao chia hê´t cho 5, vô l´ı. Suy ra tông Sn = x1 + x2 vo . sô´ n` . , sô´ tu. nhiên n không chia hê´t cho 5. √ √ ˘. t 4 α = u, 4 β = v. Khi d¯o ´ theo d¯iê`u kiên ˜ cho . 2.59. Ða ¯a . d −p = α + β = u4 + v4 , q = αβ = u4 v4 , , , √ √ ˘. t δ1 = u + v, δ2 = uv, ta biê´n d¯ôi mô´i v` a ta phai t´ınh 4 α + 4 β = u + v. Ða quan hê. n` ay vê` dang . −p = δ41 − 4δ21 δ2 + 2δ22 ,

q = δ42 .

,, ,, ,, Loai ung phuong . δ2 ta t`ım d¯uo. c phuong tr`ınh tr` √ √ δ41 − 4 4 qδ21 + 2 q + p = 0. ,, ,, Phuong tr`ınh tr` ung phuong n` ay ´ıt nhâ´t c´ o hai nghiêm a . l` r q √ √ ± 24q+ 2 q−p √ (do 2 q − p ≥ 0) v` a nhiê`u nhâ´t c´ o bô´n nghiêm a . l` r q √ √ ± 24q± 2 q−p p √ √ ,, (khi 2 4 q ± 2 q − p ≥ 0). V`ı δ1 l` a môt . sô´ duong, nên trong bô´n nghiêm . , , ,, ,`, , ´ ´ ´ l` trên chı c´ o môt an d¯o a truong ho. p c´ ac dâu công. Thât . sô duong thoa m˜ . . √ , 2 ´ vây, a p − 4q ≥ 0. V`ı thê |p| ≥ 2 q . t`u α > 0, β > 0 suy ra p < 0, q > 0 v`

, , ,, , `i b` 2.10. Go. i ´y v` a tra lo ai tâp . chuong 2

123

p √ p √ √ √ , ngh˜ıa l` a −p ≥ 2 q v` a 2 q − p ≥ 4 q = 2 4 q. Nhu vây . trong dâ´u , c˘ an phai lâ´y dâ´u (+): r q p √ √ √ 4 4 α + β = 2 4 q + 2 q − p. . 2.64. Ta c´ o 2x4 + 7x3 y + 9x2 y2 + 7xy3 + 2y4 = = 2S4 + 7δ2 S2 + 9δ22 = 2(δ41 − 4δ21 δ2 + 2δ22 ) + 7δ2 (δ21 − 2δ2 ) + 9δ22 = = 2δ41 − δ21 δ2 − δ22 = (δ2 + 2δ21 )(δ21 − δ2 ) = = (xy + 2(x + y)2 )((x + y)2 − xy) = = (2x2 + 5xy + 2y2 )(x2 + xy + y2 ) = = (x + 2y)(2x + y)(x2 + xy + y2 ). , ,, , (Th`ua sô´ cuô´i c` ung không phân t´ıch d ¯uo. c trên tâp . sô´ thu. c). . 2.65. Ta c´ o 18x4 − 21x3 y − 94x2 y2 − 21xy3 + 18y4 = = 18S4 − 21δ2 S2 − 94δ22 = = 18(δ41 − 4δ21 δ2 + 2δ22 ) − 21δ2 (δ21 − 2δ2 ) − 94δ22 = = 18δ41 − 93δ21 δ2 − 16δ22 = (3δ21 − 16δ2 )(6δ21 + δ2 ) = = (3(x + y)2 − 16xy)(6(x + y)2 + xy) = = (3x2 − 10xy + 3y2 )(6x2 + 13xy + 6y2 ) = = (x − 3y)(3x − y)(2x + 3y)(3x + 2y). . 2.66. Ta c´ o 3x4 − 8x3 y + 14x2 y2 − 8xy3 + 3y4 = 3S4 − 8δ2 S2 + 14δ22 = = 3(δ41 − 4δ21 δ2 + 2δ22 ) − 8δ2 (δ21 − 2δ2 ) + 14δ22 = 3δ41 − 20δ21 δ2 + 36δ22 . ,, , ´,c bâc ´,i δ2 không c´ Ta nhân o nghiêm . d¯uo. c tam thu . hai d¯ô´i vo . thu. c. V`ı thê´ ta , ,, ´, hai d¯ê phân t´ıch ra th`u,a sô´ (phu,o,ng ph´ ´p dung a phuong ph´ ap thu ap hê. sô´ .

,, ´,c d ´,ng hai biê´n Chuong 2. Ða thu ¯ô´i xu

124 ˘. t bâ´t d¯inh). Ða .

3x4 − 8x3 y + 14x2 y2 − 8xy3 + 3y4 = = (Ax2 + Bxy + Cy2 )(Cx2 + Bxy + Ay2 ) ,, 2 a A+B+C = Cho gi´ a tri. x = y = 1, ta nhân . d¯uo. c (A + B + C) = 4, ngh˜ıa l` , `˘ng A + B + C = 2. ±2. Theo l´ı luân o thê cho ra . trong phâ`n v´ı du. ta c´

,, ´,i x = 1, y = −1 ta Cho x = 0, y = 1 ta t`ım d¯uo. c A.C = 3. Cuô´i c` ung vo ,, , nhân a A + B + C = ±6. Nhu vây o hê. ¯uo. c (A − B + C)2 = 36 ngh˜ıa l` . d . ta c´ ,, phuong tr`ınh   A + B + C = 2, A − B + C = ±6,   A.C = 3.

, ,, ´, hai ta lâ´y dâ´u (+), th`ı t`u, phu,o,ng Nê´u trong vê´ phai phuong tr`ınh thu ,, ´, nhâ´t B = −2. T`u, d ˘. c l` ´ d˜ê t´ınh d tr`ınh thu aC = ¯o ¯uo. c A = 1, C = 3 (hoa , , , 1, A = 3). Cuô´i c` ung ta nhân ¯uo. c su. phân t´ıch . d 3x4 − 8x3 y + 14x2 y2 − 8xy3 + 3y4 = (x2 − 2xy + 3y2 )(3x2 − 2xy + y2 ). . 2.67. Ta c´ o 2x4 − x3 y + 3x2 y2 − xy3 + 2y4 = 2S4 − δ2 S2 + 3δ22 = = 2(δ41 − 4δ21 δ2 + 2δ22 ) − δ2 (δ21 − 2δ2 ) + 3δ22 = = 2δ41 − 9δ21 δ2 + 9δ22 = (δ21 − 3δ2 )(2δ21 − 3δ2 ) = = ((x + y)2 − 3xy)(2(x + y)2 − 3xy) = = (x2 − xy + y2 )(2x2 + xy + 2y2 ). , ,, , ´,c sau c` Nh˜ung tam thu ung không phân t´ıch d¯uo. c trong tâp ac sô´ thu. c. . c´

, , ,, , `i b` 2.10. Go. i ´y v` a tra lo ai tâp . chuong 2

125

. 2.68. Ta c´ o (x + y)5 − x5 − y5 = δ51 − S5 = δ51 − δ51 + 5δ31 δ2 − 5δ1 δ22 = = 5δ1 δ2 (δ21 − δ2 ) = = 5xy(x + y)(x2 + xy + y2 ). , ´,c d¯a ˜ cho . 2.74. Ta biê´n d ¯ôi theo công thu (x + y)7 − x7 − y7 δ7 − S7 7δ5 δ2 − 14δ31 δ22 + 7δ1 δ32 = 51 = 1 = 5 5 5 (x + y) − x − y δ1 − S5 5δ31 δ2 − 5δ1 δ22 7δ1 δ2 (δ41 − 2δ21 δ2 + δ22 ) 7δ1 δ2 (δ21 − δ2 )2 7 = = (δ21 − δ2 ) = 5 5δ1 δ2 (δ21 − δ2 ) 5δ1 δ2 (δ21 − δ2 ) 7 7 = ((x + y)2 − xy) = (x2 + xy + y2 ). 5 5

=

, ´,c d¯a ˜ cho . 2.75. Ta biê´n d ¯ôi theo công thu     1 1 1 1 2 1 + + + = (a + b)2 a2 b2 (a + b)3 a b =

2 δ1 2 δ2 − 2δ2 1 S2 + 3 = 1 2 2 + 2 = 2 2 δ1 δ2 δ1 δ2 δ1 δ2 δ1 δ2

δ21 − 2δ2 + 2δ2 δ21 1 1 = = 2 = 2 2. 2 2 2 2 a b δ1 δ2 δ1 δ2 δ2 , , ´ c d¯a ˜ cho . 2.76. Ta biê´n d¯ôi theo công thu =

(x + y)4 + x4 + y4 = δ41 + S4 = δ41 + δ41 − 4δ21 δ2 + 2δ22 = = 2δ41 − 4δ21 δ2 + 2δ22 = 2(δ41 − 2δ21 δ2 + δ22 ) = = 2(δ21 − δ2 )2 = 2(x2 + xy + y2 )2 . , ´,c d¯a ˜ cho . 2.77. Ta biê´n d¯ôi theo công thu (x + y)7 − x7 − y7 = δ71 − S7 = δ71 − (δ71 − 7δ51 δ2 + 14δ31 δ22 − 7δ1 δ32 ) = = 7δ51 δ2 − 14δ31 δ22 + 7δ1 δ32 = 7δ1 δ2 (δ41 − 2δ21 δ2 + δ22 ) = = 7δ1 δ2 (δ21 − δ2 )2 = 7xy(x + y)(x2 + xy + y2 )2 .

126

,, ´,c d ´,ng hai biê´n Chuong 2. Ða thu ¯ô´i xu

, , ´,i δ1 = δ2 = 1. Theo công . 2.78. Ta k´ı hiêu a gi´ a tri. cua tông Sn vo . an l` ,, ´,c (2.1) vo ´,i gi´ thu a tri. δ1 = δ2 = 1, ta nhân ¯uo. c . d an = an−1 − an−2 v` a an−1 = an−2 − an−3 . , , ´,c trên cho kê´t qua an = −an−3 . Tu,o,ng tu.,, ˘ng thu Công theo vê´ hai d¯a . ,, an−3 = −an−6 . V`ı thê´ ta nhân . d¯uo. c an = an−6

(n ≥ 6).

, 2 2 ´ ´,c (x + y)n − xn − yn = δn ´,i Ðê d ¯a thu 1 − Sn chia hêt cho x + xy + y , vo , , , ´,c (theo δ1 = δ2 = 1 th`ı d¯iê`u kiên a d¯u l` a tông c´ ac hê. sô´ cua d ¯a thu . câ`n v` , `˘ng không (xem v´ı du. 2.72), ngh˜ıa l` δ1 , δ2 ) phai ba a 1 − an = 0, hay n´ oi c´ ach kh´ ac an = 1. V`ı an = an−6 = an−12 = ... v` a δ1 = δ2 = 1 ta c´ o: a6k+1 = a1 = S1 = 1, a6k+2 = a2 = S2 = −1, a6k+3 = a3 = S3 = −2, a6k+4 = a4 = −a1 = −1, a6k+5 = −a2 = 1, a6k = a6 = −a3 = 2. , , ˘. c Nhu vây, o khi v` a chı khi nê´u n c´ o dang . quan hê. trên, an = 1 c´ . 6k + 1 hoa l` a 6k + 5 (ngh˜ıa l` a 6k ± 1). 2 ´,c P(x) bâc . 2.79. Cho d¯a thu a . m chia hê´t cho x + x + 1, ngh˜ıa l` , , , , 2 ´ c Q(x) l` ´ c bâc ´ P(x) = (x + x + 1)Q(x), o d¯ây d¯a thu a d¯a thu ¯o . m − 2. Khi d    2    x x x x P = + +1 Q . 2 y y y y ,, ´,i ym , nhân Nhân hai vê´ vo . d¯uo. c     x x 2 2 m−2 m = (x + xy + y )y Q y P y y

, , ,, , `i b` 2.10. Go. i ´y v` a tra lo ai tâp 127 . chuong 2     , , ˜ ´,c ym P yx v` ´,a y o, mâu Biêu thu a ym−2 Q yx không chu sô´, ngh˜ıa l` a hai d¯a     ,, x x m 2 2 m ´ ´ ´,c. Nhu, vây thu y P chia hê t cho x + xy + y . Ngu o c l ai nê u y P . . . y y , ,, ´,i y = 1 ta nhân ´ vo chia hê´t cho x2 + xy + y2 , th`ı t`u d ¯o . d¯uo. c P(x) chia hê´t , 2 ´,c P(x) bâc cho x2 + x + 1. Nhu vây, a . d¯a thu . m chia hê´t cho x + x + 1 khi v` , , x m 2 2 ´ c y P y chia hê´t cho x + xy + y . chı khi d¯a thu , , , ´,c x2n + xn yn + y2n chia hê´t Nhu vây a tri. n d ¯ê d¯a thu . ta câ`n phai t`ım gi´ , cho x2 + xy + y2 . Nhung x2n + xn yn + y2n = S2n + δn 2, , , , ´,c n` v` a d¯ê d¯a thu ay chia hê´t cho x2 + xy + y2 th`ı d ad a tông ¯iê`u kiên ¯u l` . câ`n v` , `˘ng 0, vo ´,c n` ´,i δ1 = δ2 = 1, ngh˜ıa l` c´ ac hê. sô´ cua d¯a thu ay (theo δ1 , δ2 ) ba a , , , a2n + 1 = 0, hay l` a a2n = −1. Theo kê´t qua cua b` ai 2.78 suy ra phai c´ o , ˘. c l` ˘. c 2n = 6k + 2 hoa a 2n = 6k + 4. N´ oi c´ ach kh´ ac, phai c´ o n = 3k + 1 hoa , , 2n n 2 ´ c x + x + 1 chia hê´t cho x + x + 1 khi v` n = 3k + 2. Ða thu a chı khi n , , không phai bôi . sô´ cua 3. , , , , ´,c (x + y)n + xn + yn chia . 2.80. Ta phai t`ım nh˜ung gi´ a tri. cua n d ¯ê d ¯a thu , , , ˘ng d¯inh hê´t cho x2 + xy + y2 (xem b` ai 2.79). T`u kê´t qua b` ai sô´ 2.78, kha . , , , ´, cua b` ai to´ an c´ o d¯uo. c voi n = 6k ± 2, n = 6k ± 4. , , , , ´,c co, so, cua x, y l` . 2.81. Ta k´ı hiêu a δ1 , δ2 , cua u, v l` a τ1 , τ2 : ¯a thu . nh˜ung d δ1 = x + y,

δ2 = xy,

τ1 = u + v, τ2 = uv.

Theo d ai ra ¯iê`u kiên . b` δ1 = τ1 v` a δ21 − 2δ2 = τ21 − 2τ2 . , ,, , ´,c bâ´t k`ı ´,i d¯a thu ´ vo T`u d¯ây ta nhân ¯uo. c δ1 = τ1 , δ2 = τ2 . Nhung khi d ¯o . d ϕ(z1 , z2 ), ta c´ o ϕ(δ1 , δ2 ) = ϕ(τ1 , τ2 ). Theo d a môt ¯inh . l´ı 2.1, nê´u P(x, y) l` . , , , , ˜ ´ ´ ´c d ´ ng bât k`ı v` d¯a thu a P(x, y) = ϕ(δ1 , δ2 ) l` a biêu diên cua n´ o theo c´ ac ¯ôi xu ,, , , , ´ ´c d ´ ng co so, th`ı d¯a thu ¯ôi xu P(x, y) = ϕ(δ1 , δ2 ) = ϕ(τ1 , τ2 ) = P(u, v).

,, ´,c d ´,ng hai biê´n Chuong 2. Ða thu ¯ô´i xu

128 ,, , ´,i moi Tru`ong ho. p riêng vo . n

xn + yn = un + vn . , ,, ,´, ˜ cho c´ . 2.82. Phuong tr`ınh d o thê viê´t duo i dang ¯a . δ1 = δ21 − 3δ2 . , ,, , Boi v`ı x, y phai l` a sô´ thu. c, nên 4δ2 ≤ δ21 v` a v`ı thê´ δ21 − δ1 = 3δ2 ≤

3 2 δ , 4 1

ngh˜ıa l` a 1 2 δ − δ1 ≤ 0 hay δ1 (δ1 − 4) ≤ 0. 4 1 , ,, `˘ng 3δ2 = δ2 − δ1 , ta nhân ` ´, ra Tu d ¯ây suy ra 0 ≤ δ1 ≤ 4. Nên nho ¯uo. c . d 1 , , nh˜ung kha n˘ ang sau d¯ây δ1 = 0,

δ2 = 0;

δ1 = 1,

δ2 = 0;

2 δ1 = 2, δ2 = ; δ1 = 3, δ2 = 2; δ1 = 4, δ2 = 4. 3 , , ,, , , ´, ba bi. loai Tru`ong ho. p thu a δ1 , δ2 phai . tr`u do x, y phai nguyên, ngh˜ıa l` , ,, ,, , nguyên. Nh˜ung tru`ong ho. p c` on lai ac hê. phuong tr`ınh sau: . cho ta c´     x + y = 0, x + y = 1, x + y = 3, x + y = 4, xy = 0; xy = 0; xy = 2; xy = 4. , ,, ˘. p nghiêm Giai c´ ac hê. phuong tr`ınh cho ta c´ ac ca .       x1 = 0, x2 = 1, x3 = 0, x4 = 2, x5 = 1, x6 = 2, y1 = 0; y2 = 0; y3 = 1; y4 = 1; y5 = 2; y6 = 2. , , , , ,, `˘ng c´ ˘. p trên thu. c su. l` ac ca a nghiêm Ba ach kiêm tra lai, . ta thâ´y c´ . cua phuong ˜ cho. tr`ınh d¯a , `˘ng d¯a thu ´,ng minh ra ´,c d¯ô´i xu ´,ng . 2.83. Ta phai chu (a + b)n − an − bn − 3(ab)

n−1 2

(a + b)

, , ,, , `i b` 2.10. Go. i ´y v` a tra lo ai tâp . chuong 2

129

, `˘ng 0 (xem v´ı ´,c (theo δ1 , δ2 ) c´ ´,i δ1 = δ2 = 1 l` vo ad o tông c´ ac hê. sô´ ba ¯a thu , ˜ giai), ngh˜ıa l` ´ n = 6k + 3 (b` du. d a 1 − an − 3 = 0 hay an = −2. Do d ai ¯a ¯o , , , ´ ng minh. tâp ¯iê`u câ`n chu . 2.78). T`u gia thiê´t suy ra d . 2.89. Ta c´ o 10z6 + z5 − 47z4 − 47z3 + z2 + 10z = = z(10z5 + z4 − 47z3 − 47z2 + z + 10). , , , ,, ´,c trong ngoa ´,c bâc ´i hê. sô´ tu., do kh´ ˘. c o vê´ phai l` Ða thu a d¯a thu ac không, . le vo , ´ ´,c (theo nên theo d¯inh o chia hêt cho z + 1. Ta thu. c hiên . l´ı 2.4 n´ . chia d¯a thu ,, , so d¯ô` Horner) v` ad ¯uo. c: 10z6 + z5 − 47z4 − 47z3 + z2 + 10z = = z(z + 1)(10z4 − 9z3 − 38z2 − 9z + 10) =       1 1 3 2 − 38 = = z (z + 1) 10 z + 2 − 9 z + z z = z3 (z + 1)(10(δ2 − 2) − 9δ − 38) = = z3 (z + 1)(10δ2 − 9δ − 58). , `˘ng không ta t`ım d¯u,o.,c hai nghiêm ´,c sau c` ˘ng thu Cho d ung ba ¯a . z1 = z2 = ,, z3 = 0, z4 = −1 v` a phuong tr`ınh bâc . hai 10δ2 − 9δ − 58 = 0. , ,, Nghiêm ay l` a δ = −2, δ = . cua phuong tr`ınh n`

29 10 .

, ˜ d¯ê´n phu,o,ng T`u d¯ây dân

tr`ınh 1 1 29 = −2, z + = . z z 10 , ,´, ,, ´,i hai nghiêm ˜ nhân ´, ta nhân C` ung vo cd ¯a ¯o ¯uo. c tâ´t ca 6 nghiêm . d . truo . d . ,, cho phuong tr`ınh ban d¯â`u: z+

z1 = z2 = z3 = 0, z4 = z5 = z6 = −1, z7 =

5 2 , z8 = . 2 5

,, ´,c d ´,ng hai biê´n Chuong 2. Ða thu ¯ô´i xu

130 . 2.90. Ta c´ o

      1 1 +c = az4 + bz3 + cz2 + bz + a = z2 a z2 + 2 + b z + z z = z2 (a(δ2 − 2) + bδ + c) = z2 (aδ2 + bδ + (c − 2a)). , , ,, V`ı a 6= 0 nên z không l` a nghiêm a ta giai ¯â`u v` . cua phuong tr`ınh ban d ,, ´,i δ: phuong tr`ınh d¯ô´i vo aδ2 + bδ + (c − 2a) = 0. , , ,, ˜ biê´t thoa m˜ Nghiêm an bô´n ph´ep t´ınh sô´ hoc a c˘ an ¯a . cua phuong tr`ınh d . v` , , , , , , , , , `˘ng hai phuong tr`ınh ´ c. Nghiêm thu ¯â`u d¯uo. c giai ba . cua phuong tr`ınh ban d z+

1 = δ, z

, ,, ´, nhâ´t hoa ´, hai cua phu,o,ng tr`ınh trên. ˘. c nghiêm od a nghiêm ¯ây δ l` . thu . thu , , ,, ,, , Nghiêm ay c˜ ung t`ım d¯uo. c nh`o bô´n ph´ep t´ınh v` a . cua nh˜ung phuong tr`ınh n` , ´ c. c˘ an thu , ,, ,, . 2.91. Vê´ tr´ ai cua phuong tr`ınh chia hê´t cho z + 1 (d¯inh a thuong . l´ı 2.4) v` , ´,c hê. sô´ d ´,ng bâc ´,i hê. sô´ cua sô´ hang ´,n l` a môt c´ o bâc ¯ô´i xu . d¯a thu . bô´n (vo . . lo ,, ´,c n` ´p dung nhâ´t l` a a). V`ı thê´ b` ai tâp cho d¯a thu ay v` a nghiêm . 2.90 d¯uo. c a . . , , ,, , , ´ ˜ ´ c. ˜ cho biêu diên boi bôn ph´ep t´ınh v` phuong tr`ınh d¯a a c˘ an thu , ´,ng c´ ´,i δ = z + 1z , phu,o,ng tr`ınh hê. sô´ d o sô´ hang . 2.92. Ta d¯ua biê´n mo ¯ô´i xu . , ,, , ` ,, , ´ ´ ´i δ. Nêu z1 l` tu. do kh´ ac không d a ¯uo. c d ¯ua vê phuong tr`ınh bâc . ba d¯ôi vo , ,, 1 ´ ` ˜ biêt cua phuong tr`ınh ban d môt a ¯âu (z1 6= 0), th`ı δ = z1 + z1 l` . nghiêm . d¯a , ,, ,, , , ´ ´ nghiêm d¯inh ¯ua . l´ı B´ezout ta d . cua phuong tr`ınh bâc . ba d¯ôi voi δ. Su dung . ,, ,, , ´ ` ´ phuong tr`ınh bâc a n´ o c´ o nghiêm . ba d¯ôi voi δ vê phuong tr`ınh bâc . hai m` . , , ´ , ˜ ´ ´ biêu diên nhu bôn ph´ep t´ınh sô hoc a c˘ an thuc. . v` , , , , , ´ c hê. sô´ d ´ ng bâc ´,i sô´ hang . 2.93. Ða thu tu. do a0 6= 0) c´ o thê ¯ô´i xu . le (vo . ,´, viê´t duo i dang . P(z) = a0 (z2k+1 + 1) + a1 (z2k + z) + a2 (z2k−1 + z2 ) + · · ·

, , ,, , `i b` 2.10. Go. i ´y v` a tra lo ai tâp . chuong 2

131

, ´,c tai T´ınh gi´ a tri. cua d ¯a thu . z = −1: P(−1) = a0 (−1 + 1) + a1 (1 − 1) + a2 (−1 + 1) + · · · = 0.

, ´,c d ˜ cho chia hê´t cho z + 1. Nhu vây ¯inh ¯a thu ¯a . l´ı B´ezout d . P(−1) = 0. Theo d

, , CHUONG

3

, , , ´ ´ c cua cac ´ da ˜ y sô´ ˘ ng thu Bât d¯a , , ´ tu. dô ¯ `ng thu

, , ˘. p th´ ˘. c ba sô´ . . . . . . . . . . . . . . . . . . . . . . . . . . . . . . . . . . . . . . . . . . . . . u tu. hai hoa 3.1. Ca , ´ hoa ´ .......................................................... 3.2. Tông quat ,, , , ´ d˜ 3.3. Su dung cac ay sô´ dô u tu. . . . . . . . . . . . . . . . . . . . . . . . . . . . . . . . . . . . . . . . ¯ `ng th´ . , , ˘ ng th´ 3.4. Chuyên dê uc Karamata . . . . . . . . . . . . . . . . . . . . . . . . . . . . . . ¯ ` vê` bâ´t d¯a

, ´,c Karamata . . . . . . . . . . . . . . . ˘ng thu 3.4.1. H` am lô`i v` a bâ´t d ¯a , ,, , ´ c Karamata . . . . . . . . . . . . . . . . . ˘ng thu 3.4.2. Su dung bâ´t d¯a . , , ´ c kh´ ˘ng thu ac . . . . . . . . . . . . . . . . . . . . . . . . 3.4.3. Môt . sô´ bâ´t d¯a , , ´,c trong tam gi´ ˘ng thu 3.4.4. Nh˜ung bâ´t d¯a ac . . . . . . . . . . . . . .

, , ,, , ` tra l`oi bai ` tâp 3.5. Go. i y´ va . chuong 3 . . . . . . . . . . . . . . . . . . . . . . . . . . . . . . . . . . . . . .

134 143 149 156

156 163 166 168 171

,, ,, ,´, ,, ˜ su dung c, ta d¯a râ´t nhiê`u d¯ai Trong hai chuong truo . . luo. ng bâ´t biê´n , , ,, , ´,p b` ˘. c ˜ ra v` ´ ta giai d¯uo. c nh˜ung lo trong b` ai to´ an d a du. a v` ao d ai to´ an d¯a ¯a ¯o , , , , ` ´ ´ ´ c m` ´ ng minh môt ˘ng thu ´ a trong d¯o trung. Chuyên d ay ta chu ¯ê n` ¯a . sô bât d , , , ´ ` o nhiêu b` c´ ac biên tham gia c´ o vai tr` o nhu nhau. C´ ai to´ an c´ o nh˜ung biêu , , , , , ,, , , ´ ´ ´ c không d¯ôi d ´i su. chuyên d thu ac biên cho nhau. Trong tru`ong ho. p ¯ôi vo ¯ôi c´ , ,, ,, , ´˘p xê´p theo môt ´, tu., n` ´ nhu vây, ao d¯o . ngu`oi ta thu`ong cho nh˜ung biê´n sa . thu , , , , ,, , ´ ´ ´i kêt luân m` a không anh huong g`ı to ai to´ an. Viêc ac b` ai to´ an bât . cua b` . giai c´ , , , , , , , ´ ´ ´ ´ ´ c c´ ´ tu. n` ˘ng thu ˘p xêp theo môt ´ l` d¯a o nh˜ung biên sô d¯uo. c sa ao d¯o a râ´t . thu

133 , , ,, thuân a c´ o thê tông qu´ ay, ta nghiên at h´ oa d¯uo. c. Trong chuyên d ¯ê` n` . tiên . v` , , , , ,, ´ , ´ u viêc ´ ng minh bâ´t d ´ c vo ´i k˜ı thuât ˘ng thu ˘p cu ac bô. sô´ d ¯a ¯uo. c sa . chu . tao . ra c´ , , , , , , , ´ tu. v` ´ ng minh môt ´ c nôi ˘ng thu ´ nh`o k˜ı thuât thu a sau d¯o ay chu . n` . sô´ bâ´t d¯a , tiê´ng nhu : Cauchy, Chebyshev, ... , ,, ,, ,, ao Ta lâ´y môt o t´ınh châ´t thu. c tê´: Mu`oi ngu`oi xê´p h` ang lâ´y nuo´c v` . v´ı du. c´ , ,´, , ˜ ´ ´ ` ` `i hoi th`oi gian lây d c´ ac th` ung. Môi th` ung d ac nhau. Cân sa ˘p xê´p ¯o ¯ây nuoc kh´ , , , , ,, , , , , , , , , , a thu´ tu. h` ang d ao d ¯o. i nhu thê´ n` ¯ê tô ho. p th`oi gian ch`o d ¯o. i cua mu`oi ngu`oi l` , ´ nho nhât? , , , , ´, tu., t˘ Môt ac cho ta l`oi khuyên l` a xê´p theo thu ang cua th`oi gian . tru. c gi´ , , , ,´, ,´, l` am d¯â`y nuo c c´ ac th` ung nuo c. Ðê kiêm nghiêm ac n` ay ta xem x´et . cam gi´ , ,´, , nhu sau: K´ı hiêu ac th`oi gian câ`n thiê´t l` am d¯â`y nuo c . T1 < T2 < ... < T10 c´ c´ ac th` ung. , , ,, ,´, ,, ´ , , , ˘p nhu l`oi khuyên trên th`ı tô ho. p th`oi gian Nê´u ngu`oi lâ´y nuo c d¯uo. c sa , , , , ´, tu., h` ´,i nh˜u,ng thu ch`o d a T = 10T1 + 9T2 + · · · + T10 . Vo ang d¯o. i ¯o. i phai cho l` , , , , , , , , kh´ ac, tô ho. p cua th`oi gian ch`o d¯o. i s˜e l` a S = 10S1 + 9S2 + · · · + S10 , o d ¯ây , (S1 , S2 , ..., S10 ) l` a môt an vi. cua (T1 , T2 , ..., T10 ). . ho´ , Hai bô. 10 sô´ kh´ ac nhau luôn luôn tô`n tai . môt . chı sô´ i sao cho Si 6= Ti . 0 0 ´,i sô´ j > i n` ´ Sj = Ti < Si vo ´. Ta g´ Khi d¯o ao d¯o an lai a . Si = Sj , Sj = Si v` , 0 0 0 0 0 ´ ˘. t S = 10S1 + 9S2 + · · · + S10 . Khi d ´ Sk = Sk voi k 6= i, j. Ða ¯o S − S 0 = (11 − i)(Si − Si0 ) + (11 − j)(Sj − Sj0 ) = = (Si − Sj )(j − i) > 0. , , , , , , ´, tu., theo c´ ´ su. thay d Do d ach trên cho kê´t qua th`oi gian ch`o giam d ¯o ¯ôi thu ¯i , , thu. c su. . , 0 ´, tu., trên Nê´u (S10 , S20 , ..., S10 ) 6= (T1 , T2 , ..., T10 ), th`ı qu´ a tr`ınh thay d ¯ôi thu , , ,, ´,i (T1 , T2 , ..., T10 ) nhiê`u nhâ´t l` ˘. p lai. d¯uo. c la a sau 9 lâ`n. V`ı tô ho. p . Ta s˜e d¯at . to , , , , , ` ˜ bu,o ´, tu.,, nên T thu.,c ´,c thu.,c hiên th`oi gian ch`o d¯o. i d ¯êu giam sau môi . d¯ôi thu , , , , , su. l` a gi´ a tri. nho nhâ´t cho th`oi gian ch`o d ¯o. i.

134

, , ,, ´,c cua c´ ´, tu., ˘ng thu Chuong 3. Bâ´t d¯a ac d˜ ay sô´ d ¯ô`ng thu

, ,´, Truo c khi d¯i v` ao tông qu´ at h´ oa b` ai to´ an n` ay ta x´et b` ai to´ an c´ o sô´ biê´n , , , ´ ng dung ´ıt hon v` a c´ ac u cua n´ o. .

, , ˘. p th´ ˘. c ba sô´ 3.1. Ca u tu. hai hoa , , , , , , ´,c d ˘ng thu ´p Xuâ´t ph´ at t`u môt a d˜ê hiêu ch´ ung ta c´ o thê a ¯a ¯on gian v` . bâ´t d , dung v` a giai h` ang loat ai to´ an. . . b` ˜,ng sô´ thu.,c. Nê´u a1 ≥ a2 v` Mênh d a nhu a ¯ê` 3.1. Cho a1 , a2 , b1 , b2 l` . b1 ≥ b2 , th`ı

a1 b1 + a2 b2 ≥ a1 b2 + a2 b1 . , , , , Bâ´t d ˘ng thu´c xay ra dâ´u b˘ ang khi v` a chı khi a1 = a2 hoa ˘. c b1 = b2 . ¯a , ´,ng minh. Chu ´,ng minh d¯u,o.,c suy ra t`u, bâ´t d ´,c (a1 − a2 )(b1 − ˘ng thu Chu ¯a , , , ´,c xay ra khi v` ˘ng thu ˘. c b1 = b2 . b2 ) ≥ 0. D˜ê thâ´y d¯a a chı khi a1 = a2 hoa

J

, ´,i a1 ≤ a2 v` ´ ng vo Mênh d¯ê` trên c` on d¯u a b1 ≤ b2 . Ðê thuân . . tiên, . hai bô. , , , , sô´ (a1 , a2 ) v` a (b1 , b2 ) goi ad ung thoa m˜ an d¯ô`ng th`oi ¯ô`ng thu´ tu. , nê´u ch´ . l` , ˘. c d a1 ≥ a2 v` a b1 ≥ b2 hoa a b1 ≤ b 2 . ¯ô`ng th`oi a1 ≤ a2 v` , ´, ´,i nh˜u,ng bô. (a1 , a2 ) hoa ˘. c (b1 , b2 ) d T`u mênh d¯ê` 3.1 cho thâ´y vo ¯ô`ng thu . , , , , , `˘ng c´ tu. , ta thay d¯ôi nh˜ung sô´ hang ach thay d ¯ ôi . trong tông a1 b1 + a2 b2 ba , , , , , ,, , , ,, , , c´ ac phâ`n tu c` ung bô. th`ı tông s˜e nho d¯i. Chı t`u nh˜ung ´y tuong d¯on gian nhu , , , ´p dung vây, o thê a giai c´ ac b` ai to´ an. . nhung ta c´ . , , , V´ı du. 3.1. Chu´ng minh bâ´t d ˘ng thu´c ¯a a3 + b3 ≥ a2 b + b2 a,

,, ˜,ng sô´ du,o,ng. a nhu od ¯ây a, b l`

, , , `˘ng sô´ c` `,i giai. Môt ´,n ho,n hoa ˘. c ba a b s˜e lo on lai. Lo . trong nh˜ung sô´ a v` . Biêu , , , , ´,c o, vê´ tr´ ´,c d¯a ´,ng vo ´,i biê´n ˘ng thu ˜ cho d¯ê`u d thu ai v` a vê´ phai cua bâ´t d¯a ¯ô´i xu

´, tu., hai hoa ˘. p thu ˘. c ba sô´ 3.1. Ca

135

, , , ˜ , ´,c không thay d ˘ng thu (ngh˜ıa l` a bâ´t d¯a cua a v` a b cho ¯ôi khi ta thay d¯ôi chô , , , , 2 2 ´ a ≥ b v` nhau), nên ta chı câ`n x´et tru`ong ho. p a ≥ b. Khi d¯o a suy ra bô. ,, , , ,, 2 2 ´ tu. . Su dung sô´ (a, b) v` a (a , b ) l` a d¯ô`ng thu mênh d a ta nhân ¯ê` 3.1 v` ¯uo. c . . . d

J

a3 + b3 = a2 a + b2 b ≥ a2 b + b2 a.

˜,ng sô´ du,o,ng. Chu´,ng minh bâ´t V´ı du. 3.2 (IMO 1964). Cho a, b, c l` a nhu , , d¯a ˘ng thu´c a3 + b3 + c3 + 3abc ≥ a2 b + ab2 + b2 c + bc2 + c2 a + ca2 . , , `,i giai. Bâ´t d¯a ´,c d ´,ng theo c´ ˘ng thu Lo ac thông sô´ a, b, c (ngh˜ıa l` a bâ´t ¯ô´i xu , , , ˜ , ,, , ´ c không biê´n d¯ôi khi ta thay d ˘ng thu d¯a ac biê´n). Boi thê´ ta c´ o ¯ôi chô cua c´ , ` ˘ng a ≥ b ≥ c. Khi d ´ thê cho ra ¯o a3 + b3 + c3 + 3abc = a(a2 + bc) + b(b2 + ac) + c(c2 + ab). ´, tu.,(a, b) v` ´p dung Ta a mênh d¯ê` 3.1 cho c´ ac bô. sô´ d¯ô`ng thu a (a2 + bc, b2 + . . , , , , , ´ c trên l` ˘ng thu ac) c˜ ung nhu (c, a) v` a (c, b). Vê´ bên phai cua d¯a a a(a2 +bc)+b(b2 +ac)+c(c2 +ab) ≥ a2 b+ca2 +ab2 +b2 c+bc2 +c2 a.

J

, V´ı du. 3.3 (IMO 1984). Cho a, b, c l` ad ai c´ ac canh cua môt ac bâ´t ¯ô. d` . . tam gi´ , k`ı. Chu´ng minh ra ˘`ng a2 b(a − b) + b2 c(b − c) + c2 a(c − a) ≥ 0. ,, , , H˜ ay x´ ac d ˘`ng. ¯inh . tru`ong ho. p xay ra dâ´u ba , , , ,, ,, `,i giai. Bâ´t d¯a ´,c d¯â`u b` ´,c ´,i bâ´t d¯a ˘ng thu ˘ng thu ai tuong d¯uong vo Lo a3 b + b3 c + c3 a ≥ a2 b2 + b2 c2 + c2 a2 .

(3.1)

, , ´,c n` ´,c ˘ng thu ˘ng thu Bâ´t d¯a ay l` a v` ong quanh theo c´ ac biê´n (ngh˜ıa l` a bâ´t d ¯a , , ˜ v` không thay d ong tr` on theo c´ ac biê´n). Do vây, ¯ôi khi d¯ôi chô . không mâ´t , , `˘ng trong bô. ba (a, b, c) sô´ a l` ´,n nhâ´t. T`u, t´ınh tông qu´ at ta c´ o thê cho ra a lo

, , ,, ´,c cua c´ ´, tu., ˘ng thu Chuong 3. Bâ´t d¯a ac d˜ ay sô´ d ¯ô`ng thu

136

, ´ d˜ê thâ´y trong bô. (bc, ac, ab) sô´ nho nhâ´t l` d¯o a bc. Trong bô. (a2 + bc, b2 + , , ,, ´,n nhâ´t l` ac, c2 + ab) sô´ lo a a2 + bc, d¯iê`u n` ay d ¯uo. c suy ra t`u nh˜ung bâ´t , , , ´,c tam gi´ ´,i ch´ ˘ng thu d¯a ac (vo u ´y a, b, c l` a nh˜ung d¯ô. d` ai c´ ac canh cua môt . . tam gi´ ac) a2 + bc ≥ b2 + ac ⇔ (a − b)(a + b − c) ≥ 0,

a2 + bc ≥ c2 + ab ⇔ (a − c)(a + c − b) ≥ 0. , ´,c Ta x´et biêu thu

A = bc(bc + a2 ) + ac(ac + b2 ) + ab(ab + c2 ). , , ˜ , , , , ´,c o, sô´ hang ´, hai v` ´, ba ta d¯ôi chô Nê´u trong biêu thu a thu cua nh˜ung th`ua . thu , ,, ´,n ho,n, v`ı bô. sô´ sô´ ac v` a ab, th`ı theo mênh d¯ê` 3.1 ta s˜e nhân . . d¯uo. c tông lo , ´, tu., (d (ac, ab) v` a (ab + c2 , ac + b2 ) l` a d¯ô`ng thu ay d˜ê kiêm tra, ban ¯iê`u n` . , ´ d¯oc am). Khi d ¯o . tu. l` A ≤ bc(bc + a2 ) + ab(ac + b2 ) + ac(ab + c2 ). , , , , ´, nhâ´t v` ´, hai Bây gi`o ta lai thu a thu . thay d¯ôi nh˜ung th`ua sô´ trong sô´ hang . , ´,c v` ´,i nh˜u,ng bô. sô´ (bc, ab) v` ˘ng thu ´p dung trong d a a mênh d¯ê` 3.1 vo a ¯a . . , , 2 2 ´ tu. : (ac + b , bc + a ) l` a d¯ô`ng thu A ≤ ab(bc + a2 ) + bc(ac + b2 ) + ac(ab + c2 ).

,, Ta nhân ¯uo. c . d

A = b2 c2 + c2 a2 + a2 b2 + a2 bc + ab2 c + abc2 ≤ a3 b + b3 c + c3 a + a2 bc + ab2 c + abc2 . , , ´,c (3.1). ˘ng thu T`u d¯ây suy ra bâ´t d ¯a , , , , , ˜ ˜, , ´,c chı xay ra ˘ng thu ˜ thu. c hiên Su. d¯ôi chô nhung sô´ hang d¯a . . hai lâ`n. Ða , ,, , ˘. c l` trong nh˜ung tru`ong ho. p: 1) ac = ab hoa a ac + b2 = ab + c2 v` a 2) ˘. c l` ab = bc hoa a bc + a2 = ac + b2 . , , , , , ´,c chı xay ra khi v` ˘ng thu ´ suy ra a = b = c, ngh˜ıa l` T`u d¯o ad a chı khi ¯a ˜ cho l` tam gi´ ac d¯a a d¯ê`u.

J

´, tu., hai hoa ˘. p thu ˘. c ba sô´ 3.1. Ca

137

, ˜,ng d ai c´ ac canh cua tam gi´ ac bâ´t k`ı. H˜ ay V´ı du. 3.4. Nê´u a, b, c l` a nhu ¯ô. d` . , , , chu´ng minh bâ´t d ˘ng thu´c ¯a   a b c b a c 2 + + ≥ + + + 3. b c a c b a , , , , `,i giai. Ða ´,c l` ˘. t vê´ tr´ ˘ng thu Lo ai cua bâ´t d¯a a B v` a ta biê´n d anh dang: ¯ôi th` .   2 2 2 a b c a + bc c + ab b + ac B=2 + + + + . = b c a ab ac bc , , ´,c c´ ˘ng thu Bâ´t d¯a o t´ınh v` ong quanh theo c´ ac biê´n. Ta c´ o thê cho a l` a canh . , , , , , ´n nhâ´t. Vo ´i b, c ta x´et hai tru`ong ho. p: b ≥ c v` lo a b ≤ c.   1 1 ,`, , , ˜ ` ´ ˘ng nh˜ung bô. Truong ho. p b ≥ c: Ta dê thây ra , v` a (c2 + ac bc , 1 1 , , ´, tu.,. Khi d¯o ´ ta thay d¯ôi nh˜ung th`ua sô´ v` a ab, b2 + ac) l` a d¯ô`ng thu ac bc , , , , ˜ d ´, hai v` ´, ba trong tông B s˜e dân a thu trong sô´ hang ¯ê´n tông nho hon. . thu   c a a bc b2 a2 + bc c2 + ab b2 + ac + + = + + +1+ + B≥ . ab bc ac b c b ab ac   ,, 1 1 , , ˜ ´, tu., v` ` Bây gio lai a (bc, b2 ) l` a d¯ô`ng thu a su dung . x´et nhung bô. ab , ac v` . a b ,, ´,i ch´ mênh d ung vo u ´y + ≥ 2, ta nhân ¯ê` 3.1 c` . d¯uo. c . b a c a a b a c b B ≥ + + + 1 + 2 ≥ + + + 3. b c b a c b a , ,`, , , , , Truong ho. p b ≤ c: Thay d ¯ôi nh˜ung th`ua sô´ câ`n thu. c hiên . trong sô´ hang . , , , , ´ nhâ´t v` ´ hai cua tông B, ta c˜ ´,ng minh. thu a thu ung suy ra d¯iê`u câ`n chu , ,, , `˘ng mênh Ta thâ´y ra d¯ê` 3.1 d at biêu cho hai biê´n sô´, nhung n´ o c´ o ¯uo. c ph´ . , , , , , , , ´ c c´ ˘ng thu ´p dung o ba biê´n sô´. Bây gi`o ta x´et tuong thê a cho nh˜ung bâ´t d¯a . , tu. mênh d ¯ê` 3.1 cho bô. ba sô´. . , , Hai bô. ba sô´ α = (a1 , a2 , a3 ) v` a β = (b1 , b2 , b3 ) goi ad ¯ô`ng thu´ tu. , . l` , , ˘. c d¯ô`ng nê´u ch´ ung thoa m˜ an d a b1 ≥ b2 ≥ b3 hoa ¯ô`ng th`oi a1 ≥ a2 ≥ a3 v` , th`oi a1 ≤ a2 ≤ a3 v` a b1 ≤ b2 ≤ b3 . Hai bô. ba sô´ α = (a1 , a2 , a3 ) v` aβ=

J

138

, , ,, ´,c cua c´ ´, tu., ˘ng thu Chuong 3. Bâ´t d¯a ac d˜ ay sô´ d ¯ô`ng thu

, , , , ung thoa m˜ an a1 ≥ a2 ≥ a3 (b1 , b2 , b3 ) goi a nghich ¯ao thu´ tu. , nê´u ch´ . d . l`

˘. c a1 ≤ a2 ≤ a3 v` v` a b1 ≤ b2 ≤ b3 hoa a b1 ≥ b2 ≥ b3 . , , , ´˘p xê´p ch´ V´ı du: o thê sa ung . Cho ba sô´ nguyên duong a, b, c bâ´t k`ı th`ı ta c´ , , ´ ´ ˘. c a ≤ b ≤ c). Khi d¯o ´ hai bô. ba theo thu tu. a ≥ b ≥ c (hoa  c)  sô (a, b, 1 1 1 , , 3 3 3 ´ tu. , c` , , l` a v` a (a , b , c ) l` a d¯ô`ng thu on hai bô. ba sô´ (a, b, c) v` a a b c , ´, tu.,. nghich ¯ao thu . d , Mênh d¯ê` 3.2. Cho hai bô. ba sô´ thu. c (a1 , a2 , a3 ) v` a (b1 , b2 , b3 ). K´ı hiêu . . , ,, S(i1 ,i2 ,i3 ) = a1 bi1 + a2 bi2 + a3 bi3 , o d ây (i , i , i ) l` a ho´ a n v i c ua (1, 2, 3). ¯ 1 2 3 . ´ Khi d o ¯ , , , , ´ng (a) Nê´u hai bô. ba sô´ l` ad ac bâ´t d ˘ng thu´c sau d ¯ô`ng thu´ tu. th`ı c´ ¯a ¯u S(1,2,3) ≥ S(i1 ,i2 ,i3 ) ≥ S(3,2,1) . , , , , , ´ng (b) Nê´u hai bô. ba sô´ l` a nghich ac bâ´t d ˘ng thu´c sau d ¯ao thu´ tu. th`ı c´ ¯a ¯u . d S(1,2,3) ≤ S(i1 ,i2 ,i3 ) ≤ S(3,2,1) . , , , , C´ ac bâ´t d ˘ng thu´c trên xay ra dâ´u ba ˘`ng khi v` a chı khi a1 = a2 = a3 hoa ˘. c ¯a , , , (i1 , i2 , i3 ) = (1, 2, 3) (d oi bâ´t d ˘ng thu´c vê´ tr´ ai) v` a (i1 , i2 , i3 ) = (3, 2, 1) ¯ô´i v´ ¯a , , , , (d oi bâ´t d ˘ng thu´c vê´ phai). ¯ô´i v´ ¯a , Ch´ u ´y: Ta hiêu hai bô. sô´ (x1 , x2 , ..., xn ) = (y1 , y2 , ..., yn ) khi xi = yi

´,i i = 1, 2, ..., n. vo , , ´,ng minh. (a) Gia su, hai bô. sô´ d ´, tu.,: a1 ≤ a2 ≤ a3 v` Chu a b1 ≤ ¯ô`ng thu , ,, , ` ` ` ` ˜ ´, tu.,: ´p dung b2 ≤ b3 . Tu mênh d¯ê 3.1, ta a lân luo. t cho nhung bô. d ¯ông thu . . , (a2 , a3 ) v` a (b2 , b3 ); (a1 , a2 ) v` a (b1 , b3 ); (a2 , a3 ) v` a (b1 , b2 ), suy ra nh˜ung , ´,c ˘ng thu bâ´t d¯a S(1,2,3) = a1 b1 + a2 b2 + a3 b3 ≥ a1 b1 + a2 b3 + a3 b2 ≥ a1 b3 + a2 b1 + a3 b2 ≥ a1 b3 + a2 b2 + a3 b1 = S(3,2,1) , S(1,2,3) ≥ S(1,3,2) ≥ S(3,1,2) ≥ S(3,2,1) .

´, tu., hai hoa ˘. p thu ˘. c ba sô´ 3.1. Ca

139

, ,, , ,, , ´,c sau ˘ng thu Tuong tu. ta c˜ ung nhân ¯uo. c nh˜ung bâ´t d¯a . d S(1,2,3) ≥ S(2,1,3) ≥ S(2,3,1) ≥ S(3,2,1) .

, , ,, `˘ng c´ ´,i s´ Nhu vây au tông d¯uo. c tao ac bô. a1 ≤ a2 ≤ a3 v` a b1 ≤ b2 ≤ b3 . ba . vo , , , , ´n nhâ´t l` ta kê´t luân a S(1,2,3) = a1 b1 + a2 b2 + a3 b3 , tông nho nhâ´t . tông lo l` a S(3,2,1) = a1 b3 + a2 b2 + a3 b1 . Suy ra S(1,2,3) ≥ S(i1 ,i2 ,i3 ) ≥ S(3,2,1) . , , , ´,c xay khi v` ˘ng thu Ban a chı khi (i1 , i2 , i3 ) = (1, 2, 3) (d¯ô´i ¯a . d¯oc . d˜ê thâ´y d , , ´,c vê´ tr´ ´,c ´,i bâ´t d ´,i bâ´t d¯a ˘ng thu ˘ng thu vo ai) v` a (i1 , i2 , i3 ) = (3, 2, 1) (d¯ô´i vo ¯a , vê´ phai). , , ´, (b) Nê´u nh˜ung bô. ba sô´ (a1 , a2 , a3 ) v` a (b1 , b2 , b3 ) l` a nghich . d¯ao thu , , ´, tu. , th`ı (a1 , a2 , a3 ) v` a (b10 , b20 , b30 ) = (b3 , b2 , b1 ) l` a nh˜ung bô. ba d ¯ô`ng thu , , tu. . Theo phâ`n (a), tông S(1,2,3) = a1 b1 + a2 b2 + a3 b3 = a1 b30 + a2 b20 + a3 b10 , , l` a nho nhâ´t v` a tông S(3,2,1) = a1 b3 + a2 b2 + a3 b1 = a1 b10 + a2 b20 + a3 b30

J

´,ng minh. ´,n nhâ´t. Tu,o,ng tu., phâ`n (a) suy ra d¯iê`u câ`n chu l` a lo , , ´, tu., ´,i bô. ba d T`u mênh d ut ra nh˜ung ch´ u ´y sau d¯ây: Vo ¯ê` 3.2 ta r´ ¯ô`ng thu . , ´,n nhâ´t. (a1 , a2 , a3 ) v` a (b1 , b2 , b3 ), tông S(1,2,3) = a1 b1 + a2 b2 + a3 b3 l` a lo , , , , , , ˜ d¯ê´n Su. chuyên d a nh˜ung sô´ trong c` ung môt ¯ôi nh˜ung th`ua sô´ l` . bô. ba dân , , , tông nho hon. , , ´p dung ˜ x´et: mênh d¯ê` 3.2 cho môt Bây gi`o ta c´ o thê a . . . sô´ v´ı du. d¯a , , ´, hai cho v´ı du. 3.2: Viê´t lai ´,c câ`n chu ´,ng minh ˘ng thu C´ ach giai thu . bâ´t d¯a ,´, duo i dang: . a2 (b + c − a) + b2 (c + a − b) + c2 (a + b − c) ≤ 3abc

, , ,, ´,c cua c´ ´, tu., ˘ng thu Chuong 3. Bâ´t d¯a ac d˜ ay sô´ d ¯ô`ng thu

140

, ,, ´ v` a gia su a ≥ b ≥ c. Khi d ¯o c(a + b − c) ≥ b(c + a − b) ≥ a(b + c − a),

, ´,c thu ´, ˘ng thu v`ı c(a + b − c) − b(c + a − b) = (b − c)(b + c − a) ≥ 0, bâ´t d¯a ´,ng minh tu,o,ng tu.,. Nhu, vây hai chu a (c(a + b − c), b(c + . hai bô. sô´ (a, b, c) v` , , ` ` ´ tu. . Theo mênh a − b), a(b + c − a)) d d o ¯ông thu ¯ê 3.2, ta c´ . a2 (b + c − a) + b2 (c + a − b) + c2 (a + b − c) ≤ ba(b + c − a) + cb(c + a − b) + ac(a + b − c), 2

a (b + c − a) + b2 (c + a − b) + c2 (a + b − c) ≤ ca(b + c − a) + ab(c + a − b) + bc(a + b − c).

, , , , ´,c trên, vê´ phai r´ ˘ng thu ´, Công ut gon on 6abc. T`u d¯o . theo vê´ cua hai bâ´t d¯a . c` , , , ´ c câ`n chu ´ ng minh. ˘ng thu suy ra bâ´t d ¯a , , , ´ hai cho v´ı du. 3.3: T`u, a, b v` C´ ach giai thu a c l` a c´ ac canh cua tam gi´ ac, ta . , , ,, c´ o thê gia su a ≥ b ≥ c. ´,ng ´ a(b + c − a) ≤ b(c + a − b) ≤ c(a + b − c). Ðiê`u n` Khi d¯o ay chu 1 1 1 , , ≤ ≤ . Nhu vây, minh nhu phâ`n trên. Ta c˜ ung c´ o . hai bô. sô´ (a(b + a b c 1 1 1 ´, tu.,. Theo mênh c − a), b(c + a − b), c(a + b − c)) v` a , , l` a d¯ô`ng thu . a b c d¯ê` 3.2, ta c´ o

J

1 1 1 a(b + c − a) + b(c + a − b) + c(a + b − c) c a b 1 1 1 ≤ a(b + c − a) + b(c + a − b) + c(a + b − c) a b c = a + b + c. , 1 1 1 ´,c trên th` ˘ng thu R´ ut gon anh a(b − a) + b(c − b) + c(a − c) ≤ 0, . bâ´t d¯a c a b , , ,, ,, ´,c n` ´,c câ`n chu ´,ng minh. ´,i bâ´t d¯a ˘ng thu ˘ng thu bâ´t d¯a ay tuong d¯uong vo

J

Ta x´et v´ı du. sau:

´, tu., hai hoa ˘. p thu ˘. c ba sô´ 3.1. Ca

141

, ,, , ˘`ng v´ oi moi V´ı du. 3.5. Chu´ng minh ra . sô´ duong a, b, c th`ı a4 + b4 + c4 ≥ a3 b + b3 c + c3 a.

, , , , , `,i giai. Do t´ınh d¯ô´i xu ´,ng cua c´ ´,c, ta c´ ˘ng thu Lo ac biê´n trong bâ´t d o thê gia ¯a ´, tu., v` ´ bô. ba (a, b, c) v` thiê´t l` a a ≤ b ≤ c. Khi d¯o a (a3 , b3 , c3 ) l` ad a ¯ô`ng thu , , , , ˜ theo mênh d¯ê` 3.2 su. thay d ung môt ¯ôi nh˜ung th`ua sô´ trong c` . . bô. dân d¯ê´n , , , tông nho hon. Suy ra a4 + b4 + c4 = a3 a + b3 b + c3 c ≥ a3 b + b3 c + c3 a. , , , ´,c xay ra khi v` ˘ng thu Ða a chı khi a = b = c.

J

, ,, , V´ı du. 3.6. Chu´ng minh ra ˘`ng v´ oi moi . sô´ duong a, b, c th`ı a5 b + b5 c + c5 a ≥ a4 bc + ab4 c + abc4 .

, , , , `,i giai. Do t´ınh d ´,ng cua c´ ´,c, ta c´ ˘ng thu Lo ac biê´n trong bâ´t d¯a o thê ¯ô´i xu , , ´ nh˜ung bô. ba (a4 , b4 , c4 ) v` gia thiê´t l` a a ≤ b ≤ c. Khi d¯o a (bc, ac, ab) l` a , , , , 4 4 4 ´ tu. . Theo mênh nghich d o tông S = a bc + ab c + abc l` a ¯ao thu ¯ê` 3.2 c´ . d . , nho nhâ´t. Suy ra a4 bc + ab4 c + abc4 ≤ a4 ab + b4 bc + c4 ac = a5 b + b5 c + c5 a. , , , ´,c xay ra khi v` ˘ng thu Ða a chı khi a = b = c.

J

, ,, , V´ı du. 3.7. Chu´ng minh ra ˘`ng v´ oi moi . sô´ duong x, y, z th`ı xy3 + yz3 + zx3 ≥ x2 yz + xy2 z + xyz2 .

, , , , , `,i giai. Do t´ınh d¯ô´i xu ´,ng cua c´ ´,c, ta c´ ˘ng thu Lo ac biê´n trong bâ´t d o thê gia ¯a , ´ nh˜ung bô. ba (x2 , y2 , z2 ) v` thiê´t x ≤ y ≤ z. Khi d¯o a (yz, xz, xy) l` a nghich . , , , , , 2 2 2 ` ´ tu. . Theo mênh d¯ao thu o tông S = x yz + y xz + z xy l` a nho nhâ´t. . d¯ê 3.2 c´ , , , , Môt ach chuyên th`ua sô´ cho ta tông xy3 + yz3 + zx3 . Suy ra . trong nh˜ung c´ xy3 + yz3 + zx3 ≥ x2 yz + y2 xz + z2 xy. , , , ´,c xay ra khi v` ˘ng thu Ða a chı khi x = y = z.

J

, , ,, ´,c cua c´ ´, tu., ˘ng thu Chuong 3. Bâ´t d¯a ac d˜ ay sô´ d ¯ô`ng thu

142

, ,, , ˘`ng v´ oi moi V´ı du. 3.8. Chu´ng minh ra . sô´ duong a, b, c th`ı a b c a 2 b2 c2 + + ≤ 2 + 2 + 2. b c a b c a   , , 1 1 1 `,i giai. Nh˜u,ng bô. ba (a, b, c) v` ´, tu.,. Theo Lo a , , l` a nghich . d¯ao thu a b c mênh d ¯ê` 3.2 suy ra . a·

1 1 a b c 1 +b· +c· ≤ + + , a b c b c a

ngh˜ıa l` a Ngo` ai ra 

a b c + + ≥ 3. b c a a b c + + b c a

2

a2 b 2 c2 = 2 + 2 + 2 +2 b c a

(3.2) 

a c b + + c b a

 ·

`˘ng c´ ´, tu., ´,i nh˜u,ng bô. d¯ô`ng thu ´p dung Ba ach a mênh d ¯ê` 3.2 d¯ô´i vo . .   a b c ,, v` a , , ta nhân ¯uo. c . d b c a

(3.3) 

a b c , , b c a



c2 a b b c c a a c b a2 b2 + 2 + 2 ≥ · + · + · = + + . (3.4) 2 b c a b c c a a b c b a , T`u (3.2), (3.3) v` a (3.4) suy ra    2  2  a c b a c b a b2 c2 3 + + ≤ + + ≤3 + + . b a c b a c b2 c2 a2 , , ,, ,, ´,c câ`n chu ´,ng minh. ´c hai vê´ cho 3 ta nhân ˘ng thu Sau khi gian uo ¯a . d¯uo. c bâ´t d

J

` tâp Bai . , , `˘ng phu,o,ng ph´ ´, tu.,, h˜ Ba ap nh˜ung bô. d˜ ay sô´ d¯ô`ng thu ay l` am nh˜ung b` ai , ,, , , , , , , ´i a, b, c l` tâp a nh˜ung sô´ duong (l`oi giai v` a go. i ´y o trang 171): . sau vo `˘ng aa bb ≥ ab ba . ´,ng minh ra . 3.9. Chu

, 3.2. Tông qu´ at h´ oa

143

`˘ng ´,ng minh ra . 3.10. Chu b a a+b ≥ a a+b b a+b . 2

`˘ng 3(a3 + b3 + c3 ) ≥ (a + b + c)(a2 + b2 + c2 ). ´,ng minh ra . 3.11. Chu `˘ng 2a15 + 3b10 ≥ 5a6 b6 . ´,ng minh ra . 3.12. Chu `˘ng 4a21 + 3b14 ≥ 7a12 b6 . ´,ng minh ra . 3.13. Chu

, ´ hoa ´ 3.2. Tông quat , Trong phâ`n n` ay ta tông qu´ at mênh d a mênh d¯ê` 3.2 cho hai d˜ ay ¯ê` 3.1 v` . . , , , , , ´ ˜ ´ huu han ao d¯inh ¯ua v` . ngh˜ıa: . sô thu. c. Truoc tiên ta d , Ðinh ngh˜ıa 3.1. Hai bô. sô´ thu. c (a1 , a2 , ..., an ) v` a (b1 , b2 , ..., bn ) (n ≥ 2) . , , , , ´ ` ` goi ad ung thoa m˜ an d a ¯ông thu´ tu. , nêu ch´ ¯ông th`oi a1 ≥ a2 ≥ · · · ≥ an v` . l` , ` ` ˘. c d b1 ≥ b2 ≥ · · · ≥ bn hoa a b1 ≤ b2 ≤ ¯ông thoi a1 ≤ a2 ≤ · · · ≤ an v`

· · · ≤ bn .

, Hai bô. sô´ thu. c (a1 , a2 , ..., an ) v` a (b1 , b2 , ..., bn ) (n ≥ 2) goi a nghich . . l` , , , , , d ung thoa m˜ an d¯ô`ng th`oi a1 ≥ a2 ≥ · · · ≥ an v` a b1 ≤ ¯ao thu´ tu. , nê´u ch´ , ˘. c d b2 ≤ · · · ≤ bn hoa a b1 ≥ b 2 ≥ · · · ≥ bn . ¯ô`ng th`oi a1 ≤ a2 ≤ · · · ≤ an v` ,, K´ı hiêu ¯ây bô. sô´ . S(i1 ,i2 ,...,in ) = a1 bi1 + a2 bi2 + · · · + an bin , o d , (i1 , i2 , ..., in ) l` a ho´ an vi. cua bô. sô´ (1, 2, ..., n). , Ðinh a (b1 , b2 , ..., bn−1 , . l´ı 3.1. (a) Cho hai bô. sô´thu. c (a1 , a2 , ..., an−1 , an ) v` , , , , ´ ta c´ bn ) (n ≥ 2) d o c´ ac bâ´t d¯a ˘ng thu´c sau ¯ô`ng thu´ tu. . Khi d ¯o S(1,2,...,n) ≥ S(i1 ,i2 ,...,in ) ≥ S(n,n−1,...,1) , , , v´ oi moi a ho´ an vi. cua (1, 2, ..., n). . (i1 , i2 , ..., in ) l` , (b) Cho hai bô. sô´ thu. c (a1 , a2 , ..., an−1 , an ) v` a (b1 , b2 , ..., bn−1 , bn )

144

, , ,, ´,c cua c´ ´, tu., ˘ng thu Chuong 3. Bâ´t d¯a ac d˜ ay sô´ d ¯ô`ng thu

, , , , , ´ ta c´ ˘ng thu´c sau (n ≥ 2) nghich o c´ ac bâ´t d ¯o ¯a . d¯ao thu´ tu. . Khi d S(1,2,...,n) ≤ S(i1 ,i2 ,...,in ) ≤ S(n,n−1,...,1) , , , v´ oi moi a ho´ an vi. cua (1, 2, ..., n). . (i1 , i2 , ..., in ) l` , , , , ,, C´ ac bâ´t d¯a ˘ng thu´c o (a) v` a (b) xay ra dâ´u ba ˘`ng khi v` a chı khi a1 = a2 = , , , ... = an hoa ˘. c (i1 , i2 , ..., in ) = (1, 2, ..., n) (d oi bâ´t d ˘ng thu´c vê´ tr´ ai) v` a ¯ô´i v´ ¯a , , , , ´ ´ ´ (i1 , i2 , ..., in ) = (n, n − 1, ..., 2, 1) (d oi bât d¯a ˘ng thu´c vê phai). ¯ôi v´ ´,ng minh. Ta chu ´,ng minh cho tru,`o,ng ho.,p (a) vo ´, ´,i hai bô. sô´ d Chu ¯ô`ng thu , ,, , tu. a1 ≤ a2 ≤ · · · ≤ an v` a b1 ≤ b2 ≤ · · · ≤ bn , c` on c´ ac tru`ong ho. p kh´ ac , , , , , , , ` ´ ng minh tuong tu. . Ta s˜e chu ´ ng minh d¯inh ˘ng phuong ph´ chu l´ı ba ap quy . , , , , , ´ ng minh trong mênh ´i n = 2, kha ˘ng d¯inh nap d ¯uo. c chu ¯ê` 3.1. . d . theo n. Vo . , , ´i n − 1 (n ≥ 3). ´ ng vo Ta gia thiê´t d ¯inh ¯u . l´ı d , , ´ ng minh tông lo ´,n nhâ´t l` Ta s˜e chu a S(1,2,...,n) = a1 b1 + a2 b2 + · · · + an bn . , Thât o . vây, . nê´u trong tông S(i1 ,i2 ,...,in ) = a1 bi1 + a2 bi2 + · · · + an bin c´ , , ´ `, , ´ ˘ng d¯inh a i1 = 1, th`ı kha . suy ra tru. c tiêp tu gia thiêt quy nap. . Cho i1 6= 1 v` , ,, ` ` tu mênh d¯ê 3.1 ta nhân ¯uo. c . . d ´,i j1 = 2, 3, 4, .., n. a1 bi1 + aj1 b1 ≤ a1 b1 + aj1 bi1 vo , , , ´,c xay ra khi v` ˘ng thu ˘. c (bi1 , b1 ) = (b1 , bi1 ). Theo Ða a chı khi a1 = aj1 hoa , , ´ ´ gia thiêt quy nap ay sô a2 ≤ ... ≤ an−1 ≤ an v` a . cho nh˜ung d˜ b2 ≤ ... ≤ bn−1 ≤ bn ta c´ o

a2 bi2 + · · · + aj1 bi1 + · · · + an bin ≤ a2 b2 + a3 b3 + · · · + an bn . , , , ´,c xay ra khi v` ˘ng thu ˘. c (bi2 , ..., bin ) = Ða a chı khi a2 = a3 = ... = an hoa

, 3.2. Tông qu´ at h´ oa

145

´ (b2 , b3 , ..., bn ). Khi d ¯o a1 bi1 + a2 bi2 + · · · + aj1 b1 + · · · + an bin ≤ ≤ a1 b1 + a2 bi2 + · · · + aj1 bi1 + · · · + an bin ≤ a1 b1 + a2 b2 + · · · + an bn = S(1,2,...,n) .

, , ,, , ´,ng minh tông lo ´,n nhâ´t l` ˜ chu Nhu vây a S(1,2,...,n) . Kê´t ho. p c´ ac tru`ong . ta d¯a , , , , ´,c xay ra khi v` ˘ng thu ˘. c ho. p d¯a a chı khi a1 = a2 = ... = an hoa (bi1 , bi2 , ..., bin ) = (b1 , b2 , ..., bn ). , , ,, , ´,ng minh d¯u,o.,c tông nho nhâ´t l` Ho` an to` an tuong tu. ta c˜ ung chu a

J

S(n,n−1,...,1) = a1 bn + a2 bn−1 + · · · + an b1 . , ˜,ng sô´ thu.,c v` Hê. qua 1: Cho a1 , a2 , ..., an l` a nhu a (a10 , a20 , ..., an0 ) l` a ho´ an vi. , ´ cua (a1 , a2 , ..., an ). Khi d ¯o a21 + a22 + · · · + a2n ≥ a1 a10 + a2 a20 + · · · + an an0 . , ˜,ng sô´ thu.,c du,o,ng v` a Hê. qua 2: Cho a1 , a2 , ..., an l` a nhu a (a10 , a20 , ..., an0 ) l` , ´ ho´ an vi. cua (a1 , a2 , ..., an ). Khi d ¯o a0 a0 a10 + 2 + · · · + n ≥ n. a1 a2 an , V´ı du. 3.14. Cho x1 ≤ x2 ≤ ... ≤ xn v` a y1 ≤ y2 ≤ ... ≤ yn l` a hai bô. sô´ thu. c. , , Chu´ng minh ra ˘`ng nê´u z1 , z2 , ..., zn l` a ho´ an vi. bâ´t k`ı cua c´ ac sô´ y1 , y2 , ..., yn , th`ı n n X X (xi − yi )2 ≤ (xi − zi )2 . i=1

i=1

, , `,i giai. Theo gia thiê´t ta c´ Lo o y21 + y22 + · · · + y2n = z21 + z22 + · · · + z2n .

146

, , ,, ´,c cua c´ ´, tu., ˘ng thu Chuong 3. Bâ´t d¯a ac d˜ ay sô´ d ¯ô`ng thu

, , ,, ,, ´,c d¯a ´,i ˘ng thu ˜ cho tuong d a r´ ut gon, Sau khi khai triên v` ¯uong vo . bâ´t d¯a n X i=1

xi yi ≥

n X

xi zi

i=1

, ´,c trên d¯u ˘ng thu ´ ng. v` a theo d¯inh . l´ı 3.1 bâ´t d¯a

J

, ,, , V´ı du. 3.15. Cho 2n sô´ duong a1 < a2 < ... < a2n . Chu´ng minh ra ˘`ng tông S = a1 a2n + a2 a2n−1 + ... + an an+1 , , ˜,ng ca ˜,ng sô´ d ˜ cho. c´ o gi´ a tri. nho nhâ´t trong sô´ tông nhu ˘. p trong nhu ¯a

, ,, , `,i giai. 1) C´ ´, tu., ´,i hai bô. sô´ d¯ô`ng thu Lo ach thu´ nhâ´t: Su dung d¯inh . l´ı 3.1 vo . , , ´,ng minh. (a1 , a2 , ..., an ) v` a (an+1 , ..., a2n ) cho kê´t qua phai chu , ´, ,, , 2) C´ ach thu´ hai: C˜ ung nhu chu ng minh d l´ı 3.1, ta d` ung phuong ¯inh . ph´ ap quy nap an hoc: . to´ . , , , , ´i n = 1 kha ˘ng d¯inh ´ ng l` Vo a hiên nhiên v`ı không c´ o ho´ an vi. kh´ ac cua . d¯u c´ ac sô´. , , , `˘ng kha ´,ng minh kha ´,i 2n sô´. Ta s˜e chu ˘ng d¯inh ˘ng ´ ng vo Gia thiê´t ra d¯u . ´,i 2(n + 1) sô´: a0 < a1 < a2 < ... < a2n < a2n+1 . ´ ng vo d¯inh ung d¯u . c˜ , , ,, , ´,ng minh d D˜ê chu a a2n+1 nhân a tri. nho hon l` a ¯uo. c kê´t ho. p hai sô´ a0 v` . gi´ , ´, ´ ´ ch´ ung kêt ho. p voi c´ ac sô kh´ ac. Thât . vây, . a0 a2n+1 +ak a` < a0 ak +a2n+1 a` ,, boi v`ı (a` − a0 )(a2n+1 − ak ) > 0. , , , , V`ı thê´ tông nho nhâ´t cua nh˜ung t´ıch hai sô´ c´ o dang a0 a2n+1 + .... . , , , , , ´i 2n sô´ th`ı tông cua t´ıch c´ ˘. p sô´ c´ Nhung theo gia thiê´t quy nap: ac ca o gi´ a . Vo , , , ˘ng d ´ ng tri. nho nhâ´t l` a a1 a2n + a2 a2n−1 + ... + an an+1 . Nhu vây ¯inh ¯u . d . kha , ´i moi vo . n. `˘ng c´ ´,ng minh tu,o,ng tu., hoa ´,ng ˘. c d` ´: Ba Ch´ uy ach chu ung d l´ı 3.1 ta chu ¯inh . , , , ,, , , ,, ˘. p sô´ duong c´ ˘ng d¯inh: minh d¯uo. c nh˜ung kha Tông cua nh˜ung t´ıch c´ ac ca o . , ´n nhâ´t l` gi´ a tri. lo a

J

S = a1 a2 + a3 a4 + ... + a2n−1 a2n .

, 3.2. Tông qu´ at h´ oa

147

, , , ´, `˘ng phu,o,ng ph´ ˘. p tông Ba ap nhu chu ng minh b` ai trên cho t´ıch cua c´ ac ca , hai sô´ cua d˜ ay sô´. , ,, , V´ı du. 3.16. Cho 2n sô´ duong a1 < a2 < ... < a2n . Chu´ng minh ra ˘`ng tông S = (a1 + a2n )(a2 + a2n−1 )...(an + an+1 ) , ˜,ng ca ˜,ng sô´ d ˜ cho. c´ o gi´ a tri. l´ on nhâ´t trong sô´ t´ıch nhu ˘. p trong nhu ¯a , , ,, `,i giai. D` ´,i n = 1, kha ˘ng d¯inh ´ ng Lo ung phuong ph´ ap quy nap ¯u . d . theo n. Vo , , , , , l` a hiên nhiên v`ı không c´ o tô ho. p kh´ ac cua tông. , , ,, , ´,ng minh kha ´,i 2n sô´. Ta s˜e chu ˘ng d¯inh ˘ng d ´ ng vo Gia su kha ung ¯inh . d¯u . c˜ , ´ ´i 2(n + 1) sô: a0 < a1 < a2 < ... < a2n < a2n+1 . ´ ng vo d¯u , ´,ng minh kê´t ho.,p tông hai sô´ a0 v` ´,n ho,n l` D˜ê chu a a2n+1 nhân a tri. lo a . gi´ , ´, , ´ ´ ´ ch´ ung kêt ho. p voi c´ ac sô kh´ ac. Thât o . vây, . voi k, ` ∈ {1, 2, ..., 2n} ta c´ (a0 + a2n+1 )(ak + a` ) − (a0 + ak )(a2n+1 + a` ) = (a0 ak + a0 a` + a2n+1 ak + a2n+1 a` )− − (a0 a2n+1 + a0 a` + ak a2n+1 + ak a` ) = a0 (ak − a2n+1 ) + a` (a2n+1 − ak ) = (a2n+1 − ak )(a` − a0 ) ≥ 0. , , ´,n nhâ´t cua nh˜u,ng tông hai sô´ c´ V`ı thê´ t´ıch lo o dang . (a0 +a2n+1 )...(an + , , , , , ´i 2n sô´ th`ı t´ıch cua tông c´ ˘. p an+1 ). Nhung theo gia thiê´t quy nap: ac ca . Vo , , ´n nhâ´t l` sô´ c´ o gi´ a tri. lo a (a1 + a2n )(a2 + a2n−1 )...(an + an+1 ). Nhu vây . , , ´i moi ˘ng d¯inh ´ ng vo kha ¯u . d . n. , , , , `˘ng c´ ´ ng minh tu,o,ng tu., cho kha ˘ng d ´ : Ba ach chu T´ıch cua nh˜ung Ch´ uy ¯inh: . , , ,, ˘. p sô´ duong c´ tông theo c´ ac ca o gi´ a tri. nho nhâ´t l` a

J

S = (a1 + a2 )(a3 + a4 )...(a2n−1 + a2n ). ˜,ng sô´ nguyên du,o,ng kh´ V´ı du. 3.17 (IMO 1978). Cho c1 , c2 , ..., cn l` a nhu ac , nhau. Chu´ng minh ra ˘`ng c2 cn 1 1 c1 + 2 + · · · + 2 ≥ 1 + + · · · + . 2 n 2 n

148

, , ,, ´,c cua c´ ´, tu., ˘ng thu Chuong 3. Bâ´t d¯a ac d˜ ay sô´ d ¯ô`ng thu

, , ,, `,i giai. Cho a1 , a2 , ..., an l` ´, tu., t˘ a nh˜ung sô´ ci d Lo ang dâ`n. V`ı ¯uo. c xê´p theo thu , , , ai l` a nh˜ung sô´ nguyên duong kh´ ac nhau, ta c´ o a1 ≥ 1, a n 2 ≥ 2, ..., an ≥  1 1 1 1 , a 1, 2 , · · · , 2 v` a 1 > 2 > · · · > 2 . Nhu vây . hai bô. (a1 , a2 , ..., an ) v` 2 n 2 n , , , ´ l` a nghich d ao th u t u . Theo d inh l´ ı 3.1 ta c´ o ¯ ¯ . . .

J

c2 cn a2 an 2 n + · · · + 2 ≥ a1 + 2 + · · · + 2 ≥ 1 + 2 + · · · + 2 . 22 n 2 n 2 n , , ˜,ng sô´ du,o,ng, th`ı bâ´t d V´ı du. 3.18 (M˜y 1974). Nê´u a, b, c l` a nhu ˘ng thu´c ¯a ´ng sau d ¯u c1 +

aa bb cc ≥ (abc)

a+b+c 3 .

, , , `,i giai. Bâ´t d¯a ´,c d ´,ng theo c´ ˘ng thu ´ ta c´ Lo ac thông sô´ a, b, c, do d¯o o thê ¯ô´i xu `˘ng a ≤ b ≤ c, khi d¯o ´ ln a ≤ ln b ≤ ln c. Theo d¯inh cho ra . l´ı 3.1: a ln a + b ln b + c ln c ≥ a ln a + b ln b + c ln c, a ln a + b ln b + c ln c ≥ a ln b + b ln c + c ln a, a ln a + b ln b + c ln c ≥ a ln c + b ln a + c ln b. , , ,, ´,c trên v` ˘ng thu Công ac bâ´t d a ta nhân ¯a . theo vê´ cua c´ . d¯uo. c: (a + b + c)(ln a + ln b + ln c) . 3 , , , ,, am sô´ m˜ u s˜e nhân T`u d¯ây biê´n d ¯ôi theo h` . d¯uo. c kê´t qua. a ln a + b ln b + c ln c ≥

` tâp Bai .

, , , (Goi a tra l`oi t`u trang 172) . ´y v`

`˘ng nê´u a, b, c > 0, th`ı ´,ng minh ra . 3.19. Chu a3 b3 c3 a 2 + b2 + c 2 + + ≥ . b+c c+a a+b 2 , ,, `˘ng ´,ng minh ra . 3.20. Cho a, b, c l` a nh˜ung sô´ duong. Chu bc ca ab 1 + + ≤ (a + b + c). b+c c+a a+b 2

J

,, ´, tu., 3.3. Su dung c´ ac d˜ ay sô´ d¯ô`ng thu .

149

, ,, `˘ng ´,ng minh ra . 3.21. Cho x1 , x2 , ..., xn l` a nh˜ung sô´ duong. Chu x1 + x2 + · · · + xn ≤

x21 x22 x2 + + ··· + n. x2 x3 xn

, ,, ,, ´,ng . 3.22. Cho a, b, c l` a nh˜ung sô´ duong v` a n l` a sô´ nguyên duong, chu `˘ng minh ra bn cn an−1 + bn−1 + cn−1 an + + ≥ . b+c c+a a+b 2 , , . 3.23. (M˜y 1999) Cho a1 , a2 , a3 , ... (n > 3) l` a nh˜ung sô´ thu. c sao cho a1 + a2 + · · · + an ≥ n v` a a21 + a22 + · · · + a2n ≥ n2 .

`˘ng max(a1 , a1 , ..., an ) ≥ 2. ´,ng minh ra Chu

, `˘ng radian) cua môt . 3.24. Cho A, B, C l` a d¯ô. d¯o g´ oc (ba ac, a, b, c . tam gi´ , a+b+c , `˘ng ´ ng minh ra ˘. t p = l` ad ai c´ ac canh cua tam gi´ ac v` ad . Chu ¯ô. d` ¯a . 2 B C 3π A + + ≥ . p−a p−b p−c p

,, , , ´ d˜ 3.3. Su dung cac ay sô´ dô u tu. ¯ `ng th´ . , ,, ,, ´, tu., d¯a ˜ giai d¯uo. c râ´t nhiê`u b` Phuong ph´ ap d` ung d˜ ay sô´ d ai tâp ¯ô`ng thu . vê` , , , , , , ´ c. Nhung n´ ´ ng minh môt ˘ng thu o c` on l` a môt ap chu bâ´t d ¯a . phuong ph´ . sô´ bâ´t , , , , , , , , , ´ ´ ´ c Chebyshev v` ´ c Cauchy. ´ c co ban nhu bât d¯a ˘ng thu ˘ng thu ˘ng thu a bât d d¯a ¯a , ,, , Ðinh ngh˜ıa 3.2. Môt ay sô´ thu. c a1 , a2 , ..., an d¯uo. c goi a d˜ ay d ¯o n d ¯iêu . . d˜ . l` . ,, , ´ ´ t˘ ang, nêu a1 ≤ a2 ≤ ... ≤ an . D˜ ay sô a1 , a2 , ..., an d¯uo. c goi a d˜ ay d ¯o n . l` , d ¯iêu . giam, nê´u a1 ≥ a2 ≥ ... ≥ an .

, , ˜, ´,c Chebyshev). Cho hai d˜ ˘ng thu Ðinh l´ı 3.2 (Bâ´t d ay sô´ thu. c hu u han ¯a . . a1 , a2 , ..., an v` a b1 , b2 , ..., bn . Ða ˘. t A = a1 b1 + a2 b2 + · · · + an bn v` a B = a1 bn + a2 bn−1 + · · · + an b1 ..

, , ,, ´,c cua c´ ´, tu., ˘ng thu Chuong 3. Bâ´t d¯a ac d˜ ay sô´ d ¯ô`ng thu

150

, ang (hoa ˘. c (a) Nê´u hai d˜ ay a1 , a2 , ..., an v` a b1 , b2 , ..., bn c` ung d ¯iêu ¯o n d . t˘ , , c` ung d ¯on d ¯iêu . giam), th`ı (a1 + · · · + an )(b1 + · · · + bn ) ≥ B; n , , (b) Nê´u d˜ ay a1 , a2 , ..., an l` ad ang v` a b1 , b2 , ..., bn l` ad ¯on d¯iêu ¯o n d ¯iêu . t˘ . , , , , giam (hoa ˘. c a1 , a2 , ..., an d a b1 , b2 , ..., bn d ang), th`ı ¯o n d ¯iêu ¯on d¯iêu . giam v` . t˘ (a1 + · · · + an )(b1 + · · · + bn ) A≤ ≤ B. n , , , ,, , , C´ ac bâ´t d ˘ng thu´c trong (a) v` a (b) tro th` anh d ˘ng thu´c khi v` a chı khi ¯a ¯a A≥

a1 = a2 = ... = an hoa ˘. c b1 = b2 = ... = bn . ´,ng minh. Ta chu ´,ng minh cho tru,`o,ng ho.,p (a) vo ´,i hai d˜ ˜ cho l` Chu ay d¯a a c` ung ,`, , , , , , , ´ ng minh ho` d¯on d¯iêu ang, c` on c´ ac truong ho. p kh´ ac chu an to` an tuong tu. . . t˘ , ,, ˘ng Theo d¯inh l´ı 3.1 ho´ an vi. v` ong quanh theo bi , ta nhân ¯uo. c n bâ´t d¯a . . d , ´c thu A ≥ a1 b1 + a2 b2 + · · · + an bn ≥ B, A ≥ a1 b2 + a2 b3 + · · · + an b1 ≥ B, ········· A ≥ a1 bn + a2 b1 + · · · + an bn−1 ≥ B. , , , , , ´,c trên v` ´,c ˘ng thu ˘ng thu a biê´n d Công theo vê´ cua nh˜ung bâ´t d¯a ¯ôi vê` bâ´t d ¯a . Chebyshev. , , , , , ´,c khi v` ´,c tro, th` ˘ng thu ˘ng thu a chı khi anh d¯a C˜ ung t`u d ¯inh . l´ı 3.1 bâ´t d¯a ˘. c b1 = b2 = ... = bn . a1 = a2 = ... = an hoa , ´,c Chauchy). Nê´u nhu ˜,ng sô´ a1 , a2 , ..., an l` ˜,ng ˘ng thu Ðinh a nhu . l´ı 3.3 (Bâ´t d¯a , sô´ thu. c không âm, th`ı

J

√ a1 + a2 + · · · + an ≥ n n a1 a2 ...an . , , , , Ða ˘ng thu´c xay ra khi v` a chı khi a1 = a2 = ... = an .

,, ´, tu., 3.3. Su dung c´ ac d˜ ay sô´ d¯ô`ng thu .

151

, √ ´,ng minh. Ða ´,c l` ˘. t G = n a1 a2 ...an . Nê´u G = 0 th`ı bâ´t d¯a ˘ng thu Chu a , a1 a2 a1 ˘. t b1 = ´ d , b2 = , ..., bn = hiên nhiên. X´et G 6= 0, khi d ¯o ¯a G G2 , a1 a2 ...an = 1. Theo hê. qua 2 ta c´ o Gn an b1 b2 bn a1 a2 + + ··· + , n≤ + + ··· + = b n b1 bn−1 G G G , , a + a2 + · · · + an ,, ,, ´,c xay ra khi ´,i 1 ˘ng thu d¯iê`u n` ay tuong d¯uong vo ≥ G. Ða n , v` a chı khi b1 = b2 = ... = bn , suy ra a1 = a2 = ... = an .

J

, ˜,ng sô´ du,o,ng, th`ı V´ı du. 3.25. Chu´ng minh ra ˘`ng nê´u a1 , a2 , ..., an l` a nhu √ n

a1 a2 ...an ≥

n . 1 1 1 + + ··· + a1 a2 an

, , , , Ða ˘ng thu´c xay ra khi v` a chı khi a1 = a2 = ... = an . , a1 a1 a2 a1 a2 ...an √ `,i giai. Ða ˘. t G = n a1 a2 ...an , b1 = , b2 = , ..., bn = = Lo 2 G G Gn , 1. Theo hê. qua 2 ta c´ o b1 b2 bn G G G + + ··· + = + + ··· + , b2 b3 b1 a1 a2 an ,, ,, ´,i d¯iê`u n` ay tuong d ¯uong vo n G≥ . 1 1 1 + + ··· + a1 a2 an , , , ´,c xay ra khi v` ˘ng thu Ða a chı khi b1 = b2 = ... = bn , suy ra a1 = a2 = n≤

... = an . , ˜,ng sô´ du,o,ng, th`ı V´ı du. 3.26. Chu´ng minh ra ˘`ng nê´u a1 , a2 , ..., an l` a nhu r a21 + a22 + · · · + a2n a1 + a2 + · · · + an ≥ . n n , , , , Ða ˘ng thu´c xay ra khi v` a chı khi a1 = a2 = ... = an .

J

152

, , ,, ´,c cua c´ ´, tu., ˘ng thu Chuong 3. Bâ´t d¯a ac d˜ ay sô´ d ¯ô`ng thu

, , `,i giai. Theo hê. qua 1, ta c´ Lo o a21 + a22 + · · · + a2n ≥ a1 a2 + a2 a3 + · · · + an a1 , a21 + a22 + · · · + a2n ≥ a1 a3 + a2 a4 + · · · + an a2 , ··· ≥ ··· a21 + a22 + · · · + a2n ≥ a1 an + a2 a1 + · · · + an an−1 . , ´,c trên v` ˘ng thu Công theo vê´ c´ ac bâ´t d¯a a công thêm v` ao hai vê´ c` ung d¯ai . . . ,, ,, 2 2 2 luo. ng a1 + a2 + · · · + an , ta nhân . d¯uo. c n(a21 + a22 + · · · + a2n ) ≥ (a1 + a2 + · · · + an )2 , , , , ,, ,, ´,ng minh. Ða ´,c xay ra khi ´,i kê´t qua câ`n chu ˘ng thu d ay tuong d ¯iê`u n` ¯uong vo , v` a chı khi a1 = a2 = ... = an . , ´,c Cauchy-Schwarz). Cho a1 , a2 , ..., an , b1 , b2 , ..., bn ˘ng thu V´ı du. 3.27 (Bâ´t d ¯a ˜,ng sô´ thu.,c. Khi d ´ l` a nhu ¯o

J

(a1 b1 + a2 b2 + · · · + an bn )2 ≤ (a21 + a22 + · · · + a2n )(b21 + b22 + · · · + b2n ), , , , , ´ sao cho ai = kbi d ˘ng thu´c xay ra khi v` a chı khi tô`n tai ˘`ng sô´ k n` ao d ¯a ¯o . môt . ha , hoa ˘. c bi = kai v´ oi i = 1, n. , `,i giai. Nê´u a1 = a2 = ... = an = 0 hoa ˘. c b1q= b2 = ... = bn = 0 Lo , ,, ´ ˘. t S = a21 + a22 + · · · + a2n v` th`ı kêt luân a hiên nhiên. Nguo. c lai a ¯a . l` . ta d q , , , T = b21 + b22 + · · · + b2n . V`ı hai d¯ai ac không, ta c´ o thê . luo. ng trên d¯ê`u kh´ , ai bi , ´i i = 1, n. Theo hê. qua 1, ˘. t xi = d¯a v` a xn+i = vo S T a2 + a22 + · · · + a2n b21 + b22 + · · · + b2n 2= 1 + = x21 + x22 + · · · + x22n S2 T2 ≥ x1 xn+1 + x2 xn+2 + · · · + xn x2n + xn+1 x1 + xn+2 x2 + · · · + x2n xn = 2(a1 b1 + a2 b2 + · · · + an bn ) , ST , , , ,, ,, ´,c câ`n chu ´,ng minh. Ða ´,c xay ´,i bâ´t d¯a ˘ng thu ˘ng thu d¯iê`u n` ay tuong d ¯uong vo , ´,i i = 1, n, hay l` ´,i i = 1, n. ra khi v` a chı khi xi = xn+i vo a ai T = bi S vo =

J

,, ´, tu., 3.3. Su dung c´ ac d˜ ay sô´ d¯ô`ng thu .

153

, ˜,ng sô´ du,o,ng v` a abc = 1. Chu´ng V´ı du. 3.28 (IMO 1995). Cho a, b, c l` a nhu minh ra ˘`ng 1 1 1 3 + + ≥ . a3 (b + c) b3 (c + a) c3 (b + a) 2 , , 1 1 1 `,i giai. Ða ´,c câ`n ˘. t x = bc = , y = ca = , z = ab = . Bâ´t d¯a ˘ng thu Lo a b c , ´,ng minh tro, th` chu anh y2 z2 3 x2 + + ≥ . z+y x+z y+x 2 , , ,, , , `˘ng x ≤ ´ c l` ´ ng theo c´ ˘ng thu Boi bâ´t d ad ac biê´n, nên ta gia thiê´t ra ¯a ¯ô´i xu 1 1 1 , ´ x2 ≤ y2 ≤ z2 v` y ≤ z, khi d a ≤ ≤ . Nhu vây ¯o . hai bô. z+y x+z y+x (x2 , y2 , z2 ) v` a   1 1 1 ´, tu.,. Theo d , , l` a d¯ô`ng thu o ¯inh . l´ı 3.1 ta c´ z+y x+z y+x y2 z2 x2 y2 z2 x2 + + ≥ + + , z+y x+z y+x y+x z+y x+z x2 y2 z2 x2 y2 z2 + + ≥ + + . z+y x+z y+x x+z y+x z+y , ,, ´,c n` ˘ng thu Công ac bâ´t d ay theo vê´ v` a chia cho 2, ta nhân ¯a ¯uo. c . c´ . d   x2 y2 z2 1 y2 + x2 z2 + y2 x2 + z2 + + ≥ + + . z+y x+z y+x 2 y+x z+y x+z , , (a + b)2 ´,c a2 + b2 ≥ ´,c Cauchy, ta ˘ng thu ˘ng thu ´p dung Ta a bâ´t d¯a v` a bâ´t d¯a . 2 ,, nhân . d¯uo. c   x2 y2 z2 1 y+x z+y x+z x+y+z + + ≥ + + = z+y x+z y+x 2 2 2 2 2 √ 3 · 3 xyz 3 ≥ = . 2 2 , V´ı du. 3.29. Cho 0 ≤ ak < 1 v´ oi k = 1, 2, ..., n v` a S = a1 + a2 + · · · + an . , ` H˜ ay chu´ng minh ra ˘ng n X ak nS ≥ . 1 − ak n−S

J

k=1

, , ,, ´,c cua c´ ´, tu., ˘ng thu Chuong 3. Bâ´t d¯a ac d˜ ay sô´ d ¯ô`ng thu

154

, , , , ,, `,i giai. Không mâ´t t´ınh tông qu´ Lo at ta c´ o thê gia su l` a a1 ≥ a2 ≥ · · · ≥ an ≥ 0. ´ Khi d ¯o 0 < 1 − a1 ≤ 1 − a2 ≤ · · · ≤ 1 − an v` a

a1 a2 an ≥ ≥ ··· ≥ . 1 − a1 1 − a2 1 − an

, ´,c Chebyshev: ˘ng thu Theo bâ´t d¯a a2 an a1 (1 − a1 ) + (1 − a2 ) + · · · + (1 − an ) 1 − a1 1 − a2 1 − an n n n 1 X ak X n − S X ak ≤ (1 − ak ) = . n 1 − ak n 1 − ak

S=

k=1

k=1

J

k=1

˜,ng sô´ du,o,ng bâ´t k`ı. Chu´,ng minh bâ´t V´ı du. 3.30. Cho x1 , x2 , ..., xn l` a nhu , , d ˘ng thu´c ¯a (x1 + x2 + ... + xn )3 ≤ n2 (x31 + x32 + ... + x3n ). , , , , `,i giai. Do bâ´t d ´,c l` ´,ng theo c´ ˘ng thu Lo a d¯ô´i xu ac biê´n nên ta c´ o thê gia thiê´t ¯a `˘ng ra , ´,c Chebyshev ta c´ ˘ng thu x1 ≤ x2 ≤ ... ≤ xn . Theo bâ´t d o ¯a ! n ! n n X X X n x3i ≥ xi x2i i=1

i=1

i=1

v` a n

n X i=1

x2i



n X

!2 xi

.

i=1

, , , , ´,c trên ta d¯u,o.,c bâ´t d ´,c câ`n phai chu ´,ng ˘ng thu ˘ng thu Nhân theo vê´ cua hai d ¯a ¯a

J

minh. , ˜,ng sô´ du,o,ng, th`ı V´ı du. 3.31. Chu´ng minh ra ˘`ng nê´u x1 , x2 , ..., xn l` a nhu (x1 x2 ...xn )

x1 +x2 +...+xn n

≤ xx11 xx22 ...xxnn .

,, ´, tu., 3.3. Su dung c´ ac d˜ ay sô´ d¯ô`ng thu .

155

, , ,, ,, `,i giai. Bâ´t d¯a ´,c d¯a ´,i ˘ng thu ˜ cho tuong d¯uong vo Lo x1 + x2 + ... + xn lg x1 + lg x2 + ... + lg xn · ≤ n n x1 lg x1 + x2 lg x2 + ... + xn lg xn ≤ . n , , ,, , ´,c n` ´,c Chebyshev (hoa ´,ng ˘ng thu ˘ng thu ˘. c ta chu Bâ´t d ay d¯uo. c suy ra t`u bâ´t d¯a ¯a , ´,ng minh d minh nhu trong chu ¯inh . l´ı Chebyshev).

J

, V´ı du. 3.32. Cho a, b, c > 0, n ∈ N. Chu´ng minh ra ˘`ng

an+1 + bn+1 + cn+1 a+b+c ≥ . an + bn + cn 3

, , , , `,i giai. Gia su, 0 < a ≤ b ≤ c, suy ra an ≤ bn ≤ cn . Theo bâ´t d¯a ´,c ˘ng thu Lo

Chebyshev

(a + b + c)(an + bn + cn ) ≤ 3(an+1 + bn+1 + cn+1 ). , , ´,ng minh. T`u d¯ây suy ra d ¯iê`u phai chu

J

, V´ı du. 3.33. Cho tam gi´ ac nhon ˘`ng . ABC. Chu´ng minh ra sin 2A + sin 2B + sin 2C ≤ sin A + sin B + sin C. , , , `,i giai. Gia su, A ≤ B ≤ C. Ta x´et hai bô. sô´ (sin A, sin B, sin C) v` Lo a , π (cos A, cos B, cos C), do tam gi´ ac l` a nhon . nên A, B, C ∈ (0, 2 ), trong khoang , , , n` ay h` am sin d ang, c` on cos d¯on d¯iêu ¯on d¯iêu . t˘ . giam, suy ra sin A ≤ sin B ≤ , ´,c Chebyshev ˘ng thu ´a ´p dung sin C v` a cos A ≥ cos B ≥ cos C. Sau d¯o bâ´t d¯a . , cho kê´t qua.

J

` tâp Bai .

, , , ,, (Go. i ´y v` a l`oi giai o trang 174).

, ,, ´,ng . 3.34. Cho a, b, c, d l` a nh˜ung sô´ duong v` a S = a2 + b2 + c2 + d2 . Chu `˘ng minh ra a3 + b3 + c3 a3 + b3 + d3 a3 + c3 + d3 b3 + c3 + d3 + + + ≥ S. a+b+c a+b+d a+c+d b+c+d

156

, , ,, ´,c cua c´ ´, tu., ˘ng thu Chuong 3. Bâ´t d¯a ac d˜ ay sô´ d ¯ô`ng thu

, ,, . 3.35. Cho a1 , a2 , ..., an l` a nh˜ung sô´ duong v` a a = a1 + a2 + · · · + an . , ` ´ ng minh ra ˘ng Chu n X ai n . ≥ 2a − ai 2n − 1 i=1

,, `˘ng vo ´,ng minh ra ´,i moi ´,i moi . 3.36. Chu a vo . sô´ nguyên duong n v` . x ∈ R, , , ´ c sau d ˘ng thu ´ ng bâ´t d¯a ¯u sin2n x + cos2n x ≥

1 2n−1

.

, . 3.37. Cho ai v` a bi (i = 1, 2, ..., n) l` a c´ ac sô´ thu. c sao cho a1 ≥ a2 ≥ ... ≥ ´,ng minh an > 0 v` a b1 ≥ a1 , b1 b2 ≥ a1 a2 , ..., b1 b2 ...bn ≥ a1 a2 ...an . Chu `˘ng ra b1 + b2 + ... + bn ≥ a1 + a2 + ... + an . `˘ng ´,ng minh ra . 3.38. Cho tam gi´ ac nhon . ABC. Chu sin A + sin B + sin C 1 ≤ (tgA + tgB + tgC). cos A + cos B + cos C 3

, , ˘ ng th´ 3.4. Chuyên dê uc Karamata ¯ ` vê` bâ´t d¯a , , ` lô`i va ` bâ´t d¯a ˘ ng th´ 3.4.1. Ham uc Karamata , , ,, , ,´, Chuyên d¯ê` n` ay mo rông nh˜ung kê´t qua cua phâ`n truo c, trong chuyên . ´˘c lai ai niêm am lô`i v` ad ac d¯inh am lô`i, d¯ê` ta c˜ ung nha ¯inh . l´ı x´ . môt . kh´ . vê` h` . h` ,, , , , d¯ua ra c´ ac v´ı du. môt am co so l` a nh˜ung h` am lô`i. . sô´ h` , ,, Ðinh ngh˜ıa 3.3. H` am sô´ gi´ a tri. thu. c f(x) x´ ac d ¯inh . trên d¯oan . [a, b] d¯uo. c . goi a h` am lô`i nê´u . l` f(λx + (1 − λ)y) ≤ λf(x) + (1 − λ)f(y) ´,i moi vo a λ ∈ [0, 1]. . x, y thuôc . [a, b] v`

, ´,c Karamata ˘ng thu 3.4. Chuyên d ¯ê` vê` bâ´t d ¯a

157

,, H` am sô´ f(x) d¯uo. c goi a h` am lô`i cha ˘. t nê´u . l` f(λx + (1 − λ)y) < λf(x) + (1 − λ)f(y) ´,i moi vo a λ ∈ (0, 1). . x, y thuôc . [a, b], x 6= y v` ´: 1. H` Ch´ uy am sô´ f(x) goi a h` am l˜ om (h` am l˜ om cha ˘. t) trên [a, b] nê´u −f(x) . l` , , ˘. t). H`ınh 3.1 thê hiên l` a h` am lô`i (h` am lô`i cha am lô`i, c` on h`ınh 3.2 thê hiên . h` . h` am l˜ om.

y

y ·C

·B A· A· · C 0

H`ınh 3.1. H` am lô`i

·B

x

x

0

H`ınh 3.2. H` am l˜ om

, ˘. t c´ ˘. t 2. H` am lô`i cha o ´y ngh˜ıa h`ınh hoc am sô´ f(x) l` a lô`i cha . nhu sau: H` , , , ´i moi khi v` a chı khi vo a B = (y, f(y)) trên d¯ô` thi. . hai d¯iêm A = (x, f(x)) v` , , ,´, ` `˘m gi˜u,a x v` ´,i moi ˘m duoi d¯oan ay cua f th`ı d ¯iêm C = (z, f(z)) na . AB vo . z na (h`ınh 3.1). ,, ´,ng 3. Nê´u h` am sô´ f(x) l` a h` am liên tuc . trên d¯oan . [a, b], th`ı ngu`oi ta chu , ,, ` ´,i moi ˘ng f(x) l` minh d a h` am lô`i khi v` a chı khi vo ¯uo. c ra . x, y ∈ [a, b]   x+y f(x) + f(y) f ≤ 2 2 , ˘. t khi v` v` a n´ o l` a h` am lô`i cha a chı khi   x+y f(x) + f(y) f < 2 2

158

, , ,, ´,c cua c´ ´, tu., ˘ng thu Chuong 3. Bâ´t d¯a ac d˜ ay sô´ d ¯ô`ng thu

, ´,i tâ´t ca x, y ∈ [a, b], x 6= y. vo ´ l` C´ o râ´t nhiê`u t` ai liêu oi vê` h` am lô`i, môt a cuô´n s´ ach [11] . n´ . trong sô´ d¯o , , ´ ´ ´ c´ o b` an toi v` a liêt o. . kê môt . sô t´ınh chât cua n´ ´,u t´ınh châ´t v` Ta không d¯i sâu nghiên cu a c´ ac d am ¯inh . l´ı liên quan d¯ê´n h` , , ˜˘n nhu sau: lô`i, ta chı d` ung môt o sa ¯inh . l´ı c´ . d Ðinh l´ı 3.4. Cho h` am sô´ f(x) c´ od am bâc ¯ao . h` . hai trên [a, b]. . , 00 ´ l` • Nê´u d am bâc oi moi am d a h` am lô`i; ¯ao ¯o . h` . hai f (x) ≥ 0 v´ . x ∈ [a, b], th`ı h` , 00 ´ l` • Nê´u d¯ao am bâc oi moi am d a h` am lô`i ¯o . h` . hai f (x) > 0 v´ . x ∈ (a, b), th`ı h` cha ˘. t. ´: 1. Môt Ch´ uy am tuyê´n t´ınh f(x) = ax + b (x ∈ R) l` a môt am lô`i v` a . h` . h`

c˜ ung l` a h` am l˜ om. , 2. Tông hai h` am lô`i (h` am l˜ om) l` a môt am lô`i (h` am l˜ om). . h` , , , ˘. c l˜ T`u d¯inh o thê kiêm tra c´ ac h` am sau d¯ây l` a lô`i hoa om: . l´ı trên ta c´ ´,i x > 0. 1. H` am sô´ f(x) = xα vo ´,i 0 < α < 1 h` Ta c´ o f 00 (x) = α(α − 1)xα−2 , vây am sô´ n` ay l˜ om, c` on . vo , ´i α < 0 v` vo a α > 1 h` am sô´ n` ay lô`i. , x ´i a > 0, a 6= 1. 2. H` am sô´ f(x) = a vo 2 00 x ´,i moi ˜ cho lô`i. Ta c´ o f (x) = a ln a > 0 vo am sô´ d ¯a . x, suy ra h` , ´i x > 0, a > 0, a 6= 1. 3. H` am sô´ f(x) = lga x vo 1 00 ´,i a < 1, f(x) l` ´,i a > 1 Ta c´ o f (x) = − 2 , nên vo a h` am lô`i, c` on vo x ln a n´ o l` a h` am l˜ om. ´,i x > 0. 4. H` am sô´ f(x) = x ln x vo 1 Ta c´ o f 00 (x) = > 0, nên h` am f(x) l` a h` am lô`i. x , , ,´, ´, tu., v` ´,ng dung Nh˜ung phâ`n truo c ta x´et c´ ac bô. sô´ d a c´ ac u cua ¯ô`ng thu . , n´ o. Phâ`n n` ay ta x´et c´ ac bô. sô´ c´ o t´ınh châ´t gâ`n nhu vây: . , , Ðinh ngh˜ıa 3.4. Cho hai bô. sô´ gô`m nh˜ung sô´ thu. c (a1 , a2 , ..., an ) v` a .

, ´,c Karamata ˘ng thu 3.4. Chuyên d ¯ê` vê` bâ´t d ¯a

159

´,i i = 1, 2, ..., n − 1. Ta (b1 , b2 , ..., bn ) sao cho ai ≥ ai+1 , bi ≥ bi+1 vo , `˘ng bô. sô´ (a1 , a2 , ..., an ) trôi n´ oi ra . hon bô. sô´ (b1 , b2 , ..., bn ), nê´u   a 1 ≥ b1 ,      a 1 + a2 ≥ b1 + b2 ,  (3.5) ...    a + a + · · · + a ≥ b + b + · · · + b ,  1 2 n−1 1 2 n−1    a + a + ··· + a = b + b + ··· + b . 1 2 n 1 2 n ˘. c l` K´ı hiêu a (a1 , a2 , ..., an )  (b1 , b2 , ..., bn ) hoa a . l`

(b1 , b2 , ..., bn ) ≺ (a1 , a2 , ..., an ). Ta x´et môt . sô´ v´ı du: .

1 1 1 1 1 ´,i n ≥ 2, (1, 0, ..., 0)  ( , , 0, ..., 0)  · · ·  ( , , ..., ). 1. D˜ê thâ´y vo | {z } | 2 2 {z n n n } {z } | n sô´ ´ ´ n sô n sô ,, a không bô. sô´ 2. Bô. sô´ (5, 5, 0) v` a (6, 2, 2) không so s´ anh d ¯uo. c, ngh˜ıa l` , n` ao trôi ao. . hon bô. sô´ n` , , , 3. Cho a, b, c l` a d¯ô. d` ai c´ ac canh cua môt ac bâ´t k`ı. Ta c´ o thê gia . . tam gi´ ,, ´ ta luôn luôn c´ su a ≥ b ≥ c. Khi d o ¯o   a+b b+c c+a (a, b, c)  , , . 2 2 2 x1 + x2 + · · · + xn 4. Nê´u x1 ≥ x2 ≥ · · · ≥ xn v` ax= , th`ı n (x1 , x2 , ..., xn )  (x, | x,{z..., x}). n sô´ , , `˘ng x1 + x2 + ´,ng minh ra ´i k = 1, 2, ..., n − 1 ta chı câ`n chu Thât . vây, . vo

· · · + xk ≥ kx. Ta c´ o (n−k)(x1 +· · ·+xk ) ≥ (n−k)kxk ≥ k(n−k)xk+1 ≥ k(xk+1 +· · ·+xn ),

, nhu vây . (n−k)(x1 +· · ·+xk ) ≥ k(xk+1 +· · ·+xn ). Công . k(x1 +· · ·+xk ) , , , , , ´ ´ ´ c trên, ta nhân ˘ng thu v` ao hai vê cua bât d¯a ¯uo. c n(x1 + · · · + xk ) ≥ . d ´ x1 + · · · + xk ≥ kx. k(x1 + · · · + xn ) = knx. Do d ¯o

, , ,, ´,c cua c´ ´, tu., ˘ng thu Chuong 3. Bâ´t d¯a ac d˜ ay sô´ d ¯ô`ng thu

160

Ðinh l´ı 3.5. Cho f(t) l` a môt am lô`i trên (a, b) v` a x1 , x2 , ..., xn , y1 , y2 , ..., . h` . ´ yn ∈ (a, b) sao cho (x1 , x2 , ..., xn )  (y1 , y2 , ..., yn ). Khi d ¯o f(x1 ) + f(x2 ) + · · · + f(xn ) ≥ f(y1 ) + f(y2 ) + · · · + f(yn ).

(3.6)

Nê´u f(x) l` a h` am l˜ om th`ı f(x1 ) + f(x2 ) + · · · + f(xn ) ≤ f(y1 ) + f(y2 ) + · · · + f(yn ). , , ´,c 3.6 goi ´,c Karamata1 . Chu ´,ng minh bâ´t ˘ng thu ˘ng thu Bâ´t d¯a a bâ´t d¯a . l` , , , ´,c n` ˘ng thu d ay du. a v` ao hai mênh d¯ê` phu. tro. sau: ¯a . , Mênh d¯ê` 3.3. Nê´u f(t) l` a h` am lô`i v` a x1 + x2 = y1 + y2 v´ oi (y1 ≥ x1 ≥ .

x2 ≥ y2 ), th`ı f(y1 ) + f(y2 ) ≥ f(x1 ) + f(x2 ). ´,ng minh. Mênh Chu d ay l` a ¯ê` n` . , , ˘ng han hiên nhiên, cha . ta x´et ,`, , , truong ho. p nhu h`ınh 3.3. Ta thâ´y `˘m trên CD, suy ra d¯oan . AB na , , trung d¯iêm cua AB l` a E c˜ ung , , ` ˘m trên trung d¯iêm cua CD l` na a , , F. Tung d ay ¯ô. cua nh˜ung d¯oan . n`

·B

E ·

A· C·

F ·

y1 x 1 G

·D

x2 y2

l` a H`ınh 3.3 f(y1 ) + f(y2 ) , 2 f(x1 ) + f(x2 ) FG = . 2 , , ´,c trên suy ra d¯iê`u câ`n chu ´,ng minh. ˘ng thu T`u c´ ac d ¯a EG =

J

, , , Ðinh ngh˜ıa 3.5. Ph´ep dich a viêc . chuyên cua bô. sô´ (x1 , x2 , ..., xn ) l` . thu. c . , , ,, , ˜ bao to` hiên ang xi v` a giam xj môt an ¯ô`ng th`oi t˘ ¯ai . d . d . luo. ng sao cho vân , , tông cua hai sô´ n` ay, khi xi ≥ xj . 1 Jovan

Karamata (1902-1967): Viên an hoc . s˜ı to´ . Serbia-Montenegro.

, ´,c Karamata ˘ng thu 3.4. Chuyên d ¯ê` vê` bâ´t d ¯a

161

, ´,n ho,n sô´ 5 v` ´ sô´ 7 lo V´ı du: a ta thu. c hiên ¯o . Cho bô. sô´ (3, 7, 8, 9, 5, 4) khi d . , , , ph´ep dich a giam: 7 − 2 v` a t˘ ang: 5 + 2. Sau khi dich . chuyên l` . chuyên bô. sô´ ,, tro th` anh (3, 5, 8, 9, 7, 4). , , ´,i ph´ep dich Theo mênh d ¯ê` 3.3 th`ı vo . chuyên cua bô. sô´ (x1 , x2 , ..., xn ), . ,, d¯ai ang. . luo. ng f(x1 ) + f(x2 ) + · · · + f(xn ) không t˘ , , `, môt ˜,ng ph´ep dich Mênh d oi nhu ¯ê` 3.4. Tu . bô. sô´ (b1 , b2 , ..., bn ) v´ . chuyên liên . , , , , tiê´p c´ o thê d oi bô. sô´ (a1 , a2 , ..., an ) khi v` a chı khi ¯ua t´ (a1 , a2 , ..., an )  (b1 , b2 , ..., bn ). , ´,ng minh. Phu,o,ng ph´ ´,ng minh cua J. Karamata, G.H. Hardy, J.E. Chu ap chu Littlewood v` a G. Polya. , 1. Ðiê`u kiên a hiên . câ`n: Ðiê`u kiên . (a1 , a2 , ..., an )  (b1 , b2 , ..., bn ) l` , , , , , nhiên sau nh˜ung ph´ep dich chuyên. Thât chuyên t`u . . . vây, . su. dich , ˜ lâ`n thu.,c hiên ´,i (a1 , a2 , ..., an ), sau môi (b1 , b2 , ..., bn ) to . chuyên . ph´ep dich ,, ,´, , , ´, ngh˜ıa l` ta nhân c d¯o a trôi ¯â`u . d¯uo. c bô. sô´ trôi . hon bô. sô´ truo . hon bô. sô´ d tiên (b1 , b2 , ..., bn ). , 2. Ðiê`u kiên ¯u: . d , `˘ng nê´u (a1 , a2 , ..., an )  (b1 , b2 , ..., bn ), th`ı t`u, bô. ´,ng minh ra Ta phai chu , `˘ng ´,ng minh ba ´,i (a1 , a2 , ..., an ). Ta chu sô´ (b1 , b2 , ..., bn ) ta c´ o thê tiê´n to , , , quy nap an hoc . to´ . theo sô´ luo. ng cua bô. sô´. , , ´,i n = 1, kha ˘ng d a hiên nhiên. a) Vo ¯inh . l` ´,i d ˜ c´ ´ ng: vo b) Cho môt od ¯iê`u sau d¯u ¯iê`u kiên . sô´ bâ´t k`ı k, k ≤ n − 1 d¯a . , , ` ´ ˘ng (a1 , a2 , ..., ak )  (b1 , b2 , ..., bk ) thoa m˜ an, th`ı t`u bô. sô (b1 , b2 , ..., bk ) ba , , , , ´ ´ ´,ng ´i bô. sô (a1 , a2 , ..., ak ). Ta phai chu c´ ac ph´ep dich o thê tiên to . chuyên c´ , ´,i k = n. Trong bô. sô´ (b1 , b2 , ..., bn ) ta ˘ng d¯inh ´ ng vo minh kha ung d¯u . trên c˜ , , ´,i b1 , bn (b1 l` ´,n nhâ´t trong liên tuc a sô´ lo . chuyên d¯ô´i vo . thu. c hiên . ph´ep dich , , , , , ´ vê´ bên phai cua cua tâ´t ca bô. sô´, bn l` a sô´ nho nhâ´t trong bô. sô´). Khi d ¯o

, , ,, ´,c cua c´ ´, tu., ˘ng thu Chuong 3. Bâ´t d¯a ac d˜ ay sô´ d ¯ô`ng thu

162

, , , ´,c trong (3.5) d¯ê`u t˘ ´,i môt ˘ng thu ´ bâ´t d¯a ang, ngh˜ıa l` a tiê´n to ao d ¯iêm n` ¯o . th`oi d , , ,, , , ´ c tro th` ´c ˘ng thu ˘ng thu c´ o môt anh d¯a ¯a . bâ´t d a1 + a2 + · · · + a` = b∗1 + b2 + · · · + b` ,

, ,, ´,i cua b1 . od a môt ¯ây b∗1 l` . biê´n mo ,, , , ´,c n` ˘ng thu T`u d¯a ay hê. (3.5) tro th` anh hai phâ`n   ∗    a`+1 ≥ b`+1 ,  a1 ≥ b 1 ,    a  a + a ≥ b∗ + b , `+1 + a`+2 ≥ b`+1 + b`+2 , 1 2 2 1  ...  ...       a`+1 + · · · + an = b`+1 + · · · + b∗n . a1 + · · · + a` = b∗1 + · · · + b` , , , ∗ ´,i (a1 , a2 , ..., a` ) Theo gia thiê´t quy nap, o thê tiê´n to . t`u bô. sô´ (b1 , b2 , ..., b` ) c´ , , ´,i (a`+1 , ..., an ) (v`ı ta luôn c´ o thê tiê´n to o v` a t`u (b`+1 , b`+2 , ..., b∗n ) c´ (a1 , a2 , ..., a` )  (b∗1 , b2 , ..., b` ) v` a (a`+1 , ..., an )  (b`+1 , b`+2 , ..., b∗n )). V`ı , , ´,i nh˜ung d¯iê`u trên ta thâ´y ngay bô. sô´ (b1 , b2 , ..., bn ) c´ o thê tiê´n to ´,ng minh. (a1 , a2 , ..., an ). д o l` ad ¯iê`u câ`n chu , , ,, ´ ng minh d¯inh Ðê chu hai mênh d¯ê` trên. . l´ı 3.5, ta su dung . . , , , V`ı (a1 , a2 , ..., an )  (b1 , b2 , ..., bn ) nên ta c´ o thê thu. c hiên . ph´ep chuyên , ´,i bô. sô´ (a1 , a2 , ..., an ) (mênh dich d ¯ê` 3.4), . t`u bô. sô´ (b1 , b2 , ..., bn ) tiê´n to . , , , ´i c´ Vo ac ph´ep chuyên dich ang (mênh . tông f(x1 ) + f(x2 ) + · · · + f(xn ) t˘ .

J

d ¯ê` 3.3).

, , ´,c Fuchs). Cho f(x) l` ˘ng thu Hê. qua 3.1 (Bâ´t d a h` am lô`i, p1 , p2 , ..., pn l` a ¯a , , , ˜ ng sô´ duong, x1 ≥ x2 ≥ · · · ≥ xn , y1 ≥ y2 ≥ · · · ≥ yn sao cho nhu k X i=1

pi x i ≥

k X i=1

n n X X , pi yi v´ oi 1 ≤ k ≤ n − 1 v` a pi x i = pi y i . i=1

i=1

´ Khi d ¯o p1 f(x1 ) + p2 f(x2 ) + · · · + pn f(xn ) ≥ p1 f(y1 ) + p2 f(y2 ) + · · · + pn f(yn ).

, ´,c Karamata ˘ng thu 3.4. Chuyên d ¯ê` vê` bâ´t d ¯a

163

˘. t p1 + p2 + · · · + pn = 1 v` Nê´u ta d a ¯a y1 = y2 = ... = yn = p1 x1 + p2 x2 + · · · + pn xn , , , , ´,c Fuchs2 tro, th` ´,c Jensen3 . ˘ng thu ˘ng thu th`ı bâ´t d¯a anh bâ´t d ¯a , , ´,c Jensen). Cho f(x) l` ˘ng thu Hê. qua 3.2 (Bâ´t d a môt am lô`i trên d ¯oan ¯a . h` . , , ˜ ng sô´ trên d ˜ [a, b], x1 , x2 , ..., xn l` a nhu o an [a, b], p , p , ..., p l` a nh u ng sô´ ¯ . 1 2 n , ,, ´ duong bâ´t k`ı c´ o tông ba ˘`ng 1. Khi d ¯o f(p1 x1 + · · · + pn xn ) ≤ p1 f(x1 ) + · · · + pn f(xn ).

, , , , , ´,c trên suy ra h` ´,c nôi ˘ng thu ˘ng thu T`u nh˜ung bâ´t d¯a ang loat ac bâ´t d¯a . c´ , , ´,c Cauchy, Chebyshev, ... ˘ng thu tiê´ng kh´ ac nhu bâ´t d¯a , ,, , ˘ ng th´ 3.4.2. Su dung bâ´t d¯a uc Karamata . , , , ˜,ng sô´ du,o,ng bâ´t k`ı a, b v` V´ı du. 3.39. V´ oi nhu a c ta c´ o bâ´t d ˘ng thu´c ¯a 1 1 1 1 1 1 + + ≤ + + . a+b b+c c+a 2a 2b 2c , , , , , `,i giai. V`ı vai tr` Lo o cua a, b, c nhu nhau, nên ta c´ o thê gia thiê´t a ≥ b ≥ c. ,, , ´ d˜ê thâ´y nh˜ung bô. sô´ (2a, 2b, 2c)  (a + b, a + c, b + c). Su dung Khi d¯o . , 1 , ´,c Karamata cho h` ´i hai bô. sô´ trên suy ra d¯iê`u câ`n ˘ng thu bâ´t d¯a am lô`i vo x ´,ng minh. chu

J

, ,, V´ı du. 3.40 (Ðê` thi Olympic to´ an khu vu. c Th´ ai B`ınh Duong 1996). Cho , , ˜,ng d a, b v` a c l` a nhu ai c´ ac canh cua môt ac. Chu´ng minh ra ˘`ng khi ¯ô. d` . . tam gi´ , , ´ ta c´ ´ng d o bâ´t d ˘ng thu´c sau d ¯o ¯a ¯u √ √ √ √ √ √ a + b − c + b + c − a + c + a − b ≤ a + b + c.

, , , , , `,i giai. Do vai tr` ´,c, nên ta c´ ˘ng thu Lo o a, b, c nhu nhau trong bâ´t d¯a o thê gia , ´ suy ra (a + b − c, c + a − b, b + c − a)  (a, b, c). thiê´t a ≥ b ≥ c. T`u d ¯o 2 Lazarus 3 Johan

Immanuel Fuchs (1833-1902): Nh` a to´ an hoc . Ba Lan. Ludwig William Valdemar Jensen (1859-1925): Nh` a to´ an hoc . Ðan Mach. .

, , ,, ´,c cua c´ ´, tu., ˘ng thu Chuong 3. Bâ´t d¯a ac d˜ ay sô´ d ¯ô`ng thu

164

, √ ,, ´,c Karamata cho h` ˘ng thu Su dung a hai bô. sô´ trên s˜e cho am l˜ om x v` ¯a . bâ´t d , kê´t qua.

J

, ,, , ˜, V´ı du. 3.41 (IMO 2000). V´ oi nhu ng sô´ duong a, b, c > 0 thoa m˜ an d ¯iê`u kiên . , , , ` ´ng abc = 1. Chu´ng minh ra ˘ng bâ´t d ˘ng thu´c sau d¯u ¯a     1 1 1 b−1+ c−1+ ≤ 1. a−1+ b c a , ,, `,i giai. T`u, d¯iê`u kiên Lo ac sô´ duong x, y, z sao cho . abc = 1, suy ra tô`n tai . c´ , x y z ´,c d¯a ˘ng thu ´ bâ´t d¯a ˜ cho c´ a = , b = , c = . Khi d¯o o dang . y z x (x − y + z)(y − z + x)(z − x + y) ≤ xyz.

(3.7)

,, ,, , , , X´et tru`ong ho. p ca ba th`ua sô´ x − y + z, y − z + x, z − x + y d¯ê`u duong. Ta lâ´y lôgarit hai vê´ ln(x − y + z) + ln(y − z + x) + ln(z − x + y) ≤ ln x + ln y + ln z. , , , , ´,c nên ta gia thiê´t x ≥ y ≥ z. ˘ng thu Vai tr` o cua x, y, z nhu nhau trong bâ´t d¯a ,, ´ ta c´ Khi d¯o o c´ ac bô. sô´ (y − z + x, x − y + z, z − x + y)  (x, y, z). Su dung . , , , , ´ ´ ´ ´ ´ ˘ng ˘ng thuc Karamata voi h` am l˜ om ln x v` a hai bô. sô trên suy ra bât d¯a bât d¯a , , ´, ,, , ´,c (3.7) vo ´,c (3.7). Ban ´,i c´ ˘ng thu ac tru`ong ho. p thu ng minh bâ´t d ¯oc ¯a . d . tu. chu

J

c` on lai. .

, ´,c Szeg). Cho ϕ(x) l` ˘ng thu V´ı du. 3.42 (Bâ´t d¯a a h` am lô`i v` a a1 ≥ a2 ≥ ... ≥ ´ a2n−1 , khi d ¯o

ϕ(a1 ) − ϕ(a2 ) + ϕ(a3 ) − · · · + ϕ(a2n−1 ) ≥ ϕ(a1 − a2 + a3 − · · · + a2n−1 ). , , `,i giai. Ta d ´,c Szeg4 ˘. t a = a1 − a2 + a3 − a4 + · · · + a2n−1 . Bâ´t d¯a ˘ng thu Lo ¯a , d¯ua vê` dang . ϕ(a1 ) + ϕ(a3 ) + · · · + ϕ(a2n−1 ) ≥ ϕ(a) + ϕ(a2 ) + · · · + ϕ(a2n−2 ). 4 G´ abor

,, Szeg (1895-1985): Nh` a to´ an hoc . ngu`oi Hungari.

, ´,c Karamata ˘ng thu 3.4. Chuyên d ¯ê` vê` bâ´t d ¯a

165

, , ´,ng minh Nhu vây on chu . ta chı c` (a1 , a3 , ..., a2n−3 , a2n−1 )  (a2 , a4 , ..., a2n−4 , a2n−2 , a), , , , `˘ng nhau v` ´,i ´ l` nhung d a hiên nhiên v`ı tông c´ ac sô´ ba a a2k−1 ≥ a2k vo ¯iê`u d¯o

J

moi . k = 1, 2, ..., n.

, V´ı du. 3.43 (IMO 1999). Cho n ≥ 2. H˜ ay x´ ac d ˘`ng sô´ C nho nhâ´t sao ¯inh . ha , , cho bâ´t d¯a ˘ng thu´c !4 n X X 2 2 xi xj (xi + xj ) ≤ C xi 1≤i 2, ta c´ o x1 + x2 + · · · + xn > 0 (v`ı nê´u x1 + x2 + · · · + , , ´,c l` ˘ng thu ´ bâ´t d¯a a hiên nhiên). xn = 0 th`ı moi . xi = 0, i = 1, 2, ..., n. Khi d¯o xi ´,i i = 1, 2, ..., n, khi d¯o ˘. t ai = ´ a1 +a2 +· · ·+an = 1. vo Ða x1 + x2 + · · · + xn , ´,c câ`n chu ´,ng minh v` ˘ng thu Thay xi = ai (x1 + x2 + · · · + xn ) v` ao bâ´t d¯a a r´ ut

gon anh dang: . th` .

X

ai aj (a2i + a2j ) ≤ C. 1≤i 0 trên 0, . 2 3

3

4

166

, , ,, ´,c cua c´ ´, tu., ˘ng thu Chuong 3. Bâ´t d¯a ac d˜ ay sô´ d ¯ô`ng thu

, , , ´,c nhu, nhau, không mâ´t t´ınh tông qu´ ˘ng thu at V`ı vai tr` o cua ai trong bâ´t d ¯a , , ta c´ o thê gia thiê´t a1 ≥ a2 ≥ · · · ≥ an .   , 1 1 1 ˘ng Nê´u a1 ≤ , th`ı ta c´ o , , 0, ..., 0  (a1 , a2 , ..., an ). Theo bâ´t d¯a 2 2 2 ´,c Karamata, ta c´ thu o     1 1 1 +f + f(0) + · · · + f(0) = . f(a1 ) + f(a2 ) + · · · + f(an ) ≤ f 2 2 8   1 Nê´u a1 > , th`ı 1 − a1 , a2 , ..., an ∈ 0, 21 . Ta x´et hai bô. sô´ gô`m n − 1 2 , ,, ´,c Karamata ˘ng thu phâ`n tu v` a (1 − a1 , 0, ..., 0)  (a2 , a3 , ..., an ), theo bâ´t d ¯a

suy ra

f(a2 ) + · · · + f(an ) ≤ f(1 − a1 ) + f(0) + · · · + f(0). , ,, ,, , ´,c trên vo ´,i f(a1 ) v` ˘ng thu Công ao hai vê´ bâ´t d¯a a su dung . v` . tru`ong ho. p n = 2 ta c´ o 1 f(a1 ) + f(a2 ) + · · · + f(an ) ≤ f(a1 ) + f(1 − a1 ) ≤ . 8 , , , `˘ng nhau v` ´,c xay ra khi v` ˘ng thu Ða a chı khi hai biê´n ba a n − 2 biê´n kh´ ac d¯ê`u ` ˘ng 0. ba

J

, , ´ ˘ ng th´ 3.4.3. Môt uc khac . sô´ bâ´t d¯a , , , , 12 12 12 V´ı du. 3.44. T`ım gi´ a tri. cu. c d ¯ai . cua biêu thu´c a + b + c , nê´u 1 −1 ≤ a, b, c ≤ 1 v` aa+b+c=− . 2 , , `˘ng h` `,i giai. Ta ch´ Lo u ´y ra am f(x) = x12 l` a liên tuc a lô`i trên khoang [−1, 1], . v` , , , v`ı f 00 (x) = 132x10 ≥ 0 trên (−1, 1). Vai tr` o cua a, b, c nhu nhau trong biêu , , , ´,c, nên không mâ´t t´ınh tông qu´ thu at ta c´ o thê gia thiê´t −1 ≤ c ≤ b ≤ a ≤ 1, 1 `˘ng (1, − 1 , −1)  (a, b, c). ´,ng minh ra ´,i a + b + c = − , ta s˜e chu c` ung vo 2 2 , , `˘ng nhau, ho,n n˜u,a Thât . vây, . d˜ê thâ´y tông cua hai bô. ba d¯ê`u ba 1 ≥ a v` a

1 1 1 = 1 − ≥ −c − = a + b. 2 2 2

, ´,c Karamata ˘ng thu 3.4. Chuyên d ¯ê` vê` bâ´t d ¯a

167

, ´,c Karamata ta c´ ˘ng thu Theo bâ´t d¯a o 1 1 a12 + b12 + c12 = f(a) + f(b) + f(c) ≤ f(1) + f(− ) + f(−1) = 2 + 12 . 2 2 1 1 , ,, Vây a tri. cu. c d¯ai a 2 + 12 d a c = −1. ¯at ¯uo. c khi a = 1, b = − v` . gi´ . l` . d 2 2 , , , ´,c tap ´,ng minh thoa m˜ Nh˜ung v´ı du. sau d ut khi chu an ¯ây phu . hon môt . ch´ , , , , ´ c Karamata. ˘ng thu nh˜ung d ¯iê`u kiên . cua bâ´t d¯a

J

, V´ı du. 3.45. Cho x1 , x2 , ..., xn ∈ [− π6 , π6 ]. Chu´ng minh ra ˘`ng cos(2x1 − x2 ) + cos(2x2 − x3 ) + · · · + cos(2xn − x1 ) ≤ cos x1 + · · · + cos xn .

, , , π π `,i giai. V`ı h` Lo am cos x l˜ om trên d¯oan ¯iê`u kiên . . [− 2 , 2 ], ta chı câ`n kiêm tra d , , ´ ´ ´ ˘ (3.5) l` ad u v o i hai b ô sô (2x − x , ..., 2x − x ) v` a (x , x , ..., x ). Ta s a p ¯ 1 2 n 1 1 2 n . xê´p lai ay th` anh c´ ac bô. sô´ (2xm1 − xm1 +1 , ..., 2xmn − xmn +1 ) . hai bô. sô´ n` , , , ´,c: ˘ng thu v` a (xk1 , xk2 , ..., xkn ) thoa m˜ an nh˜ung bâ´t d¯a 2xm1 − xm1 +1 ≥ 2xm2 − xm2 +1 ≥ ... ≥ 2xmn − xmn +1 , xk1 ≥ xk2 ≥ ... ≥ xkn , ,, , , , ´,i quy u,o ´,c xn+1 = x1 ). Ðiê`u n` (vo ay thu. c hiên . d¯uo. c. Hon n˜ua nhung ta thâ´y `˘ng 2xm1 − xm1 +1 ≥ 2xk1 − xk1 +1 ≥ xk1 v` a ra (2xm1 − xm1 +1 ) + (2xm2 − xm2 +1 ) ≥ (2xk1 − xk1 +1 ) + (2xk2 − xk2 +1 ) ≥ xk1 + xk2 . , , , ,, , , ´, nhâ´t không nho ho,n tông ` Tuong tu. tông ` sô´ d ¯â`u tiên cua bô. sô´ thu , , , ´, hai. Thu.,c tê´ n´ sô´ cua bô. sô´ thu o không nho hon (2xk1 − xk1 +1 ) + · · · + , , , , , (2xk` −xk` +1 ), nhung tông n` ay không nho hon xk1 +xk2 +· · ·+xk` (c´ o thê , `˘ng quy nap). ´,ng minh ba Nhu vây, chu . . (2xm1 − xm1 +1 , ..., 2xmn − xmn +1 )  (xk1 , xk2 , ..., xkn ).

˜,ng sô´ du,o,ng. Chu´,ng minh ra V´ı du. 3.46. Cho a1 , a2 , ..., an l` a nhu ˘`ng a31 a32 a3 + + · · · + n ≥ a21 + a22 + · · · + a2n . a2 a3 a1

J

168

, , ,, ´,c cua c´ ´, tu., ˘ng thu Chuong 3. Bâ´t d¯a ac d˜ ay sô´ d ¯ô`ng thu

, , , `,i giai. Thu.,c hiên ´,c câ`n chu ´,ng ˘ng thu Lo a viê´t lai ¯ôi biê´n xi = ln ai v` ¯a . d . bâ´t d minh e3x1 −x2 + e3x2 −x3 + · · · + e3xn −x1 ≥ e2x1 + e2x2 + · · · + e2xn .

, ,, , , ,´, ´, T`u d an to` an tuong tu. nhu b` ai to´ an truo c voi h` am lô`i f(x) = ex ¯ây l´ı luân . ho` v` a bô. sô´ (3x1 − x2 , ..., 3xn − x1 )  (2x1 , 2x2 , ..., 2xn ).

J

, , , ´ ˘ ng th´ 3.4.4. Nh˜ung bâ´t d¯a uc trong tam giac , , , ´,c Karamata ta c´ ´,c ´,i bâ´t d¯a ˘ng thu ˘ng thu Vo o thê s´ ang tao ang loat . h` . bâ´t d¯a ,, , ´,i nhau, sau d¯ây ch´ vê` c´ ac phâ`n tu trong môt ac vo ung ta d¯ua ra môt . tam gi´ . , , , ,`, , ` ´ ´ ´ ˘ng thuc m` ˘ng c´ ˜ t`ım ra ba sô bât d a nguoi ta d¯a ach l` am nhu trên. Ðê cho ¯a , , ´ ´ ˜ thông nhât ta k´ı hiêu a nhung g´ oc cua tam gi´ ac; a, b, c l` a d¯ô. d` ai . α, β, γ l` , ,, a+b+c , , ´, , ´i g´ l` a nua chu vi cua tam c´ ac canh tuong ung vo oc trên; p = . 2 gi´ ac; x = p − a, y = p − b, z = p − c. , V´ı du. 3.47. Cho tam gi´ ac nhon ˘`ng . ABC, chu´ng minh ra 1 < cos α + cos β + cos γ ≤

, , v` a x´ ac d ao th`ı c´ od ˘ng thu´c? ¯inh ¯a . khi n`

3 2

, , , π `,i giai. Không mâ´t t´ınh tông qu´ ´α≥ Lo at ta gia thiê´t α ≥ β ≥ γ. Khi d ¯o 3 , π π π 2π , , , ´ ´ c > α ≥ , π > α + β(= π − γ) ≥ ˘ng thu v` a γ ≤ . T`u nh˜ung bât d , ¯a 3 2 3 3 suy ra π π  π π π , , 0  (α, β, γ)  , , . h πi 2 2 3 3,3 ´ ´,c Karamata ta c´ ˘ng thu o V`ı f(x) = cos x l` a h` am l˜ om trên 0, , theo bât d¯a 2 π π 1=f +f + f(0) 2 2 ≤ f(α) + f(β) + f(γ) = cos α + cos β + cos γ π π π 3 ≤f +f +f = . 3 3 3 2

, ´,c Karamata ˘ng thu 3.4. Chuyên d ¯ê` vê` bâ´t d ¯a

169

, , `˘ng xay ra v`ı trong môt ´,c vê´ tr´ ˘ng thu Bâ´t d¯a ai không c´ o dâ´u ba ac . tam gi´ , , , , , ` ´ c vê´ phai dâ´u ba ˘ng thu ˘ng xay ra khi v` không thê c´ o hai g´ oc vuông. Bâ´t d a ¯a , chı khi ABC l` a tam gi´ ac d¯ê`u.

J

, , ´,c vê` c´ ˘ng thu I. Nh˜ ung bâ´t d¯a ac canh: . , , , , , 1. Vai tr` o cua c´ ac canh cua môt ac nhu nhau, nên ta c´ o thê gia thiê´t . . tam gi´ , ´ nh˜ung quan hê. sau d¯u ´ ng cho tam gi´ a ≥ b ≥ c. Khi d¯o ac bâ´t k`ı (ban . d¯oc . , , tu. kiêm tra).   p p p 2p 2p 2p , , , , . (p, p, 0)  (a, b, c)  , (p, 0, 0)  (z, y, x)  3 3 3 3 3 3 , ´,c sau: ˘ng thu Ta c´ o c´ ac bâ´t d¯a 1 a 2 + b 2 + c2 1 a) ≤ < ; 3 (a + b + c)2 2 1 bc + ca + ab 1 b) < ≤ ; 4 (a + b + c)2 3 √ √ √ √ √ c) p < p − a + p − b + p − c < 3p; p p p √ d) a(p − a) + b(p − b) + c(p − c) ≤ 2p; 1 (b + c)(c + a)(a + b) 8 e) < ≤ ; 4 (a + b + c)3 27 9 1 1 1 f) ≤ + + . p p−a p−b p−c   , `˘ng (a, b, c)  a + b , c + a , b + c , ta c´ 2. D˜ê kiêm tra thâ´y ra o 2 2 2 8abc ≤ (b + c)(c + a)(a + b). ´,i (2x, 2y, 2z) ≺ (a, b, c) ta c´ 3. Vo o a) 8xyz ≤ abc; 2xyz abc b) ≤ ; xy + yz + zx ab + bc + ca c) 32xyz(xy + yz + zx) ≤ abc(ab + bc + ca); , ´,c vê` c´ ˜,ng bâ´t d ˘ng thu II. Nhu ac g´ oc: ¯a

170

, , ,, ´,c cua c´ ´, tu., ˘ng thu Chuong 3. Bâ´t d¯a ac d˜ ay sô´ d ¯ô`ng thu

, , ´ ta c´ Ta c´ o thê gia thiê´t α ≥ β ≥ γ. Khi d o quan hê. ¯o π π π (π, 0, 0)  (α, β, γ)  , , . 3 3 3 ,, ´,i k < 0; l` ´,i 0 < k ≤ 1 trên 1. Su dung h` am f(x) = sink x l` a lô`i vo a l˜ om vo . , π ´,c sau d¯u ´,i k ≥ 2 trên [0, ]. Nh˜u,ng bâ´t d ˘ng thu ´ ng [0, π] v` a lô`i vo ¯a 4 k 31+ 2 ´,i k < 0, sink α + sink β + sink γ ≥ ; a) Vo 2k k 31+ 2 , k k k ´i 0 < k ≤ 1, 0 < sin α + sin β + sin γ ≤ b) Vo ; 2k 3 α β γ ´,i k < 0, k ≤ sink + sink + sink ; c) Vo 2 2 2 2 γ 3 , k α k β ´i 0 < k ≤ 1, 1 < sin d) Vo + sin + sink ≤ k ; 2 2 2 2 ,, 2. H` am sô´ f(x) = lg sin x l` a h` am l˜ om trên (0, π), o d¯ây lg l` a k´ı hiêu am . h` , , , , ´ ´ c sau d ˘ng thu ´ ng: lôgarit co sô 10. Nh˜ung bâ´t d¯a ¯ây d ¯u √ 3 3 0 < sin α sin β sin γ ≤ . 8 π ´,i k < 0; l` ´,i 0 < k ≤ 1 trên [0, ] v` a 3. H` am sô´ f(x) = cosk x l` a lô`i vo a l˜ om vo 2 , π ´,c sau d¯u ´,i k ≥ 2 trên [0, ]. Nh˜u,ng bâ´t d¯a ˘ng thu ´ ng l˜ om vo 4 k 31+ 2 α β γ ´,i k < 0, a) Vo ≤ cosk + cosk + cosk ; k 2 2 2 2 k α β γ 31+ 2 ´,i 0 < k ≤ 1, 2 < cosk + cosk + cosk ≤ b) Vo ; 2 2 2 2k , π ´,c sau d¯ây ˘ng thu 4. H` am sô´ f(x) = lg cos x l` a h` am l˜ om trên (0, ). Bâ´t d¯a 2 ´ ng: d¯u √ α β γ 3 3 0 < cos cos cos ≤ . 2 2 2 8

` tâp Bai . . 3.48. Nê´u n ≥ 2 l` a môt a 0 < a1 < a2 < ... < a2n+1 l` a . sô´ nguyên v`

, , ,, , `i b` 3.5. Go. i ´y v` a tra lo ai tâp . chuong 3

171

, , `˘ng ´,ng minh ra nh˜ung sô´ thu. c, chu √ n

a1 −

√ n

a2 +

√ n

a3 − · · · −

√ n

√ a2n + n a2n+1 < √ < n a1 − a2 + a3 − · · · − a2n + a2n+1 .

, , ,, ´,ng minh bâ´t d ´,c sau d ´,i x, y, z l` ˘ng thu ´ ng vo . 3.49. Chu a nh˜ung sô´ duong ¯a ¯u r r r xy xz zy xy xz zy + + ≥ + + . z2 y2 x2 z2 y2 x2 , ,, `˘ng ´,ng minh ra . 3.50. Cho a, b, c, d l` a nh˜ung sô´ duong. Chu a4 + b4 + c4 + d4 + 2abcd ≥ a2 b2 + a2 c2 + a2 d2 + b2 c2 + b2 d2 + c2 d2 . , ,, `˘ng ´,ng minh ra . 3.51. Cho a1 , a2 , ..., an l` a nh˜ung sô´ duong. Chu       a22 a2n a21 1+ ··· 1 + . (1 + a1 )(1 + a2 ) · · · (1 + an ) ≤ 1 + a2 a3 a1 , ´,ng minh bâ´t d ´,c Karamata c´ ˘ng thu . 3.52. Chu o trong ¯a . sô´:

m1 f(x1 ) + m2 f(x2 ) + · · · + mn f(xn ) ≥ m1 f(y1 ) + m2 f(y2 ) + · · · + mn f(yn ), ,, o d¯ây f l` a môt am lô`i v` a hai bô. sô´ (x1 , x2 , ..., xn ) v` a (y1 , y2 , ..., yn ) c` ung . h` , , , d¯on d¯iêu a thoa m˜ an hê. sau . giam v`   m1 x1 ≥ m1 y1 ,       m1 x1 + m2 x2 ≥ m1 y1 + m2 y2 , ...    m1 x1 + · · · + mn−1 xn−1 ≥ m1 y1 + · · · + mn−1 yn−1 ,     m x + ··· + m x = m y + ··· + m y . 1 1 n n 1 1 n n

(mi ∈ R).

, , ,, , ` tra l`oi bai ` tâp 3.5. Go. i y ´ va . chuong 3 , ,, , ´ ln a ≥ ln b. Nhu vây . 3.9. Gia su a ≥ b, khi d¯o a (ln a, ln b) d¯ô`ng . (a, b) v` , ´, tu.,. T`u, mênh ´ suy ra thu d o a ln a + b ln b ≥ a ln b + b ln a. T`u d¯o ¯ê` 3.1, ta c´ . a b b a a b ≥a b .

, , ,, ´,c cua c´ ´, tu., ˘ng thu Chuong 3. Bâ´t d¯a ac d˜ ay sô´ d ¯ô`ng thu

172

, , ,, ´,c câ`n chu ´,ng minh tu,o,ng ˘ng thu . 3.10. Do a, b l` a nh˜ung sô´ duong, bâ´t d ¯a ,, ´,i d¯uong vo a+b  a+b ≥ a b ba . 2 , , , , a+b √ ´ ng minh bâ´t d¯a ´,c trên ta chı câ`n ˘. t kh´ ˘ng thu ≥ ab, nên d Ma ac, ¯ê chu 2 , ´,ng minh bâ´t d ´,c sau: ˘ng thu chu ¯a √ a+b ab ≥ a b ba . , , ,, ,, ´,ng minh bâ´t d ´,c aa bb ≥ ab ba , d ´,i chu ˘ng thu Tuong d¯uong vo a kê´t qua ¯a ¯ây l` , cua b` ai 3.9. , ,, ´ hai bô. sô´ (a, b, c) v` . 3.11. Gia su a ≥ b ≥ c. Khi d¯o a (a2 , b2 , c2 ) l` a d¯ô`ng ,, , , ´ tu. . Su dung thu mênh d¯ê` 3.2 ta c´ o . . a3 + b3 + c3 ≥ aa2 + bb2 + cc2 , a3 + b3 + c3 ≥ ab2 + bc2 + ca2 , a3 + b3 + c3 ≥ ac2 + ba2 + cb2 .

, , ,, ´,c trên ta nhân ´,ng minh. ˘ng thu Công ac bâ´t d¯a ¯uo. c kê´t qua câ`n chu . theo vê´ c´ . d , ,, ´,c Cauchy cho 5 sô´ du,o,ng: ˘ng thu . 3.12. Su dung bâ´t d¯a . a15 , a15 , b10 , b10 , b10 . , ,, ´,c Cauchy cho 7 sô´ du,o,ng: a21 , a21 , a21 , ˘ng thu . 3.13. Su dung bâ´t d¯a . a21 , b14 , b14 , b14 . , , , ,, ´,c d¯ô´i xu ´,ng theo a, b, c nên ta c´ ˘ng thu . 3.19. Do bâ´t d o thê gia su ¯a 1 1 1 ´ a + b ≤ c + a ≤ b + c. Do d¯o ´ a ≤ b ≤ c. Khi d¯o ≤ ≤ . b+c c+a a+b Theo d¯inh . l´ı 3.1: a3 b3 c3 a3 b3 c3 + + ≤ + + , a+b b+c c+a b+c c+a a+b a3 b3 c3 a3 b3 c3 + + ≤ + + . c+a a+b b+c b+c c+a a+b

, , ,, , `i b` 3.5. Go. i ´y v` a tra lo ai tâp . chuong 3

173

, ,, ´,c trên, rô`i chia cho 2, ta nhân ˘ng thu Công ¯a . theo vê´ hai bâ´t d . d¯uo. c   b3 c3 a3 1 a3 + b3 b3 + c3 c3 + a3 + + + + . ≤ 2 a+b b+c c+a b+c c+a a+b , x2 + y2 x 3 + y3 ´,c ˘ng thu = x2 − xy + y2 ≥ , ta c´ o Do bâ´t d ¯a x+y 2   a2 + b2 + c2 1 a2 + b2 b2 + c2 c2 + a2 = + + 2 2 2 2 2  3  3 3 3 3 1 a +b b +c c + a3 ≤ + + 2 a+b b+c c+a ≤

a3 b3 c3 + + . b+c c+a a+b

, ,, 1 1 1 ´ . 3.20. Gia su 0 < a ≤ b ≤ c. Khi d ≤ ≤ v` a bc ≥ ¯o b + c c + a a + b   1 1 1 ´ hai bô. ba ca ≥ ab. Do d¯o , , v` a (bc, ca, ab) l` a nghich . b+c c+a a+b , , , ´ tu. . Theo d d¯ao thu ¯inh . l´ı 3.1 ca ab ab bc ca bc + + ≤ + + , b+c c+a a+b b+c c+a a+b bc ca ab ca ab bc + + ≤ + + . b+c c+a a+b b+c c+a a+b , ,, ´,c trên, ta nhân ˘ng thu Công ¯a . theo vê´ hai bâ´t d . d¯uo. c   bc ca ab 2 + + ≤ a + b + c. b+c c+a a+b , ,, . 3.21. Gia su x1 ≥ x2 ≥ ... ≥ xn > 0, suy ra x21 ≥ x22 ≥ ... ≥ x2n v` a  1 1 1 1 1 1 ´ hai bô. sô´ (x21 , x22 , ..., x2n ) v` ≤ ≤ ... ≤ . Do d¯o a , , ..., x1 x2 xn x x xn , , 1 2 , , ´ ´ ´p dung l` a nghich aa d ¯inh . d¯ao thu tu. v` . l´ı 3.1 s˜e cho kêt qua. . , ,, ´ hai bô. sô´ (an , bn , cn ) v` . 3.22. Gia su a ≤ b ≤ a   c. Khi d¯o 1 1 1 ´, tu.,. , , l` a d¯ô`ng thu b+c c+a a+b , , . 3.23. Ban ¯oc . d . tu. giai.

174

, , ,, ´,c cua c´ ´, tu., ˘ng thu Chuong 3. Bâ´t d¯a ac d˜ ay sô´ d ¯ô`ng thu

, ,, ´ hai bô. sô´ (A, B, C) v` . 3.24. Gia su A ≤ B  ≤ C. Khi d a ¯o  1 1 1 , , ´ tu. v` ´p dung , , d¯ô`ng thu aa môt . l´ı 3.1 s˜e . . sô´ lâ`n d¯inh p−a p−b p−c , cho kê´t qua. , , ,, ´,c Chebyshev cho ˘ng thu ´p dung . 3.34. Gia su a ≥ b ≥ c ≥ d v` aa bâ´t d ¯a . , ˜ bô. ba sô´ d¯o,n d¯iêu môi ac sô´ trên rô`i công ung lai. . giam trong c´ . ch´ . , , ,, 1 1 ´ ≥ ≥ . 3.35. Ta c´ o thê gia su a1 ≥ a2 ≥ · · · ≥ an . Khi d¯o 2a − a1 2a − a2 1 ´,i quy u,o ´,c an+i = ai . Vo ´,i m = 0, 1, 2, ..., n − 1, theo d¯inh ··· ≥ . Vo . 2a − an , , l´ı 3.1, ta nhân d u o c ¯ . . n n X X am+i ai ≤ . 2a − ai 2a − ai i=1

i=1

, Pn nai ai , , Pn ´,c n` ˘ng thu Công n bâ´t d ≤ i=1 . ay, ta nhân ¯a ¯uo. c i=1 . . d 2a − ai 2a − ai a 1 1 ai , ,, T`u = + , ta nhân ¯uo. c . d 2a − ai 2 2 2a − ai n n X n 1 X ai ai + ≤n . 2 2 2a − ai 2a − ai i=1 i=1 , , ´,c câ`n chu ´,ng minh. ˘ng thu T`u d¯ây suy ra bâ´t d ¯a ,, ´,i a, b ≥ 0 v` ˘. p sô´ (a, b) v` . 3.36. Vo a n l` a sô´ nguyên duong, ca a (an−1 , bn−1 ) , , , , ,, ´ c Chebyshev, ta nhân ´ tu. . Do d ˘ng thu ´, theo bâ´t d¯a l` a d¯ô`ng thu ¯uo. c ¯o . d

1 (a + b)(an−1 + bn−1 ). 2 , ,, ´,c Chebyshev, cuô´i c` ˘. p lai ˘ng thu ´p dung ung nhân La bâ´t d ¯a . d¯uo. c . viêc . a . an + bn ≥

an + bn ≥

1 (a + b)n . 2n−1

, `˘ng c´ ´,c trên s˜e ˘. t a = sin2 x v` ˘ng thu Ba ach d¯a a b = cos2 x v` a thay v` ao bâ´t d ¯a , ´,ng minh. cho kê´t qua câ`n chu bk ´,i ˘. t ck = . 3.37. Ða v` a dk = (c1 − 1) + (c2 − 1) + · · · + (ck − 1) vo ak

, , ,, , `i b` 3.5. Go. i ´y v` a tra lo ai tâp . chuong 3

175

, , ´,c Cauchy v` ˘ng thu ˜ cho, ta c´ 1 ≤ k ≤ n. Theo bâ´t d a gia thiê´t d o ¯a ¯a √ c1 + c2 + · · · + ck ≥ k c1 c2 ...ck ≥ 1, k , ´ suy ra dk ≥ 0. Cuô´i c` t`u d¯o ung ta c´ o (b1 + b2 + · · · + bn ) − (a1 + a2 + · · · + an ) = (c1 − 1)a1 + (c2 − 1)a2 + · · · + (cn − 1)an = d1 a1 + (d2 − d1 )a2 + · · · + (dn − dn−1 )an = d1 (a1 − a2 ) + d2 (a2 − a3 ) + · · · + dn an ≥ 0. , ,, , . 3.38. Gia su 0 < A ≤ B ≤ C < π2 . X´et hai bô. (tgA, tgB, tgC) d ¯ on , , , ´,c ˘ng thu ´p dung d¯iêu ang, (cos A, cos B, cos C) l` ad aa bâ´t d ¯on d¯iêu ¯a . t˘ . giam v` . Chebyshev.

, , CHUONG

4

,, ` Phuong tr`ınh ham

,, , ´ thê´ nh˜ung gia ´ tr.i dô 4.1. Phuong phap ¯ ´i sô´ . . . . . . . . . . . . . . . . . . . . . . . . . . . . . . . . . , ,, ´ diê 4.2. Phuong phap .......................................... ¯ m bâ´t d¯ông . , ´ da ´ ................................ 4.3. Chuyên dê uc giao hoan ¯ ` vê` cac ¯ th´

4.3.1. Ðinh . ngh˜ıa . . . . . . . . . . . . . . . . . . . . . . . . . . . . . . . . . . . . . . . ´,c Chebyshev . . . . . . . . . . . . . . . . . . . . . . . . . . . . . . . 4.3.2. Ða thu , ´,c giao ho´ 4.3.3. B` ai to´ an tông qu´ at vê` c´ ac d¯a thu an . . . . . . .

, , ,, , ` tra l`oi bai ` tâp 4.4. Go. i y´ va . chuong 4 . . . . . . . . . . . . . . . . . . . . . . . . . . . . . . . . . . . . . .

176 187 194

194 197 200 203

, , ,, ,, ,, Phuong tr`ınh h` am l` a phuong tr`ınh m` a ân sô´ l` a c´ ac h` am sô´. Giai phuong , , , ,, ˜ cho tr`ınh h` am ngh˜ıa l` a t`ım tâ´t ca nh˜ung h` am sô´ thoa m˜ an phuong tr`ınh d¯a ,´, ,, , ´ ` ´ truoc. C´ o rât nhiêu dang ac nhau nhu trong cuôn s´ ach [5] . phuong tr`ınh kh´ ˜ liêt d¯a . kê.

,, , ´ thê´ nh˜ung gia ´ tr.i dô 4.1. Phuong phap ¯ ´i sô´ , , ,, ,, , Phuong ph´ ap chung d¯ê giai c´ ac phuong tr`ınh h` am l` a không c´ o. Nhung , , ,, ,, , c´ o phuong ph´ ap thê´ c´ ac gi´ a tri. biê´n v` ao phuong tr`ınh d¯ê giai, thu. c châ´t , ,, ˜ cho khi ta thay gi´ d¯ây c˜ ung l` a dang a tri. biê´n . bâ´t biê´n cua phuong tr`ınh d¯a , ,, , ,, ,´, ˜ d ´ ng. C˜ kh´ ac nhau phuong tr`ınh vân ung nhu chuong truo c, khi ta thay d¯ôi ¯u , , , ´,c d ´,ng không d vai tr` o cua c´ ac biê´n cho nhau th`ı d a tri. v` a t`u ¯a thu ¯ô´i xu ¯ôi gi´

,, ˜,ng gi´ 4.1. Phuong ph´ ap thê´ nhu a tri. d¯ô´i sô´

177

, ´, ´,c d ´,ng v` ´ d¯a ˜ t`ım ra nh˜ung u d¯o ng dung a cho t´ınh bâ´t ¯ô´i xu . râ´t hay cho d¯a thu , , , , ´ c d¯ô´i xu ´ ng. Trong muc ´i d¯a thu biê´n d¯ô´i vo ay ta c˜ ung lo. i dung t´ınh bâ´t . n` . , , , , , , ´ c khi thay c´ ˘ng thu biê´n cua d¯a ac gi´ a tri. cua biê´n kh´ ac nhau d ¯ê giai. , ,, V´ı du. 4.1. Giai phuong tr`ınh h` am   1 = x, f(x) + f 1−x

(x 6= 0, x 6= 1).

(4.1)

, `˘ng `,i giai. Trong (4.1) ta thay x ba Lo

1 ,, , ta nhân . d¯uo. c 1−x     1 x−1 1 f +f = . 1−x x 1−x

1 ,, , ta nhân ¯uo. c . d 1−x   x−1 x−1 f + f(x) = . x x , ,, Lâ´y (4.3) tr`u d¯i (4.2), ta nhân ¯uo. c . d   x2 − x + 1 1 f(x) − f = . 1−x x(x − 1)

(4.2)

, `˘ng T`u (4.2) ta lai . thay x ba

(4.3)

(4.4)

Công a (4.4) ta c´ o . (4.1) v` f(x) =

x3 − x + 1 . 2x(x − 1)

,, , ,, ,, , `˘ng c´ ˜ cho, h` Ba ach thu tru. c tiê´p v` ao phuong tr`ınh h` am d am t`ım d¯uo. c thu. c ¯a , , su. l` a nghiêm . cua (4.1). ,, `˘ng c´ ´,i c´ ach thê´ v` ao phuong tr`ınh h` am vo ac gi´ a tri. kh´ ac nhau v` a sau Ba , , , , ´ c n` ˘ng thu ´ kê´t ho. p c´ d¯o ac d¯a ay s˜e t`ım ra nghiêm. X´et môt ¯iên h`ınh: . . v´ı du. d

J

`, phu,o,ng tr`ınh sau V´ı du. 4.2. H˜ ay t`ım h` am f(x) tu af(αx + β) + bf(−αx − β) = cx,

(α 6= 0).

(4.5)

,, ,, Chuong 4. Phuong tr`ınh h` am

178

, `˘ng x th`ı phu,o,ng tr`ınh (4.5) c´ `,i giai. Ta thê´ αx + β ba Lo o dang . af(x) + bf(−x) =

c (x − β). α

(4.6)

`˘ng x th`ı phu,o,ng tr`ınh (4.5) c´ Ta thê´ −αx − β ba o dang . c af(−x) + bf(x) = − (x + β). (4.7) α , , ´, , ´ tr`u d¯i cho nhau cho ta Nhân (4.6) v` a (4.7) lâ`n luo. t vo i a v` a b v` a sau d¯o , kê´t qua c (a2 − b2 )f(x) = [(a + b)x − (a − b)β]. α , ,, T`u d¯ây ta nhân ¯uo. c: . d   , c 1 β , a) Nê´u a2 6= b2 , th`ı f(x) = x− . Ta kiêm tra tru. c tiê´p α a−b a+b , ,, ˜ cho. thâ´y h` am trên l` a nghiêm ¯a . cua phuong tr`ınh d , , ˜ cho không c´ b) Nê´u a2 − b2 = 0, c 6= 0, phuong tr`ınh d o nghiêm. ¯a . , , , Nê´u c 6= 0, a = b = 0, th`ı phuong tr`ınh (4.5) c´ o nghiêm a tâ´t ca c´ ac . l` h` am.

,, Nê´u c 6= 0, a = ±b 6= 0, th`ı phuong tr`ınh (4.5) không c´ o nghiêm. . , , c) Nê´u c = 0, a = b 6= 0, th`ı phuong tr`ınh (4.5) c´ o nghiêm . , ´ f(x) = −f(−x), nghiêm l` a m oi h` a m sô l e. . . ,, ´ d) Nêu c = 0, a = −b 6= 0, th`ı phuong tr`ınh (4.5) c´ o nghiêm . ˜ ´ ˘n. f(x) = f(−x), nghiêm a moi am sô cha . l` . h` , , , e) Nê´u c = a = b = 0, th`ı phuong tr`ınh (4.5) c´ o nghiêm a tâ´t ca c´ ac . l` h` am. , ,, ˜ x´et hai v´ı du. d¯iên h`ınh d` Ta d¯a ung phuong ph´ ap thê´ c´ ac gi´ a tri. kh´ ac , , , `˘ng h` ´ c. L` ˘ng thu ˘. c nhiên l` ˜ cho ra nhau d¯ê`u cho d am nhu vây a ta d am sô´ ¯a ¯a . ma , , , ˜ tô`n tai, nghiêm o c´ o thu. c ¯a . d . nên khi t`ım ra nghiêm . rô`i phai kiêm tra lai . n´ , su. l` a nghiêm . hay không. , , , ´,i moi V´ı du. sau d ao t´ınh bâ´t biê´n cua h` am sô´ l` a vo a tri. cua ¯ây du. a v` . gi´

J

,, ˜,ng gi´ 4.1. Phuong ph´ ap thê´ nhu a tri. d¯ô´i sô´

179

`˘ng sô´ (bâ´t biê´n) v` ´ l` biê´n sô´, h` am sô´ d¯ê`u nhân a tri, a tri. d¯o a ha a ta . gi´ . môt . gi´ , ,, ` , , ˘ng sô´ n` ´ l` t`ım d¯uo. c ha ay, sau d¯o a t`ım ra nghiêm am. . cua phuong tr`ınh h` , , , ˜,ng h` V´ı du. 4.3. H˜ ay t`ım tâ´t ca nhu am sô´ x´ ac d oi moi a thoa m˜ an ¯inh . v´ . x v` ,, phuong tr`ınh sau d ¯ây (x − y)f(x + y) − (x + y)f(x − y) = 4xy(x2 − y2 ).

(4.8)

, ,, `,i giai. Ða ˘. t u = x + y, v = x − y v` Lo a thay v` ao (4.8), ta nhân ¯uo. c . d vf(u) − uf(v) = uv(u2 − v2 ). (4.9) , , f(u) f(v) 2 2 ´,i u 6= 0, v 6= 0, t`u, (4.9) ta nhân Vo . d¯uo. c u − v = u − v , hay f(v) f(u) − u2 = − v2 . (4.10) u v f(x) , ´,i T`u (4.10) suy ra h` am sô´ − x2 nhân ung môt a tri. (bâ´t biê´n) vo . c` . gi´ x , f(x) , ´ a tri. kh´ ac nhau cua d¯ô´i sô´, ngh˜ıa l` a − x2 = c v` a khi d nh˜ung gi´ ¯o x 3 f(x) = x + cx, (4.11) ,, `˘ng sô´ bâ´t k`ı. od a môt ¯ây c l` . ha ,, ˘. c u = 0, v 6= 0) ta nhân Nê´u trong (4.9) thay v = 0, u 6= 0 (hoa ¯uo. c . d , ´,i x = 0. Suy f(0) = 0. Nhung moi am c´ o dang a tri. 0 vo . h` . (4.11) d¯ê`u nhân . gi´ , , , ra nê´u h` am f(x) thoa m˜ an (4.8) n´ o câ`n thoa m˜ an dang (4.11). D˜ê kiêm . , ,, tra thâ´y moi am c´ o dang an phuong tr`ınh (4.8). . h` . (4.11) thoa m˜ , , , , ´,i ´y tu,o,ng cua b` ´ l` Vo ai trên ta c´ o thê cho môt a tri. cua h` am sô´ sau d¯o a . gi´ , ,, giai phuong tr`ınh: , ,, V´ı du. 4.4. H˜ ay giai phuong tr`ınh h` am sau

J

f(x + y) − 2f(xy) − 3f(x) + (2x2 − 1)f(y) = 2x(xy − 1) − 5.

(4.12)

, , , , `,i giai. Gia su, f(x) l` ˘. t f(0) = c. Trong (4.12) Lo a nghiêm a ta d¯a . cua (4.12) v` ´ ta cho y = 0, khi d ¯o 2f(x) = 2cx2 + 2x + 5 − 3c.

,, ,, Chuong 4. Phuong tr`ınh h` am

180

, T`u d o 2c = 5 − 3c, suy ra c = 1. Suy ra f(x) = x2 + x + 1. ¯ây cho x = 0, ta c´ , , ,, , , , Kiêm tra tru. c tiê´p h` am t`ım d an (4.12). ¯uo. c thu. c su. thoa m˜ , V´ı du. 4.5 (IMO 1999). T`ım tâ´t ca h` am f : R → R sao cho

J

f(x − f(y)) = f(f(y)) + xf(y) + f(x) − 1 , v´ oi moi . x, y ∈ R. , ,, ,, `,i giai. Ða ˘. t f(0) = c. Thay x = y = 0 v` Lo ao phuong tr`ınh trên v` a nhân ¯uo. c . d f(−c) = f(c) + c − 1. , , ´,i x = f(y) ∈ A, ta c´ Nhu vây a miê`n gi´ a tri. cua f, th`ı vo o . c 6= 0. Cho A l` c = f(0) = f(x) + x2 + f(x) − 1. , c + 1 x2 ,, ,, − . Giai phuong tr`ınh trên ta t`ım d ¯uo. c f(x) = 2 2 ,, , ´i moi ´ ng vo Ta d am d¯u ¯i t`ım c. Phuong tr`ınh h` . x, y, nên ta thay x = y 6= 0 ,, ,, ˜ cho, ta d¯uo. c: v` ao phuong tr`ınh d¯a f(x − f(x)) = f(f(x)) + xf(x) + f(x) − 1. , c + 1 x2 ,, − v` ao phuong tr`ınh trên cho kê´t qua: 2 2   c + 1 (x − f(x))2 c + 1 (f(x))2 c + 1 x2 c + 1 x2 − = − +x − + − − 1. 2 2 2 2 2 2 2 2

Thay f(x) =

−(x − f(x))2 = −(f(x))2 + x(c + 1 − x2 ) + c + 1 − x2 − 2. −x2 + 2xf(x) − (f(x))2 = −(f(x))2 + x(c + 1 − x2 ) + c + 1 − x2 − 2. 2xf(x) = x(c + 1 − x2 ) + c − 1.   c + 1 x2 = x(c + 1 − x2 ) + c − 1. 2x − 2 2 x2 , , ´,c trên suy ra c = 1. Vây ´,i x ∈ A, f(x) = 1 − ˘ng thu T`u d¯a . H` am t`ım . vo 2 2 , x ,, ´,i moi d¯uo. c x´ ac d o thê thay h` am f(x) = 1 − x´ ac d¯inh ¯inh . vo . . x ∈ R. Ta c´ 2

,, ˜,ng gi´ 4.1. Phuong ph´ ap thê´ nhu a tri. d¯ô´i sô´

181

, ,, , , ´,i moi ˜ cho v` vo ao phuong tr`ınh d¯a a thâ´y n´ o thu. c su. l` a nghiêm . x ∈ R v` . cua ,, phuong tr`ınh. , , ,, , ,, ,, ´ l` C` on phuong ph´ ap n˜ua d¯ê giai phuong tr`ınh h` am d¯o a phuong ph´ ap , , ,, ,, ` Cauchy: Phuong ph´ ap n` ay giai phuong tr`ınh h` am trên miên x´ ac d¯inh . cua ,, , , `˘ng c´ n´ o ba ach lâ`n luo. t x´et c´ ac c´ ac biê´n l` a sô´ tu. nhiên, sô´ nguyên, sô´ h˜uu , , , , , ,, ˜ d` tı v` a nh˜ung sô´ thu. c. Trong khi giai nh˜ung phuong tr`ınh n` ay ta vân ung , , , ´ ´ ´ ´ ´ c´ ach thê c´ ac gi´ a tri. kh´ ac nhau cua d am sô. ¯ôi sô d¯ê t`ım ra công thuc h`

J

, ,, V´ı du. 4.6. H˜ ay giai phuong tr`ınh h` am f(x + y) = f(x) + f(y)

(4.13)

,, ,, ,, , o d¯ây f(x) l` a h` am liên tuc oi moi a phuong tr`ınh . v´ . x (phuong tr`ınh trên goi . l` Cauchy). , ,, `,i giai. Thê´ x = y = 0 v` Lo ao (4.13) ta nhân a . d¯uo. c f(0) = 2f(0), ngh˜ıa l` f(0) = 0. ,, Thê´ y = −x v` ao (4.13) ta nhân a nê´u f(x) ¯uo. c f(x) = −f(−x) ngh˜ıa l` . d , , , l` a nghiêm o phai l` a môt am le. . cua (4.13), th`ı n´ . h` ,, ´,i y = 2x, ta Thê´ y = x v` ao (4.13) ta nhân on vo ¯uo. c f(2x) = 2f(x), c` . d ,, nhân . d¯uo. c f(3x) = f(x) + f(2x) = f(x) + 2f(x) = 3f(x). `˘ng phu,o,ng ph´ ´,ng minh d¯u,o.,c f(nx) = nf(x) vo ´,i moi Ba ap quy nap . ta chu . sô´ , tu. nhiên n. ,, , ˜ t`ım d¯uo. c cua h` Nê´u m l` a sô´ nguyên âm, do c´ ac t´ınh châ´t d am f(x) ta c´ o ¯a f(mx) = −f(−mx) = −(−m)f(x) = mf(x). , , ´,ng minh d¯a ´,c ˘ng thu ˜ chu Nhu vây ¯a . ta d f(`x) = `f(x)

(4.14)

,, ,, Chuong 4. Phuong tr`ınh h` am

182 ´,i moi ´ ng vo d¯u . sô´ nguyên `.

,, , ´,c ´,i x = 1, t`u, (4.14) ta nhân ˘ng thu Nê´u ta k´y hiêu ¯a . f(1) = c vo . d¯uo. c d f(`) = c`

(4.15)

´,i moi ´ ng vo d¯u . sô´ nguyên `. , Nê´u p l` a môt on q l` a môt ac 0 th`ı theo (4.14) . sô´ nguyên, c` . sô´ tu. nhiên kh´ ,, v` a (4.15) ta nhân ¯uo. c . d     p p qf =f q = f(p) = cp q q   , p p , ´,c ˘ng thu ngh˜ıa l` af = c. . T`u d¯ây suy ra d¯a q q f(x) = cx (4.16) , , , , , , , ´i moi ´ ng vo d¯u a tri. h˜uu tı cua x. Cho môt a d˜ ay sô´ h˜uu tı . gi´ . sô´ vô tı x v` a1 , a2 , a3 , ..., an , ... sao cho lim an = x. Theo (4.16) ta c´ o n→∞

f(an ) = can . (4.17) , , ,, Khi n → ∞ v` a v`ı t´ınh liên tuc am f(x), t`u (4.17) ta nhân . cua h` . d¯uo. c f(x) = cx. , , , `˘ng sô´ bâ´t k`ı, l` `˘ng h` am f(x) = cx, o d¯ây c l` a môt a nghiêm D˜ê thâ´y ra . ha . cua ,, phuong tr`ınh (4.13). , , ,, , , Hai v´ı du. sau d¯ây l` a nh˜ung b` ai to´ an phuong tr`ınh h` am co ban c´ o thê , , , , `˘ng c´ d¯ua vê` dang ai to´ an v`ua giai ba ac ph´ep thê´ th´ıch ho. p. . b` , ,, V´ı du. 4.7. H˜ ay giai phuong tr`ınh h` am

J

f(xy) = f(x) + f(y), ,, , od am f(x) l` a h` am liên tuc oi moi ¯ây h` . v´ . x > 0.

(4.18)

, ,, `,i giai. Ta thê´ x = cu v` Lo a y = cv v` ao (4.18). Ta nhân . d¯uo. c f(cu+v ) = f(cu ) + f(cv ), ngh˜ıa l` a g(u + v) = g(u) + g(v),

(4.19)

,, ˜,ng gi´ 4.1. Phuong ph´ ap thê´ nhu a tri. d¯ô´i sô´

183

,, ,, od a dang a môt am liên tuc ¯ây g(u) = f(cu ) l` . (4.19) l` . . h` . theo u. Boi v`ı dang , ,´, ˜ giai b` (4.13) d¯a ai truoc, nên g(u) = f(cu ) = au. ,, V`ı x = cu , ngh˜ıa l` a u = lgc x nên ta t`ım d ¯uo. c f(x) = a.lgc x, ,, , `˘ng sô´ bâ´t k`ı, c > 0, c 6= 1. od a nh˜ung ha ¯ây a, c l` , ,, an (4.18). D˜ê thâ´y h` am t`ım d¯uo. c thoa m˜ , ,, V´ı du. 4.8. Giai phuong tr`ınh h` am   f(x) + f(y) x+y = , f 2 2 ,, , od am f(x) l` a h` am liên tuc oi moi ¯ây h` . v´ . x.

(4.20)

J (4.21)

, ,, `,i giai. Thê´ x = u + v v` Lo a y = 0 ta nhân . d¯uo. c   u+v f(u + v) + f(0) f = , 2 2 m` a vê´ tr´ ai theo (4.21) l` a f(u + v) + f(0) f(u) + f(v) = , 2 2 hay f(u + v) − f(0) = (f(u) − f(0)) + (f(v) − f(0)), ,, ´,i g(x) = f(x) − f(0). d¯ây ch´ınh l` a dang . phuong tr`ınh (4.13) vo ´ g(x) = ax v` Khi d a suy ra ¯o f(x) = ax + f(0) = ax + b, ,, `˘ng sô´ bâ´t k`ı v` od a ha a v` a b = f(0). ¯ây a l` , , , Kiêm tra tru. c tiê´p ta thâ´y f(x) = ax + b l` a nghiêm . cua (4.21). , ,, ,, Môt ap kh´ ac giai phuong tr`ınh h` am l` a t`ım ra môt . phuong ph´ . nghiêm . , , ` ´ ˘ng c´ ´ ba am phai t`ım f(x). Sau d ach thê ψ(x) = f(x)−ϕ(x) riêng ϕ(x) cua h` ¯o

J

,, ,, Chuong 4. Phuong tr`ınh h` am

184

, ,, ,, ,, ´,i ˜ cho v` v` ao phuong tr`ınh h` am d¯a a giai phuong tr`ınh h` am nhân ¯uo. c d¯ô´i vo . d , , , ,, , ´ phuong tr`ınh h` ψ(x). Khi d¯o am phai giai s˜e d ¯on gian râ´t nhiê`u. , ,, V´ı du. 4.9. Giai phuong tr`ınh h` am f(x + y) = f(x) + f(y) + xy, , ,, od am f(x) l` a h` am liên tuc ¯ây h` ¯iêm x = 0. . tai . d

(4.22)

, ,, `,i giai. Vo ´,i y = 0, t`u, (4.22) ta nhân ´,i moi ´ vo Lo a tri. ¯o . d¯uo. c f(0) = 0. Khi d . gi´ , , thu. c cua x ta c´ o lim [f(x) + f(y) + xy] = f(x) + lim f(y) + 0 = f(x) + f(0) + 0 = f(x),

y→0

y→0

, `˘ng f(x) (nê´u ngh˜ıa l` a theo (4.22) lim f(x + y) = f(x), d¯iê`u n` ay chı ra ra y→0 ´,i moi tô`n tai) a h` am liên tuc . l` . d¯ô´i vo . x. , , , 1 ,, Ta c´ o thê kiêm tra h` am ϕ(x) = (x2 + x) l` a nghiêm cua phuong . 2 tr`ınh (4.22). ´,i moi ˘. t ψ(x) = f(x) − ϕ(x). Khi d ´ h` Ða am ψ(x) l` a h` am liên tuc a ¯o . vo . x v` ψ(x + y) = f(x + y) − ϕ(x + y) = = [f(x) + f(y) + xy] − [ϕ(x) + ϕ(y) + xy] = = [f(x) − ϕ(x)] + [f(y) − ϕ(y)] = = ψ(x) + ψ(y). ,, ,, Ðây l` a dang . phuong tr`ınh (4.13), ta t`ım d¯uo. c nghiêm . ψ(x) = ax. Suy ra 1 1 1 f(x) = ψ(x) + ϕ(x) = ax + (x2 + x) = x2 + (a + )x, 2 2 2 ,, 1 2 ˜ `˘ng sô´ bâ´t k`ı. Dê thâ´y ra `˘ng d ngh˜ıa l` a f(x) = 2 x + bx, o d a ha ¯ây b l` ¯ây ch´ınh , ,, ˜ cho. l` a nghiêm . chung cua phuong tr`ınh d¯a , , , V´ı du. 4.10. H˜ ay t`ım tâ´t ca c´ ac h` am f(x) thoa m˜ an tâ´t ca c´ ac d ¯iê`u kiên . sau

J

d ¯ây:

,, ˜,ng gi´ 4.1. Phuong ph´ ap thê´ nhu a tri. d¯ô´i sô´

185

, 1) f(−x) = −f(x), v´ oi moi . x; , 2) f(x + 1) = f(x) + 1, v´ oi moi . x;   1 1 , 3) f oi moi = 2 f(x), v´ . x 6= 0. x x , , , , `˘ng c´ `˘ng ϕ(x) = x thoa m˜ `,i giai. Ba Lo ach kiêm tra tru. c tiê´p ta thâ´y ra an tâ´t , , , ca c´ ac d ai to´ an, suy ra l` a môt ai to´ an. ¯iê`u kiên . cua b` . nghiêm . riêng cua b` ˘. t ψ(x) = f(x) − x. Khi d ´ Ða ¯o ψ(−x) = f(−x) + x = −f(x) + x = −ψ(x), ψ(x + 1) = f(x + 1) − (x + 1) =

(4.23) (4.24)

= f(x) + 1 − (x + 1) = f(x) − x = ψ(x),     1 1 1 1 1 =f − = 2 f(x) − = (4.25) ψ x x x x x 1 1 = 2 [f(x) − x] = 2 ψ(x). x x ,, ´,i x = 0, t`u, (4.23) ta nhân a ψ(0) = 0. Vo . d¯uo. c ψ(0) = −ψ(0), ngh˜ıa l` , , , , ´i x = −1, t`u (4.24) ta nhân Vo . d¯uo. c ψ(−1) = ψ(0) = 0. ,, ´,i x 6= 0 v` Vo a x 6= −1, theo (4.23), (4.24) v` a (4.25) ta nhân . d¯uo. c     1 −1 ψ(x) = ψ(x + 1) = (x + 1)2 ψ = −(x + 1)2 ψ x+1 x+1     x −1 + 1 = −(x + 1)2 ψ = −(x + 1)2 ψ x+1 x+1     2 x x+1 1 2 = −(x + 1)2 ψ = −x ψ 1 + (x + 1)2 x x   1 1 = −x2 ψ = −x2 2 ψ(x) = −ψ(x), x x ngh˜ıa l` a 2ψ(x) = 0 hay l` a ψ(x) = 0. , , ´,i moi T`u ψ(x) = 0 vo am sô´ f(x) = x l` a nghiêm . x, suy ra h` . duy nhâ´t cua ,, ˜ cho. phuong tr`ınh d¯a

J

,, ,, Chuong 4. Phuong tr`ınh h` am

186

, ,, , a thoa m˜ an phuong tr`ınh V´ı du. 4.11. T`ım c´ ac h` am x´ ac d¯inh oi moi . x v` . v´ xf(y) + yf(x) = (x + y).f(x)f(y)

(4.26)

, v´ oi x, y bâ´t k`ı. , ,, , `˘ng x, ta t`ım d `,i giai. T`u, (4.26) thay y ba Lo ¯uo. c 2xf(x) = 2x(f(x))2 . T`u d ¯ây , , , 2 ´i x 6= 0 ta t`ım d ˘. c l` vo a f(x) = 0 hoa a f(x) = 1. ¯uo. c f(x) = (f(x)) , ngh˜ıa l` , , , , ´i x = a 6= 0. Khi d¯o ´ t`u (4.26) ta nhân Cho f(a) = 1 vo ¯uo. c af(y) + y = . d , ´ (a + y)f(y) hay y = yf(y), ngh˜ıa l` a f(y) = 1 voi y 6= 0. , , ,, ´ tâ´t ca nghiêm Ta k´ı hiêu a: . f(0) = c. Khi d¯o . cua phuong tr`ınh (4.26) l` a) H` am sô´ f(x) = 0; , b) Nh˜ung h` am sô´

 f(x) =

´,i x 6= 0 1 vo ´,i x = 0. c vo

,

, , ` tâp Bai (Go. i ´y v` a tra l`oi b` ai tâp . sau tai . trang 203). . , ,, . 4.12. Giai phuong tr`ınh h` am sau   1 f(x) + af = x, x , ,, . 4.13. Giai phuong tr`ınh h` am

a 6= ±1.

f(x + y) + f(x − y) = 2f(x) cos y. , ,, . 4.14. Giai phuong tr`ınh h` am f(x + y) − f(x − y) = 4xy. , ,, . 4.15. Giai phuong tr`ınh h` am f(x + y) − 2f(x − y) + f(x) − 2f(y) = y − 2.

J

, ,, 4.2. Phuong ph´ ap d¯iêm bâ´t d¯ông .

187

, ,, . 4.16. Giai phuong tr`ınh h` am f(x + y) = f(x)f(y). , ,, . 4.17. Giai phuong tr`ınh h` am f(xy) = f(x)f(y). , ,, . 4.18. Giai phuong tr`ınh h` am f(x + y) + f(x − y) = 2[f(x) + f(y)]. , ,, . 4.19. Giai phuong tr`ınh h` am f(x) +

x 1 = f( ). sin x 2

, ,, . 4.20. Giai phuong tr`ınh h` am f(x + y) = f(x)f(y) + xy − xf(y) − yf(x) + x + y.

, ,, ´ diê 4.2. Phuong phap ¯ m bâ´t d¯ông . , , , ´, ˜ d¯uo. c u L´ı thuyê´t d¯iêm bâ´t d¯ông d¯a ng dung râ´t nhiê`u trong to´ an cao . . , , , , , , ´ ´ câp nhu giai t´ıch thu. c, giai t´ıch phuc v` a giai t´ıch h` am,... trong muc ay ta . n` , , , ,, , ,, , ´ ´ ´ ´ lo. i dung ¯ai . d¯iêm bât d¯ông . nhu môt . d . luo. ng bât biên d¯ê giai môt . sô phuong tr`ınh h` am. , , , Môt ad am f(x) nê´u f(x0 ) = x0 . ¯iêm bâ´t d¯ông ¯iêm x0 goi . cua h` . d . l` ´,i n = 2, 3, 4, ..., h` ˘. t f1 (x) = f(x) v` Ða a fn (x) = f(fn−1 (x)) vo am sô´ fn (x) , , goi a ph´ep la ˘. p thu´ n cua h` am f(x). . l` , , n K´ı hiêu a tâp o t´ınh châ´t sau d¯ây: ¯iêm bâ´t d ¯ông . Sn l` . d . cua f (x). Ta c´ , , , 1. Nê´u x l` a môt cua fn (x), th`ı f(x) l` a c˜ ung l` ad ¯ông ¯iêm bâ´t . d¯iêm bâ´t d . , n d¯ông . cua f (x). Thât . vây . fn (f(x)) = fn+1 (x) = f(fn (x)) = f(x).

,, ,, Chuong 4. Phuong tr`ınh h` am

188

, ´nh xa. Sn v` Nhu vây, ao ch´ınh Sn . . f(x) a , ´,i a, b ∈ Sn , th`ı ´nh trên Sn . V`ı nê´u f(a) = f(b) vo 2. f l` ad ¯on a a = fn (a) = fn−1 (f(a)) = fn−1 (f(b)) = fn (b) = b. , , T`u hai t´ınh châ´t trên suy ra nê´u Sn l` a tâp a ph´ep ho´ an . h˜uu han . th`ı f(Sn ) l` vi. trong Sn .

, , V`ı f(x) = x k´eo theo f2 (x) = f(f(x)) = f(x) = x, nhu vây ¯iêm bâ´t d¯ông . d . , , , 2 , , ´, 2 4 8 cua f c˜ ung l` a d¯iêm bâ´t d ông c ua f . Ta x´ e t d˜ a y f, f , f , f , ... tu o ng u ng ta ¯. ,, nhân ¯uo. c . d S1 ⊆ S2 ⊆ S4 ⊆ S8 ⊆ ... , , V´ı du. 4.21. T`ım tâ´t ca c´ ac h` am f : R → R sao cho f(f(x)) = x2 − 2 v´ oi moi . x ∈ R. , , , , `,i giai. Gia su, h` Lo am f(x) tô`n tai. ac tâp a S4 nhu sau: . Ta t´ınh c´ . S2 v` , , , , ,, Nh˜ung d cua h` am f2 l` a nghiêm ¯iêm bâ´t d¯ông . . cua phuong tr`ınh x = , , 4 , x2 − 2, ngh˜ıa l` a S2 = {−1, 2}. Nh˜ung d a nghi êm ¯iêm bâ´t d ¯ông .  cua f (x) l` √ . , −1 ± 5 ,, cua phuong tr`ınh x = x4 − 4x2 + 2, ngh˜ıa l` a S4 = −1, 2, . Ta 2 √ √ −1 + 5 −1 − 5 ˘. t c = d¯a ,d = . V`ı f l` a ho´ an vi. trong S2 v` a c, d ∈ S4 \ S2 , 2 2 ˘. c f(c) = d (do f a ´nh xa. S4 v` nên f(c) = c hoa ao S4 ). , , Nê´u f(c) = c, th`ı f2 (c) = c k´eo theo c l` a d¯iêm bâ´t d¯ông cua f2 , d¯iê`u . n` ay vô l´y. ´ c = f(d) = f(f(c)) = f2 (c), c˜ Nê´u f(c) = d, th`ı f(d) = c. Khi d¯o ung vô , , , , ` l´y nhu trên. Nhu vây am f n` ao thoa m˜ an b` ai to´ an. . không thê tôn tai . môt . h`

J

, V´ı du. 4.22 (IMO 1983). T`ım tâ´t ca c´ ac h` am f : R+ → R+ sao cho , + f(xf(y)) = yf(x) v´ oi moi a f(x) → 0 khi x → +∞. . x, y ∈ R v`

, ,, 4.2. Phuong ph´ ap d¯iêm bâ´t d¯ông .

189

, , ,, `,i giai. 1. Thay x = y = 1 v` ´,c d¯a ˘ng thu ˜ cho ta nhân Lo ao d ¯a ¯uo. c . d , ,, ´,c d ˘ng thu ˜ cho ta nhân f(f(1)) = f(1). Thay x = 1, y = f(1) v` ao d¯a ¯a ¯uo. c . d ´ f(f(f(1))) = [f(1)]2 . Khi d¯o [f(1)]2 = f(f(f(1))) = f(f(1)) = f(1) ⇒ f(1) = 1 (v`ı f(1) ∈ R+ ). , , , Nhu vây a d¯iêm bâ´t d ¯ông . 1 l` . cua f. ,, , 2. Lâ´y y = x, nhân ung l` a . d¯uo. c f(xf(x)) = xf(x). Nhu vây . w = xf(x) c˜ , , , + ´ ´i moi d ¯iêm bât d ¯ông . cua f vo . x∈R . , ,, , ,´, 3. Gia su f c´ od x > 1. Theo buo c 2, xf(x) = x2 c˜ ung l` a ¯iêm bâ´t d ¯ông . , , , 2 2 4 2n ´ ´ d x f(x ) = x c˜ ung l` a d¯iêm bât d ... Nhu vây a ¯iêm bât d ¯ông, ¯ông, . . . x l` , , , 2n 2n 2n ´ ˜ nhung d¯iêm bât d V`ı x > 1 nên x → +∞, nhung f(x ) = x → ∞ ¯ông. . , , , , ´ chu không phai l` a 0 nhu gia thiê´t. Ðiê`u n` ay vô l´ı. , ,, , ´ 4. Gia su f c´ o d¯iêm bâ´t d¯ông . x ∈ (0, 1). Khi d¯o         1 1 1 1 1 x =f f(x) = xf ⇒f = , 1=f x x x x x , , 1 ´,i phâ`n 3 o, trên. Nhu, vây ngh˜ıa l` a f c´ o d¯iêm bâ´t d > 1, tr´ ai vo ¯ông . . f không x , , c´ o d¯iêm bâ´t d¯ông ao trong khoang (0, 1). . n` , , , ,´, 5. Sau c´ ac buoc trên chı c` on lai cua f l` a 1. Theo phâ`n 2 ¯iêm bâ´t d ¯ông . d . 1 , + ´i moi ta c´ o xf(x) = 1 suy ra f(x) = vo . x∈R . x   ,, 1 x y 1 , ´ Thu lai: V o i f(x) = , f(xf(y)) = f = = yf(x) v` a f(x) = → 0 . x y x x , 1 ,, ˜ cho. a nghiêm am d¯a khi x → +∞. Vây . cua phuong tr`ınh h` . f(x) = x l` , , V´ı du. 4.23 (IMO 1996). T`ım tâ´t ca c´ ac h` am thoa m˜ an f : N0 → N0 sao cho , f(m + f(n)) = f(f(m)) + f(n) v´ oi moi . m, n ∈ N0 .

J

, ,, `,i giai. 1. Cho m = n = 0, nhân Lo . d¯uo. c f(f(0)) = f(f(0)) + f(0), suy ,, ra f(0) = 0. Lai a f(n) ¯uo. c f(f(n)) = f(n), ngh˜ıa l` . lâ´y m = 0, ta nhân . d

190

,, ,, Chuong 4. Phuong tr`ınh h` am

, , ,, ,, ´,i moi ´ phuong tr`ınh tro th` l` a d¯iêm bâ´t d anh cua f vo ¯ông . n ∈ N0 . Khi d¯o .

f(m + f(n)) = f(m) + f(n). , , , `˘ng kw c˜ ´ ta chı ra ra 2. Nê´u w l` a môt cua f, khi d¯o ung l` a . d¯iêm bâ´t d¯ông . , , , , ` ´ ´ ´ ˘ng quy nap d¯iêm bât d¯ông cua f voi moi an . . k ∈ N0 . Ta chung minh ba . to´ , ,, , ,`, , ´ ´ ˜ biêt d ´ ng. Gia su kw l` hoc: a d¯iêm bât d¯ông, khi ¯u . Truong ho. p k = 0 ta d¯a . , ´ f(kw + w) = f(kw + f(w)) = f(f(kw)) + f(w) = kw + w v` d¯o a nhu vây . , , ´ (k + 1)w c˜ ung l` a d¯iêm bât d¯ông . cua f. , , ´,i moi ´ f(n) = 0 vo 3. Nê´u 0 l` a d¯iêm bâ´t d¯ông duy nhâ´t cua f, khi d¯o . . , , ,´, ,, `˘ng không l` n ∈ N0 theo buo c 1. Hiên nhiên, h` am ha a nghiêm c ua phu o ng . ˜ cho. tr`ınh d¯a , , , `˘ng w l` Nê´u f c´ o ´ıt nhâ´t môt w > 0 v` a gia thiê´t ra ad ¯ông ¯iêm . d¯iêm bâ´t d . , , , , ,, , `˘ng chı c´ bâ´t d duong nho nhâ´t. Ta s˜e chı ra ra o nh˜ung d¯iêm bâ´t d¯ông ¯ông . . , ,, , ´ ´ kw, k ∈ N0 l` a nghiêm. Th ât v ây, Gi a s u x l` a m ôt d iê m bâ t d ông n` a o d ¯ ¯ ¯o . . . . . ,, ,, duong. Theo thuât an chia, x = kw + r, o d¯ây 0 ≤ r < w. Ta c´ o . to´ x = f(x) = f(r + kw) = f(r + f(kw)) = = f(r) + f(kw) = f(r) + kw. , , , , ,, Nhu vây a d¯iêm bâ´t d¯ông duong nho nhâ´t, nên . f(r) = x − kw = r. T`u w l` . r = 0 v` a x = kw. , , ,´, ´,i moi T`u f(n) l` ad vo c 1), nên f(n) = ¯iêm bâ´t d ¯ông . . n ∈ N0 (theo buo , ´i cn ∈ N0 . Ta c´ cn w vo o c0 = 0. ´,i n ∈ N0 , theo thuât 4. Vo an chia, ta c´ o n = kw + r, 0 ≤ r < w. Vây . to´ . f(n) = f(r + kw) = f(r + f(kw)) = f(r) + f(kw) = hni )w. = cr w + kw = (cr + k)w = (cr + w ,, ˜ w > 0, lâ´y c0 = 0 v` ´,i môi Thu lai: a c1 , c2 , ..., cw−1 ∈ N0 bâ´t k`ı. X´et h` am . vo , , , , , n ` f(n) = (cr + [ w ])w, o d¯ây r l` a phân du cua ph´ep chia n cho w (v` a ca h` am

, ,, 4.2. Phuong ph´ ap d¯iêm bâ´t d¯ông .

191

`˘ng không). Ta viê´t m = kw + r, n = `w + s vo ´,i 0 ≤ r, s < w. Khi d ´ ha ¯o f(m + f(n)) = f(r + kw + (cs + `)w) = = cr w + kw + cs w + `w = f(f(m)) + f(n).

J

, V´ı du. 4.24. T`ım tâ´t ca c´ ac h` am f : N0 → N0 sao cho f(f(m)+f(n)) = m+n , v´ oi moi . m, n ∈ N0 . , , `˘ng f(x) = x l` `˘ng quy nap, `,i giai. D˜ê thâ´y ra Lo a môt Ba . nghiêm. . . ta s˜e chı ra `˘ng b` ra ai to´ an không c´ o nghiêm ac. . kh´ , ,, , ´ ng minh f(1) = 1: Gia su f(1) = t > 1. Cho s = f(t−1) > 0. Thâ´y Ta chu `˘ng nê´u f(m) = n, th`ı f(2n) = f(f(m) + f(m)) = 2m. Nhu, vây ra . f(2t) = 2 ´ 2s + 2t = f(f(2s) + f(2t)) = f(2t) = 2, suy ra v` a f(2s) = 2t − 2. Khi d ¯o ´ f(1) = 1. t < 1, d¯iê`u n` ay vô l´ı. Do d¯o , ,, ´ f(n + 1) = f(f(n) + f(1)) = n + 1. Do d ´ Gia su f(n) = n. Khi d¯o ¯o , ´i moi f(n) = n vo . n ∈ N0 .

J

, V´ı du. 4.25 (IMO 1987). Chu´ng minh ra ˘`ng không tô`n tai am f : N0 → N0 . h` sao cho f(f(n)) = n + 1987. , , , , , `,i giai. Gia su, tô`n tai ´ f l` ´nh, ngh˜ıa l` Lo am f nhu vây. a d a ¯on a . h` . Khi d¯o f(a) = f(b), suy ra a + 1987 = f(f(a)) = f(f(b)) = b + 1987, k´eo theo a = b. , ,, ´,i k gi´ Gia su f(n) kh´ ac vo a tri. kh´ ac nhau c1 , ..., ck trong N0 , ngh˜ıa l` a , , ´i moi ´i 2k gi´ ´ f(f(n)) kh´ f(n) 6= c1 , ..., ck vo ac vo a tri. kh´ ac ¯o . n ∈ N0 . Khi d , nhau c1 , ..., ck v` a f(c1 ), ..., f(ck ) trong N0 (nh˜ung gi´ a tri. f(cj ) kh´ ac nhau ,, , , ´ ´nh). Bây gi`o nêu w 6= c1 , c2 , ..., ck , f(c1 ), f(c2 ), ..., f(ck ), boi v`ı f l` a d¯on a ` ´ tôn tai ´ khi d ¯o . m ∈ N0 sao cho f(f(m)) = w. V`ı w 6= f(cj ), m 6= cj , do d¯o , ` ` ` ˘ng ´ f(f(n)) = w. Ðiêu n` tôn tai ay chı ra ra . n ∈ N0 sao cho f(n) = m, khi d¯o , f(f(n)) kh´ ac chı c´ o 2k gi´ a tri. c1 , c2 , ..., ck , f(c1 ), ..., f(ck ) v` a không kh´ ac

,, ,, Chuong 4. Phuong tr`ınh h` am

192

, gi´ a tri. n` ao n˜ua. V`ı n + 1987 kh´ ac 1987 gi´ a tri: a 2k 6= 1987, . 0, 1, ..., 1986 v` d¯iê`u n` ay vô l´ı.

J

, , , ,, V´ı du. 4.26. H˜ ay t`ım tâ´t ca c´ ac d a nghiêm ¯a thu´c P(x) l` . cua phuong tr`ınh P(Q(x)) = Q(P(x)), (4.27) ,, , , od a môt am x´ ac d¯inh oi moi a Q(x) > x v´ oi moi a ¯ây Q(x) l` . h` . v´ . x v` . x ≥ 0 v` P(0) = 0. , `,i giai. Cho Q(x) l` ´,i moi ´,i moi Lo a h` am x´ ac d¯inh a Q(x) > x vo . vo . x v` . x ≥ 0. Ta tao ay . d˜

a0 , a1 , a2 , ..., an , ...

(4.28)

theo c´ ach sau: a0 = 0, a1 = Q(0), a2 = Q(a1 ), ..., ak = Q(ak−1 ), ... ,, Ta nhân a a1 > a0 ; Q(a1 ) > a1 ngh˜ıa l` a a2 > a1 ; ¯uo. c Q(0) > 0, ngh˜ıa l` . d , ` ` ˘ng d˜ Q(a2 ) > a2 ngh˜ıa l` a a3 > a2 ; ... d¯iêu n` ay chı ra ra ay (4.28) t˘ ang v` a , ´ ˜ suy ra n´ o c´ o vô han ac nhau. . nhung sô hang . kh´ , , , , , , ´ c P(x) phai t`ım phai thoa m˜ ´,c: ˘ng thu Ða thu an nh˜ung d¯a P(a0 ) = P(0) = 0 = a0 , P(a1 ) = P(Q(0)) = a1 P(a2 ) = P(Q(a1 )) = Q(P(a1 )) = Q(a1 ) = a2 , P(a3 ) = P(Q(a2 )) = Q(P(a2 )) = Q(a2 ) = a3 , ······ , , , Tâ´t ca c´ ac sô´ hang ay (4.28) thoa m˜ an P(an ) = an . . cua d˜ ´,c H(x) = x c˜ ´,i d Vo ung c´ o H(an ) = an . ¯a thu , ´,c tr` ´,i vô han Khi hai d¯a thu ung nhau vo ac gi´ a tri. kh´ ac nhau cua d ¯ô´i sô´ . c´ th`ı ch´ ung tr` ung nhau.

, ,, 4.2. Phuong ph´ ap d¯iêm bâ´t d¯ông .

193

, , ,, ,, Nhu vây a nghiêm ¯uo. c P(x) = x l` . ta t`ım d . duy nhâ´t cua phuong tr`ınh

J

(4.27).

` tâp Bai . , , , Môt a tra l`oi b` ai tâp . sô´ go. i ´y v` . sau tai . trang 204. , . 4.27. T`ım tâ´t ca c´ ac h` am f : R → R sao cho ´,i moi a) f(x + y) = f(x) + f(y) + 2xy vo a . x, y ∈ R v` f(x) b) limx→0 = 1. x , . 4.28. (1986 IMO). T`ım tâ´t ca c´ ac h` am f : [0, ∞) → [0, ∞) sao cho ´,i x, y ≥ 0 v` a) f(xf(y))f(y) = f(x + y) vo a , ´i 0 ≤ x < 2. b) f(2) = 0 v` a f(x) 6= 0 vo

, ,, . 4.29. Gia su f : R → R+ sao cho p ´,i moi f( x2 + y2 ) = f(x)f(y) vo . x, y ∈ R. , ´,i x ∈ Q trong d¯o ´ c´ T`ım tâ´t ca f(x) vo o sô´ hang . f(1). , , ,, , . 4.30. (1990 IMO). Cho Q+ l` a tâp ac sô´ h˜uu tı duong. H˜ ay xây du. ng . c´ h` am sô´ f : Q+ → Q+ sao cho f(x) , + ´i moi vo . x, y ∈ Q . y , , ´, , . 4.31. (1994 IMO). Cho S l` a tâp n hon −1. T`ım tâ´t ca c´ ac h` am . sô´ thu. c lo f(xf(y)) =

f : S → S sao cho ´,i moi a) f(x + f(y)) = y + f(x) + yf(x) vo a . x, y ∈ S v` f(x) ´,i −1 < x < 0 v` ´,i 0 < x. ˘. t vo b) t˘ ang nga a vo x , . 4.32. (1992 IMO). T`ım tâ´t ca c´ ac h` am f : R → R sao cho ´,i moi f(x2 + f(y)) = y + (f(x))2 vo . x, y ∈ R.

194

,, ,, Chuong 4. Phuong tr`ınh h` am

, . 4.33. T`ım tâ´t ca c´ ac h` am f : Q → Q sao cho f(2) = 2 v` a   x+y f(x) + f(y) , ´i x 6= y. f vo = x−y f(x) − f(y)

, ´ da ´ 4.3. Chuyên dê uc giao hoan ¯ ` vê` cac ¯ th´ 4.3.1. Ð.inh ngh˜ıa , , , ,, ´,c d¯a ˜d ach cua t´ ac gia [8] c´ o nhiê`u Nh˜ung d ¯a thu ¯uo. c x´et trong cuô´n s´ , ,, , , , , , ´ ng dung ´ c o chuo,ng truo ´,c l` u ai tâp an. Nh˜ung d a d¯a ¯a thu . trong viêc . giai b` . to´ , , , , , ´ ´ ´ c hai biên d¯ôi xu ´ ng, ngh˜ıa l` ´ c không thay d¯ôi khi ta d thu a d¯a thu ¯ôi hai biê´n , , ´ c giao ho´ cho nhau. Ta x´et hai d an nhu d ¯a thu ¯inh . ngh˜ıa sau d¯ây. ´,c môt Ðinh ngh˜ıa 4.1. Hai d¯a thu a Q(x) goi a giao ho´ an, nê´u . biê´n P(x) v` . l` . P(Q(x)) = Q(P(x)). , , , ´,c trong d ´,i moi ˘ng thu ´ ng vo Ða a tri. thu. c cua x, ngh˜ıa l` a ¯inh . ngh˜ıa d¯u . gi´ , , ´ , ´ ´ ´ c P(Q(a)) = Q(P(a)). ˘ng thu nêu a l` a môt o d¯a . sô thu. c bât k`ı th`ı ta luôn c´ , , , , ´ c, t`ım d ´ c giao ho´ B` ai to´ an cho môt an cua n´ o không phai ¯a thu . d¯a thu , ,, , d¯on gian, ta d¯i x´et lâ`n luo. t c´ ac v´ı du. sau: , , V´ı du. 4.34. Cho α l` a sô´ bâ´t k`ı, h˜ ay t`ım tâ´t ca d an c´ o bâc ¯a thu´c giao ho´ . , , , , 2 ´ không l´ on hon 3 d oi d ¯ôi v´ ¯a thu´c P(x) = x − α. , `,i giai. T`ım d¯a thu ´,c Q(x) = x3 + ax2 + bx + c. T`u, d ˜ cho Lo ¯iê`u kiên . d¯a P(Q(x)) = Q(P(x)) ta c´ o: (x3 + ax2 + bx + c)2 − α = (x2 − α)3 + a(x2 − α)2 + b(x2 − α) + c. , , , `˘ng vê´ Khai triên l˜ uy th`ua hai vê´ v` a nh´ om lai anh c´ ac hê. sô´: Ta thâ´y ra . d¯ê so s´ , , , , ,´, 5 , ˜˘n, c` ´,a l˜ phai chı chu uy th`ua bâc on o vê´ tr´ ai, hê. sô´ truo c x l` a 2a. Ngh˜ıa . cha ,, ,´, 3 ` ´ ´ ˘ng 2c, ngh˜ıa l` ´ c˜ l` a a = 0. Sau d¯o ung o vê tr´ ai, hê. sô truoc x ba a c = 0. V`ı

´,c giao ho´ 4.3. Chuyên d ac d¯a thu an ¯ê` vê` c´

195

, ˘. c v` thê´ c` on Q(x) = x3 + bx. Khai triên ngoa a so s´ anh c´ ac hê. sô´ c` on lai . ta ,, nhân . d¯uo. c   2b = −3α, b2 = 3α2 + b,   α = α3 + bα. , ,, ´, hai trong hê. ta c´ ˘. t α = 2γ, khi d ´ b = −3γ. T`u phuong tr`ınh thu Ða o ¯o ˘. c l` 9γ2 = 12γ2 − 3γ, hay l` a γ2 − γ = 0. Ngh˜ıa l` a γ = 0 hoa a γ = 1. Suy ra , ˜ ˘ ´ α = 0 hoa. c α = 2. Dê kiêm tra hai gi´ a tri. n` ay l` a nghiêm ¯ung. . d , ´,c bâc ´,c P(x) = x2 −α ´,i d Nhu vây, an vo ¯a thu . tô`n tai . d¯a thu . ba Q(x) giao ho´ ´,i α = 0: P(x) = x2 , Q(x) = x3 ; vo ´,i α = 2: ˘. c α = 2. Vo nê´u α = 0 hoa P(x) = x2 − 2, Q(x) = x3 − 3x. ,, , ´,ng minh d¯u,o.,c nê´u Q(x) l` ´,c bâc Tuong tu. c˜ ung chu a d¯a thu . hai, th`ı , ´ c bâc P(x) = Q(x), c` on nê´u Q(x) l` ad ¯a thu . môt, . th`ı Q(x) = x.

J

, , V´ı du. 4.35. Chu´ng minh ra ˘`ng trong c´ ac d¯a thu´c bâc a . k, tô`n tai . không qu´ , , , ´ ´ ˜ cho P(x). môt an v´ oi d¯a thuc bâc ¯a thuc giao ho´ ¯a . d . hai d , , , `,i giai. Cho d¯a thu ´,c P(x) = x2 + px + q, phai t`ım c´ Lo ac hê. sô´ a1 , ..., ak cua ´,c sau: d¯a thu

, ´,c ˘ng thu Ta c´ o d¯a

Q(x) = xk + a1 xk−1 + a2 xk−2 + · · · + ak .

(xk + a1 xk−1 + · · · + ak )2 + p(xk + a1 xk−1 + · · · + ak ) + q− − (x2 + px + q)k − a1 (x2 + px + q)k−1 − · · · − ak = 0. ,, `˘ng c´ `˘ng 0 nh˜u,ng hê. sô´ tru,o ´,c x2k , x2k−1 , ..., x, x0 . Ta nhân Ba ach cho ba ¯uo. c . d , ,, ´,i a1 , a2 , ..., ak , p, q. Ði giai hê. phu,o,ng tr`ınh môt ¯ô´i vo . hê. phuong tr`ınh d , , ,, , `˘ng c´ n` ay không phai l` a d˜ê. Ta tiê´n h` anh ba ach kh´ ac: Gia su nh˜ung hê. sô´ ,´, 2k−1 2k−2 ,, ,, ´,c truo cx ,x , ..., xk lâ`n luo. t l` a b1 , b2 , ..., bk , ta nhân . d¯uo. c công thu

,, ,, Chuong 4. Phuong tr`ınh h` am

196 sau b1 = 2a1 + R1 (p, q) = 0,

b2 = 2a2 + R2 (p, q, a1 ) = 0, ... bk = 2ak + Rk (p, q, a1 , ..., ak−1 ) = 0, , , ,, ´,c d od a môt ¯ây Ri l` ¯ai . biêu thu . sô´ cua p, q, a1 , ..., ai−1 . , , , , ´,c d¯â`u tiên cua hê. trên suy ra a1 biêu di˜ên qua p, q, T`u, ˘ng thu T`u d¯a , , , ´,c thu ´, hai suy ra a2 biêu di˜ên qua p, q v` ˘ng thu d¯a a a1 , ngh˜ıa l` a n´ o c˜ ung chı , , , , ´,c thu ´, ba a3 c˜ ˘ng thu biêu di˜ên qua p, q. T`u d ung biêu di˜ên qua p, q, a1 , a2 , ¯a , , , ,, ngh˜ıa l` a n´ o c˜ ung biêu di˜ên chı qua p, q, ..., tiê´p tuc . nhu vây . s˜e nhân . d¯uo. c , , , , c´ ac hê. sô´ a1 , a2 , ..., ak d¯ê`u biêu di˜ên chı qua p, q. Nhu vây, ac hê. sô´ cua . c´ , , ´, ´,c Q(x) giao ho´ ´,i P(x) x´ d¯a thu an vo ac d¯inh ng thông qua p, q. д o l` a . tuong u , ´ ng minh. d¯iê`u câ`n chu

J

, ˜,ng d¯a thu´,c bâc V´ı du. 4.36. H˜ ay t`ım tâ´t ca nhu a bâc a n´ o giao ho´ an . 4 v` . 8 m` , , ˜ cho P(x). v´ oi d ¯a thu´c bâc ¯a . hai d , `˘ng d¯a thu `,i giai. Ta s˜e chu ´,c Q(x) = P(P(x)) giao ho´ ´,ng minh ra ´,i an vo Lo ´,c Q(x) c´ P(x). Thât o bâc a . vây, . Q(P(x)) = P(P(P(x))) = P(Q(x)). Ða thu . l` , , , , ´ c giao ho´ ´i P(x). Tuong 4 v` a theo v´ı du. 4.35 th`ı n´ o l` a d¯a thu an duy nhâ´t vo , , , ´ ng minh cho d¯a thu ´ c R(x) = P(P(P(x))) duy nhâ´t bâc tu. ta chu an . 8 giao ho´ , ´i P(x). vo

J

, , , V´ı du. 4.37. Chu´ng minh ra ˘`ng hai d a R(x) giao ho´ an v´ oi d¯a ¯a thu´c Q(x) v` , , , thu´c thu´ ba P(x) bâc ung giao ho´ an v´ oi nhau. . 2, th`ı ch´ , `,i giai. Ða ´,i ˘. t S(x) = Q(R(x)) v` Lo a T (x) = R(Q(x)). V`ı P(x) giao ho´ an vo Q(x) v` a R(x), nên P(S(x)) = P(Q(R(x))) = Q(P(R(x))) = Q(R(P(x))) = S(P(x)).

´,c giao ho´ 4.3. Chuyên d ac d¯a thu an ¯ê` vê` c´

197

´,c S(x). Tu,o,ng tu., P(x) c˜ ´,i d Ngh˜ıa l` a P(x) giao ho´ an vo ung giao ho´ an ¯a thu , , , ˜ ` ´ c c` ´i T (x). Dê thâ´y ra ˘ng S(x) v` vo a T (x) l` a nh˜ung d¯a thu ung bâc, . (nê´u Q(x) , v` a R(x) c´ o bâc a k v` a `, th`ı bâc a T (x) l` a k`). Theo v´ı du. 4.35 . l` . cua S(x) v` suy ra S(x) = T (x), ngh˜ıa l` a Q(R(x)) = R(Q(x)).

J

, 4.3.2. Ða th´ uc Chebyshev , , Ðinh ˘`ng tô`n tai ay d ¯a thu´c P1 , P2 , ..., Pk , ... giao . môt . d˜ . l´ı 4.1. Chu´ng minh ra , ho´ an c` ung nhau sao cho bâc a k v` a P2 (x) = x2 − 2. . cua Pk (x) l` , , ,, `˘ng môt `,i giai. Ta giai b` Lo ai to´ an ba ap. . sô´ phuong ph´ , ,, ˘. t x = t + t−1 . Khi d¯o ´ d˜ê d` Phuong ph´ ap 1: Ða ang kiêm tra xk c´ o dang . xk = (t + t−1 )k = = (tk + t−k ) + a1 (tk−1 + t−(k−1) ) + a2 (tk−2 + t−(k−2) )+ + · · · + ak−1 (t + t−1 ) + ak ,

,, , `˘ng quy nap od a nh˜ung sô´ cô´ d¯inh. Ba an hoc ¯ây a1 , a2 , ..., ak l` . . to´ . theo k suy , , , , k −k ˜ ´i dang ra t + t c´ o thê biêu diên duo . xk + b1 xk−1 + · · · + bk−1 x + bk ,

,, , ´. K´ı hiêu od a nh˜ung sô´ cô´ d ao d ¯ây b1 , b2 , ..., bk l` ¯inh ¯o . n` . Pk (x) = xk + b1 xk−1 + · · · + bk . −1 k −k Theo d¯inh . ngh˜ıa th`ı Pk (t + t ) = t + t sao cho

Pk (P` (t + t−1 )) = Pk (t` + t−` ) = tk` + t−k` = Pk` (t + t−1 ). , ´,c bâ´t k`ı ˘. p d ´ môt Nhu vây, ¯a thu . Pk (P` (x)) = Pk` (x) = P` (Pk (x)). Do d¯o . ca , , ,, ˜ xây du. ng d¯uo. c d˜ t`u d˜ ay Pk (x) d¯ê`u giao ho´ an. V`ı P2 (x) = x2 − 2, ta d¯a ay , , ´ c câ`n t`ım. nh˜ung d¯a thu , ,, ,, ´,c Pk (x) thông qua d˜ Phuong ph´ ap 2: C´ o thê nhân ay d ay d ¯uo. c d˜ ¯a thu ¯a . d , , , , ´ c Chebyshev Tk (x) (c´ ´ c Chebyshev trong thu ac ban o thê tham khao d¯a thu . c´

,, ,, Chuong 4. Phuong tr`ınh h` am

198

, , , , ´,c n` [12]). Ta s˜e không d¯ua ra d ngh˜ıa tru. c tiê´p d ay, nhung d ¯inh ¯a thu ¯ua . , ,, ´,c n` d¯ê´n phuong tr`ınh h` am m` ad ay thoa m˜ an: Tk (cos t) = cos kt. Ta ¯a thu , , , , , ˜ ` ´ ng minh ra ´ c bâc ´,c ˘ng Tk l` c´ o thê chu a d¯a thu ¯a thu . k. Dê kiêm tra nh˜ung d ´,i nhau: Tk (Tm (x)) = Tkm (x) = Tm (Tk (x)) (do su., Chebyshev giao ho´ an vo , , , , biê´n d¯ôi cua cos k(mt) = cos kmt = cos m(kt)). Nhung Tk không phai l` a , ,, , , , ´ ´ ´ ´ ´ c d¯uo. c goi a unita, nêu hê. sô o sô hang unita (môt . l` . bâc . cao nhât cua . d¯a thu , ,, , k−1 ` ´ ´ ´ ˘ng 1), v`ı hê. sô o sô hang n´ o ba a2 . Nhung d ay c´ o thê ¯iê`u n` . bâc . cao nhât l` , ,, , ´˘c phuc ´,i c´ ˘. t n` ˘. t Pk (x) = 2Tk ( x2 ). Vo ach d¯a ay kha a d¯a ¯uo. c nh`o viêc . d . d¯ôi biê´n v` , , , , , , ˜ ` ´ c vân d¯uo. c bao to` ˘ng d˜ th`ı t´ınh giao ho´ an cua c´ ac d¯a thu an. Nhu vây ay . ba , , , , ` ´ c Chebyshev ta c˜ d¯a thu ung du. ng d ay theo yêu câu. Môt u ´y quan ¯uo. c d˜ . ch´ , , , ´ ` ´ c Chebyshev c´ ´ ng dung trong l` a nh˜ung d¯a thu ou rât nhiêu trong l´ı thuyê´t . . , , , `˘ng d¯a thu ´ c. xâ´p xı nh˜ung h` am ba , ,, , , , , ´: Nh˜ung phuong ph´ Ch´ uy ap trên xuâ´t ph´ at t`u ´y tuong sau d¯ây: Cho môt . , , ´ ´ h` am sô´ f nhân vô h an c´ a c gi´ a tr i v` a sao cho v o i m oi sô t u nhiên n, f(nt) = . . . . . ,, , ´ ´,i nhau. ´ Qn (f(t)), o d¯ây Qn (x) l` ad a th u c. Khi d o Q v` a Q giao ho´ an vo ¯ ¯ m n ,, ´,c giao ho´ V´ı du, am f(t) = et th`ı ta nhân ay c´ ac d¯a thu an ¯uo. c d˜ . nê´u cho h` . d , , , ´ c Pn nhu, Qn (x) = xn . Nê´u f(t) = et + e−t , th`ı ta nhân ay d¯a thu ¯uo. c d˜ . d , `˘ng trong trong v´ı du. d ay (chı c´ o kh´ ac l` a thay t ba ¯â`u tiên trong chuyên d ¯ê` n` ,, , , , , t ´ c Tn nhu o trên. e ). Nê´u f(t) = cos t, th`ı ta nhân ay d¯a thu ¯uo. c d˜ . d , ,, ´,c Pk bâc Phuong ph´ ap 3: T`u v´ı du. 4.35, suy ra tô`n tai ¯a thu . duy nhâ´t d . k , , , 2 ´ c nhu vây: ´i P2 (x) = x − 2. Ta viê´t môt giao ho´ an vo . sô´ d¯a thu . P1 (x) = x, P2 (x) = x2 − 2, P3 (x) = x3 − 3x, P4 (x) = x4 − 4x2 + 2, P5 (x) = x5 − 5x3 + 5x, P6 (x) = x6 − 6x4 + 9x2 − 2,

´,c giao ho´ 4.3. Chuyên d ac d¯a thu an ¯ê` vê` c´

199

,, , `˘ng c´ od a P5 (x) t`ım ba ach nhu ¯ây P4 (x) = P2 (P2 (x)), P6 (x) = P2 (P3 (x)) v` , , , , , ´,c trên ta c´ ´,c ˘ng thu trong v´ı du. 4.34. T`u nh˜ung d o thê gia thiê´t d˜ ay d ¯a ¯a thu , ´,c hô`i quy sau: thoa m˜ an công thu Pk+1 (x) = xPk (x) − Pk−1 (x). , ´,ng minh công thu ´,c n` ´,i moi ´ ng vo Ta c´ o thê chu ay d ac d¯inh ay ¯u . d˜ . k > 1. X´ , 2 ` ´ c P1 , P2 , ..., Pn , ... ba ´,i ˘ng quy nap: d¯a thu on vo . P1 (x) = x, P2 (x) = x − 2, c` , `˘ng tâ´t ca d¯a thu ´,ng minh ra ´,c k > 1, Pk+1 (x) = xPk (x) − Pk−1 (x). Ta s˜e chu , ´ c P2 ; khi d¯o ´,i d ´ ch´ Pk giao ho´ an vo ung giao ho´ an c` ung nhau (do v´ı du. ¯a thu 4.37). `˘ng phu,o,ng ph´ ´,ng minh ba Ta chu ap quy nap. a P2 (x) giao . D˜ê thâ´y P1 (x) v` , ,, , , , ´ ´ ´ ho´ an voi P2 (x). Gia su tât ca c´ ac d an ¯a thuc P1 (x), P2 (x), ..., Pk (x) giao ho´ , ´,ng minh Pk+1 (x) c˜ ´,i P2 (x), ta phai chu ´,i P2 (x). Ngh˜ıa l` vo ung giao ho´ an vo a , , , , ´ ng minh công thu ´ c giao ho´ ta phai chu an gi˜ua Pk+1 (x) v` a P2 (x) = x2 − 2 l` a: Pk+1 (x2 − 2) = (Pk+1 (x))2 − 2. , ,, ´,i Pk+1 v` Thât ao v` a biê´n d ¯ôi vê` dang . vây, . ta thay phuong tr`ınh hô`i quy vo . , ´ ng minh l` ta câ`n chu a (x2 − 2)Pk (x2 − 2) − Pk−1 (x2 − 2) = (xPk (x) − Pk−1 (x))2 − 2. , , ,, `˘ng c´ Ba ach khai triên v` a d¯ua vê` c´ ac sô´ hang . chung ta nhân . d¯uo. c −2([Pk (x)]2 − xPk (x)Pk−1 (x) + [Pk−1 (x)]2 ) = 2x2 − 8 ngh˜ıa l` a [Pk (x)]2 − xPk (x)Pk−1 (x) + [Pk−1 (x)]2 = 4 − x2 .

(4.29)

, , ´,ng minh Sk (x) không Ta k´ı hiêu ai cua (4.29) l` a Sk (x). Ta chı c` on chu . bên tr´

,, ,, Chuong 4. Phuong tr`ınh h` am

200

`˘ng 4 − x2 . Thât ´ phu. thuôc ao k v` a ba ¯o . v` . vây, . cho k > 2. Khi d Sk (x) = [Pk (x)]2 − xPk (x)Pk−1 (x) + [Pk−1 (x)]2 = = Pk (x)[Pk (x) − xPk−1 (x)] + [Pk−1 (x)]2 = = −Pk (x)Pk−2 (x) + [Pk−1 (x)]2 = = [Pk−2 (x)]2 − xPk−1 (x)Pk−2 (x) + [Pk−1 (x)]2 = Sk−1 (x), ´,i moi ngh˜ıa l` a Sk (x) = Sk−1 (x). V`ı S2 (x) = 4 − x2 , nên Sk (x) = 4 − x2 vo . , , ,, , , ´ ´ ´ an voi P2 (x) voi k ≥ 2. Nhu vây ¯a chung minh d ¯uo. c Pk (x) giao ho´ . ta d moi . k.

J

, , ´ vê` cac ´ da ´ ` toan ´ tông quat uc giao hoan 4.3.3. Bai ¯ th´ , , ´,c P(x) (không nhâ´t thiê´t phai l` Khi cho d¯a thu a unita) b` ai to´ an tông , , ´,c Q(x) (không nhâ´t thiê´t phai l` ˘. t ra l` qu´ at d¯a a t`ım nh˜ung d a unita) ¯a thu , ,`, , , , , ´ ´ giao ho´ an voi n´ o. Truong ho. p d a c´ ac d o bâc a 1 th`ı ban ¯on gian l` ¯a thuc c´ . l` . , , ´ ´ d¯oc ay giai b` ai tâp a b` ai tâp a Q(x) . h˜ . 4.38 v` . 4.39. Tiêp tuc, . ta gia thiêt P(x) v` , , ` ´ d¯êu c´ o bâc . lon hon 1. Cho P(x) = a0 xk + a1 xk−1 + · · · + ak−1 x + ak

v` a

Q(x) = b0 x` + b1 x`−1 + · · · + b`−1 + b` . , , , `˘ng 0, ´,c trên vê` dang Ta c´ o thê d¯ua nh˜ung d c´ ac hê. sô´ a1 v` a b1 b a ¯a thu . , ,, ´,c nhu, vây ´,c tinh nh˜ung d a d¯a thu ¯a thu ¯uo. c tiê´n . tiê´n d . tiê´n. Viêc . goi . l` . tinh , , ´,c mo ´,i n´ ´,i h` anh nhu sau: Ta cô´ d¯inh a vo o ta xây du. ng môt ¯a thu . môt . sô´ a v` . d , , ´,c d¯a ´,c: R(a) (x) = R(x − a) + a. Ðê ˜ cho R(x) theo công thu R(a) t`u d¯a thu ´,c R(x) t`u, R(a) (x) , ta d` ´,c R(x) = R(a) (x + a) − a. lâ´y lai ung công thu . d¯a thu V´ı du. R(x) = 2x2 + 3x + 5 th`ı R(a) (x) = 2(x − a)2 + 3(x − a) + 5 + a = , , 2x2 − 4ax + 2a2 + 3x − 3a + 5 + a, d¯ê a1 = 3 − 4a = 0 suy ra ta phai cô´ d¯inh . a=

3 4

v` a R(a) (x) = 2x2 +

37 8 .

´,c giao ho´ 4.3. Chuyên d ac d¯a thu an ¯ê` vê` c´

201

`˘ng nê´u P(x) v` ´,ng minh d¯u,o.,c ra ´,c giao ho´ D˜ê chu a Q(x) l` a hai d¯a thu an th`ı P(a) (x) v` a Q(a) (x) c˜ ung giao ho´ an. , `˘ng P(x) v` ´,c tinh Phâ`n sau d a Q(x) l` a nh˜ung d¯a thu tiê´n, ¯ây ta cho ra . ,, ngo` ai ra ch´ ung c` on l` a unita, ngh˜ıa l` a ch´ ung c´ o hê. sô´ o sô´ hang bâc . . cao , , , ,´, , ` ` ´ ` ˘ng 1. Ba ˘ng c´ nhât ba ac l´ı luân an to` an tuong tu. nhu phân truoc ta t`ım . ho` ,, , , `˘ng ±1. Ta liêt ´ ´ ´ d¯uo. c d˜ ay d an voi hê. sô bâc ¯a thuc giao ho´ . cao nhâ´t ba . kê lai . môt . sô´ v´ı du: .

´,c bâc ´,c P0 (x) = x, P1 (x) = A. Cho R(x) l` a môt ay d ¯a thu . d¯a thu . r. X´et d˜ R(x), P2 (x) = R(R(x)), P3 (x) = R(R(R(x))), ... Pi+1 (x) = Pi (R(x)). , `˘ng c´ ´,c Pi (x) giao ho´ ´,i nhau v` D˜ê thâ´y ra ac d an vo a bâc ¯a thu . cua Pi (x) `˘ng ri . ba , ´,c Fk (x) = xk . Tâ´t ca d ´,c Fk (x) giao ho´ B. Cho d˜ ay d an d¯ôi môt ¯a thu ¯a thu . , ` ˘ng k. v` a bâc . cua Fk (x) ba , −1 k −k ´ c dang C. D˜ ay d an d a bâc ¯a thu ¯ôi môt . Pk (t + t ) = t + t giao ho´ . v` . , `˘ng k. cua Pk (t) ba `˘ng công thu ´,c Hk (t − t−1 ) = tk + t−k (k D. Ðinh ay H1 , H2 , .... ba . ngh˜ıa d˜ , , ´,ng minh d¯u,o.,c d˜ ´,c trên tô`n tai l` a sô´ le). D˜ê chu ay d a x´ ac d Nh˜ung ¯a thu ¯inh. . . v` , ´,c n` d¯a thu ay c˜ ung giao ho´ an d¯ôi môt a bâc a k. . v` . cua Hk (x) l` ´,u vê` d¯a thu ´,c ngu,`o,i ta c˜ E. Trong d¯ai ung hay nghiên . sô´, khi nghiên cu , , , , , , , ´ ´ ´ u nh˜ung d ´ c c´ ´ c (ban cu o hê. sô l` a nh˜ung sô phu o thê tham khao ¯a thu . d¯oc . c´ , ´,c λ sao trong cuô´n s´ ach [8]). Ta cô´ d¯inh a môt . môt . sô´ tu. nhiên m v` . sô´ phu , ´,c th`ı vo ´,i moi cho λm = 1 (theo t´ınh châ´t cua sô´ phu . m, tô`n tai . m − 1 sô´ , nhu vây ac 1). . kh´ , , ˘. p sô´ (u, v) sao cho uv = λ, v` ˘. t x = u + v. D˜ê kiêm Ta x´et nh˜ung ca a d¯a ,, tra d¯uo. c uk + vk = xk + a1 xk−1 + · · · + ak , ,, , ˘. t Pk (x) = ´ phu. thuôc o d¯ây a1 , a2 , ..., ak l` a nh˜ung sô´ n` ao d¯o ao λ. Ða . v`

202

,, ,, Chuong 4. Phuong tr`ınh h` am

, `˘ng d ´,c Pk (x) d ˘. c trung ba xk +a1 xk−1 +· · ·+ak . Ða thu ¯a ¯iê`u kiên: . Pk (u+v) = k k u + v nê´u uv = λ. , ´ u` v` = (uv)` = λ` = λ, Cho sô´ ` sao cho n´ o chia cho m c` on du 1. Khi d¯o , ` ` kl kl vây a ` chia cho m du 1 th`ı . Pk (u + v ) = u + v . Vây . th`ı nê´u k v` Pk (P` (u + v)) = ukl + vkl = P` (Pk (u + v)), ngh˜ıa l` a Pk (x) v` a P` (x) giao , ,, , , ´ ˜ ´,c giao ´ ˜ xây du. ng d¯uo. c theo sô λ d˜ ho´ an voi nhau. Ta d¯a ay nhung d¯a thu , , ´,c n` ho´ an P1 (x), Pm+1 (x), P2m+1 (x), ... tâ´t ca nh˜ung d¯a thu ay l` a unita, tinh . , ´ tiên v` a bâc a k. . cua Pk (x) l` , , , ´i m = 1, λ = 1 ta c´ ´,i m = 2, λ = −1 ˘. c biêt Ða o kê´t qua cua phâ`n A, vo . vo , , ta c´ o kê´t qua cua phâ`n D. , , , C´ ac v´ı du. t`u phâ`n A d ay bao h` am tâ´t ca nh˜ung ¯ê´n phâ`n E trong phâ`n n` , , , `˘ng nê´u P(x) v` ´,c giao ho´ d¯a thu an. Ta c´ o thê gia thiê´t ra a Q(x) l` a nh˜ung d¯a , ´,c giao ho´ ´,c, ´,n ho,n 1 vo ´,i hê. sô´ l` thu an, unita, tinh a nh˜ung sô´ phu . tiê´n, bâc . lo , ,, , , ,, , , `˘m o, môt th`ı ch´ ung na ay d ¯uo. c xây du. ng o phâ`n A, B, E o . trong nh˜ung d˜ trên.

` tâp Bai . `˘ng hai d¯a thu ´,ng minh ra ´,c bâc ´,i . 4.38. Chu a Q(x) giao ho´ an vo . nhâ´t P(x) v` , ˘. c l` ˘. c l` nhau khi v` a chı khi hoa a P(x) = x + α, Q(x) = x + β, hoa a tô`n tai . môt . , x0 sao cho P(x0 ) = Q(x0 ) = x0 (d ¯iêm bâ´t d ¯ông). . , ´,c bâc ´,c Q(x) giao . 4.39. Cho P(x) l` a d¯a thu ay t`ım tâ´t ca c´ ac d ¯a thu . 1, h˜ ´,i P(x). ho´ an vo `˘ng nê´u hai d¯a thu ´,ng minh ra ´,c P(x) v` . 4.40. Chu a Q(x) giao ho´ an th`ı , (a) (a) ´ c giao ho´ P (x) v` a Q (x) c˜ ung l` a hai d an. ¯a thu

, , ,, , `i b` 4.4. Go. i ´y v` a tra lo ai tâp . chuong 4

203

, , ,, , ` tra l`oi bai ` tâp 4.4. Go. i y ´ va . chuong 4 , x2 − a `˘ng 1 . H` am kê´t qua l` a f(x) = . . 4.12. Thay x ba x (1 − a2 )x ,, `˘ng: x = 0, y = z; . 4.13. Thay lâ`n luo. t c´ ac gi´ a tri. x v` a y ba π π π π ,, ,, x = + z, y = ; x = , y = + z v` a nhân ¯uo. c phuong tr`ınh . d 2 2 2 2 π f(z) = f(0) cos z + f( ) sin z. 2 , ,, ,, Giai phuong tr`ınh h` am n` ay, ta nhân . d¯uo. c nghiêm . f(x) = a cos x + b sin x, ,, , ` ´ ´ ˜ ˘ od a nhung hang sô bât k`ı. ¯ây a, b l` ,, , . 4.14. D` ung phuong ph´ ap bâ´t biê´n nhu trong b` ai 4.3. Nghiêm a f(x) = . l` 2 x + c. ´,i: x = 0, y = z; x = 0, y = −z; x = z, y = 0 . 4.15. Thay x v` a y liên tiê´p vo ,, ,, , v` a phuong tr`ınh nhân ¯ua vê` f(z) = z + 1. . d¯uo. c d h x i2 x x , am f1 (x) = 0 . 4.16. T`u f(x) = f( + ) = f( ) suy ra f(x) ≥ 0. H` 2 , 2,, 2 , ´i f1 (x) = 0. Khi d ´ l` a môt Gia su c´ o nghiêm ac f(x) kh´ ac vo ¯o . nghiêm. . . kh´ , ´ ´ f(x) > 0 voi moi o môt a tri. x = x0 sao cho f(x0 ) = 0, . x. Thât . vây, . nêu c´ . gi´ , ,, ,, ` ˜ cho ta nhân tu phuong tr`ınh d¯a ¯uo. c . d f(x0 + y) = f(x0 ).f(y) = 0.f(y) = 0 , , ´,c cuô´i c` ´,i moi ´,i moi ˘ng thu vo ung suy ra f(x) = 0 vo a ¯a . y. T`u d . x, ngh˜ıa l` , ` ´i d ˘. t ra. f(x) = f1 (x) vô l´ı vo ¯iêu ta d¯a ,, , ´ ˜ cho v` Lôgarit co sô 10 hai vê´ phuong tr`ınh h` am d a h` am sô´ lgf(x) ¯a , , ,, ,, thoa m˜ an phuong tr`ınh (4.13). Nghiêm a f(x) = 0 v` a . cua phuong tr`ınh l` f(x) = ax . , ,, . 4.17. H` am lgf(x) thoa m˜ an phuong tr`ınh (4.18) v` a c´ o nghiêm . f(x) = 0 a v` a f(x) = x . ,, . 4.18. D` ung phuong ph´ ap Cauchy v` a nghiêm a f(x) = a.x2 . . l`

204

,, ,, Chuong 4. Phuong tr`ınh h` am

, , ,, . 4.19. Môt a ϕ(x) = cotgx. Kê´t qua l` a . nghiêm . riêng cua phuong tr`ınh l` f(x) = a + cotgx. , , ,, . 4.20. Môt a ϕ(x) = x. Kê´t qua f(x) = . nghiêm . riêng cua phuong tr`ınh l` x + ax . ´,i n ∈ R, x´et y = x, y = 2x, ..., y = (n − 1)x. . 4.27. Vo , , , . 4.31. H˜ ay chı ra f chı c´ o d¯iêm bâ´t d¯ông . duy nhâ´t tai . 0. , ,, , ´ chı ra x > 0 k´eo theo f(x) > 0 v` . 4.32. Gia su f(0) = 0, khi d a f t˘ ang. ¯o ,, , , ´i y = cx vo ´i c kh´ . 4.33. Thu vo ac nhau thuôc a y = x − 2. . Q v`

, , CHUONG

5

, , ˜ ` ´ HOC NHUNG TRO CHOI TOAN .

5.1. Chiê´n thuât . bâ´t biê´n . . . . . . . . . . . . . . . . . . . . . . . . . . . . . . . . . . . . . . . . . . . . . . . . . . . , , , ´ vo ` choi . . . . . . . . . . . . . . . . . . . . . . . . . . . . . . . . . . . . . . . . . . . . . ´ i tro 5.2. Kiê´n th´ uc toan , , ` toan ´ thi hoc 5.3. Nh˜ung bai . sinh gioi . . . . . . . . . . . . . . . . . . . . . . . . . . . . . . . . . . . . . . . , ` choi Nim . . . . . . . . . . . . . . . . . . . . . . . . . . . . . . . . . . . 5.4. Chuyên dê ¯ ` chiê´n thuât . tro

5.4.1. Ðinh a v´ı du. . . . . . . . . . . . . . . . . . . . . . . . . . . . . . . . . ngh˜ıa v` , 5.4.2. Chiê´n thuât trong tr` o choi Nim . . . . . . . . . . . . . . . . . . . . , ´,ng minh bang chiê´n thuât ´,ng dung 5.4.3. Chu au n´ o .... . v` .

206 210 214 225

225 229 236

, ` liêu Tai . tham khao . . . . . . . . . . . . . . . . . . . . . . . . . . . . . . . . . . . . . . . . . . . . . . . . . . . . . . . . . . , Nh˜ung k´ı hiêu . ...............................................................

241

Muc . luc . .....................................................................

242

240

, , , , ´˘ng hoa ˘. c `i hoi nh˜ung chiê´n thuât C´ o râ´t nhiê`u tr` o choi to´ an hoc ¯o . d . d¯ê tha , , , , ,´, , , `i hoi môt d¯ê bao vê. buoc d¯i trong cuôc. o choi n` ay c´ o thê d ¯o . Nh˜ung tr` . sô´ , , , , ˘. c nh˜ung ph´ep suy ra cua t´ınh châ´t l´ı thuyê´t cua tâp ¯ê` hoa . ho. p, lôgic mênh . d , , , ,´, , , to´ an tô ho. p. Môt o choi c´ o thê c´ o vô han c d¯i, lai o nh˜ung tr` o . sô´ tr` . buo . c´ , , , , , , , ´ ´ ` ´c. Môt ˘ng l` choi phai d`ung sau h˜uu han a môt . buo . chiên thuât . tha . so d¯ô cho ,´, , ´ ,´, ,`, ´ , ,, ˘ng luôn luôn thu. c hiên a buo c ph´ep nguoi tha ¯i, trong thu. c tê l` . d¯uo. c buoc d , , , ,´, ´ ´ ´ ´ ˘. n buoc d ˘ng cua d d¯i cuôi c` ung. Môt o thê t`ım ¯i tha ¯ôi thu c´ . chiên thuât . cha , , , ` ´ ´ ` ´ c to´ ˘ng kiên thu thây ba an hoc. o choi to´ an hoc . ngh˜ıa vê tr` . Môt . d¯inh . nhu sau:

,, , ˜,ng tr` Chuong 5. Nhu o choi to´ an hoc .

206

,, ,, , , an hoc Ðinh o choi to´ . thu`ong gô`m môt . sô´ ngu`oi choi sao . tr` . ngh˜ıa 5.1. Môt cho: , ,, , , ,´, , 1) Nh˜ung ngu`oi choi thay phiên nhau thu. c hiên c d¯i trên . nh˜ung buo , ,, ,, ,´, ´ ´ ´ ˜ ˘c cho truoc; huu han a theo môt ¯ôi tuo. ng d¯êm d¯uo. c v` . d . quy ta ,`, , , , ,, ´ , , ´c d¯i l` ˘ng cuôc. 2) Nguoi cuô´i c` ung thu. c hiên a ngu`oi tha ¯uo. c buo . d . , , , ´,u hai kh´ıa Nhu vây, ai to´ an tr` o choi ta phai nghiên cu . khi xem x´et môt . b` ´˘c cho,i nhu, thê´ n` ´, nhâ´t l` ´, hai l` canh quan trong: Thu a quy ta ao; thu a chiê´n . . , , ´ , , , , ˜ ´i môi ngu`oi choi. C´ ˘ng han thuât ao d o râ´t nhiê`u v´ı du, ¯ô´i vo . nhu thê n` . cha . , , nhu tr` o choi sau: , , , , C´ o ba d ¯ô´ng soi gô`m d¯ô´ng 1 viên soi, d ¯ô´ng 3 viên soi, d ¯ô´ng 5 viên soi. Hai , , , ,, ,, ´. ngu`oi lâ`n luo. t lâ´y d¯i môt ˘. c môt ao d ¯o . d¯ô´ng soi hoa . sô´ viên soi cua môt . d¯ô´ng n` , ,`, , , ´ ´ ´ ´ ´ Nguoi n` ao lây viên soi cuôi c` ung l` a ngu`oi ây chiên tha ˘ng. , , , , , ´, , Tr` o choi c´ o thê thu. c hiên a ngu`oi thu nhâ´t v` a II l` a . nhu sau: K´ı hiêu . I l` ,`, ´, nguoi thu hai. I

II

I

II

I

(1, 3, 5) −→ (1, 3, 2) −→ (1, 1, 2) −→ (1, 1, 0) −→ (1, 0, 0) −→ (0, 0, 0),

, , , ´, ,, , , ´˘ng. Ðây l` a dang o choi Nim d oi k˜ı v` a ngu`oi thu nhâ´t tha ¯on gian s˜e d¯uo. c n´ . tr` , , , trong chuyên d ay. ¯ê` cua chuong n` ,, , , ´, ˘. t ra l` Thu`ong môt ai to´ an d¯a a t`ım môt nhâ´t . b` . chiê´n thuât . cho ngu`oi thu ,`, , , , , ´˘ng. Phâ`n thu ´ hai tha ´ nhâ´t ta x´et nh˜ung b` ˘. c nguoi thu hoa ai to´ an tr` o choi , ,, , tuong d¯ô´i d¯on gian.

5.1. Chiê´n thuât . bâ´t biê´n , , , Môt ac luât a thu. c hiên am sao . chiê´n thuât . trong khi thu. c hiên . c´ . choi l` . l` , ` ˜, , , , , , , , ´ ´ d¯ua vê nhung d˜u kiên o lo. i cho ngu`oi choi. д o c˜ ung l` a nh˜ung l`oi . bât biên c´ , , , giai cua c´ ac b` ai to´ an tr` o choi to´ an hoc. .

5.1. Chiê´n thuât . bâ´t biê´n

207

,, , , oi 2001 hat a lâ´y V´ı du. 5.1. A v` a B tiê´n h` anh tr` o choi v´ ¯âu. . d . Môt . nuo´c d¯i l` , , ,, , khoi d ˘. c 3 hat a thay phiên nhau. Ngu`oi ¯ô´ng hat ¯âu . d . 1, 2 hoa . d¯âu. . A d¯i truo´c v` ,, ,, ,, n` ao lâ´y d ung l` a ngu`oi chiê´n tha ˘´ng. Vây ao c´ o chiê´n ¯uo. c hat ¯âu . d . cuô´i c` . ngu`oi n` , , ´ , ´ ´ ´ nhu thê n` thuât ˘ng trong cuôc a chiên thuât ao? ¯ê luôn tha ¯o . d . choi v` . d , ,, ´˘ng nê´u A thu.,c hiên `,i giai. A luôn luôn tha Lo . chiê´n thuât . sau: Khoi d¯â`u A ,, ,´, ´ lâ´y 1 hat c tiêp theo A s˜e lâ´y d a sô´ hat ¯i 4 − x hat, ¯ây x l` ¯âu . d¯âu, . nuo . od . d . B ,, , , , , ´ ` ` ´c d ´c d ´. Thât ˜ lây o nuo d¯a on lai ¯i truo ¯o . vây, . sau khi A d¯i lân d¯âu tiên, c` . 2000 ˜ ´ ´ ` ` ´ hat ¯âu. . d . Tiêp theo, theo chiên thuât . trên th`ı sau môi lân B rôi d¯ên A d¯i, , ` ´ ´ ˘ng bôi d¯ông hat on lai ung d ¯ê´n . d¯âu . luôn c` . sô hat . ba . sô´ cua 4. Do vây, . cuô´i c` ,´, , ,, ,, c on lai u B thu. c hiên ach n` ao th`ı A c˜ ung d luo. t B th`ı c` ¯i d ¯uo. c nuo . 4 hat. . D` . c´ ´ ´ ´ ´ ´ ˘ng cuôc. cuôi s˜e lây d¯i hêt sô hat a A tha . d¯âu . v` .

J

, , , V´ı du. 5.2 (M˜y 1999). Tr` o choi Y2K trên dai b˘ ang c´ o ô vuông k´ıch c˜o 1×2000. ,, ,, , ` , ˜,ng ô trô´ng hoa ao nhu ˘. c l` a S hoa ˘. c l` a O. Ngu`oi choi d Hai ngu`oi choi viê´t v` ¯â u , , ´ ´ ´ ´ ˜ SOS s˜e tha ˜ tiên tao anh chu ˘ng cuôc. ac ô d ¯a . ra ba ô liên tiêp th` . Nêu tât ca c´ , ,, , ` ` d a không tao o choi goi a h` oa. Chu´ng minh ¯iên d ¯ây m` ¯uo. c SOS th`ı tr` . ra d . l` ,`, , ´, ` ´ ´ ra ˘ng nguoi choi thu hai c´ o môt ˘ng. . chiên thuât . tha , `,i giai. Goi ˘. t S hoa ˘. c O v` Lo a xâ´u nê´u khi d ao ô n` ay s˜e tao ¯a . môt . ô trô´ng l` . cho ,`, ,, ,, , ´ ´ nguoi choi tiêp theo nhân ¯i tiêp theo. . d¯uo. c SOS trong luo. t d , , Bô d¯ê`: Môt a xâ´u khi v` a chı khi n´ o na ˘`m trong khô´i 4 ô vuông . ô vuông l` ,, liên tiê´p c´ o dang a môt ¯ây ∗ l` . S ∗ ∗S, o d . ô trô´ng. , ,, Thât a xâ´u. Nguo. c lai, . vây, . d˜ê thâ´y nh˜ung ô vuông trong S ∗ ∗S l` . nê´u ô vuông l` a xâ´u, th`ı khi ta viê´t O s˜e tao . d¯iê`u kiên . viê´t SOS trong lâ`n tiê´p , , ,, ,`, , , theo cua nguoi choi kh´ ac. Nhu vây o môt v` a môt . ô xâ´u phai c´ . S o bên canh . . ,, ,, , , ´ ´ ´ ô trô´ng o ph´ıa bên canh kia. Khi ta viê t S s˜ e b i thua ngay o bu o c tiê p theo, . . , , , ˘. t k´ı tu. S kh´ ngh˜ıa l` a ta phai d ac bên canh kia cua ô trô´ng, tao ¯a . . ra dang . , S ∗ ∗S l` a nh˜ung ô xâ´u. ,, , , ´˘ng ´, hai c´ Bây gi`o ta quay lai ai to´ an, ngu`oi choi thu o chiê´n thuât . b` . tha

208

,, , ˜,ng tr` Chuong 5. Nhu o choi to´ an hoc .

, , , ´, , , ´, , ˜ choi, ngu`oi thu nhu sau: Sau khi ngu`oi thu nhâ´t d¯a hai viê´t S tai ach . vi. tr´ı c´ , , , , ˘. c t`u ph´ıa ô d¯â`u tiên m` xa ´ıt nhâ´t 4 ô trô´ng t`u ph´ıa d ang hoa a ¯â`u cua dai b˘ ,`, ´, , , , , , , , ´ hai, ngu`oi thu ´ hai s˜e d¯a ´c thu ˘. t S c´ ˜ choi. Ðê´n buo nguoi thu nhâ´t d¯a ach xa , ,, , , , ,´, , , ` ´ ´ ´c d¯o ´ sao cho o gi˜ua l` ˜ choi cua m`ınh lân truo ba ô t`u ô d a trông. (Nêu buo c ¯a , , , , , , , , , ´ ` ´ hai cua ngu`oi thu ´ nhât o bên canh ˘. c c´ d¯i thu hoa ach môt . . ô t`u ô choi lân , , ,`, ´, ,`, ´, , ` ´ hai o, canh ˘. t S thu d¯âu cua nguoi thu hai, th`ı nguoi thu hai s˜e d bên kia.) ¯a . , , , ,´, , , , , ` ´ ´ hai cua ngu`oi thu ´ hai, tôn tai Sau buoc thu ang. Nhu . 2 ô xâu trên giai b˘ , ,, , , ´ trong hai ngu`oi choi phai d¯iê`n v` vây ung ai d ao nh˜ung ô n` ay v` a tr` o ¯o . cuô´i c` , choi s˜e không h` oa. ,´, ,, , , ´, hai d¯ê´n lu,o.,t, luôn luôn Tai c choi kh´ ac, khi ngu`oi choi thu . bâ´t k`ı buo , , ,, , ´, ˜˘n ô xâ´u, nhu, vây c´ o sô´ le ô trô´ng v` a sô´ cha hai c´ o thê luôn . ngu`oi choi thu , , luôn choi v` ao ô không phai l` a ô xâ´u.

J

, ,, , V´ı du. 5.3. C´ o môt an vuông v´ oi sô´ ô vuông trên c´ ac canh l` a le. Hai ngu`oi . b` . , ,, ,, , , ˜,ng d ao choi lâ`n luo. t d ˘. t nhu ao c´ ac ô vuông cua b` an. Ngu`oi choi n` ¯a ¯ô`ng xu v` , , , ,, không c` on chô˜ d ˘. t d a thua trân. ay chı ra ra ˘`ng ngu`oi ¯ê d¯a ¯ô`ng xu coi nhu l` . H˜ , , , choi thu´ nhâ´t c´ o thê luôn luôn chiê´n tha ˘´ng. , , , `,i giai. V`ı c´ ac canh c´ o sô´ le ô vuông nên tô`n tai Lo . . môt . ô vuông tai . tâm cua ,, ,, ,, , ´, , ´, ˘. t d¯ô`ng xu tai b` an. Ngu`oi choi thu nhâ´t d an. Nê´u ngu`oi choi thu ¯a . ô o tâm b` , ,, ` , , ´, ˘. t d ˘. t d¯ô`ng xu nhâ´t c´ o thê d hai c` on d¯a ao môt ¯a ¯uo. c d ¯ông xu v` . ô, th`ı ngu`oi thu , , , , ´ ng qua d¯ô`ng xu o tâm b` ´, hai ´i vi. tr´ı ngu,`o,i thu tai an (d¯ô´i diên . vi. tr´ı d¯ô´i xu . vo , , , , ˘. t d¯ô`ng xu). V`ı diên ˘. t b` ˜d d an c` on trô´ng giam, tr` o choi phai ¯a ¯a . t´ıch cua ma ,, , ´, ´˘ng. kê´t th´ uc. Ngu`oi choi thu nhâ´t s˜e tha

J

,, , , , V´ı du. 5.4 (Tr` o choi Bachet). Khoi d an c´ o n quân c`o v´ oi n > 0. ¯â`u, trên b` ,`, , ˜ ` , ´ , ´ ` Hai nguoi choi môi lân choi lây d a nhiêu nhâ´t k quân ¯i ´ıt nhât 1 quân c`o v` , , ,, ,, ,, , , c`o. Ngu`oi cuô´i c` ung c´ o thê lâ´y d on lai an s˜e d ¯uo. c tâ´t ca quân c`o c` ¯uo. c . trên b` , ,, , ,, ´ , , ˜,ng gi´ goi a ngu`oi tha ˘ng cuôc. oi nhu a tri. n` ao cua n ngu`oi choi thu´ nhâ´t c´ o . l` . V´ , , , , , , , ´ ´ ˜ ng gi´ môt ˘ng cuôc? oi nhu a tri. n` ao cua n ngu`oi choi thu´ hai . chiên thuât . tha . V´

5.1. Chiê´n thuât . bâ´t biê´n

209

c´ o môt ˘´ng cuôc? . chiê´n thuât . tha . , , ,, , , , , `˘ng `,i giai. Ta c´ ´i nh˜u,ng tru,`o,ng ho.,p d Lo o thê thu vo ¯on gian cua n, ta thâ´y ra , , , , ´˘ng, ´ nhâ´t c´ ´ ngu`oi choi thu nê´u n 6≡ 0 (mod k + 1), khi d¯o o chiê´n thuât . tha , , ,, ,`, ´, ´˘ng. Ðê chu ´ ng minh d nguo. c lai o chiê´n thuât ay, ¯iê`u n` . nguoi thu hai s˜e c´ . tha , cho nt l` a sô´ quân c`o trên b` an. ´, nhâ´t c´ ´,i 0 < rt < k + 1, th`ı ngu,`o,i cho,i thu Nê´u nt = (k + 1)qt + rt vo o , , ˜ ´ ` ` ˘ng ba ˘ng c´ ˘ng do rt > 0 nên thê tha ach môi lâ`n lâ´y d¯i rt quân c`o. (Ch´ u ´y ra , , , , , , , , ˜ ´ nhâ´t lâ´y nhu vây ´ hai xuâ´t ph´ ´,i bôi môi lâ`n ngu`oi thu at vo . th`ı ngu`oi thu . cua , , , , , ´, , , k + 1 quân c`o v` a lâ`n n` ao c˜ ung nhu vây hai chı c´ o thê lâ´y d ¯i t`u 1 . ngu`oi thu , d¯ê´n k quân c`o). ,, , ´, nhâ´t c´ Nê´u n = (k + 1)q, th`ı ngu`oi thu o lâ´y bao nhiêu quân c`o d¯i, , ,, , ´˘ng ba `˘ng ´, hai c´ ˘. ng han cha o thê tha . m quân c`o (1 ≤ m ≤ k) th`ı ngu`oi thu , , ˜ lâ`n choi sao cho m + q = k + 1. c´ ach lâ´y q quân c`o môi

J

` tâp Bai . , , , ˜ ngu,`o,i cho,i c´ . 5.5. Trong tr` o choi Bachet, nê´u luât o . choi thay d¯ôi cho môi , ˜ , ` ´ ´ ˘. c 2 hoa ˘. c 4 hoa ˘. c 8 hoa ˘. c 16 hoa ˘. c ... (môt thê môi lân lây sô quân c`o l` a 1 hoa . , , , ´ ´ ˘ng? sô l˜ uy th`ua cua 2), th`ı ai c´ o thê tha , , , , ˜ ngu,`o,i c´ . 5.6. Trong tr` o choi Bachet, nê´u luât o thê . choi thay d¯ôi cho môi , , ˜ ` ´˘ng? ˘. c môt lâ´y d lân, th`ı ai c´ o thê tha ¯i môt . hoa . sô´ nguyên tô´ quân c`o môi ,, , , . 5.7. Khoi d o môt oc trên b` an c`o n × n ô vuông. Hai ¯â`u c´ . quân c`o tai . môt . g´ , ,, , , ngu`oi choi A v` a B thay nhau di chuyên quân c`o sang (môt . ô vuông) bên , , , ,´, ,`, , ´ ˘. c xuông duoi. Nguoi choi không thê di chuyên tr´ ai, bên phai, lên trên hoa , ,, ,, ´,i ô vuông d¯a ˜ d¯i qua rô`i. Ngu`oi thua cuôc to a ngu`oi cuô´i c` ung không thê . l` ,, , ,, ´˘ng khi n l` ˜˘n? Ngu,`o,i n` ´˘ng khi d ao s˜e tha a sô´ cha ao s˜e tha ¯i d¯uo. c n˜ua. Ngu`oi n` , , ,, ´˘ng nê´u quân c`o, kho,i d¯â`u t`u, môt n l` a sô´ le? ngu`oi n` ao s˜e tha . ô vuông bên ,, , canh ô vuông o g´ oc b` an c`o? .

210

,, , ˜,ng tr` Chuong 5. Nhu o choi to´ an hoc .

,, , , , ´, ´,i hai d¯ô´ng quân c`o, gô`m p v` . 5.8. Khoi d¯â`u vo a q quân c`o tuong u ng. A , , , , , ,´, , , , ´c d¯i. Ngu`oi choi thu. c hiên v` a B thay nhau thu. c hiên cd ¯i . nh˜ung buo . buo , , , , ´ v` bao gô`m lâ´y d¯i môt ao d a chia nh˜ung quân c`o cua d¯ô´ng ¯ô´ng quân c`o n` ¯o . d , ` ´ ´ ´ ˘ng nhau). kia th` anh hai d¯ông (không nhât thiêt hai d o sô´ quân c`o ba ¯ô´ng c´ , ,`, ,`, ´ , , , ´ Nguoi thua cuôc a nguoi d¯ên luo. t không c` on d¯ông quân c`o c´ o thê chia . l` , ,, ,, , ´˘ng? (Go.,i ´y: Kê´t qua phu. thuôc ˜˘n d¯uo. c. Ngu`oi n` ao s˜e tha ao nh˜ung t´ınh cha . v` , , le cua p v` a q).

, , , ´ vo ` choi ´ i tro 5.2. Kiê´n th´ uc toan , ,, , ˜, sô´ d V´ı du. 5.9. Hai ngu`oi choi thay phiên nhau viê´t liên tiê´p c´ ac chu ¯ê cuô´i , ,, ˜, ˜, sô´, theo quy d c` ung ta c´ o môt o 10 chu Ngu`oi thu´ nhâ´t viê´t chu ¯inh: . con sô´ c´ . , , , ,, , ,, ˜, sô´ thu´, hai, lai sô´ thu´ nhâ´t, ngu`oi thu´ hai viê´t chu . ngu`oi thu´ nhâ´t viê´t sô´ thu´ , ,, ,, , ba,... Ngu`oi thu´ hai muô´n c´ o môt on ngu`oi thu´ nhâ´t chô´ng . sô´ chia hê´t cho 7, c` , , ,, , , lai ay. Vây a choi nhu thê´ n` ao d ¯ê d ¯at ¯uo. c ´y muô´n cua m`ınh? . ´y muô´n n` . ai v` . d , , `,i giai. Ngu,`o,i thu ´, hai l` Lo a viê´t sô´ cuô´i c` ung quyê´t d o ¯inh . kê´t qua sô´ â´y c´ , , , , , ´ nhâ´t viê´t ch˜u sô´ cuô´i c` chia hê´t cho 7 không. Khi ngu`oi thu ung cua m`ınh, , ,`, ´, ,, , ´, mu,`o,i, ngu,`o,i th`ı nguoi thu hai nhân o 9 ch˜u sô´. Ðê t`ım sô´ thu . d¯uo. c môt . sô´ c´ , , ´, ´, hai thêm sô´ 0 v` ´ (ngh˜ıa l` ´ thu ao sô´ d a nhân sô´ thu d¯uo. c vo i 10), sau d¯o ¯o , , , d o sô´ du l` a r, th`ı sô´ câ`n ¯em chia cho 7. Kê´t qua thu d ¯uo. c sau ph´ep chia c´ , , ,, , , ´ hai l` viê´t thêm v` ao cua ngu`oi thu a 7 − r. Ðây c˜ ung l` a chiê´n thuât . ngu`oi d¯i ´˘ng. sau luôn luôn tha

J

, ,, ˜ cho sa V´ı du. 5.10. Trên bang d ˘˜n phuong tr`ınh ¯a ∗x2 + ∗x + ∗ = 0.

,, , ,, , ,, Ngu`oi thu´ nhâ´t trong hai ngu`oi choi d uy ´y v` a sau ¯uo. c quyê`n chon . ra ba sô´ t` ,, ,`, ´, ´ ` ´ ´ nguoi thu hai d ´d d ac sô d ao c´ ac dâu ∗ trong phuong tr`ınh trên ¯o ¯em c´ ¯o ¯iên v` , ,, ,`, ´, ´ ´ môt ach t` uy ´y. Nguoi thu nhât s˜e tha ˘ng cuôc ¯uo. c mong muô´n . c´ . nê´u thu. c hiên . d

, , ´,c to´ 5.2. Kiê´n thu an vo´i tr` o choi

211

, ,, ,, ˜,u tı phân o hai nghiêm l` a cuô´i c` ung nhân ¯uo. c môt . phuong tr`ınh bâc . hai c´ . hu . d , ,, , ,, , ,, ´ cua ngu`oi thu´ nhâ´t. Ngu`oi biêt. ach chô´ng lai ¯inh ¯o . Ngu`oi thu´ hai t`ım c´ . ´y d . d , , , , , , ,, thu´ nhâ´t s˜e phai chon ac sô´ nhu thê´ n` ao d ¯ê thu. c hiên . c´ . d¯uo. c mong muô´n cua m`ınh? , , , , ´, `,i giai. Goi Lo a c´ ac sô´ m` a ngu`oi thu nhâ´t chon, . a, b, c l` . thê´ th`ı anh ta phai `˘ng 0 th`ı bi. ´,i a, b, c 6= 0, (v`ı nê´u môt chon . sao cho a + b + c = 0 vo . sô´ ba ,`, , , , , ´ hai lâ´y sô´ d¯o ´ nhâ´t v` ´ g´ nguoi thu an v` ao hê. sô´ thu a phuong tr`ınh không ,, , , , , ´ nhâ´t s˜e thua). Nhu, vây, c` on l` a phuong tr`ınh bâc a ngu`oi thu . hai n˜ua v` . , , ´, ,, ˜ nhân ´, dâ´u ∗ n` ngu`oi thu hai d` u c´ o thê´ a, b, c v` ao bâ´t cu ao th`ı ta vân d u . ¯ o. c , , ,, môt a theo hê. qua cua d l´ı Vi´ete th`ı luôn c´ o ¯inh . . phuong tr`ınh bâc . hai m` , c , ´ hai c´ ˘. c nghiêm hai nghiêm o thê l` a . phân biêt . x1 = 1, x2 = a (hoa . thu b c a , , ´, x2 = , x2 = , x2 = ,... phu. thuôc ao ngu`oi thu hai g´ an a, b, c v` ao vi. . v` a b b tr´ı ∗ n` ao).

J

, , V´ı du. 5.11. Trong môt od a trong môt ¯u. ng 15 qua câ`u xanh, v` . chiê´c hôp . c´ . , ,`, ,, , ˜ ´ ` ´ hôp ac c´ o 12 qua câu tra ˘ng. Hai nguoi choi, trong môi luo. t d ¯i, ho. buôc . kh´ . lây , , ,`, ´ ,`, ´ , , ´ ` ` d ˘. c 2 qua câu tra ˘ng. Nguoi tha ˘ng cuôc a nguoi lây d ¯i 3 qua câu xanh hoa ¯uo. c . l` , , , ,`, ´, ´ , ` ´ ´ ˜ nhung qua câu cuôi c` ung. Vây ao . nguoi thu nhât phai choi theo chiên thuât . n` , ´ d ˘ng cuôc? ¯ê tha .

, , `,i giai. Ngu,`o,i thu ´, nhâ´t trong lu,o.,t cho,i d Lo ¯â`u tiên, câ`n lâ´y ra 2 qua câ`u , , , ,´, , ´˘ng t`u, hôp ´, hai, v` tra a sau buo c n` ay th`ı tı lê. gi˜ua qua câ`u xanh v` a qua . thu ´˘ng l` câ`u tra a 15 : 10 = 3 : 2. , ,, ´, hai lâ´y 3 qua câ`u xanh t`u, hôp ´, nhâ´t hoa ˘. c 2 Nê´u l´ uc â´y, ngu`oi thu . thu , , ,, , , , , , , , , , , ´ ´ hai th`ı ngu`oi thu ´ nhâ´t phai lâ´y nguo. c lai ´c ˘ng o hôp qua mâ`u tra . thu . o buo , , ,, ,, , ´ ´ hai hoa ´, ˘ng o hôp ˘. c 3 qua câ`u xanh o hôp tiê´p theo, 2 qua câ`u tra . thu . thu , , , , , , , nhâ´t d ¯ê d¯am bao su. bâ´t biê´n cua tı lê. 3 : 2 d ¯ê´n cuô´i cuôc . choi.

J

,, , , V´ı du. 5.12. Hai muoi cô g´ ai ngô`i quanh môt an v` a choi môt o choi . chiê´c b` . tr`

212

,, , ˜,ng tr` Chuong 5. Nhu o choi to´ an hoc .

, ,, , ac quân b` ai. Trong môt ai. Khoi d v´ oi n quân b` ¯â`u, môt . cô câ`m tâ´t ca c´ . lâ`n , ´ ´ c´ choi, nêu l´ uc d o ´ıt nhâ´t môt ai câ`m ´ıt nhâ´t hai quân b` ai, môt ¯o . cô g´ . trong , , , , , ˜ ˜ ng cô g´ nhu ai n` ay phai chuyên môt ai cho môi ngu`oi ngô`i bên canh . . quân b` , , ´ ˜ ` ` ´ m`ınh. Tr` o choi kêt th´ uc khi v` a chı khi môi cô g´ ai câm nhiêu nhât môt . quân b` ai. , , , , ` ng. a) Chu´ng minh ra ˘`ng nê´u n ≥ 20 th`ı tr` o choi không thê du , , , b) Chu´ng minh ra ˘`ng nê´u n < 20 th`ı tr` o choi phai c´ o kê´t th´ uc. , `,i giai. a) Nê´u n > 20 th`ı theo nguyên l´ı Dirichlet1 , tô`n tai Lo . ´ıt nhâ´t môt . cô , , , g´ ai câ`m ´ıt nhâ´t hai quân b` ai tai o choi không kê´t th´ uc. . moi . th`oi d¯iêm. Vây . tr` Nê´u n = 20, th`ı g´ an nh˜ an cho c´ ac cô g´ ai l` a G1 , G2 , ..., G20 theo chiê`u , , a G1 l` a cô g´ ai d ac quân b` ai. Ðinh kim d¯ô`ng hô` v` ¯â`u tiên gi˜u tâ´t ca c´ . ngh˜ıa , , ` ˘m trong tay cô g´ gi´ a tri. hiên ai l` a i nê´u n´ o na ai Gi (gi´ a tri. . th`oi cua quân b` , , , , ` ´ tu. ta g´ ˘ng ch´ınh sô´ thu ˘. t S tông gi´ quân b` ai ba an cho c´ ac cô g´ ai). Ða a tri. cua , ,, ,, , nh˜ung quân b` ai. Khoi d ai, ¯â`u S = 20 (v`ı G1 khoi d¯â`u câ`m tâ´t ca 20 quân b` ˜ ` ˘ng 1). môi quân b` ai d¯ê`u c´ o g´ ai tri. ba , ,´, ˜ ngu,`o,i ngô`i bên canh. Ta x´et truoc v` a sau Gi chuyên quân b` ai cho môi . , , , ´ Nêu i = 1 th`ı S t˘ ang lên −1 − 1 + 2 + 20 = 20 (v`ı G1 chı c´ o thê chuyên cho , , ´, ´,i sô´ d¯a ´ gi´ ˜ G2 v` a G20 m` a thôi, khi d a tri. quân b` ai t˘ ang lên tuong ung vo ¯o , ´ g´ an cho c´ ac cô g´ ai v` a mât d¯i gi´ a tri. cua G1 , cô g´ ai c´ o quân b` ai c´ o gi´ a tri. l` a , ´ 1). Nêu 1 < i < 20 th`ı S không thay d¯ôi g`ı, v`ı −i−i+(i−1)+(i+1) = 0 (cô , ˜ chuyên quân b` g´ ai Gi d ai c´ o gi´ a tri. i th` anh i − 1 v` a i + 1 cho hai cô ngô`i ¯a , canh a Gi+1 ). Nê´u i = 20 th`ı S giam d ¯i 20 v`ı −20−20+1+19 = −20 . Gi−1 v` , ,, , , , ,´, ` (tuong tu. nhu phân trên). Nhu vây a sau khi chuyên quân b` ai, . truoc khi v` , , ,, , ˜ ´ ´ môi cô g´ S luôn luôn l` a bôi o choi c´ o kêt th´ uc. Khi d¯o ai . cua 20. Gia su tr` , ` ´ câm môt ai v` a S = 1 + 2 + · · · + 20 = 210, sô n` ay không phai l` a sô´ . quân b` , , , , , , bôi o choi không thê kê´t th´ uc d¯uo. c. . cua 20, vô l´ı. Nhu vây . tr` 1 Johann

, Peter Gustav Lejeune Dirichlet (1805-1859): Nh` a to´ an hoc uc. . д

, , ´,c to´ 5.2. Kiê´n thu an vo´i tr` o choi

213

, , , , ,, , `i hoi hai b) Ðê thâ´y d¯uo. c tr` o choi phai kê´t th´ uc nê´u n < 20, ta phai d¯o , ´nh dâ´u lai cô g´ ai d¯a ai khi lâ`n d¯â`u tiên môt ai chuyên . quân b` . trong hai cô g´ , ,, ,, quân b` ai cho ngu`oi kia. Khi m` a môt ai chuyên môt ai cho ngu`oi . cô g´ . quân b` , , , `i hoi cô g´ ˜ d¯a ´nh dâ´u gi˜ua hai ngô`i canh m`ınh, d¯o ai phai d` ung quân b` ai d¯a . ,, ˜ c´ ngu`oi nê´u d o (v`ı theo d¯iê`u kiên ai to´ an khi c´ o hai quân b` ai trong tay ¯a . b` , , , , , ´˘c lai ´i d ´ mo ˜d ´nh dâ´u s˜e ta th`ı cô g´ ai d¯o ai d¯a ¯uo. c chuyên). Nhu vây ¯a . quân b` . , ´ ´ ` ˘. p hai cô g´ ˘. p canh ˜d ´nh dâu n´ gi˜ua ca ai d¯a o. Nêu n < 20, s˜e tôn tai ¯a . môt . ca . , , ´ ´nh dâu, do d ´ không bao gi`o nhau m` a hai cô n` ay không c´ o quân b` ai d¯ê d¯a ¯o , trao d¯ôi môt ai n` ao cho nhau. . quân b` , , ,, , ´ ´, ˜ cô g´ Gia su tr` o choi tiên toi vô han ai chuyên . lâ`n, ta x´et sô´ lâ`n môi , , , , nh˜ung quân b` ai. V`ı tr` o choi không thê d`ung, nên không c´ o môt ai n` ao . cô g´ , , , , ´ ´ ` ` ˜ ´ ˘ ˘ chuyên quân b` ai môt sô h u u h an lâ n. Nhu ng ta x´ e t b a t d â u v o i c a p ¯ . . . cô , , , ˜ cô g´ g´ ai m` a không c´ o su. chuyên d¯ôi n` ao (môi ai n` ay c´ o nhiê`u nhâ´t môt . , , , , , , , quân b` ai, v`ı nê´u hon th`ı phai chuyên d¯uo. c sang hai ngu`oi bên canh) v` a di . , , , ` ˜ chuyên theo chiê`u kim d¯ô`ng hô` môt cô g´ a i m ôt lâ n (c´ o thê nh u ng cô g´ ai . . , , , ˘. p Gi v` n` ay chı chuyên h˜uu han ung tô`n tai a Gi+1 sao . lâ`n), cuô´i c` . môt . ca , , , cho Gi chuyên quân b` ai h˜uu han a Gi+1 chuyên quân b` ai vô han . lâ`n v` . lâ`n , , , , ˘. p n` (nê´u không c´ o ca ao nhu vây ac cô g´ ai chı chuyên h˜uu han a . th`ı c´ . lâ`n v` , , , ´ , tr` o choi kêt th´ uc). Ðiê`u n` ay r˜ o r` ang không d ung s˜e d`ung ¯uo. c v`ı Gi cuô´i c` , , , , chuyên quân b` ai v` a c˜ ung d`ung nhân ai t`u Gi+1 . . nh˜ung quân b`

J

` tâp Bai . , , ´˘t d ´,i môt . 5.13. Ba a B thay nhau thu. c hiên ¯â`u vo . d¯ô´ng n quân c`o. A v` . , ,´, ,´, ` , , ` nh˜ung buo c d¯i. Buo c d¯âu tiên, A lâ´y ´ıt nhâ´t môt quân c o , nhu ng không . , ,`, , , , ´y bâ´t k`ı sô´ quân c`o,, m` ´, môt hê´t quân c`o. T`u khi d ngu o i cho i c´ o thê lâ a ¯o . , , , , , , , , , , , , ´ ´c sô´ cua sô´ quân c`o d¯a ´c truo ´c d¯o ˘ng ˜ lâ´y o buo ´. Ngu`oi tha sô´ n` ay l` a môt . uo , ,`, ,, ,´, , , ´ ˘ng cuôc? cuôc a nguoi c´ o thê d ung. Ai s˜e l` a ngu`oi tha ¯i d ¯uo. c buoc cuô´i c` . l` . (Phu. thuôc v` a o n.) .

214

,, , ˜,ng tr` Chuong 5. Nhu o choi to´ an hoc .

, , ,, , . 5.14. A v` a B thay nhau tô mâ`u t`ung ô vuông cua b` an c`o 4 × 4 ô. Ngu`oi , ,, ,, thua cuôc a ngu`oi tô mâ`u ho` an chınh h`ınh vuông 2 × 2 ô vuông. Ngu`oi . l` , , ´˘ng? B c´ n` ao c´ o kha n˘ ang tha o thê c´ o môt oa không. . chiê´n thuât . h` , . 5.15. A v` a B thay phiên nhau d a trên b` an c`o 1994 × 1994 ô ¯i quân m˜ , , ,´, vuông. A chı thu. c hiên cd ¯i theo chiê`u ngang (x, y) → (x ± 2, y ± 1), B . buo , ´˘t d¯â`u chon chı d¯i theo chiê`u doc . (x, y) → (x ± 1, y ± 2). A ba . môt . ô vuông , , , , , ´c d¯i. Nh˜ung ô vuông quân m˜ ˜ d¯i qua th`ı trên b` an c`o v` a thu. c hiên a d¯a . buo , ,, ,`, ,`, ,, , a nguoi không thê d¯i d¯uo. c n˜ua. không d¯uo. c ph´ep d¯i lai. . Nguoi thua cuôc . l` `˘ng A c´ ´˘ng. ´,ng minh ra Chu o môt . chiê´n thuât . tha

, , ` toan ´ thi hoc 5.3. Nh˜ung bai sinh gi oi . , ,, `˘ng râ´t nhiê`u b` Ta thâ´y ra ai to´ an d¯uo. c ra trong c´ ac k`ı thi hoc . sinh gioi tai . , , ,´, ,, , , , ´ ´ môt ai to´ an d ac phuong ¯iên h`ınh nhât. C´ . sô nuoc, o d¯ây chı liêt . kê nh˜ung b` , , , ´ ph´ ap giai c˜ ung rât phong ph´ u, ban ay tham khao v` a t`ım ra c´ ach giai . d¯oc . h˜ , cua m`ınh. ,, V´ı du. 5.16 (Liên Xô 1984). Cho môt a hai . khung h`ınh khô´i lâp . phuong v` , ,`, , , , loai au d a m` au xanh, hai nguoi quyê´t d o choi nhu sau: ¯o v` ¯inh . son m` . môt . tr` , , , ,`, ´, ´ , Nguoi thu nhât chon a tô ch´ ung ba ˘`ng m` au d ¯o. . ba canh . cua khô´i lâp . phuong v` , ,, , ˜,ng canh ´ ngu`oi thu´ hai chon Sau d on lai ¯o . ba canh . trong sô´ nhu . c` . cua khô´i lâp . ,, ,, ,`, ´, ´ ´ o t ngu o i th u nhâ t ch on ba c anh phuong v` a tô ch´ ung m` au xanh. Lai d ê n lu ¯ . . . . , , ,`, ´, ´ ´ trong sô´ c` on lai tô m` a u d o. Cuô i c` u ng ngu o i th u hai tô nô t m` a u xanh c ua ba ¯ . ,`, ´ ,`, ` , , canh c` on lai. ˘ng cuôc a nguoi ba ˘ng m` au son cua m`ınh tô tron . . Nguoi tha . l` . ven . , , , , , , , ´ ´ ´ ´n ´ ` 4 canh c ua m ôt m a ˘ t khô i l âp phu o ng. Ngu o i th u nhâ t s˜ e th a ˘ ng v´ o i m ôt chiê . . . . . , ´ng d thuât ˘´n trong cuôc ¯u ¯a . d . choi không? , ,, , , `˘ng vo `,i giai. Ta s˜e chu ´,ng minh ra ´,i môt Lo . chiê´n thuât . th´ıch ho. p ngu`oi choi , , ´˘ng d¯u,o.,c. Ta thâ´y ra `˘ng d¯ê không bi. thua, ngu,`o,i cho,i ´, nhâ´t không thê tha thu

, ˜,ng b` 5.3. Nhu ai to´ an thi hoc . sinh gioi

215

, ´˘t ch´eo nhau (không giao nhau) l` `˘ng c´ ´, hai tô ba canh thu va a d¯u, v`ı ba ach . ,`, ´, ,, , ,`, ´, ˜ ˜ ˘. t. Nhu vây, n` ay nguoi thu hai tô d ¯uo. c môi canh . trên môi ma . nguoi thu nhâ´t , , , , , , , ´˘t ch´eo ´ hai tô ba canh ´c tô d¯â`u tiên phai ng˘ ˘. n ngu`oi thu ngay t`u buo an cha . va , , , ´, , `˘ng d ´,ng minh ra nhau. Ta s˜e chu ay ngu`oi thu nhâ´t không thê thu. c hiên ¯iê`u n` . , , ,, , ´ ´ ˘t ch´eo c` d¯uo. c. Thât va ung nhau. Tông . vây, . ta x´et tât ca bô. ba nh˜ung canh . , , ˜ ´ ´ tham gia trong hai bô. ba nhu sô l` a 8 bô. ba nhu vây, trong d¯o . v`ı môi canh . , ,, ´, nhâ´t c´ vây o). Môt ngu`oi thu o thê tô ¯oc . (ban . d . v˜e h`ınh s˜e r˜ . lâ`n tô 3 canh . , , , , ` ` ´ ´, hai môt nhiêu nhât l` a trong 6 bô. . phân nh˜ung canh . . Suy ra ngu`oi choi thu , , ´˘t ch´eo c` c´ o thê tô môt ac canh va ung nhau c` on lai a . trong nh˜ung bô. ba c´ . . v` , , ´ nhu vây oa. . kêt qua s˜e h`

J

,, ´,c 1984). Trên môt V´ı du. 5.17 (Ðu ang d ¯uo. c viê´t liên tiê´p 2n k´ı hiêu . h` . x canh . ,`, , ` nhau. Hai nguoi choi thay phiên nhau thay môt ˘ng môt . k´ı hiêu . x ba . trong , , , , , , , ´ ´ ˜ ng chu ˜ sô 1, 2, 3, 4, 5, 6. Ngu`oi choi thu´ hai tha nhu ˘ng khi v` a chı khi sô´ nhân . ,, ´ ` , ´ ´ ´ ˜ sô (trong hê. sô thâp d ay t`ım tâ´t ¯uo. c sô gôm 2n-chu . phân) chia hêt cho 9. H˜ , , , , , , , ˜ ng gi´ o chiê´n thuât ˘´ng. ca nhu a tri. cua n sao cho ngu`oi choi thu´ hai c´ . tha , `˘ng ngu,`o,i cho,i thu `,i giai. Ta s˜e chu ´,ng minh ra ´, nhâ´t (ngu,`o,i cho,i d Lo ¯â`u tiên) , , ´˘ng vo ´, ´i n 6≡ 0 (mod 9), c` ´i n ≡ 0 (mod 9) ngu,`o,i thu c´ o chiê´n thuât on vo . tha

´˘ng. hai c´ o chiê´n thuât . tha ,, ,, , , ´, nhâ´t chon Cho n ≡ i (mod 9), o d¯ây 0 < i ≤ 8 ngu`oi choi thu . ch˜u sô´ , , d¯â`u tiên x0 cua m`ınh theo bang sau: i x0

1 1

2 2

3 4

4 6

5 1

6 1

7 3

8 5

,, ,, , , , , ´, ´ ngu`oi n` c` on sau d ay luôn chon a ch˜u sô´ ngu`oi thu ¯o . ch˜u sô´ 7 − y, o d¯ây y l` , ,´, ,´, ,, , , , ´, , ´. Trong tru`ong ho. p nhu vây, hai v`ua chon c truo c d¯o . buo . sau khi ngu`oi thu , , ,´, , , ´ tông S hai d¯i buo c cuô´i c` ung cua m`ınh v´ı du. nhu chon . ch˜u sô´ x1 , khi d¯o , , , , cua nh˜ung ch˜u sô´ trong sô´ v`ua tao a . ra l` S = x0 + 7(n − 1) + x1 ≡ x0 + 7(i − 1) + x1

(mod 9).

216

,, , ˜,ng tr` Chuong 5. Nhu o choi to´ an hoc .

, , , `˘ng c´ Ba ach kiêm tra tru. c tiê´p d o gi´ a tri. cho ph´ep ¯ua d¯ê´n kê´t luân . không c´ , n` ao cua x1 sao cho x0 + 7(i − 1) + x1 chia hê´t cho 9. ,, ´˘ng, nê´u ngu,`o,i n` ´, hai s˜e tha ˘. t Nê´u n ≡ 0 (mod 9), ngu`oi thu ay luôn d ¯a ,, , ´ , ´ ,`, ´, ,´, , ´ ˜ ˜ ` ˜ chon chu sô 7 − y, o d a chu sô nguoi thu nhât d ¯ây y l` ¯a ¯i vua . trong buoc d , , , ,, , , ´ d¯ê´n hê´t cuôc xong. Khi d a ¯o . choi tông nh˜ung ch˜u sô´ cua sô´ nhân . d¯uo. c l` S = 7n ≡ 0 (mod 9).

J

, ˜, nhât ˜,ng ô vuông V´ı du. 5.18 (Ai-len 1996). Trên môt an c`o h`ınh chu o nhu . b` . c´ , ,, ,, ,, , , , ` ˘. t k´ıch thuo´c 5 × 9, tr` o choi sau d ¯˜ıa d¯uo. c d ¯a ¯ây d ¯uo. c choi: Khoi d ¯âu, môt . sô´ d , , ˜ ´ ` ˜ ng ô vuông, không c´ ngâu nhiên trên môt o ô vuông n` ao chu´a nhiêu . sô nhu , , , , ` ` ´ ´ ˜,ng luât hon môt ac d ¯˜ıa. Môt ¯˜ıa theo nhu . d . lân choi bao gôm lây d¯i tât ca c´ . choi sau d ¯ây: , , , ,, ˜ d i) Môi o thê chuyên t´ oi môt ¯˜ıa c´ . h`ınh vuông ph´ıa trên, ph´ıa duo´i, ph´ıa , tr´ ai hoa ˘. c ph´ıa phai; , ,, , ˜ chuyên lên hoa o ii) Nê´u môt ˘. c xuô´ng duo´i môt ¯˜ıa d ¯a . lâ`n choi rô`i, th`ı n´ . d , , , , , phai chuyên vê` bên tr´ ai hoa ˘. c bên phai, v` a nguo. c lai; . , , , , iii) Tai uc lâ`n choi, không c´ o ô vuông n` ao chu´a hai d ¯˜ıa . th`oi d¯iêm kê´t th´ , hoa ˘. c nhiê`u hon thê´. , , , , Tr` o choi kê´t th´ uc nê´u n´ o không c´ o kha n˘ ang thu. c hiên ¯â`y d ¯u môt . d . lâ`n ,, , , , , ´ ˘. t trên b` an, tr` o choi tiêp. Chu´ng minh ra ˘`ng nê´u khoi d ˘`ng 33 d ¯â`u ba ¯˜ıa d¯uo. c d¯a , ,, , , ` ´ ´ ` choi cuôi c` ung phai kêt th´ uc. Chu´ng minh ra ˘ng khoi d ˘. t 32 d ¯âu ta d ¯a ¯˜ıa trên , b` an th`ı tr` o choi s˜e k´eo d` ai vô tân. . , , ,, ,´, , `,i giai. Nê´u 32 d ˘. t trên b` Lo an g´ oc duo i ph´ıa phai chiê´m h`ınh ch˜u ¯˜ıa d¯uo. c d ¯a , , , ,´, nhât c 4 × 8; ch´ ung c´ o thê tâ´t ca di chuyên lên trên, sang bên tr´ ai, . k´ıch thuo , , , ,´, ` ´ ˘. p d¯i la ˘. p lai. ˘ng tr` xuông duoi v` a sang bên phai môt ach la o . c´ . Ðê chı ra ra , , , ´, ´ ´nh nh˜ choi voi 33 d ung phai d`ung, ta d¯a an cho c´ ac ô vuông trên ¯˜ıa cuôi c` , b` an nhu sau:

, ˜,ng b` 5.3. Nhu ai to´ an thi hoc . sinh gioi 1 2 1 2 1

2 3 2 3 2

1 2 1 2 1

2 3 2 3 2

1 2 1 2 1

217 2 3 2 3 2

1 2 1 2 1

2 3 2 3 2

1 2 1 2 1

, `˘ng c´ ˜ chuyên d¯i, môt Ch´ u ´y ra o t´ am sô´ 3. Môt ¯a . d¯˜ıa tai . 1 d¯i d¯ê´n 3 sau khi 2 d . , , , ´ ´ ´ ´ ˘. c 3 tru. c tiêp v` d¯˜ıa tai a môt ¯i d¯ên 1 hoa ¯˜ıa tai ¯ên 2 tru. c tiêp. Nhu . 2d . d . 3 d¯i d , ,, , , vây a k > 8, th`ı tr` o choi d`ung lai ¯˜ıa khoi d¯â`u trên 1 v` ¯u . nê´u k d . v`ı không d , , , ˜ 3 cho nh˜u,ng d `˘ng sô´ k ≤ 8, ´i. Nhu, vây chô ay chuyên to ¯˜ıa n` . ta gia thiê´t ra ,, ,`, , ˘. c 3 khi khoi d¯â`u (do trong truong ho. p n` ay c´ o nhiê`u nhâ´t 16 d ¯˜ıa trên 1 hoa , , c´ o 8 sô´ 3 v` a k ≤ 8 l` a sô´ d¯˜ıa nhiê`u nhâ´t c´ o thê c´ o trên 1) v` a nhu vây o ´ıt . c´ , ,, , ´ ` ´ ´ ` nhât 17 d¯˜ıa trên 2. Tu 17 d¯˜ıa n` ay, nhiêu nhât 8 d¯˜ıa d¯uo. c chuyên d¯ên 3 sau , , , ` môt o ´ıt nhâ´t 9 d¯˜ıa cuô´i c` ung d¯ê´n 1. Nh˜ung d¯˜ıa n` ay . lân chuyên, nhu vây . c´ , , , , , ´ ´ ´ ´. Nhu vây không thê tât ca chuyên d o choi s˜e kêt th´ uc. ¯ên 3 sau d¯o . tr`

J

,, , , an c`o rông V´ı du. 5.19 (Israel 1995). Hai ngu`oi choi trên môt . b` . vô han . bao ,`, , ,´, , ´, ´ ` ˜ gôm nhung ô vuông k´ıch thuoc 1 × 1. Nguoi choi thu nhât chon . môt . ô vuông ,`, , ´, ` ´ ´nh dâu n´ ´ nguoi choi thu hai chon v` ad o ba ˘ng O. Khi d ac v` a ¯a ¯o . môt . ô vuông kh´ , ,`, , , , ` ´ ´ ˜ ´nh dâu ba ´nh d¯a ˘ng X. Ho. choi nhu vây ¯ên khi môt ¯a . d . trong nhung nguoi choi d ,`, ,, , ,, ` o c m ôt h` a ng ho a ˘ c m ôt c ôt 5 ô vuông liê n nhau v` a ngu o i l` a m d u o c nhu dâ´u d u ¯ . ¯ . . . . . , ,, ,, ,, , vây ˘´ng. Nê´u không ngu`oi choi n` ao c´ od ¯uo. c kê´t qua mong muô´n . truo´c s˜e tha , , ,, , , ,, , a h` oa. Chu´ng minh ra ˘`ng ngu`oi choi thu´ hai c´ o thê ng˘ an th`ı tr` o choi d ¯uo. c coi l` ,, ,`, , ´, ´ ´ cha ˘. n d ˘ng cuôc. ¯uo. c nguoi choi thu nhât tha . , , `,i giai. Ta g´ Lo an nh˜ an cho c´ ac ô vuông nhu h`ınh 5.1. ˜ sô´ d¯ê`u c´ ˘. p. Sô´ 1 v` ˘. p theo chiê`u doc, Ch´ u ´y môi o môt a sô´ 2 c´ o ca on . ca . c` ,`, , , ˜ ´ ´ ` ´ ´ nhât d ˘. p theo chiêu ngang. Môi khi nguoi choi thu ´nh sô 3 v` a sô 4 c´ o ca ¯a , ,`, , ´, ´ ´ ´ ˘. p. V`ı bât k`ı ´nh dâu ô vuông kia cua ca dâu ô vuông, nguoi choi thu hai s˜e d¯a , , , , ´ ´ a môt ˘. c theo h` ˘. p cua c` 5 ô liên tiêp theo côt ang phai chu ung nh˜ung . hoa . ca , ,, , ´, , ´ ˘ng. sô´ n` ay, nhu vây nhâ´t không bao gi`o tha . ngu`oi choi thu

J

,, , ˜,ng tr` Chuong 5. Nhu o choi to´ an hoc .

218

··· ··· ··· ··· ··· ··· ··· ···

.. . 1 1 3 4 1 1 3 4 .. .

.. . 2 2 3 4 2 2 3 4 .. .

.. . 3 4 1 1 3 4 1 1 .. .

.. . 3 4 2 2 3 4 2 2 .. .

.. . 1 1 3 4 1 1 3 4 .. .

.. . 2 2 3 4 2 2 3 4 .. .

.. . 3 4 1 1 3 4 1 1 .. .

.. . 3 4 2 2 3 4 2 2 .. .

··· ··· ··· ··· ··· ··· ··· ···

H`ınh 5.1

, ˜,ng ô vuông k´ıch thu,o´,c 1×1. V´ı du. 5.20 (IMO 1993). Trên b` an c`o vô han . nhu , , ,, , o choi nhu sau: Ðâ`u tiên xê´p n2 quân v` ao môt Ngu`oi ta thu. c hiên . môt . tr` . h`ınh , ˜ ô vuông cua h`ınh vuông gô`m n × n ô vuông canh . liê`n canh . sao cho trong môi , , , , , ,, vuông chu´a môt ach d o choi l` a quân c`o chı d ¯i trong trong tr` ¯uo. c . quân c`o. C´ , ,, , , nhay theo h` ang hoa ˘. c côt o chu´a quân c`o o ngay s´ at bên canh, . qua môt . ô c´ . , ,, , ´ ´ ´ ´ sang môt ô trô ng tiê p ngay sau d o . Khi d o quân c` o o ô b i nh ay qua s˜ e b i ¯ ¯ . . . loai . , bo. , , , , , , T`ım c´ ac gi´ a tri. cua n d o thê kê´t th´ uc tr` o choi sao cho trên b` an c`o chı ¯ê c´ , ´ng môt c` on lai . d¯u . quân c`o. , , , , ,, `,i giai. Ta x´et nh˜u,ng quân c`o, d ˘. t tai Lo o toa. d¯ô. ¯iêm luo´i (d¯iêm c´ ¯a . nh˜ung d , 2 ´i k = 0, 1, 2, d ˘. t Ck = {(x, y) ∈ Z : nguyên): Z = {(x, y) : x, y ∈ Z}. Vo ¯a , ,´, , ´,i ˘. t ak l` x + y ≡ k (mod 3)}. Ða a sô´ quân c`o trên d¯iêm luo i thuôc ao Ck vo . v`

k = 0, 1, 2. , , ,´, , , Buo cd ang l` a lâ´y môt ¯i theo h` . quân c`o tai . d¯iêm (x, y) nhay qua quân c`o , , , , ˜ ak ´i d tai ¯iêm trô´ng (x ± 2, y). Sau khi di chuyên, môi . (x ± 1, y) chuyên to , , , , ˜˘n le. ˘. c giam d t˘ ang lên hoa ¯i 1. Nhu vây ¯ôi t´ınh cha . ak thay d

, ˜,ng b` 5.3. Nhu ai to´ an thi hoc . sinh gioi

219

n2 ´˘t d ´ a0 = a1 = a2 = ngay khi ba Nê´u n chia hê´t cho 3, khi d¯o ¯â`u 3 , , , , , ˜˘n le n` ´ tai choi. Do d¯o o c` ung t´ınh cha ay. Nhu vây ¯iêm, ak d¯ê`u c´ . moi . th`oi d . , , , ´ ´ tr` o choi không thê kêt th´ uc voi môt on lai o hai ak trên toa. . quân c` . v`ı s˜e c´ , ,´, ˜ ˘n le). d¯ô. luoi 0 v` a c´ ai kia l` a 1 (không c` ung t´ınh cha , , ´,i môt Nê´u n không chia hê´t cho 3, th`ı tr` o choi c´ o thê kê´t th´ uc vo . quân , , `˘ng quy nap. ´ ng minh ba ´i n = 1 hoa ˘. c 2, d c` on lai. ay d˜ê ¯iê`u n` . Ta s˜e chu . Vo , , , , , , , ´i n ≥ 4 ta xây du. ng phuong ph´ thâ´y. Vo ap d ¯ê giam n × n quân c`o xuô´ng , (n − 3) × (n − 3) quân c`o. , , ,´, , Ta x´et nh˜ung quân c`o tai cd ¯i . vi. tr´ı (0, 0), (0, 1), (0, 2), (1, 0). Nh˜ung buo , ,, , (1, 0) → (−1, 0), (0, 2) → (0, 0), (−1, 0) → (1, 0), bo d¯i d ¯uo. c ba quân c`o , , , ´, , ` liên tiê´p trong môt a tra lai tu vê vi. tr´ı ban d o thê ¯â`u. Ta c´ . côt . v` . quân c`o thu , , , , ˘. p lai ´p dung ´y bên tr´ a ay la ai 3 × (n − 3) quân c`o cua n × n . thu thuât . n` . t`u d¯a , , , , ,´, , ´ t`u d¯a ´y duo ô vuông t`u tr´ ai qua phai, sau d i bên phai (n − 3) × 3 quân c`o ¯o , , , , , ,´, , , , ´y duo t`u d i lên d¯ınh v` a cuô´i c` ung t`u d¯ınh phai 3 × 3 quân c`o t`u phai sang ¯a , , ´,ng minh. tr´ ai. Nhu vây on lai . s˜e c` . (n − 3) × (n − 3) quân c`o ta câ`n chu

J

, , , V´ı du. 5.21 (Anh 2000). Alice choi môt o choi môt an c`o 20×20. . tr` . m`ınh trên b` , ,, , ˜ ô môt Khoi d an c`o trong môi ac dang ¯â`u Alice trai trên b` ¯ô`ng xu gô`m c´ . d . sau d a 100 quarter. Alice chon ¯ây: 100 penny, 100 nickel, 100 dime, v` . 59 d¯ô`ng xu , , ˜ ´ môi lâ`n Alice lâ´y 1 d¯ô`ng xu theo nguyên bâ´t k`ı v` a lâ´y ra khoi b` an c`o. Sau d ¯o ta ˘´c sau d ¯ây: , ,, • Môt o thê lâ´y d o bô´n h`ınh vuông bên canh ¯ô`ng peny c´ ¯i d ¯uo. c nê´u c´ . d . , , ,, ,´, , ,, , ˜ ng ô o ngo` (trên, duoi, phai, tr´ ai) bo trô´ng. Nhu ai b` an c`o không d ¯uo. c , , ˜,ng ô o, g´ t´ınh l` a bô´n ô trô´ng theo quy ta ˘´c n` ay. V´ı du, oc b` an c`o hoa ˘. c . nhu ,, , , ´ ˜ ng ô n` o bên canh b` an c`o, nhu ay thâm o ba ô bên canh trông th`ı . . tr´ı c´ . ,, ´ ` d¯ông xu tai ay c˜ ung không d ˘c n` ay. ¯uo. c t´ınh theo quy ta . ô n` , ` ´ ´ ` ´ • Môt o thê lây d ¯i nêu tôn tai . d¯ông Nikel c´ . ´ıt nhât ba ô trô´ng bên canh . , , ´ ˜ ng ô ngo` (nhu ai b` an c`o không t´ınh l` a ô trông).

,, , ˜,ng tr` Chuong 5. Nhu o choi to´ an hoc .

220

, , o ´ıt nhâ´t hai ô bên canh trô´ng • Môt o thê lâ´y d ¯i chı khi c´ ¯ô`ng Dime c´ . . d ,, , , ˜ ng ô o ngo` (nhu ai b` an c`o c˜ ung không t´ınh l` a ô trô´ng). , , • Môt o thê lâ´y d o ´ıt nhâ´t môt ¯ô`ng quarter c´ ¯i chı khi c´ . d . ô trô´ng bên canh . ,, , , ´ ˜ ng ô o ngo` (nhu ai b` an c`o c˜ ung không t´ınh l` a ô trông). , , , , , , ˜ ng d Alice tha ˘´ng nê´u Alice lâ´y d an c`o. Chu´ng minh ¯uo. c tâ´t ca nhu ¯ô`ng xu trên b` , , ra ˘`ng không c´ o kha n˘ ang tha ˘´ng cua Alice. , , ,, , `˘ng nhau. Ta x´et tai `,i giai. Phu,o,ng ph´ Lo ap 1: Gia su nh˜ung ô vuông ba . môt . , , , , , , ´ th`oi d¯iêm cua cuôc ung nh˜ung ô vuông trông (v` ung n` ay . choi, chu vi cua v` , , ,, , , ` ˘. c không liên thông). V´ı du, ˘ng tai c´ o thê liên thông hoa ¯iêm . gia su ra . th`oi d , ´˘n vo ´,i nhau tao ´ 64 ô vuông l` n` ao d¯o a trô´ng. Nê´u nh˜ung ô vuông trô´ng ga . ra ` ´ ˘ng 4.8 = 32. Ma ˘. t kh´ 8 × 8 ô vuông, chu vi s˜e ba ac, nêu không c´ o môt ao . ô n` ˜ ` ˘ng 4.64 = 256 (môi ô trong 64 ô vuông l` a bên canh nhau, th`ı chu vi s˜e ba . vuông c´ o chu vi 4). X

X

X

X

X

X

X

X

X

X

X

X

P

X

X

X

X

X

X

X X

X

X

X

X

X

X

X

X

X

X

X

X

X

X

X

X

H`ınh 5.2 , , ,, , , , Tai o choi c´ o k = 59 ô vuông trô´ng. Tai ¯iêm khoi d¯â`u tr` ¯iêm . th`oi d . th`oi d ,, , n` ay chu vi tô´i d¯a l` a 4k. Bây gi`o ta x´et c´ ac d ac nhau d ¯ô`ng xu kh´ ¯uo. c lâ´y d ¯i. , , ´,i c´ Ðê môt o phai c´ o xung quanh 4 canh vo ac ô ¯i, n´ . d¯ô`ng penny lâ´y d . , , , ` ˜ ´ vuông trông. So d n` ay: Nh˜ung ô x´ am l` a c´ ac ô ¯ô sau d ¯ây diên ta t`ınh trang . ,, , , ´ ´ ´ ` ´ trông, nh˜ung ô vuông c´ o dâu X bi. chiêm boi nh˜ung d on ô ¯ông xu bât k`ı, c`

, ˜,ng b` 5.3. Nhu ai to´ an thi hoc . sinh gioi

221

´,a d¯ô`ng penny, d¯u,`o,ng v˜e d¯âm c´ o dâ´u P l` a chu a chu vi. . l` , ´,a d¯ô`ng penny Sau khi lâ´y d ô vuông chu ¯ô`ng penny d¯i rô`i, nh˜ung canh . ,, , , ˜ lâ`n lâ´y d¯u,o.,c d không t´ınh n˜ua. Nhu vây ¯ô`ng penny d¯i th`ı chu vi d ¯uo. c . môi , giam d¯i 4. , ,, ,, Nê´u môt o kha n˘ ang n´ o d¯uo. c bao quanh boi 4 ô ¯i, c´ . d¯ô`ng nickel lâ´y d , , ,, , , vuông bên canh . ô trô´ng nhu trên, trong tru`ong ho. p nhu vây . chu vi s˜e giam , , , d¯i 4. Tuy nhiên d¯ô`ng nickel c´ o thê chı c´ o ba ô trô´ng bên canh nhu h`ınh . sau: X

X

X

X

X

X

X

X

X

X

X

X

X

X

X

X

X

N

X

X

X

X

X

X

X

X

X

X

X

X

X

X

X

X

X

H`ınh 5.3 , ,, , , ´, ho` Trong tru`ong ho. p n` ay, chu vi giam d an ¯i 2. Nhu vây, . trong bâ´t cu , , canh n` ao, sau khi lâ´y d ¯i d ¯ô`ng nickel chu vi d ¯ê`u giam d ¯i ´ıt nhâ´t 2. ,, , `˘ng l´ı luân Ba ay v˜e h`ınh) ta c´ o kê´t luân a nê´u môt . tuong tu. (h˜ . l` . d¯ô`ng , ,, ´ ˘. c 4), v` dime d¯uo. c lây d¯i, chu vi không t˘ ang lên (n´ o c˜ ung giam d¯i 2 hoa a khi , , , , môt o thê t˘ ang lên, nhung nhiê`u nhâ´t l` a ¯i, chu vi c´ . d¯ô`ng quarter d¯uo. c lâ´y d , , , ˘. c giam d ˘. c 4). 2 (n´ o c´ o thê không thay d¯ôi hoa ¯i 2 hoa , , , , `˘ng trên b` Bây gi`o ta s˜e chı ra ra an c`o không thê không c` on môt . d¯ô`ng , ,, ,, , xu n` ao. Gia su lâ´y d an c`o th`ı chu vi s˜e l` a 4.20 = 80. ¯uo. c hê´t d ¯ô`ng xu trên b` , ,, , , ´ ` ` Gia su tai at c´ o p d¯ông penny, n d¯ông nickel, d d¯ô`ng . th`oi d¯iêm xuât ph´ , ˘. t cho ˜ lâ´y d¯i (nhu vây dime, v` a q d¯ô`ng quarter d¯a . p + n + d + q = k). Ða

222

,, , ˜,ng tr` Chuong 5. Nhu o choi to´ an hoc .

, ,, , ˜ d¯uo. c lâ´y d¯i, th`ı t2 − p gon on lai ¯a . t = 10. Nê´u tâ´t ca nh˜ung d¯ô`ng xu c` . d , ,, , ´ ng 4(t2 − p). C˜ d¯ô`ng penny s˜e d¯uo. c lâ´y d¯i, chu vi giam d ung nhu vây, ¯i d ¯u . , , , , , , 2 ´ o s˜e giam d¯i ´ıt t −nd o thê d¯uo. c lây d ¯i, nhung chu vi cua n´ ¯ô`ng nickel c´ , ,, ´ 2 2 ´ ` nhât 2(t − n). (t − d) d¯ông dime d¯uo. c lây d¯i v` a chu vi giam d¯i ´ıt nhâ´t ,, `˘ng 0. Cuô´i c` ba ung, (t2 − q) d¯ô`ng quarter d¯uo. c lâ´y d¯i v` a chu vi t˘ ang nhiê`u nhâ´t l` a 2(t2 − q). , , ´˘c lai Ta nha a gi´ a tri. ban d¯â`u cua chu vi nhiê`u nhâ´t l` a 4k. Nhu vây, . l` . nê´u , , , , tâ´t ca c´ ac d¯ô`ng xu d a tri. cuô´i c` ung cua chu vi nhiê`u nhâ´t l` a ¯uo. c lâ´y d ¯i, gi´ 4k − 4(t2 − p) − 2(t2 − n) + 2(t2 − q) = 4k − 4t2 + 4p + 2n − 2q. , , , `˘ng 4(p + n + d + q) = 4k, nhu, vây ˘. c ba Nhung 4p + 2n − 2q l` a nho hon hoa . 2 2 2 ´ ` ´ chu vi cuôi c` ung nhiêu nhât l` a 4k−4t +4k = 8k−4t = 8.59−4.10 = 72, , , , ` ´ ´i thu. c tê l` ˘ng 80. tr´ ai vo a chu phai ba ,, , , ,, Phuong ph´ ap 2: Ðinh a chu vi nh˜ung v` ung trô´ng (nhu d¯inh . ngh˜ıa P l` . ngh˜ıa o , , , , ´, c´ ach giai trên) v` a cho q, d, n l` a sô´ nh˜ung d ng quarter, dime, v` a ¯ô`ng tuong u , , , , , ´d nickel trên b` an c`o tai at. Khi d¯o ¯iêm xuâ´t ph´ ¯ai . th`oi d . luo. ng P−4q−2d+2n , , , , l` a không d a n´ o không t˘ ang (không thay d ¯ôi d¯on d¯iêu: ¯ôi khi nh˜ung . Ngh˜ıa l` , ,, ,, ,, d¯ô`ng xu d¯uo. c ph´ep lâ´y d¯i). Gi´ a tri. khoi d¯â`u cua d¯ai . luo. ng trên nhiê`u nhâ´t , , ´ gi´ l` a 72, khi d a tri. m` a tâ´t ca c´ ac ô trô´ng trên b` an c`o l` a 80. Vô l´ı, d ay ¯o ¯iê`u n` , , , ˜ d dân an c`o không bao gi`o tâ´t ca c´ ac ô d¯ê`u trô´ng. ¯ê´n trên b`

J

,, ´ ` , , V´ı du. 5.22 (Anh 2002). Môt o choi hai ngu`oi ba ˘t d oi môt ¯âu v´ . tr` . côt . tiê`n n , , , , ˜ c´ d o th` anh hai côt ¯ô`ng xu (n ≥ 3). Ngu`oi choi thu´ nhâ´t chia côt ¯a . tiê`n d . tiê`n ,`, , ´, ˜ c´ tu`y ´y. Nguoi choi thu hai chon ac côt o trên b` an) ¯a . môt . côt . tiê`n (trong c´ . tiê`n d , ,`, ´ ` ´ v` a lai o ra th` anh hai côt uy ´y. Tiêp tuc ˘ng cuôc . chia n´ . tiên t` . nhu vây, . nguoi tha . , , ,`, ,´, , ´ ´ ` ` l` a nguoi choi d¯ên buoc d am cho tât ca côt o môt ˘. c hai d ¯i l` ¯ông . tiên chı c´ . hoa , , ,´, ` ,`, , ´ ` ` tiên. V´ oi gi´ a tri. ban d ao cua n th`ı nguoi d¯i buoc d ˘ng, gia ¯âu n` ¯âu tiên s˜e tha , ,, ,, , , su hai ngu`oi choi d¯ê`u t` ai gioi nhu nhau? , , ´˘ng khi v` ˜˘n; `,i giai. Ngu,`o,i cho,i thu ´, nhâ´t tha ˘. c n cha Lo a chı khi n = 3 hoa

, ˜,ng b` 5.3. Nhu ai to´ an thi hoc . sinh gioi

223

, , , ,, , ´, ´˘ng vo ´,i moi ngu`oi choi thu hai tha a n > 3. Ta c´ o thê kiêm tra kê´t luân . n le v` . , ` ´ ng minh ba ˘ng quy nap. ´ ta chu trên d¯ê´n n = 6 v` a sau d¯o . , ˜˘n, ngu,`o,i thu ´, nhâ´t tao 1) Nê´u n > 6 l` a cha o 1 d¯ô`ng xu) v` a . ra côt . c˜o 1 (c´ , , , , , ´ ` ´ ´ ˜ ` ˘ng côt on−1d a le, nguoi thu nhât s˜e tha ¯ông xu). V`ı n − 1 l` . co n − 1 (c´ , ,, ,`, ,`, ´, , ´, ´ ´ theo gia thiêt quy nap anh nguoi thu hai v` a . (v`ı nguoi choi thu nhât tro th` , ,, ´ ´ ` o vi. tr´ı xuât ph´ at c´ o sô le d¯ông xu). , ,, , ´, ˜˘n 2) Nê´u n ≥ 7 l` a sô´ le, ngu`oi choi thu nhâ´t tao o sô´ cha . ra môt . côt . c´ , , , , , ´ hai lai d¯ô`ng xu v` a môt o sô´ le d¯ô`ng xu. Ngu`oi choi thu . côt . c´ . chia côt . sô´ , , ,, ˜˘n d cha o sô´ le d ach nhu vây, ¯ô`ng xu ra hai côt ¯ô`ng xu. Tiê´p tuc . d¯ê`u c´ . c´ . ngu`oi , ,´, ,, , , ´, , ´, ´p lai choi thu hai luôn luôn d¯a ac buo cd nhâ´t v` ad ¯i cua ngu`oi choi thu ¯ua . c´ , , ,, , ´, ´,i nh˜u,ng sô´ le d¯ô`ng xu. Khi ngu`oi choi thu nhâ´t v` ao t`ınh thê´ chı c´ o sô´ côt . vo , ´,i c˜o, 1 (chı c` c´ ac côt on 1 d¯ô`ng xu), ch´ ung không c` on liên quan d¯ê´n . tiê´n to , ,, ,´, , , , , , buoc choi. Vâ´n d¯ê` nguy kich a c´ o c˜o 3 o môt . cho ngu`oi choi l` . côt . tiê`n: Chı , ,, ,, , ´, , c´ o môt ach duy nhâ´t m` a ngu`oi choi thu hai c´ o thê thua l` a nê´u ngu`oi choi . c´ ,, , ´, , ´, hai tao thu nhâ´t môt a râ´t nhiê`u . cho ngu`oi choi thu . côt . duy nhâ´t c˜o 3 (v` , , , , , , , , , ´, nhâ´t phai côt ¯ô`ng xu). Nhung trong tru`ong ho. p nhu vây . 1d . ngu`oi choi thu ˘. c l` ˘. c l` tao a môt a môt a ¯ô`ng xu, hoa . ra hoa . côt . duy nhâ´t hai d¯ô`ng xu v` . côt . 3d , , , , , , , ´ hai môt u trong tru`ong ho. p n` ao, ngu`oi choi thu . côt . duy nhâ´t 4 d¯ô`ng xu. D` , , , , , , , , `˘ng c´ ´˘ng o ngay buo ´c d¯i tiê´p theo ba ach l` am giam d tha ¯i: Trong tru`ong ho. p ,, ´, nhâ´t, côt thu a côt a . ba d¯ô`ng xu d¯uo. c chia ra côt . 1 d¯ô`ng xu v` . hai d¯ô`ng xu, v` ,`, , , , , , ´ hai, ngu`oi thu ´ hai chia côt trong truong ho. p thu anh hai côt ¯ô`ng xu th` . 4d . , , , , , ˜ ´ ` ´ ´ ` ˘ môi côt hai d ô ng xu. T´ o m l ai, chiê n thu ât th a ng c ua ngu o i cho i th u hai l` a ¯ . . . , , , , ˜ luôn luôn tao an l` a nh˜ung côt ¯ê´n t`ınh thê´ tâ´t ca . ra to` . tiê`n le tr`u khi dân d , ,, c´ ac côt a môt o3d ¯ô`ng xu v` ¯ô`ng xu; trong tru`ong . chı môt . d . côt . duy nhâ´t c´ , , ,, , , , ´˘ng d ho. p nhu vây ¯uo. c thu. c hiên . th`ı chiê´n thuât . chiê´n tha . nhu mô ta trên.

J

,, , , ` ˜,ng d¯ô`ng xu tr` V´ı du. 5.23. Hai ngu`oi choi môt o choi ba ˘ng nhu on b´ an k´ınh . tr` , , , , , , ˜ ˜ nhât 1, trên ma ˘. t b` an h`ınh chu o k´ıch thuo´c m × n. Môi ngu`oi choi c´ o quyê`n . c´

224

,, , ˜,ng tr` Chuong 5. Nhu o choi to´ an hoc .

d ˘. t môt an sao cho không d ac. Khi ba ˘´t d ¯a ¯ô`ng xu trên b` ¯`e lên d ¯ô`ng xu kh´ ¯â`u . d , ,, , ´. Gia su moi `i hoi vô han ma ˘. t b` an không c´ o môt ao trên d ¯ô`ng xu n` ¯o ¯o . d . d . vê` , , , , , , , , , , ˜ ng gi´ ´p u´ng, v´ d oi nhu a tri. n` ao cua m v` a n ngu`oi choi thu´ ¯ô`ng xu d ¯ê`u d ¯uo. c d ¯a nhâ´t c´ o chiê´n thuât ˘´ng? . tha , , `,i giai. Nê´u m < 2 hoa ˘. c n < 2, th`ı không thê d¯a ˘. t môt Lo ao trên . d¯ô`ng xu n` , ,`, ´, ˘. t kh´ b` an nhu vây, ac, nê´u n ≥ 2 v` a m ≥ 2, . vây . nguoi thu nhâ´t thua ngay. Ma , , , ,`, , ´, ` ´ ˘ng nguoi choi thu nhâ´t c´ ˘ng. Goi ta s˜e chı ra ra o thê tha a tâm cua b` an, . O l` , ,`, ´, , ´ nhâ´t cua n´ ˘. t d¯ô`ng xu thu v` a cho nguoi thu nhâ´t d o sao cho tâm d¯ô`ng xu ¯a , ´ ng qua tâm (ngh˜ıa l` ch´ınh l` a O. Câ´u h`ınh c` on lai a d¯ô´i xu a phâ`n c` on lai a . l` . l` , ,´, , , , , , , ◦ ´ hai d ´i O). Bây gi`o ngu`oi choi thu ˘. t nhu nhau duoi ph´ep quay 180 d ¯ô´i vo ¯a , , , , , , , ` ´ nhâ´t d¯a ´i tâm A n` ´c d ˘ng buo ´. Ngu`oi choi thu ´p lai d¯ô`ng xu vo ao d a ¯o ¯i l` . ba , , , , , ◦ ` ´ ´i O. Ngu`oi choi ˘. t d¯ông xu c´ d¯a o tâm tai a ph´ep quay 180 cua A d ¯ôi vo . B, l` , , , , , , , `˘ng c´ ´ ´ nhât s˜e luôn luôn thu. c hiên ´c d ˘. t d thu ach d¯a ¯uo. c buo ¯i cua m`ınh ba ¯ô´i . d , , , `˘ng bu,o ´,ng qua tâm. Ta phai chı ra ra ´,c d¯i nhu, vây xu a luôn luôn thu. c hiên . l` . ,, ´ ng luât d¯uo. c (d ¯u . cho ph´ep). ´,ng nên bu,o ´,c ˘. t d ´ ng luât, V`ı viêc a d¯u ¯a ¯ô`ng xu tai ¯ô´i xu . d . tâm A l` . do t´ınh d , ,, ´, hai ˘. t d¯ô`ng xu tai ´ ng luât. d¯i d¯a ung d ¯u . tâm B c˜ . Nhu vây . sau khi ngu`oi thu ,, ,´, ,, , , ´, ˘. t d¯ô`ng xu tai d nhâ´t d c ho. p lê. tai a d¯ô`ng ¯a ¯i d¯uo. c buo . B m` . A, ngu`oi choi thu , , ´i d¯ô`ng xu c´ xu c´ o tâm tai o tâm tai . B không thê giao vo . A. Thât . vây, . nê´u P , , l` a môt a BP ≤ 1, vây a trung . d¯iêm giao, th`ı AP ≤ 1 v` . AB ≤ 2. Nhung O l` , , AB ` ˘m trên ma ˘. t d¯ô`ng ≤ 1. Ðiê`u n` ay ngh˜ıa l` a A na d¯iêm cua AB, nên AO = 2 ,, , ´c d ˘. t d ´ buo xu c´ o tâm O, vô l´ı. Do d¯o o tâm tai a thu. c hiên ¯i d ¯a ¯ô`ng xu c´ . B l` . ,, d¯uo. c.

J

` tâp Bai . , ,, , , ´,c 1974) Peter v` . 5.24. (Ðu a Paul choi môt o choi nhu sau: Hai ngu`oi lâ`n . tr` , , , ,, ,´, ´ ˜ sô´ tu., nhiên cua : 1, 2, 3, ... ´,n nhâ´t cua môi luo. t x´ ac d c sô chung le lo ¯inh . uo , ,´, ´ , ,, Nê´u uo c sô n` ay chia cho 4 c` on du 1, th`ı Peter mâ´t môt a Paul d¯uo. c . d¯iêm v`

, 5.4. Chuyên d o choi Nim ¯ê` chiê´n thuât . tr`

225

, , ,´, ´ , môt c sô n` ay chia cho 4 du 3, th`ı Paul mâ´t môt a Peter ¯iêm. Nê´u uo . d . d¯iêm v` , , , ,, , , `˘ng ´ ng minh ra d¯uo. c môt ¯iêm. Sau môt . d . th`oi gian ho. d`ung cuôc . choi. Chu ´˘ng. ˜ chiê´n tha Paul d¯a

, ` choi Nim 5.4. Chuyên dê ¯ ` chiê´n thuât . tro ` v´ı du. 5.4.1. Ð.inh ngh˜ıa va , ,, , ,, , , Tr` o choi Nim l` a tr` o choi gi˜ua hai ngu`oi trên môt . sô´ d¯ô´ng soi (o d¯ây ta , , , ˜ ´˘c: Lâ`n lu,o.,t môi x´et cuôc o bô´n d ¯ô´ng soi nhu h`ınh v˜e 5.4) theo quy ta . choi c´ , , ,, ,, ´ hoa ngu`oi d¯uo. c ph´ep lâ´y d ao d ˘. c ¯i môt ¯o . sô´ bâ´t k`ı viên soi trong môt . d¯ô´ng soi n` , ,`, ,`, ´ , , ´ ´ ´ ´. Nguoi thua cuôc lây d ao d a nguoi d¯ên luo. t m` a không ¯i tât ca môt ¯ông n` ¯o . d . l` , , ´ c` on viên soi n` ao d¯ê lây.

8

10

7

1

H`ınh 5.4 , ,, , , , Tr` o choi Nim l` a tr` o choi dân gian cua ngu`oi Trung Quô´c, trong nh˜ung , ,, , ,, , , ´c d ˜ d¯uo. c truyê`n b´ thê´ kı truo a sang châu Âu v` a môt a tr` o choi phô ¯a . th`oi l` , ,, , ,´, , , ,, ˜ c´ biê´n o c´ ac nuo c n` ay. T`u tr` o choi n` ay d o h` ang loat o choi tuong ¯a . kiêu tr` , , , , , , , ˘n nh˜ung l´ı thuyê´t d a t`ım ra tu. ra d o ha ¯ê giai th´ıch v` ¯`oi, không nh˜ung vây, . c´ , , , chiê´n luo. c, chiê´n thuât o choi Nim. Trong phâ`n n` ay ch´ ung tôi . cho loai . tr` , , , , , ˜ ˜ ´c u ´ ng dung chı biên soan a c´ ach thu biêu diên d˜ ay sô´ . môt . phâ`n dê hiêu v` . , , ´˘ng trong tr` nhi. phân d o choi n` ay. ¯ê tao . ra chiê´n thuât . tha , , , , ,, , , ´,i tr` Ðê cu. thê ta chı d¯ê` câp o choi c´ o 4 d¯ô´ng soi v` a sô´ luo. ng nhu trong . to ,, ,, ,, , ,, ´ l` h`ınh v˜e v` a ngu`oi d¯i d a ngu`oi thu´ nhâ´t v` a ngu`oi d a ngu`oi ¯â`u tiên l` ¯i tiê´p d ¯o , , ,, , , ´˘t d ˘. t b` ˘. c thu´ hai. Sô´ luo. ng soi v` a c´ ac d¯ô´ng soi trên ma an l´ uc ba ¯â`u choi hoa

,, , ˜,ng tr` Chuong 5. Nhu o choi to´ an hoc .

226 Lâ`n d¯i 1 2 3 2 2 2 2

,`, ´, nhâ´t d¯i Nguo i thu [8, 10, 3, 1] [7, 7, 3, 1] [6, 5, 3, 1] [5, 4, 3, 1] [5, 3, 0, 1] [0, 3, 0, 1] [0, 0, 0, 1]

,`, ´, hai d Nguo i thu ¯i [8, 7, 3, 1] [7, 5, 3, 1] [6, 4, 3, 1] [5, 4, 0, 1] [2, 3, 0, 1] [0, 1, 0, 1] [0, 0, 0, 0]

, ,´, , Bang 5.1. C´ ac buo c d¯i trong tr` o choi Nim , , ,´, ,, , l` a sau môt c d¯i cua ngu`oi choi ta goi a môt th´ ai. C´ o thê môt . trang . . buo . l` . ,, , , , ,, , ˜ ´ ´n tr` phuong a o choi d¯uo. c ghi lai ac trang th´ ai nhu bang 5.1. Sau môi . voi c´ . , , , ,´, , ,`, , , ´, , ´ ` buoc d¯i cua tung nguoi choi sô viên soi v` a c´ ac d¯ô´ng soi c` on lai ng . tuong u , , , , ,´, ,, , ´ nhu trong bang trang ai. Tr` o choi không thê choi vô han . th´ . buoc v`ı sô luo. ng , , , ,, ´ ´ ˜ trong d a huu han, uc không c` on viên soi n` ao, ngu`oi n` ao ¯ông soi l` . d¯ên môt . l´ ,, d¯ê´n luo. t l` a thua. , ,, , ,´, ´˘ng d Ðê c´ o nh˜ung buo c d¯i th` anh chiê´n thuât o ¯ô´i phuong trong tr` . tha , , , , , ´˘ng d¯ô´i thu ´ u l´ı thuyê´t sau d¯ây d¯ê c´ choi n` ay ta nghiên cu o thê chiê´n tha , ˜ bu,o `˘ng c´ ´,c d¯i. Ta d¯i d¯inh ach theo d˜ oi thay d ac trang th´ ai sau môi ba ¯ôi c´ . . , , ngh˜ıa tông c´ ac sô´ trong tr` o choi Nim: , , ,, , Ðinh ngh˜ıa 5.2. Tông-Nim cua hai sô´ trong tr` o choi Nim d¯uo. c k´ı hiêu a . l` . ´ ˘c sau: ] theo hai nguyên ta , , , , , (A) Tông-Nim hai sô´ c´ o l˜ uy th`ua kh´ ac nhau cua 2 giô´ng nhu tông b`ınh ,, thu`ong trong sô´ hoc. . V´ı du. 4 ] 8 = 12. , , , `˘ng 0. V´ı du. 4 ] 4 = 0. (B) Tông-Nim hai sô´ c´ o c` ung l˜ uy th`ua cua 2 ba , ,, `˘ng moi Ch´ u ´y: Ta biê´t ra ¯ê`u biêu di˜ên theo hê. nhi. . sô´ nguyên duong d ,, phân (c´ o dang a = a0 2n + a1 2n−1 + · · · + an , o d¯ây n ≥ 0, a0 6= 0, . , , , ´,i i = 1, 2, ..., n). Do d ´ Tông-Nim cua hai sô´ không phai l˜ 0 ≤ a i < 2 vo uy ¯o

, 5.4. Chuyên d o choi Nim ¯ê` chiê´n thuât . tr`

227

, , , , , ,, ` , ˘ng c´ th`ua cua 2 c˜ ach chuyên qua biêu di˜ên co sô´ 2 ung thu. c hiên ¯uo. c ba . d , ´˘c trên. D˜ê thâ´y t`u, d¯inh rô`i công o nh˜ung t´ınh . ngh˜ıa trên ta c´ . theo nguyên ta châ´t sau: ,, (C) 0 ] a = a ] 0 = a (phâ`n tu 0).

(D) a ] b = b ] a (t´ınh giao ho´ an).

, (E) (a ] b) ] c = a ] (b ] c) (t´ınh kê´t ho. p). , , , ,, `˘ng quy ta ´˘c trên ta c´ ´, Tông-Nim cua hai sô´ bâ´t Ba o thê t´ınh d ¯uo. c bâ´t cu , `˘ng c´ ´˘c trên nhu, sau: ´p dung k`ı. V´ı du. t´ınh Tông-Nim 5 ] 3 ba ach a c´ ac quy ta . 5 ] 3 = (4 + 1) ] (2 + 1) = (4 ] 1) ] (2 ] 1) =4]61]2]61 =4]2 =6

´˘c (A) theo quy ta ´˘c (B) theo quy ta

´˘c (B) theo quy ta

´˘c (A). theo quy ta

, , , , Ta liêt ¯â`u tiên (bang 5.2, trang sau). . kê Tông-Nim cua 11 sô´ tu. nhiên d ] 0 1 2 3 4 5 6 7 8 9 10

0 0 1 2 3 4 5 6 7 8 9 10

1 1 0 3 2 5 4 7 6 9 8 11

2 2 3 0 1 6 7 4 5 10 11 8

3 3 2 1 0 7 6 5 4 11 10 9

4 4 5 6 7 0 1 2 3 12 13 14

5 5 4 7 6 1 0 3 2 13 12 15

6 6 7 4 5 2 3 0 1 14 15 12

7 6 6 5 4 3 2 1 0 15 14 13

8 8 9 10 11 12 13 14 15 0 1 2

9 9 8 11 10 13 12 15 14 1 0 3

, , , , Bang 5.2. Tông-Nim cua 11 sô´ tu. nhiên d¯â`u tiên , , , Không câ`n bang 5.2 ta c´ o thê t´ınh Tông-Nim c´ ac sô´ nguyên:

10 10 11 8 9 14 15 12 13 2 3 0

,, , ˜,ng tr` Chuong 5. Nhu o choi to´ an hoc .

228 , V´ı du. 5.25. T´ınh Tông-Nim: a) 8 ] 10 ] 7 ] 1; b) 5 ] 1 ] 3 ] 7.

, `,i giai. a) 8 ] 10 ] 7 ] 1 =6 8 ] (6 8 ] 6 2) ] (4 ] 6 2 ] 6 1) ] 6 1 = 4. Lo

J

b) 5 ] 1 ] 3 ] 7 = (6 4 ] 6 1) ] 6 1 ] (6 2 ] 6 1) ] (6 4 ] 6 2 ] 6 1) = 0. , ,, , , `˘ng n´ ´,u k˜ı d¯inh Ta nghiên cu o tuong tu. nhu . ngh˜ıa Tông-Nim th`ı thâ´y ra , , ,´, ´˘c viêc ac sô´ biêu di˜ên duo i dang ung tôi không nha . t´ınh tông c´ . nhi. phân. Ch´ , ,, , lai ach biêu di˜ên môt a c´ ach công ¯ây c´ . od . sô´ co sô´ 10 sang dang . nhi. phân v` . ,, , ´ c´ ac sô´ nhi. phân. Ta d` ung c´ ach công c´ a c sô o d ang nh i phân nhu ng không . . . , c´ o nh´ o. V´ı du. ] 8 3 6 4 9

= = = = =

23 1

22 0

1

1 1 0

21 0 1 1 0 0

20 0 1 0 0 1

, , ← l˜ uy th`ua co sô´ 2

, ← Tông-Nim.

, , , `˘ng c´ ach t´ınh Ban o thê kiêm tra lai ¯oc . d . c´ . Tông-Nim 8 ] 3 ] 6 ] 4 = 9 ba , ,, , ,´, cua v´ı du. trên. Trong khi thu. c hiên . ph´ep t´ınh duoi dang . nhi. phân o trên ta , , , ´, sang h` không d` ung ph´ep nho ang kh´ ac, chı d¯on gian l` a d¯ê´m sô´ 1 trên môt . , , , , ˜ ´ ´ ´ ´ ´ ´ ˘ côt nê u l` a sô ch a n th` ı cho kê t qu a 0 v` a nê u l` a sô l e cho kê t qu a 1 (th u . .c , ,, châ´t d¯ây l` a ph´ep to´ an XOR trong tin hoc ung). Ta kiêm tra . ngu`oi ta hay d` , ,, ´ ng không: lai on d¯u . hai kê´t qua o v´ı du. trên xem c` 3 2 1 0 ] 2 2 2 2 ] 22 21 20 8 = 1 0 0 0 5 = 1 0 1 10 = 1 0 1 0 1 = 0 0 1 7 = 0 1 1 1 3 = 0 1 1 1 = 0 0 0 1 7 = 1 1 1 4 = 0 1 0 0 0 = 0 0 0

, 5.4. Chuyên d o choi Nim ¯ê` chiê´n thuât . tr`

229

, , ,, `˘ng c´ ˜ chı ra ra Ngu`oi ta d ach t´ınh tông c´ ac sô´ theo dang nhi. phân cho ¯a . , , , ´i c´ kê´t qua tr` ung vo ach t´ınh Tông-Nim. , ` choi Nim 5.4.2. Chiê´n thuât . trong tro , , , , ,´, Ta s˜e d` ung Tông-Nim d¯ê du. d¯o´ an buo c tiê´p theo cua ta. Phâ`n c` on lai . , , , , ,, , ` ´ Tông-Nim cua c´ cua chuyên d¯ê ta d¯i x´et nh˜ung trang th´ ai o d¯o ac hat . . soi , , ` ` ´ ´ ng minh nêu Tông-Nim ˘ng 0 hoa ˘. c kh´ c` on lai a ba ac 0. Ta s˜e chu . sau lân d¯i l` ,´, ,´, ,, ´ ´ ´, nhâ´t; ˘ng cho ngu`oi thu tai ac 0 th`ı c´ o chiên thuât ¯i tha . buoc d¯i kh´ . buoc d , , , , , , `˘ng 0 th`ı ngu`oi thu ´˘ng. Tai ´ hai c´ nhung nê´u Tông-Nim ba o thê tha ai . trang . th´ , , , , , ´i Tông-Nim kh´ ´,i vo ac 0 ngu`oi ta goi a trang th´ ai m`o v` a tai th´ ai vo . . l` . trang . , , , `˘ng 0 ngu,`o,i ta goi Tông-Nim ba a trang ai r˜ o. Nhu vây o bang chiê´n . th´ . l` . ta c´ ,, , , thuât . cho hai ngu`oi choi nhu sau: , Tông-Nim 6= 0 0

Tên goi ai . trang . th´ , M`o R˜ o

,, ´ ˘ng Ngu`oi tha ,`, ´, Nguoi thu nhâ´t , , ´, Ngu`oi thu hai

, , , Bang 5.3. Bang chiê´n thuât o choi Nim . trong tr` , , , ,´, , Nh`ın v` ao bang chiê´n thuât cd ¯ua ra nh˜ung buo ¯i . trên trong khi choi s˜e d , , , , , , `˘ng 0 hoa ´c khi d ˘. c kh´ ac 0. Truo ao giai th´ıch th´ıch ho. p d¯ê cho Tông-Nim ba ¯i v` , , ˘. c biêt tai o bang 5.3, ta h˜ ay x´et môt ai d¯a . sao lai . c´ . sô´ trang . th´ . xay ra khi hai ,, , ngu`oi c` ung choi. , ´ • [0, 0, 0, ..., 0] Trang th´ ai không c´ o môt ao trên b` an, khi d ¯o . . viên soi n` , ,`, ´, ,, , , ˜, ´ ´ a thua ngay. nguoi thu nhât d¯ên luo. t v` a không thê d ¯i d ¯uo. c nua, vây . l` , ,`, ,`, , , , ` ´ ´ hai tha ˘ng 0, nguoi thu ˘ng, d¯u ´ ng nhu Truong ho. p n` ay Tông-Nim ba , , ,, bang 5.3. Trang th´ ai nhu thê´ n` ay ngu`oi ta goi a trang ai kê´t th´ uc . th´ . . l` , tr` o choi. , , , , ´, • [1, 0, 0, ..., 0] Trên b` an chı c` on môt nhâ´t s˜e lâ´y viên . viên soi, ngu`oi thu , , , ,, ´˘ng. Tông-Nim cua trang soi duy nhâ´t n` ay v` a l` a ngu`oi chiê´n tha th´ ai .

230

,, , ˜,ng tr` Chuong 5. Nhu o choi to´ an hoc .

, , , , ´, , ´˘ng. ´ ng nhu bang 5.3, ngu`oi thu n` ay l` a 1, trang ai m`o v` a d¯u nhâ´t tha . th´ , , , , ´, , • [n, 0, 0, ..., 0] Chı c´ o n viên soi tai . môt . d¯ô´ng. Theo luât . choi ngu`oi thu , , ,, ´ ,, , ˘ng. Tông-Nim trong tru`ong ho. p nhâ´t lâ´y hê´t viên soi v` a l` a ngu`oi tha , , , , ´, , ´ ng l` n` ay l` a n, trang ai n` ay c˜ ung m`o v` a nhu bang 5.3 d a ngu`oi thu ¯u . th´ ´˘ng. nhâ´t tha , , `˘ng nhau, Tông-Nim ba `˘ng • [n, n, 0, 0, ..., 0] Trên b` an c´ o hai d ¯ô´ng soi ba , , ,, 0 (công theo biêu di˜ên nhi. phân thâ´y ngay) v` a theo bang 5.3 ngu`oi . ´˘ng. Ngu,`o,i thu ´˘ng th`ı cu ´, hai s˜e tha ´, hai muô´n chiê´n tha ´, d ´ ng sô´ thu ¯i d¯u , , ,, , ,`, , , , ´ nhâ´t d viên soi nhu nguoi thu ac v` a nhu vây ¯i, nhung o d¯ô´ng soi kh´ . , ,`, ´, ` ´ ˘ng 0, nguoi thu hai s˜e tha ˘ng. tông Nim luôn luôn ba , , , ˜ ´ • [1, 1, 1, ..., 1] Trên b` an c´ o n viên soi v` a môi d o môt ¯ông soi chı c´ . viên. ,`, ,`, , , Trong truong ho. p n` ay hai nguoi choi thay phiên nhau d¯i d ¯ê´n khi , , , , , , ´˘ng ´ nhâ´t tha không c` on viên soi n` ao n˜ua, vây . th`ı nê´u n le, ngu`oi thu , , , , , `˘ng 1; nê´u n cha ˜˘n ngu`oi thu ´˘ng v`ı Tông-Nim ´ hai s˜e tha v`ı Tông-Nim ba , , `˘ng 0. Ðiê`u n` ´i bang 5.3. ´ ng vo ba ay c˜ ung d¯u , , , ´,ng minh bang 5.3 l` ´ ng, phâ`n sau Nh˜ung v´ı du. trên không phai chu ad ¯u , , , , , `˘ng ´ ng minh d ´i nh˜ung mênh ´ chi tiê´t. Ta d¯ê ´y ra vo d¯ê` cu. thê ta s˜e chu ¯iê`u d ¯o . , ,, , , , , , , ´˘ng, ta phai lu`ong truo ´c nh˜ung kha n˘ d¯ê choi d` anh chiê´n tha ang trang th´ ai . , , , , , , ´i Tông-Nim cua n´ xay ra c` ung vo o. Ngh˜ıa l` a ta phai d a ¯i t´ınh Tông-Nim v` , , , ´i Tông-Nim nhu v´ı du. sau: c´ ac trang ai vo . th´ , , ˜,ng trang V´ı du. 5.26. V´ oi nhu th´ ai sau d ay t´ınh c´ ac Tông-Nim cho c´ ac ¯ây h˜ . ,´, , buoc d ¯i ho. p lê: . a) [2, 2, 3] b) [5, 1]. , , , , `,i giai. a) Tông-Nim 2 ] 2 ] 3 = 3. Nhu,ng bu,o ´,c d Lo ¯i ho. p lê. nhu sau: [1, 2, 3], , , , ´, ´,i Tông-Nim [0, 2, 3], [2, 1, 3], [2, 0, 3], [2, 2, 2], [2, 2, 1], [2, 2, 0] v` a tuong u ng vo 0, 1, 0, 1, 2, 1 v` a 0. , ,´, , b) Tông-Nim 5 ] 1 = 4. C´ ac buo c d¯i ho. p lê. [4, 1], [3, 1], [2, 1], [1, 1], [0, 1], , , , ´, ng l` a 5, 2, 3, 0, 1 v` a 5. [5, 0] v` a c´ o Tông-Nim tuong u

J

, 5.4. Chuyên d o choi Nim ¯ê` chiê´n thuât . tr`

231

, , , Ðinh a Tông-Nim cua trang ai ban d ¯â`u . th´ . ngh˜ıa 5.3. Tông-Nim nguyên gô´c l` , , , ,´, ,´, , , truoc khi choi. Tông-Nim m´ oi l` a Tông-Nim cua trang ai sau môt . th´ . buoc d¯i ´. n` ao d ¯o , T`u v´ı du. trên ta c´ o mênh d¯ê` râ´t hay sau d ¯ây: . , , ˜,a nhu ˜,ng Mênh d ¯ê` 5.1. Tông-Nim nguyên gô´c không bao gi`o xuâ´t hiên . giu . , , Tông-Nim m´ oi.

H`ınh 5.5 , , , ´,ng minh. Vo ´,i viêc Chu ac sô´ nhi. phân ta . biêu di˜ên Tông-Nim theo tông c´ , , , , `˘ng môt ´c d¯i ho. p lê. bâ´t k`ı l` ´ ng môt thâ´y ra am thay d¯ôi d¯u . buo . sô´ hang . trong , , , , , ˘. c tâ´t ca sô´ cua n´ tông nhu h`ınh 5.5. Sô´ hang n` ay s˜e thay d o ¯ôi môt . . sô´ hoa , , , , , ´ ba cua biêu trong c´ ach biêu di˜ên nhi. phân (trong h`ınh 5.5 k´ı tu. sô´ thu , , , , di˜ên c˜ u l` a 1 chuyên th` anh 0). Moi o su. thay d¯ôi cua sô´ hang s˜e xuâ´t . côt . c´ . , , , , ,, ˜˘n le, ngh˜ıa l` hiên a sô´ 1 trong côt ay thay d¯ôi t`u sô´ luo. ng ¯ôi t´ınh cha . thay d . n` , , , , ,, ˜˘n sang le hoa ˜˘n, nhu, vây ´,i kh´ ˘. c t`u sô´ luo. ng le sang cha ac cha . Tông-Nim mo , , , ´i Tông-Nim nguyên gô´c ´ıt nhâ´t l` vo a tai ay (trong h`ınh 5.5 ch˜u . vi. tr´ı côt . n` , , ´, ba trong Tông-Nim nguyên gô´c s˜e thay d sô´ trong côt ¯ôi). . thu , , ´,ng minh trên ta chı d` Ch´ u ´y trong l´ı luân ung môt . chu . sô´ hang . thay d¯ôi , , ´, không quan tâm d¯ê´n t˘ ˘. c giam Tông-Nim. chu ang hoa

J

,, , ˜,ng tr` Chuong 5. Nhu o choi to´ an hoc .

232

, , , , ˜,ng gi´ a tri. nho hon Tông-Nim nguyên gô´c d ˘`m Mênh d¯ê` 5.2. Tâ´t ca nhu ¯ê`u na . , , trong sô´ c´ ac Tông-Nim m´ oi. , , , ´,ng minh. Ta kiêm tra kha ´,ng ˘ng d¯inh ´ ng. Chu Chu ad ¯u . qua v´ı du. 5.26 qua l` , , ,´, , minh mênh d¯ê` n` ay hoi kh´ o môt ut. Ðê d˜ê hiêu ta d¯i qua ba buo c sau: . . ch´ , , , ˜ cho sang môt 1. V´ı du. vê` giam Tông-Nim cua môt ai d ai ¯a . trang . th´ . trang . th´ , , , , ` ´ c´ o Tông-Nim nho hon ba ˘ng c´ ach thay d V´ı du. trang th´ ai ¯ôi môt . sô hang: . . , , ` ` ´ ˘ng 6, muôn t`ım trang ˘ng 3 [8, 7, 5, 12] c´ o Tông-Nim ba ai c´ o Tông-Nim ba . th´ , , ` ´ ´ ˘ng c´ ba ach thay d am thê n` ao? H`ınh 5.6 chı cho ta thâ´y ¯ôi môt . sô hang . th`ı l` , , , , biêu di˜ên cua c´ ac sô´ 8, 7, 5, 12, Tông-Nim 6 v` a Tông-Nim mong muô´n 3 , , ,´, , ´,i Tông-Nim hiên duo i dang nhi. phân. Tông-Nim mong muô´n kh´ ac vo . . th`oi ] 8 7 5 12 6 3

= = = = = =

23 1 0 0 1 0 0

22 21 20 0 0 0 1 1 1 1 0 1 1 0 0 1 1 0 0 1 1 ↑ ↑ | {z } vi. tr´ı thay

, , ←Tông-Nim hiên . th`oi , ←Tông-Nim muô´n c´ o

, , , , ,, , 2 `i hoi thay d H`ınh 5.6. Giam Tông-Nim t`u 6 xuô´ng 3 d¯o a 20 ¯ôi o côt . 2 v` , , , ,, 2 a 20 . Ðê d tai ¯at . d¯uo. c Tông-Nim mong muô´n th`ı ta phai . hai vi. tr´ı côt . 2 v` , , , chon a d¯ôi gi´ a tri. tai ay. Nhung ta chon . môt . sô´ hang . v` . nh˜ung côt . n` . sô´ hang . , , , , ´ ´ ˜ c´ n` ao trong sô c´ ac sô hang d¯a o? Ðê tra l`oi câu hoi n` ay ta x´et h`ınh 5.7 . , , , 2 0 ´ ´ ´i su. biên d¯ôi c´ vo ac sô 8, 7, 5, 12 tai ac côt a 2 . Ta ch´ u ´y l` a sô´ hang . c´ . 2 v` . , , ,´, , ´ ` 8 t˘ ang, c` on lai ac sô hang kh´ ac giam. Nhu vây ¯i ho. p lê. . c´ . . tôn tai . ba buoc d , , ` ´ ˘ng 3 l` t`u trang th´ ai [8, 7, 5, 12] d¯ên trang th´ ai c´ o Tông-Nim ba a [8, 2, 5, 12] . . ,´, , ˘. c [8, 7, 0, 12] hoa ˘. c [8, 7, 5, 9], (thêm v` ´ môt hoa ao d¯o ¯i không ho. p lê. . buoc d [13, 5, 7, 12]).

, 5.4. Chuyên d o choi Nim ¯ê` chiê´n thuât . tr` 8 7 5 12

= = = =

1 0 0 1

0 0 0 1 1 1 1 0 1 1 0 0 ↑ ↑ | {z } vi. tr´ı thay

→ → → →

1 0 0 1

1 0 0 0

233 0 1 0 0

1 0 0 1

= = = =

13 2 0 9

(t˘ ang) , (giam) , (giam) , (giam)

, , 2 H`ınh 5.7. Kê´t qua thay d¯ôi tai a 20 . côt . 2 v`

, , ,, , , ˜ nhi. phân: Ðê nhân 2. Quy ta ˘´c nhân on hon duo´i dang . biêu diên . biê´t sô´ l´ . biê´t , , ´˘c sau: T`ım vê` bên tr´ ´n hon trong dang sô´ n` ao lo nhi. phân theo nguyên ta ai . , ,, ˜ nhi. phân; sô´ c´ ´ hai sô´ kh´ nhâ´t vi. tr´ı o d ac nhau trong dang o sô´ 1 ¯o . biêu diên , , , , , ´n hon 0111(7), 11011(27) nho ho,n tai ay l` a l´ on hon. V´ı du. 1000(8) lo . vi. tr´ı n` sô´ 11101(29). , , , , , , 3. Chu´ng minh mênh ac ¯ê`: Ðê d¯ôi t`u môt . d . Tông-Nim sang môt . Tông-Nim kh´ , , , , , ,´, ˜ nho hon, truoc tiên ta so s´ anh biêu diên nhi. phân cua hai Tông-Nim v` a x´ ac , , ´ l` d¯inh a ch´ ung kh´ ac nhau, ta goi a D-vi. tr´ı. . nh˜ung vi. tr´ı m` . nh˜ung vi. tr´ı d¯o , , , , ˜ Nê´u bâ´t k`ı môt a tri. tai ¯ôi gi´ . sô´ hang . d¯uo. c thay d . môi D-vi. tr´ı, th`ı Tông-Nim , mong muô´n s˜e cho kê´t qua. Nê´u sô´ hang c´ o 0 tai ai nhâ´t, th`ı s˜e . . D-vi. tr´ı tr´ , , , ´i nh˜ung sô´ hang ´ sô´ hang thay th` anh 1 v` a khi d¯o ang; nhung vo o 1 tai . s˜e t˘ . c´ . , , ` ˘ng 0 v` ´ D-vi. tr´ı tr´ ai nhâ´t, n´ o s˜e thay ba a nhu vây s˜e giam. Do d ¯o . sô´ hang . , , , ,´, ,`, , d¯ê t`ım buoc d¯i ho. p lê. cho ph´ep thay d¯ôi trong Tông-Nim, nguoi ta chon . , , ˜ ´i sô´ 1 tai môt vo ai nhâ´t, v` a thay d o tai ¯ôi n´ . sô´ hang . . D-vi. tr´ı tr´ . môi D-vi. tr´ı. 2 0 ` (Trong phân 1 v´ı du. h`ınh 5.6 th`ı D-vi. tr´ı l` a côt a 2 . Sô´ hang 12 c´ o . 2 v` . , , , 2 ´ 1 tai a thay d o tai ¯ôi n´ . D-vi. tr´ı (côt . 2 ) v` . nh˜ung D-vi. tr´ı s˜e sinh ra sô nho , , `˘ng luôn luôn tô`n tai ´, hon). Ðê thâ´y ra c´ o 1 tai . môt . sô´ hang . . D-vi. tr´ı, ta nho , , , , ,, ,´, , lai a Tông-Nim mong muô´n nho hon Tông-Nim hiên c2o . l` . th`oi, theo buo , , , , , trên, Tông-Nim hiên o 1 v` a Tông-Nim mong muô´n phai c´ o 0 tai . th`oi phai c´ . , , 2 ´ D-vi. tr´ı tr´ ai nhât (trong v´ı du. trên tai o 1 v` a . 2 vi. tr´ı, Tông-Nim hiên . th`oi c´

,, , ˜,ng tr` Chuong 5. Nhu o choi to´ an hoc .

234

, , , Tông-Nim mong muô´n c´ o 1 tai ai o 0). V`ı Tông-Nim hiên . D-vi. tr´ı tr´ . th`oi c´ , , , , ´i 1 tai nhâ´t, nên phai tô`n tai vo ai nhâ´t . môt . sô´ le nh˜ung sô´ hang . . D-vi. tr´ı tr´ , , , ´,i 1 tai n` ay. V`ı 0 không phai l` a sô´ le, nên phai tô`n tai . ´ıt nhâ´t môt . sô´ hang . vo . , , ,´, ´ D-vi. tr´ı tr´ ai nhât n` ay, v` a nhu vây o tai ay l` a môt ¯ôi n´ . thay d . D-vi. tr´ı n` . buoc , , ´ d¯i ho. p lê. cho ra Tông-Nim mong muôn. , , , ,, V´ı du. sau chı ra k˜ı n˘ ang su dung mênh d¯ê` trên d¯ê thu. c hiên an . . . t´ınh to´ , , ´ ´ su. biên d¯ôi môt . sô hang. . , , `, trang V´ı du. 5.27. Ba ˘´t d ai tr` o choi Nim [11, 22, 33] c´ o Tông-Nim l` a ¯â`u tu . th´ , , , , , , , , , 60. H˜ ay chı ra l` am thê´ n` ao d o buo´c d ¯i ho. p lê. d ¯ê chuyên d ¯ôi Tông-Nim d ¯ê´n ¯ê c´

J

a) 55;

b) 0.

, , , `,i giai. Ta kiêm tra lai Lo . Tông-Nim: ] 11 22 33 60

= = = =

25 0 0 1 1

24 0 1 0 1

23 1 0 0 1

22 0 1 0 1

21 1 1 0 0

20 1 0 1 0

, , , , a) Ta so s´ anh hai biêu di˜ên nhi. phân cua Tông-Nim hiên o v` a Tông-Nim . c´ muô´n c´ o: 60 55

= =

25 1 1

24 1 1

23 22 21 20 1 1 0 0 0 1 1 1 ↑ ↑ ↑ | {z } ´ côt ac nhau . sô kh´

, Vi. tr´ı côt ac nhau bên tr´ ai nhâ´t l` a 23 . Trong c´ ac sô´ hang o . kh´ . 11, 22, 33 chı c´ , , ,, , ,´, , ` ´ ˘ng sô 11 c´ o 1 o vi. tr´ı n` ay v` a nhu vây o buoc d ¯i ho. p lê. cho Tông-Nim ba . d¯ê c´ , , , 3 1 0 55 l` a thay d a 2 , ngh˜ıa l` a d¯ôi t`u 11(001011)2 th` anh ¯ôi 11 tai . vi. tr´ı 2 , 2 v` 0(000000)2 .

, 5.4. Chuyên d o choi Nim ¯ê` chiê´n thuât . tr`

235

, , ,´, b) Ta so s´ anh hai sô´ biêu di˜ên duo i dang a 0: . nhi. phân cua 60 v`

60 0

= =

25 24 23 22 1 1 1 1 0 0 0 0 ↑ ↑ ↑ ↑ | {z } ´ côt sô kh´ a c nhau .

21 0 0

20 0 0

, ,, Côt ac tr´ ai nhâ´t o vi. tr´ı 25 . Trong c´ ac sô´ hang 11, 22, 33 chı c´ o 33 c´ o . sai kh´ . , , 5 4 3 2 ´ 1 tai ay v` a nêu ta thay d¯ôi tai a 2 cua 33, ta nhân . vi. tr´ı n` . vi. tr´ı 2 , 2 , 2 v` . , , , ,, ,´, , ` ´ buoc d d¯uo. c (011101)2 = 29. Do d a giam ¯o ¯i ho. p lê. chuyên Tông-Nim vê 0 l` , ´ t`u 33 xuông c` on 29. , , , ´ ng minh bang chiê´n thuât ´ ng thông qua mênh Chu o choi Nim d¯u d¯ê`: . tr` .

J

, ˜,ng mênh ´ng: Mênh d o choi Nim, nhu ¯ê` sau d ¯u ¯ê` 5.3. Trong tr` . d . , , ,´, `,. ` môt th´ ai r˜ o d¯ê`u d th´ ai mo (N1 ) Buoc d ¯i bâ´t k`ı tu ¯u a d ¯ê´n môt . trang . trang . . , ,, ,, `, bâ´t k`ı d `, môt (N2 ) Tu th´ ai mo o thê chon ¯i d¯ê´n trang ¯ê`u c´ ¯uo. c buo´c d . trang . d . . th´ ai r˜ o.

, ,, , , ˜, `, d ´ không thê c´ a) l` a môt (N3 ) Trang th´ ai kê´t th´ uc (tu o buo´c d¯i d ¯o ¯uo. c nu . . trang th´ a i r˜ o . . ,, , ´,ng minh. Mênh ˜ d¯uo. c n´ Chu oi d¯ê´n trong phâ`n nh˜ung trang ai . d¯ê` (N3 ) d¯a . th´ ,, , , ˘. c biêt d¯a o choi Nim o trên. C` on (N1 ) v` a (N2 ) d˜ê d` ang suy ra t`u . trong tr` , mênh d a mênh d¯ê` 5.2 vê` Tông-Nim. ¯ê` 5.1 v` . . , , `˘ng 0 v` a nhu vây Nê´u ta c´ o môt ai r˜ o th`ı Tông-Nim ba . theo mênh . . trang . th´ , , , , , ´c d¯i ho. p lê, d¯ê` 5.1 sau buo Tô ng-Nim không thê l` a 0, ngh˜ ıa l` a tr ang th´ ai . . , , ´i s˜e l` mo a m`o. , , ´,n ho,n 0 v` Nê´u môt ai l` a m`o th`ı Tông-Nim s˜e l` a môt a theo . trang . th´ . sô´ lo , ,, ,´, , , ` ´i trang mênh d¯ê 5.2, n´ o c´ o kha n˘ ang t`ım d ¯uo. c môt ¯i ho. p lê. to . . môt . buoc d . th´ ai r˜ o.

J

236

,, , ˜,ng tr` Chuong 5. Nhu o choi to´ an hoc .

, , , ´ ng dung ´ `u no 5.4.3. Ch´ ung minh bang chiê´n thuât . va . , , ´,ng minh bang chiê´n thuât ´ ng: Bây gi`o ta d¯i chu . 5.3 d¯u , ,, , ,, , ´ theo (N2 ) ngu`oi thu´ nhâ´t a) Gia su trang ai nguyên gô´c l` a m`o: Khi d¯o . th´ , ,´, ,, , ,´, , c´ o thê c´ o buo cd ai r˜ o; ngu`oi n` ay goi c d¯i nhu vây ¯i d¯ê´n trang . th´ . nh˜ung buo . , ,, ,, ,, , ´ nê´u ngu`oi thu´ hai c´ l` a buo´c d o thê c` on d¯i d¯uo. c (nê´u trang th´ ai ¯i tô´t. Sau d¯o . , , , , , , , không phai l` a kê´t th´ uc tr` o choi), ngu`oi n` ay theo (N1 ) chı c´ o thê d¯ua vê` , , , , ,, , , , , ´c d trang ai m`o. Tiê´p tuc ¯i tô´t, ngu`oi thu´ . thu. c hiên . buo . th´ . ngu`oi thu´ nhâ´t lai ,´, , ,, , ´, cho,i theo ´,i trang hai s˜e lai o buo cd ai m`o, ... Nê´u ngu`oi thu´ nhâ´t cu ¯i t o . c´ . th´ , , ´, ,´, , ,, i d˜ ay trang c´ ach n` ay (buo c d¯i tô´t), th`ı ngu`oi thu´ hai luôn luôn d¯ê´n luo. t vo . , , ,´, , , , ˜ ´c. Ðê´n môt th´ ai r˜ o, môi buoc d l´ u c n` ao ¯i k´eo theo giam sô´ viên soi hon truo . , ,`, ´, , ´i trang ˘. t vo ´ nguoi thu hai s˜e phai d¯ô´i ma d ai kê´t th´ uc v` a bi. thua. ¯o . th´ ,, , ´ ,´, , , , b) Tuong tu. , nêu trang ai nguyên gô´c l` a r˜ o: ngu`oi thu´ nhâ´t s˜e c´ o buo c . th´ , , , , , , , , , ´ ` ´ ´i d˜ d th´ ai m`o v` a ngu`oi thu´ hai c´ o thê dôn ngu`oi thu´ nhât vo ay ¯i d ¯ên trang . ,´, ,`, ´, ´ ´ ´ ´ buoc d th´ ai r˜ o; tât nhiên nguoi thu nhât d uc n` ao d¯o ¯i cho trang ¯ên môt . . l´ , , , , , ´ ´ ´ ´i trang ˘. t vo ˘ng. ´ ngu`oi thu´ hai s˜e tha phai d¯ôi ma ai kêt th´ uc v` a khi d ¯o . th´

, , , H`ınh 5.8. So d¯ô` b´ı quyê´t cua tr` o choi Nim , , , , T´ om lai, o choi Nim, th`ı trang th´ ai s˜e phai thay nhau . d¯ê choi tô´t tr` . , , , , ´,i khi d ´,i trang chuyên d¯ôi gi˜ua r˜ o v` a m`o cho to th´ ai kê´t th´ uc. Nê´u ¯at . to .

, 5.4. Chuyên d o choi Nim ¯ê` chiê´n thuât . tr`

237

, , ´, , không c´ o g`ı sai lâ`m th`ı nê´u trang ai nguyên gô´c l` a m`o th`ı ngu`oi thu nhâ´t . th´ , , , , , ´ ´˘ng ´ hai phai tha ˘ng, nê´u trang phai tha th´ ai nguyên gô´c l` a r˜ o th`ı ngu`oi thu . , , , , ` nhu bang chiê´n thuât a thu. c hiên ¯ô 5.8. . 5.3 v` . theo so d , ,, , `, `, nhu ˜,ng trang V´ı du. 5.28. Tu ai m`o sau d ay t`ım tâ´t ca buo´c d ¯i tô´t tu ¯ây h˜ . th´ ch´ ung: a) [3, 4, 8, 9]

b) [8, 10, 7, 1]

c) [9, 25, 49]

d) [6, 5, 3, 1].

, , , `,i giai. a) Tông-Nim 3 ] 4 ] 8 ] 9 = 6, ta kiêm tra lai: Lo . ] 3 4 8 9 6

= = = = =

23 0 0 1 1 0

22 0 1 0 0 1

21 1 0 0 0 1

20 1 0 0 1 0

, , , , , ,´, `˘ng 0, nh`ın v` Ðê c´ o buo cd a ta phai d ao bang ¯i tô´t ngh˜ıa l` ¯ôi Tông-Nim ba , , 2 trên ta phai thay d a 21 môt ac sô´ hang. Tai ¯ôi tai . vi. tr´ı côt . 2 v` . trong c´ . . vi. tr´ı , , ,´, 2 ˜ ´ ´. Do d¯o ´ buoc d¯i tô´t l` côt o sô 4 l` a c´ o biêu diên nhi. phân 1 tai a ¯o . 2 chı c´ . d , , ,, , ´ ´ ´ d¯ôi 4(= 0100) th` anh 2(= 0010). Ngh˜ıa l` a lây o d ¯ông thu hai d¯i hai viên soi. , , , , , ˘. c Meo o choi Nim: Trong khi choi tr` o n` ay không thê ke bang hoa . trong tr` , , ,, biê´n d¯ôi nhi. phân c´ ac sô´ rô`i t´ınh tông râ´t phiê`n h` a. C´ o môt ach l` a su dung . c´ . , ,, ´ , ph´ep Tông-Nim d¯uo. c tiên h` anh nhu sau (h`ınh 5.9): , , , ˜ d¯ô´ng soi 1. Chia môi lâ´y phâ`n l˜ uy th`ua cua 2 gâ`n nhâ´t, v´ı du. [3, 4, 8, 9] chia th` anh d˜ ay 1, 2, 4, 8, 8, 1. , , ,, 2. Theo Tông-Nim trong d˜ ay c´ o hai 8, 8 v` a 1, 1 d on ¯ê`u bi. truo. t tiêu, chı c` , , , , ` ` ˘ng 0 th`ı tông hai sô´ n` ˘ng lai am Tông-Nim ba ay phai ba ¯ê l` . d¯ô´ng 2, 4 d , ,, 0 ngh˜ıa l` a phai lâ´y d ¯i 2 viên o d ¯ô´ng 4 viên. , `˘ng 4: b) Tông-Nim ba

,, , ˜,ng tr` Chuong 5. Nhu o choi to´ an hoc .

238

? 4

3

] 8 10 7 1 4

8 H`ınh 5.9 23 1 1 0 0 0

= = = = =

22 0 0 1 0 1

9

21 0 1 1 0 0

20 0 0 1 1 0

, , , , ,, 2 `˘ng 0, ta phai thay d¯ôi môt Ðê Tông-Nim ba ay . sô´ hang . o côt . 2 . Trong côt . n` , , , , ´c d¯i tô´t nhâ´t l` chı c´ o 7 c´ o sô´ 1, vây a chuyên 7(= 0111) th` anh 3(= 0011), . buo , ,, , ´ ba d ngh˜ıa l` a lâ´y o d ¯ô´ng thu ¯i 4 viên soi. , `˘ng 33: c) Tông-Nim ba ] 9 25 49 33

25 0 0 1 1

= = = =

24 0 1 1 0

23 1 1 0 0

22 0 0 0 0

21 0 0 0 0

20 1 1 1 1

, , , , ,, 5 `˘ng 0, ta phai thay d¯ôi môt Ðê Tông-Nim ba o côt a môt . sô´ hang . . 2 v` . sô´ , ,, , , 0 5 ´c d¯i tô´t nhâ´t l` hang o côt o 49 c´ o sô´ 1, vây a . . 2 . Trong côt . 2 chı c´ . buo , ,, , ´ ´ ´ chuyên 49(= 110001) th` anh 16(= 010000), ngh˜ıa l` a lây o d ¯ông thu ba d¯i , 33 viên soi. , `˘ng 1: d) Tông-Nim ba ] 6 5 3 1 1

= = = = =

22 1 1 0 0 0

21 1 0 1 0 0

20 0 1 1 1 1

, 5.4. Chuyên d o choi Nim ¯ê` chiê´n thuât . tr`

239

, , , , ,, 0 `˘ng 0, ta phai thay d¯ôi môt Ðê Tông-Nim ba ay . 2 . Trong côt . n` . sô´ hang . o côt , , , ´c d c´ o ba sô´ 5, 3, 1 c´ o sô´ 1, vây o ba kha n˘ ang l` a lâ´y d ¯i tô´t nhâ´t c´ ¯i 1 viên . buo ,, ´. o môt ¯ô´ng d¯o . trong ba d

J

` tâp Bai . , ,, , , . 5.29. (Tr` o choi Nim giai ô) Môt ang giâ´y d` ai d anh nh˜ung ô ¯uo. c chia th` . b˘ vuông v` a g´ an nh˜ an 0, 1, 2, 3, ...(h`ınh 5.10).

H`ınh 5.10 , ,, , ˜ ô vuông c´ ˘. t môt ˘. c môt Trên môi o thê d¯a ai quân c`o. Hai ngu`oi lâ`n . hoa . v` , ,, , ,´, ,´, , ´ ng 1 quân vê` luo. t thu. c hiên ac buo cd cd a chuyên d ¯i: Môt ¯i ho. p lê. l` ¯u . c´ . buo , , , , , , ´˘ng cuôc ´c cuô´i c` ph´ıa tr´ ai (vê` sô´ 0). Ngu`oi tha a ngu`oi d¯i buo ung vê` chuô`ng . l` , , , , ´i tr` (ô nh˜ an sô´ 0). H˜ ay t`ım mô´i quan hê. gi˜uu tr` o choi n` ay vo o choi Nim v` a ,´, , , , ´ ´c. t`ım buoc d ai cho truo ¯i tôt t`u môt . trang . th´ , , . 5.30. (Tr` o choi Nim quân c`o) , , ,, , ˜ ô d¯u,o.,c d ˘. t môt ˘. c Tr` o choi d an c`o vua. Trên môi ¯uo. c thu. c hiên ¯a . trên b` . hoa ,`, , ,´, ,´, , , v` ai quân c`o. Hai nguoi choi thay phiên nhau thu. c hiên ¯i: Môt . buoc d . buoc , , , , ´ ´ ng môt d¯i ho. p lê. l` a chuyên d¯u ao . quân c`o t`u môt . ô sang bât k`ı ô vuông n` , , ` ` ` ˘. c vê ph´ıa tr´ vê ph´ıa trên hoa ai (nhung không d¯ông th`oi). Muc a dô`n . d¯´ıch l` , , ,, , , ´nh dâ´u l` hê´t tâ´t ca quân c`o vê` ô vuông d¯uo. c d¯a a chuô`ng (h`ınh ??) v` a ngu`oi , , ,, ´ , ˘ng cuôc. ay giai th´ıch mô´i d¯ua quân c`o cuô´i c` ung vê` chuô`ng l` a ngu`oi tha . H˜ ,´, , , , ´,i tr` liên quan tr` o choi n` ay vo o choi Nim v` a t`ım buo c d¯i tô´t t`u môt ai . trang . th´ , ˜ cho (v´ı du. nhu h`ınh ??). d¯a

, ` LIÊU TAI . THAM KHAO , , ,, [1] Kvant (Tap an hoc a tru`ong phô thông cua Liên Xô c˜ u v` a . ch´ı to´ . cho nh` ng` ay nay l` a Liên bang Nga, n˘ am 1970-2002) ˜,ng suy luân [2] G. Polya, To´ an hoc a nhu o l´ı, NXBGD, 1995. . v` . c´ [3] R. j. McEliece, R. B. Ash, C. Ash, Introduction to Discrete Mathematics, McGraw-Hill Book Co., 1989. , , [4] G. H. Hardy, J.E. Littlewood, G. Polya Bâ´t d ˘ng thu´c, NXB ÐHQG H` a ¯a Nôi, 2002. . ,, [5] Nguy˜ên V˘ an Mâu, am, NXB GD, 1998. . Phuong tr`ınh h` , , ,, , [6] Nguy˜ên H˜uu Ðiên, Phuong ph´ ap Ðirichle v` a u´ng dung, NXB KHKT, . 1999. , , ,, [7] Nguy˜ên H˜uu Ðiên, Phuong ph´ ap Quy nap an hoc, . to´ . NXB GD, 2000. , , , ,, , , [8] Nguy˜ên H˜uu Ðiên, Phuong ph´ ap Sô´ phu´c v´ oi h`ınh hoc ˘ng, NXB . pha ÐHQG, 2000. , , , , , ˜,ng phu,o,ng ph´ [9] Nguy˜ên H˜uu Ðiên, Nhu ap d an phô ¯iên h`ınh trong giai to´ thông, NXB GD, 2001. , , , , ˜,ng phu,o,ng ph´ [10] Nguy˜ên H˜uu Ðiên, Nhu ap giai b` ai to´ an cu. c tri. trong h`ınh hoc, . NXB KHKT, 2001. , , , , [11] Nguy˜ên H˜uu Ðiên, S´ ang tao an phô thông, NXB GD, 2002. . trong giai to´ , , , , [12] Nguy˜ên H˜uu Ðiên, Ða thu´c v` a u´ng dung, NXB GD, 2003. .

240

, ˜ NHUNG K´I HIÊU .

, ´,i c´ Trong cuô´n s´ ach n` ay ta d` ung nh˜ung k´y hiêu ac ´y ngh˜ıa x´ ac d¯inh . . vo , ,´, trong bang duoi d ây: ¯ N Z Q R C ≡ Ckm .. . . 6 .. USCLN BSCNN deg

J

IMO APMO

, , tâp . ho. p sô´ tu. nhiên , tâp . ho. p sô´ nguyên , , , tâp . ho. p sô´ h˜uu tı , , tâp . ho. p sô´ thu. c , ´ ´, tâp . ho. p sô phuc , dâ´u d¯ô`ng du , , , ,, tô ho. p châp . k cua m phâ`n tu ph´ep chia hê´t không chia hê´t ,´, ´ ´,n nhâ´t uo c sô chung lo , bôi . sô´, chung nho nhâ´t ´,c bâc ¯a thu . cua d , ´ ng minh Kê´t th´ uc chu International Mathematics Olympiad Asian Pacific Mathematics Olympiad

241

MUC . LUC . `,i n´ Lo oi d ¯â`u . . . . . . . . . . . . . . . . . . . . . . . . . . . . . . . . . . . . . . . . . . . . . . . . . . . . . . . . . ,, Chuong 1. Nguyên l´ı bâ´t biê´n. . . . . . . . . . . . . . . . . . . . . . . . . . . . . . . . . . . . . . ,, ,, ´,i thiêu 1.1. Gio ap d¯ai . phuong ph´ . luo. ng bâ´t biê´n . . . . . . . . . . . . . . . . . . . .

3 7 7

1.2. Ph´ at hiên ai to´ an . . . . . . . . . . . . . . . . . . . . . . . . . . . . . 9 . bâ´t biê´n trong b` , , , `˘ng d¯ai 1.3. Giai to´ an ba . luo. ng bâ´t biê´n . . . . . . . . . . . . . . . . . . . . . . . . . . . 24 , 1.4. Bâ´t biê´n d ¯on d ¯iêu . . . . . . . . . . . . . . . . . . . . . . . . . . . . . . . . . . . . . . . . . . . . 27 , 1.5. Nh˜ung b` ai to´ an nâng cao . . . . . . . . . . . . . . . . . . . . . . . . . . . . . . . . . . . . 43 1.6. Chuyên d¯ê` vê` h` am bâ´t biê´n. . . . . . . . . . . . . . . . . . . . . . . . . . . . . . . . . .

51

1.6.1. Ðinh am bâ´t biê´n trên trang ai . . . . . . . . . . . . . . . . . ngh˜ıa h` . th´ , 1.6.2. Hê. thô´ng bâ´t biê´n d ¯â`y d ¯u . . . . . . . . . . . . . . . . . . . . . . . . . . . . . . . ,, , , ´c d ´ ng hai biê´n . . . . . . . . . . . . . . . . . . . . . . . . . . . . Chuong 2. Ða thu ¯ô´i xu

52 58 64

2.1. Ðinh a t´ınh châ´t . . . . . . . . . . . . . . . . . . . . . . . . . . . . . . . . . . . . . 65 . ngh˜ıa v` , , ´,c hai biê´n . . . . . . . . . . . . . . . . . . . . . . . . . . 67 2.2. Ðinh . l´ı co ban cho d¯a thu , ,, ´,ng . . . . . . . . . . . . . . . . . . . . . . . . . . . . . . . 71 2.3. Giai hê. phuong tr`ınh d¯ô´i xu , ,, ´,ng . . . . . . . . . . . . . . . . . . . . . . 77 2.4. Ðua vê` hê. phuong tr`ınh dang ¯ô´i xu . d , ´,c d ´,ng . . . . . . . . . . . . . . . . . . . . . . . . . 82 ´,ng minh bâ´t d ˘ng thu 2.5. Chu ¯ô´i xu ¯a ´,c bâc 2.6. B` ai to´ an vê` tam thu . hai . . . . . . . . . . . . . . . . . . . . . . . . . . . . . . . . 87 ´,c d ´,ng ra th`u,a sô´ . . . . . . . . . . . . . . . . . . . . . . . 91 2.7. Phân t´ıch d¯a thu ¯ô´i xu , 2.8. Nh˜ung b` ai to´ an kh´ ac . . . . . . . . . . . . . . . . . . . . . . . . . . . . . . . . . . . . . . . . 95 ,, ´,ng . . . . . . . . . . . . . . . . . 101 2.9. Chuyên d¯ê` vê` phuong tr`ınh hê. sô´ d ¯ô´i xu , , , ´,c hê. sô´ d¯ô´i xu ´,ng . . . . . . . . . . . . . . 101 2.9.1. Ðinh . l´ı co ban cua d¯a thu 242

MUC . LUC . , , ,, 2.9.2. Nh˜ung v´ı du. giai phuong tr`ınh bâc . cao. . . . . . . . . . . . . . . . . , , , , , 2.10. Go. i ´y v` a tra l`oi b` ai tâp . chuong 2 . . . . . . . . . . . . . . . . . . . . . . . . . . , , ,, , ´, tu., . . . . . . . . . . . ´ c cua c´ ˘ng thu Chuong 3. Bâ´t d ac d˜ ay sô´ d¯ô`ng thu ¯a ´, tu., hai hoa ˘. p thu ˘. c ba sô´. . . . . . . . . . . . . . . . . . . . . . . . . . . . . . . . . . 3.1. Ca , 3.2. Tông qu´ at h´ oa . . . . . . . . . . . . . . . . . . . . . . . . . . . . . . . . . . . . . . . . . . . . . ,, ´, tu., . . . . . . . . . . . . . . . . . . . . . . . . . . . . 3.3. Su dung c´ ac d˜ ay sô´ d ¯ô`ng thu . , ´,c Karamata . . . . . . . . . . . . . . . . . . . . . ˘ng thu 3.4. Chuyên d¯ê` vê` bâ´t d¯a , ´,c Karamata . . . . . . . . . . . . . . . . . . . ˘ng thu 3.4.1. H` am lô`i v` a bâ´t d¯a , ,, ´,c Karamata . . . . . . . . . . . . . . . . . . . . . . ˘ng thu 3.4.2. Su dung bâ´t d ¯a . , ´,c kh´ ˘ng thu 3.4.3. Môt ac . . . . . . . . . . . . . . . . . . . . . . . . . . . . . sô´ bâ´t d¯a , , ´,c trong tam gi´ ˘ng thu 3.4.4. Nh˜ung bâ´t d ac . . . . . . . . . . . . . . . . . . . ¯a , ,, , , 3.5. Go. i ´y v` a tra l`oi b` ai tâp . chuong 3 . . . . . . . . . . . . . . . . . . . . . . . . . . . ,, ,, Chuong 4. Phuong tr`ınh h` am . . . . . . . . . . . . . . . . . . . . . . . . . . . . . . . . . . . ,, , 4.1. Phuong ph´ ap thê´ nh˜ung gi´ a tri. d ¯ô´i sô´ . . . . . . . . . . . . . . . . . . . . . . , ,, 4.2. Phuong ph´ ap d ¯iêm bâ´t d¯ông . ............................... ´,c giao ho´ 4.3. Chuyên d ac d an . . . . . . . . . . . . . . . . . . . . . . . . ¯ê` vê` c´ ¯a thu 4.3.1. Ðinh . ngh˜ıa . . . . . . . . . . . . . . . . . . . . . . . . . . . . . . . . . . . . . . . . . . . . ´,c Chebyshev . . . . . . . . . . . . . . . . . . . . . . . . . . . . . . . . . . . . 4.3.2. Ða thu , ´,c giao ho´ 4.3.3. B` ai to´ an tông qu´ at vê` c´ ac d an . . . . . . . . . . . ¯a thu , , , , , 4.4. Go. i ´y v` a tra l`oi b` ai tâp . chuong 4 . . . . . . . . . . . . . . . . . . . . . . . . . . . ,, , , ˜ ng tr` Chuong 5. Nhu o choi to´ an hoc . ............................. 5.1. Chiê´n thuât . bâ´t biê´n . . . . . . . . . . . . . . . . . . . . . . . . . . . . . . . . . . . . . . . , ´,c to´ ´,i tr` 5.2. Kiê´n thu an vo o choi . . . . . . . . . . . . . . . . . . . . . . . . . . . . . . . . . , , 5.3. Nh˜ung b` ai to´ an thi hoc . sinh gioi . . . . . . . . . . . . . . . . . . . . . . . . . . .

243 105 110 132 134 143 149 156 156 163 166 168 171 176 176 187 194 194 197 200 203 205 206 210 214

244

MUC . LUC .

, 5.4. Chuyên d¯ê` chiê´n thuât o choi Nim . . . . . . . . . . . . . . . . . . . . . . . . . tr`

225

5.4.1. Ðinh a v´ı du. . . . . . . . . . . . . . . . . . . . . . . . . . . . . . . . . . . . . ngh˜ıa v` , 5.4.2. Chiê´n thuât o choi Nim . . . . . . . . . . . . . . . . . . . . . . . . . trong tr` , ´,ng minh bang chiê´n thuât ´,ng dung 5.4.3. Chu au n´ o......... . v` . , T` ai liêu . tham khao . . . . . . . . . . . . . . . . . . . . . . . . . . . . . . . . . . . . . . . . . . . . . , Nh˜ung k´ı hiêu . ..................................................

225

241

Muc . luc . ........................................................

242

229 236 240

, Chiu ach nhiêm . tr´ . xuâ´t ban: , , ` ´ Chu tich am d ¯ô´c NGÔ TRÂN AI . HÐQT kiêm Tông Gi´ , , , , ˜ DUONG THUY Ph´ o Tông Gi´ am d¯ô´c kiêm Tông biên tâp . VU . Biên tâp . nôi . dung: , ` ˜ TRÂN HUU NAM Tr`ınh b` ay b`ıa: ` ´ HANH TRÂN THI. THUY . , Chê´ ban: , ˜ HU ˜,U ÐIÊN NGUYÊN ,, , Sua ban in: , ˜ HÔNG ` NGUYÊN SON

, , , , , ´ ´ ˘` ´ ÐAI ´ BANG PHUONG PHAP GIAI TOAN . LUO . NG BÂT BIÊN M˜ a sô´: 8I004T5-TTS , , In 3.000 ban (10TK), khô 14, 3 × 20, 3 cm Tai a Tây . Công ty In H` , Sô´ in: 29/TK. Sô´ xuâ´t ban: 21/727-05. , , In xong v` a nôp ang 4 n˘ am 2005. . luu chiêu th´

View more...

Comments

Copyright ©2017 KUPDF Inc.
SUPPORT KUPDF